Wikipedia:Teahouse: Difference between revisions
→Unicode or… something?: new section |
PrimeHunter (talk | contribs) →Hang on...: it could be your own views while editing the article |
||
Line 1: | Line 1: | ||
{{Short description| |
{{Short description|Community Q&A hub for new editors}}{{pp-sock|small=yes}} |
||
{{skip to top and bottom}} |
{{skip to top and bottom}} |
||
{{User:MiszaBot/config |
{{User:MiszaBot/config |
||
|archiveheader = {{Automatic archive navigator}} |
|archiveheader = {{Automatic archive navigator}} |
||
|maxarchivesize = 400K |
|maxarchivesize = 400K |
||
|counter = |
|counter = 1245 |
||
|minthreadsleft = |
|minthreadsleft = 15 |
||
|minthreadstoarchive = |
|minthreadstoarchive = 25 |
||
|algo = old(48h) |
|algo = old(48h) |
||
|archive = Wikipedia:Teahouse/Questions/Archive %(counter)d |
|archive = Wikipedia:Teahouse/Questions/Archive %(counter)d |
||
Line 12: | Line 12: | ||
{{clear}} |
{{clear}} |
||
{{Wikipedia:Teahouse/Header}} |
{{Wikipedia:Teahouse/Header}} |
||
{{clear}} |
|||
<!-- Questions go here. Please post new questions at the BOTTOM of the page. --> |
<!-- Questions go here. Please post new questions at the BOTTOM of the page. --> |
||
== |
== Technical question about the long hyphen == |
||
Hi! |
|||
Could someone please create [[SAMPLE]] as a redirect to [[SAMPLE history]] (like how [[OPQRST]] exists) and add a <nowiki>{{redirect|Sample||Sample}}</nowiki> to the latter? I can't the first half of that. Thanks [[Special:Contributions/2A0D:6FC2:6A92:3F00:0:0:0:5F9|2A0D:6FC2:6A92:3F00:0:0:0:5F9]] ([[User talk:2A0D:6FC2:6A92:3F00:0:0:0:5F9|talk]]) 22:45, 18 January 2024 (UTC) |
|||
I've been editing the timeline of Polermo where the long hyphen dominates, but I can't seem to generate one.Typing a regular hyphen, gives me just that - a regular hyphen, typing two hyphens gives me two hyphens (--) and trying to make one through the keboard shortcut which I found on internet forums (Alt+0151), just gives me one that's too long (—). So far I've been copying and pasting existing long hyphens which is kind of annoying, does anyone have any better solutions? |
|||
:{{done}}. It will be interesting to see if there will be discussion to change [[SAMPLE]] to redirect to [[Sample]] instead. [[User:GoingBatty|GoingBatty]] ([[User talk:GoingBatty|talk]]) 23:00, 18 January 2024 (UTC) |
|||
Thanks! [[User:Moonshane1933|Moonshane1933]] ([[User talk:Moonshane1933|talk]]) 14:38, 22 December 2024 (UTC) |
|||
:Hello, @[[User:Moonshane1933|Moonshane1933]]. I think you're talking about an em-dash. See [[MOS:EMDASH]] [[User:ColinFine|ColinFine]] ([[User talk:ColinFine|talk]]) 14:52, 22 December 2024 (UTC) |
|||
== red flagged/ tagged articles == |
|||
::Yes! That's what I meant! Thank you! [[User:Moonshane1933|Moonshane1933]] ([[User talk:Moonshane1933|talk]]) 15:15, 22 December 2024 (UTC) |
|||
:I don't think you could find a better character in '''"unicode table"'''. |
|||
:This "[[List of Unicode characters|article]]" is listing the most common characters. <br /> <br /> |
|||
:There are also the "[[Unicode block]]" entry on Wikipedia that can be maybe helpful. [[User:Anatole-berthe|Anatole-berthe]] ([[User talk:Anatole-berthe|talk]]) 14:54, 22 December 2024 (UTC) |
|||
::Excellent. Thank you too! [[User:Moonshane1933|Moonshane1933]] ([[User talk:Moonshane1933|talk]]) 15:16, 22 December 2024 (UTC) |
|||
:::I don't think ressources I shared with you will help you but I hope it will. [[User:Anatole-berthe|Anatole-berthe]] ([[User talk:Anatole-berthe|talk]]) 15:45, 22 December 2024 (UTC) |
|||
:Ignoring the [[Minus sign]], there are three 'horizontal line' characters most commonly used in text, the hyphen, the [[N-dash]] and the [[M-dash]]. There are various ways to insert the latter two; usually I do so with [alt]+0150 and [alt]+0151. Despite being a former professional book editor, I have not previously encountered a "long hyphen" (a term not found anywhere in Wikipedia). Note that the lengths of all these characters may look different in different typefaces: I suspect your "long hyphen" is an N-dash. [Apologies for semi-overlap with answers above.] {The poster formerly known as 897.81.230.195} [[Special:Contributions/94.1.223.204|94.1.223.204]] ([[User talk:94.1.223.204|talk]]) 17:00, 22 December 2024 (UTC) |
|||
I am wondering ...I see lots of articles with maintenance tags on them. I see they are dated. Sometimes I see in the histories people do work to repair them. My questions is...does anybody patrol the tagged pages to see if they are improved and possibly remove the tags? [[User:WikiTikiTavi63|WikiTikiTavi63]] ([[User talk:WikiTikiTavi63|talk]]) 00:14, 19 January 2024 (UTC) |
|||
::@[[User:Moonshane1933|Moonshane1933]] If you use the source editor, which you can do even if you mainly edit with the visual editor, you'll find that the N-dash and M-dash appear at the foot of the editing window, where you can click on them to insert them into text. Other useful tags like <nowiki><ref></ref></nowiki> are also available with a single click. [[User:Michael D. Turnbull|Mike Turnbull]] ([[User talk:Michael D. Turnbull|talk]]) 14:11, 24 December 2024 (UTC) |
|||
:[[User:WikiTikiTavi63|WikiTikiTavi63]], the editors making the improvements will often remove tags. Also, specific tags about certain issues (such as {{tlx|POV}}) that are not paired with any specific talk page discussion are often removed as [[WP:DRIVEBY]]s. However, tags may certainly linger longer than strictly necessary. Cheers!<span id="Remsense:1705623893433:WikipediaFTTCLNTeahouse" class="FTTCmt"> — [[User:Remsense|<span style="border-radius:2px 0 0 2px;padding:3px;background:#1E816F;color:#fff">'''Remsense'''</span>]][[User talk:Remsense|<span lang="zh" style="border:1px solid #1E816F;border-radius:0 2px 2px 0;padding:1px 3px;color:#000">诉</span>]] 00:24, 19 January 2024 (UTC)</span> |
|||
:: |
:::OOOOOOOHHHH... THANK YOU! That makes life easier! I hadn't even thought of looking at the source editor, because it always looks headache inducing to me. I'll give it a try. Thank you so much. [[User:Moonshane1933|Moonshane1933]] ([[User talk:Moonshane1933|talk]]) 13:07, 25 December 2024 (UTC) |
||
::Yes, well, the "long hyphen" is a term that I coined, simply because I lacked the knowledge of its correct name, So I would have been very surprised if it had appeared in Wikipedia. Anyway, thank you, oh mysterious IP poster, I hope our paths cross again! [[User:Moonshane1933|Moonshane1933]] ([[User talk:Moonshane1933|talk]]) 13:03, 25 December 2024 (UTC) |
|||
:::@[[User:WikiTikiTavi63|WikiTikiTavi63]] In theory yes. Finding a stale article, slapping a tag and moving in is imho lazy. If you notice a problem, try to fix it. Tags are incredibly useful when the articles are being actively worked on by other editors. But if a tree falls/article is tagged, and no one hears/sees it, does it really count? ~ 🦝 [[User:Shushugah|Shushugah]] (he/him • [[User talk:Shushugah|talk]]) 17:40, 19 January 2024 (UTC) |
|||
:::@[[User:Moonshane1933|Moonshane1933]], some Christmas goodies for you: |
|||
::::As I am a novice I do not think I should, nor do I even know how to remove a tag. I was just wondering….there seem to be a lot of tags. Thanks for your input. [[User:Shushugah]] [[User:WikiTikiTavi63|WikiTikiTavi63]] ([[User talk:WikiTikiTavi63|talk]]) 18:05, 19 January 2024 (UTC) |
|||
:::— [https://www.merriam-webster.com/grammar/em-dash-en-dash-how-to-use Merriam-Webster Dictionary] has a nice clear explanation about the both kinds of dashes and the hyphen, with good examples. |
|||
:::@[[User:WikiTikiTavi63|WikiTikiTavi63]] Anyone can remove a tag if they believe that the related issue has been fixed. On visual editor it is often as easy as clicking on a template like {citation needed} and pressing delete. There are some people who spend all of their time trying to fix these maintenance tasks, and some who spend their time identifying problems so that those people can find and fix them. I looked at [[Mater Matuta]] and did some small edits. It is not quite ready to have the notices at the top removed but it is getting closer. [[User:Reconrabbit|<span style="color:#6BAD2D">Recon</span>]][[User talk:Reconrabbit|<span style="color:#2F3833">rabbit</span>]] 18:44, 19 January 2024 (UTC) |
|||
:::— The way the two kindts of dashes is written is '''em-dash''' (for '''—)''' and '''en-dash''' ( for '''– )''', even though we pronounce the terms "''M dash''" and "''N dash''." |
|||
::::Thank you for your input. Can you tell me what still needs attention? [[User:reconrabbit]] [[User:WikiTikiTavi63|WikiTikiTavi63]] ([[User talk:WikiTikiTavi63|talk]]) 19:27, 19 January 2024 (UTC) |
|||
:::— Why these terns? Because the em-dash is exactly the width of capital ''M'' and the en-dash is exactly the width of capital ''N.'' |
|||
:::::I see the same reference being used over and over inside the same paragraph. It is usually preferred to put it at the end of the paragraph, and to only put a citation after a sentence if it is relevant to that statement or if it is specific about a stated fact. I may take another look at the article later, but there is some writing that makes it look like an essay, describing a series of events or not describing some things that may not be obvious (matera matutae?) But this is better discussed on [[Talk:Mater Matuta|the article's talk page.]] [[User:Reconrabbit|<span style="color:#6BAD2D">Recon</span>]][[User talk:Reconrabbit|<span style="color:#2F3833">rabbit</span>]] 19:33, 19 January 2024 (UTC) |
|||
:::— If you have a Macintosh, there's a real simple way to make the dashes: the '''em-dash''' by pressing Control Option Hyphen at the same time, and the '''en-dash''' by pressing Option Hyphen at the same time. |
|||
::::::thanks. [[User:WikiTikiTavi63|WikiTikiTavi63]] ([[User talk:WikiTikiTavi63|talk]]) 19:38, 19 January 2024 (UTC) |
|||
:::—Did you notice how [[User:Nick Moyes|Nick Moyes]] creatively renamed Dasher, one of Santa Claus's eight reindeer, in his "Seasonal Greetings from all at the Teahouse" post to fellow editors below? |
|||
:::—You may be pleased to know that I found [https://www.google.com/search?client=safari&rls=en&q=long+hyphen&ie=UTF-8&oe=UTF-8 an online reference to a "long hyphen."] So, then, you weren't completely alone in doing that. But as [[Special:Contributions/94.1.223.204|94.1.223.204]] commented above, in professional editing we just don't use it. Like [[User:ColinFine|ColinFine]], )I think anyone who ''did'' say "long hyphen''"'' would probably be thinking of the em-dash; though I also think what [[Special:Contributions/94.1.223.204|94.1.223.204]] said above is also technically correct, that the term would have to refer to the en-dash (that's the next size up for a hyphen, after all). [[User:Augnablik|Augnablik]] ([[User talk:Augnablik|talk]]) 06:08, 27 December 2024 (UTC) |
|||
::::@[[User:Augnablik|Augnablik]], What a great reply! I thoroughly enjoyed every bit of it! And I learned a lot (not to detract from the other contributors, each of whom taught me something new - thanks, everybody) - a special thank you for the meanings of the em-dash and en-dash (I love that type of thing), and for drawing my attention to Nick Moyes' "Seasonal Greetings", and of course for finding me an ally in calling the en-dash a "Long hyphen" (don't worry, now that I know the correct terminology I will use it and hopefully amaze my friends...). Thank you again and Merry Christmas! [[User:Moonshane1933|Moonshane1933]] ([[User talk:Moonshane1933|talk]]) 12:58, 29 December 2024 (UTC) |
|||
::::@[[User:Augnablik|Augnablik]] Not to be a naysayer, but I think the bit about em dashes being named for being 'M' width is a false etymology. I too would have loved if it were true, but I think it's actually based on the [[em (typography)|em]] unit as described in [[Dash#Em dash]].  — [[User:Kilvin the Futz-y Enterovirus|Kilvin the Futz-y Enterovirus]] ([[User talk:Kilvin the Futz-y Enterovirus|talk]]) 10:23, 30 December 2024 (UTC) |
|||
:::::Well, @Kilvin the Futz-y Enterovirus, not to be a counter-naysayer (!) but there are many online sites with support for the width of the em-dash equal to M and of the en-dash equal to N. [https://www.grammarly.com/blog/punctuation-capitalization/dash/ Here’s just one, offered by Grammarly]. (Scroll down to ''What Is an En-Dash?'' and ''What Is an Em-Dash?'') [[User:Augnablik|Augnablik]] ([[User talk:Augnablik|talk]]) 12:06, 30 December 2024 (UTC) |
|||
::::::Hm... not to be a counter-{{hairspace}}''counter''-{{hairspace}}naysayer I mean I don't really think {{'}}''many online sites''{{'}} and Grammarly (unsourced, could just be mirroring untrustworthy sources) are [[WP:RS]]. But you're right; my searches show similar results and don't really yield great answers to this matter (ideally there'd be a page like "Many people say that an em dash is named for X, but actually it is Y, and this misconception came from Z" linking to many reliable sources). I guess my personal bias is towards the people going "false etymology!" that acknowledge both X and Y rather than one group who simply assert the "fun" explanation with seemingly no awareness of the other explanation. I mean, as linked previously, the [[em (typography)|em]] and [[en (typography)|en]] are typographical units whose pages describe their origins, even addressing [[Em (typography)#Obsolete alternative definition]].  — [[User:Kilvin the Futz-y Enterovirus|Kilvin the Futz-y Enterovirus]] ([[User talk:Kilvin the Futz-y Enterovirus|talk]]) 22:53, 30 December 2024 (UTC) |
|||
:::::::Oooh, @[[User:Kilvin the Futz-y Enterovirus|Kilvin the Futz-y Enterovirus]], you’re acknowledging “personal bias” towards “false etymology?” That requires a declaration of COI! 🙂 [[User:Augnablik|Augnablik]] ([[User talk:Augnablik|talk]]) 18:09, 31 December 2024 (UTC) |
|||
::::::::Aha! I found something! In the Historical thesaurus on the [https://www.oed.com/dictionary/em-dash_n?tab=meaning_and_use#1428570550100 OED] site. And I quote: |
|||
::::::::"'''[https://www.oed.com/dictionary/em-dash_n?tab=meaning_and_use#1428570550 em dash]1836-''' A long dash ''—'', originally and usually the width of one em (see ''em'', ''n.'')." [[User:Moonshane1933|Moonshane1933]] ([[User talk:Moonshane1933|talk]]) 13:13, 1 January 2025 (UTC) |
|||
== |
== Add a page? == |
||
Hello - How can somebody submit a page for a notable person? My husband has one of the country's worst wrongful convictions in the United States and I'd love to have somebody neutral put information up regarding his wrongful conviction case. We believe he will be exonerated someday. His name is Temujin Kensu and you can google search his name to learn more about this horrible case. Thank you! [[Special:Contributions/65.111.210.82|65.111.210.82]] ([[User talk:65.111.210.82|talk]]) 06:21, 28 December 2024 (UTC) |
|||
I need help in creating a new article for an organisation that doesn't exist. I know someone who is a part of that organisation and can tell me everything about that article. I am just confused on how to do it. I found that the article is missing because I found a red link on a related organisation's page. [[User:Ultima1108|Ultima1108]] ([[User talk:Ultima1108|talk]]) 00:30, 19 January 2024 (UTC) |
|||
:Based on my Google search, I consider it almost certain that Temujin Kensu is notable and that Wikipedia ought to have an article about him. [[User:Cullen328|Cullen328]] ([[User talk:Cullen328|talk]]) 07:39, 28 December 2024 (UTC) |
|||
::For anyone interested in starting a draft [https://cse.google.com/cse?cx=007734830908295939403:galkqgoksq0#gsc.tab=0&gsc.q=Temujin%20Kensu some of these Google hits] could easily be used to pass [[WP:GNG]]. [[User:Michael D. Turnbull|Mike Turnbull]] ([[User talk:Michael D. Turnbull|talk]]) 12:21, 28 December 2024 (UTC) |
|||
:::Realistically, you may not get a volunteer. Teahouse Hosts volunteer here to advise, not to be authors or co-authors. [[User:David notMD|David notMD]] ([[User talk:David notMD|talk]]) 20:18, 28 December 2024 (UTC) |
|||
:If your husband is [[Wikipedia:Notability|notable]] enough, he will be talked about. [[User:SimpleSubCubicGraph|SimpleSubCubicGraph]] ([[User talk:SimpleSubCubicGraph|talk]]) 05:37, 1 January 2025 (UTC) |
|||
:IP editor: I have now started to draft an article at [[Draft:Temujin Kensu]]. In view of your [[WP:PSCOI|conflict of interest]] it would be best if you did not edit it directly (although that's not forbidden while it remains a draft). I would welcome your suggestions for additions and corrections at [[Draft talk:Temujin Kensu]], especially where you can provide additional published sources I can use. [[User:Michael D. Turnbull|Mike Turnbull]] ([[User talk:Michael D. Turnbull|talk]]) 12:47, 1 January 2025 (UTC) |
|||
== Isotopes lists download == |
|||
:It doesn't exist, but you know someone who's part of it. The article is missing, but someone can tell you everything about it. Please sort out your thoughts before you type here. -- [[User:Hoary|Hoary]] ([[User talk:Hoary|talk]]) 00:35, 19 January 2024 (UTC) |
|||
::I wanted to know how to create an article. I can't find the help page for it. That's why I posted it here. [[Special:Contributions/59.184.253.9|59.184.253.9]] ([[User talk:59.184.253.9|talk]]) 00:40, 19 January 2024 (UTC) |
|||
:::Maybe you meant that the organization exists but an article on it does not. If so, then [[Help:YFA]]. (Incidentally, since you have a user ID, please do make sure that you're always logged in under that user ID.) -- [[User:Hoary|Hoary]] ([[User talk:Hoary|talk]]) 00:48, 19 January 2024 (UTC) |
|||
Is there a to download these lists ? ( For example : the list in "<nowiki>https://en.wikipedia.org/wiki/Isotopes_of_sodium</nowiki>" ) |
|||
:::Meanwhile, [[User:Ultima1108|Ultima1108]]: You may not attribute material to what somebody told you (whether in conversation, in email, in Whatsapp messages, or wherever). Everything must be based on published sources. -- [[User:Hoary|Hoary]] ([[User talk:Hoary|talk]]) 00:55, 19 January 2024 (UTC) |
|||
:::Hello, Ultima1108. Please note that the very first, and absolutely necessary, step in creating any article is finding several places where people who have no connection with the subject of the article have chosen to write at some length about it, and been published in reliable places. If you cannot find such sources, then there is no point in spending any time at all in trying to write the article. [[User:ColinFine|ColinFine]] ([[User talk:ColinFine|talk]]) 11:44, 19 January 2024 (UTC) |
|||
::::EXACTLY!!!! That is what I am dealing with! [[User:PolskiSlaskiego!|PolskiSlaskiego!]] ([[User talk:PolskiSlaskiego!|talk]]) 20:26, 19 January 2024 (UTC) |
|||
I have written a c# application that describes the relations between elements, isotopes, decays, fusions ... etc.(originating from the question "Where the carbon atoms in the cafeine in your coffee come from ?") |
|||
== Draft:Single Pair Ethernet == |
|||
When you make normal modifications to these lists, it takes me about 2 weeks to refresh my database for over 3000 isotopes and 5000 decays coming from 118 pages (and subject to typing errors...) |
|||
Hi, |
|||
since years I am writing some arcticles in German Wikipedia. |
|||
Now I am starting some technical articles here. |
|||
I would like to translate these article into en.wikipedia: [[Draft:Single Pair Ethernet]] |
|||
I have tried to download one of these pages but I get one of these mumbo-jumbo network message ( about security and the correction looks like "set the web_client.Tchic_Tchac to Fling_Flang" ... and none of them works... ) |
|||
translated page|de|Single Pair Ethernet| version=240874606|insertversion=1194350781 |
|||
Do you have a suggestion ? |
|||
SPE is a small part in article [[Ethernet over twisted pair#Single-pair]] and just a short summary without IEC 63171 like in my german article about SPE. |
|||
Thank you very much [[User:Michel Béliveau|Michel Béliveau]] ([[User talk:Michel Béliveau|talk]]) 17:49, 28 December 2024 (UTC) |
|||
:Describing a message as a "mumbo-jumbo network message" is not very helpful in determining what your problem is. If you quote the error message exactly it might be more useful. In any case, I can successfully download articles using curl like this: <syntaxhighlight>curl -k https://en.wikipedia.org/wiki/Isotopes_of_sodium</syntaxhighlight> [[User:CodeTalker|CodeTalker]] ([[User talk:CodeTalker|talk]]) 19:52, 28 December 2024 (UTC) |
|||
::Thank you for this fast (and good) answer. |
|||
::The mumbo-jumbo error message was : "The request was aborted: Could not create SSL/TLS secure channel." I was not using the good approach to download the content of the web page. |
|||
::>>> However the CURL function does what I need. |
|||
::Here is want to do In my application : |
|||
::For each Element ( 118 !!! ) get the "List of Isotopes" for this Element. Then for each Isotope : get its mass, half-life, decay mode(s) and decay product(s). This yields for over 3500 isotopes and over 4500 decays. Refreshing the data took quite a long time. |
|||
::Analyzing the results of the curl command is not so hard and will eliminate typing mistakes. Even if I need a few days to program the analysis, it will be faster than re-typing the data. |
|||
::I will take a look at Wikidata. |
|||
::Thanks again. [[User:Michel Béliveau|Michel Béliveau]] ([[User talk:Michel Béliveau|talk]]) 23:02, 28 December 2024 (UTC) |
|||
:Without really understanding what you are trying to do, I would suspect that [[Wikidata]] was a more useful resource than Wikipedia for your purpose, as it is a database which contains relations between its elements. [[User:ColinFine|ColinFine]] ([[User talk:ColinFine|talk]]) 21:34, 28 December 2024 (UTC) |
|||
::@[[User:Michel Béliveau|Michel Béliveau]] Wouldn't it be easier to download from the original sources, for example [https://www-nds.iaea.org/relnsd/nubase/nubase_min.html NUBASE]? [[User:Michael D. Turnbull|Mike Turnbull]] ([[User talk:Michael D. Turnbull|talk]]) 15:48, 29 December 2024 (UTC) |
|||
:::Thank you Mr. Turnbull. |
|||
:::You are correct. It would be easier to download from the original sources. |
|||
:::I have found (and used) a NUBASE file (namely for nucleus values) . So far, I have found only 1 NUBASE file that I could use ( coming from "The Ame2020 atomic mass evaluation (I)" by W.J.Huang, M.Wang, F.G.Kondev, G.Audi and S.Naimi - Chinese Physics C45, 030002, March 2021) . |
|||
:::The purpose of my request to Wikipedia is to avoid re-typing the values. The NUBASE file allowed this. |
|||
:::Do you know other NUBASE files ? Or other sources ? (I also found some data in PeriodicTable.com) |
|||
:::Thanks again for your interest [[User:Michel Béliveau|Michel Béliveau]] ([[User talk:Michel Béliveau|talk]]) 19:19, 29 December 2024 (UTC) |
|||
{{to|Michel Béliveau}} Hello! This kind of question is something you might want to take to [[WP:RD/C|the Computing reference desk]], where you can get people with computing expertise to assist. --[[User:Slowking Man|Slowking Man]] ([[User talk:Slowking Man|talk]]) 20:39, 31 December 2024 (UTC) |
|||
== Locked out of account == |
|||
:@[[User:Ralf Moses|Ralf Moses]]: Welcome to the Teahouse! If you haven't done so already, please check out [[Wikipedia:Translation]]. You have lots of information in your draft that do not have any footnotes. Please add more footnotes for every section and resubmit. Thanks, and happy editing! [[User:GoingBatty|GoingBatty]] ([[User talk:GoingBatty|talk]]) 14:34, 19 January 2024 (UTC) |
|||
::Thanks GoingBatty for your help. I have added the footnotes now. |
|||
::I would appreciate any further suggestions to improve and publish this article. |
|||
::Thanks again. BR Ralf. [[User:Ralf Moses|Ralf Moses]] ([[User talk:Ralf Moses|talk]]) 17:18, 19 January 2024 (UTC) |
|||
:::The draft, and the sources it cites, are entirely about the technical specifications of Single Pair Ethernet. I don't see that as making an acceptable article. Does anyone use it? What for? WHat are its advantages and disadvantages? [[User:Maproom|Maproom]] ([[User talk:Maproom|talk]]) 18:36, 19 January 2024 (UTC) |
|||
I got locked out of my DooplissTTYD account because I forgot the complex password and didn’t have an email address linked to it. Is there any way that account can be renamed to something else and I change this one to DooplissTTYD? [[User:TTYDDoopliss|TTYDDoopliss]] ([[User talk:TTYDDoopliss|talk]]) 21:50, 28 December 2024 (UTC) |
|||
== Contents of an article == |
|||
:You would have to request a renaming from the account you want renamed. It can't be requested by in essence a third party(as we have no way to know who is on the other end of the computer). The best you can do is post on your current and previous user pages that you lost access to your old account and have a new one. [[User:331dot|331dot]] ([[User talk:331dot|talk]]) 00:06, 29 December 2024 (UTC) |
|||
:This is why the first thing you want to ensure you do right now is verify an e-mail address for your account, so you can restore it: [[Special:Preferences|follow this link if you haven't]]. Step 2: I highly recommend using a [[password manager]] to save passwords—as well as generate secure passwords. I suggest trying out [[Bitwarden]]. |
|||
:{{br}} |
|||
:Now in this particular case, [[Special:Contributions/DooplissTTYD|your prior account has only a few edits]]; this means if you wait a bit, say 6 months, you can [[WP:USURPNAME|request to "usurp" it]] which will accomplish what you want. ([[Paper Mario TTYD|Also, a very good game.]]) --[[User:Slowking Man|Slowking Man]] ([[User talk:Slowking Man|talk]]) 18:06, 31 December 2024 (UTC) |
|||
::I mean that’s what I did. I generated a password using a feature on my phone, and it kept saying the one I had on my phone was incorrect. |
|||
::and yes I have an email linked to this account now. 6 months is a very long time though… [[User:TTYDDoopliss|TTYDDoopliss]] ([[User talk:TTYDDoopliss|talk]]) 19:13, 31 December 2024 (UTC) |
|||
== donating - would like to donate == |
|||
:Hello, IP editor. Welcome to the Teahouse. A recent change of the 'default skin' which determines your screen's layout was made a year or so ago. The in-article Contents table was moved in the upgrade to ''outside'' of the article, on the upper left hand side of the page viewing in when desktop view. You can toggle it's position so that it either expands fully as a column on the upper left of the scren, or collapses into a pancake menu icon of three horizontal lines on the upper left of the screen. |
|||
:If you find it annoying, you can revert to using the old screen layout (=skin), but you would need to have a free, registered account and change that in your personal account preferences. I'm pretty sure an IP user cannot do that. I hope this all makes sense! Regards [[User:Nick Moyes|Nick Moyes]] ([[User talk:Nick Moyes|talk]]) 12:14, 19 January 2024 (UTC) |
|||
::Some alternatives for IP editors are explained [[Wikipedia:Vector_2022#Without_an_account|here]] and [[Wikipedia:Skin#How_to_change_the_skin|here]]. They work reasonably well (though I've reconciled myself to the new skin and have stopped using them). [[Special:Contributions/57.140.16.1|57.140.16.1]] ([[User talk:57.140.16.1|talk]]) 15:34, 19 January 2024 (UTC) |
|||
Thank yoy Nick Moyes and 57.140! The explanations and the links are fantastic! Due to browser glitches on my browser, there arent the links or icons to click on in upper left corner mentioned by NM, if and when I find another device I'll try your instructions! which are descriptive and thorough. Much thanks! 57.140, I tried "here" link #1 and the directions worked and I can see the Contents again! With practice I'll start to memirize the stuff to type at end of URL. "here" link 2 isgoing to require more practice but then i'll be able to click each time to use 2010 Contents again with my device and browser! Thank you! You guys are great! Best, [[Special:Contributions/63.248.183.70|63.248.183.70]] ([[User talk:63.248.183.70|talk]]) 11:18, 20 January 2024 (UTC) |
|||
Hail... |
|||
== Copyright question == |
|||
Would like to donate 50 quid to the cause but stop at the name and address |
|||
part. Don't really see the need for full name and address. |
|||
Just old and not particularly wise. Any suggestions? [[Special:Contributions/81.96.25.61|81.96.25.61]] ([[User talk:81.96.25.61|talk]]) 12:28, 29 December 2024 (UTC) |
|||
:Hi IP editor, the Wikimedia Foundation deals with all donation issues and questions - editors here at the Teahouse don't have any input. Please direct your query to the email address at the bottom of [[donate:Ways_to_Give|donate.wikimedia.org/wiki/Ways_to_Give]] <span style="background-color: RoyalBlue; border-radius: 1em; padding: 3px 3px 3px 3px;">'''[[User:Qcne|<span style="color: GhostWhite">qcne</span>]]''' <small>[[User talk:Qcne|<span style="color: GhostWhite">(talk)</span>]]</small></span> 12:49, 29 December 2024 (UTC) |
|||
Imagine yourself in this scenario: you come across a heavily tagged, ill-maintained article. And let's imagine you add a few new details, rectify some grammar errors without significantly altering the original content, fulfil the CN tags, and then discover that someone has added several copyright infringement that has gone undetected for over 10 years. What would you do? [[User:20 upper|20 upper]] ([[User talk:20 upper|talk]]) 13:53, 19 January 2024 (UTC) |
|||
::Just to elaborate slightly: all editors here give their time entirely voluntarily and gain absolutely no financial benefit from any contributions made to keep the broader Wikipedia projects going. So we have little knowledge of how the donation systems work - despite being grateful for everyone's contributions. The advice above is sound. [[User:Nick Moyes|Nick Moyes]] ([[User talk:Nick Moyes|talk]]) 18:21, 29 December 2024 (UTC) |
|||
:Also I'm not certain (not WMF personnel), but I am pretty confident the reason for requiring that info is fraud protection and banking and [[money laundering]] laws; it's not something the Foundation has any control over. WMF is a registered [[501(c)(3) organization]] in the US, meaning it has to comply with a bunch of laws and regulations. (For ''furrin types'', this is near-synonymous with "[[non-profit]]/[[Charity organization|charity]]" in the US, being simply the section of the [[Internal Revenue Code]] setting out the criteria said organizations have to meet to get tax exemption.) Note, a way that one can make semi-anonymous donations, is to pass them through what's called a "[[donor-advised fund]]" in the US, but this is a bit more involved and requires going through a bank or brokerage that will arrange your transfer. Alternatively, I suppose if one is accepting of the fact there are no guarantees in life one could always send cash to the WMF's front door with no return address and leave its final disposition up to them. --[[User:Slowking Man|Slowking Man]] ([[User talk:Slowking Man|talk]]) 18:38, 31 December 2024 (UTC) |
|||
== I don't find the ressource to add books wrote by someone == |
|||
:remove it, if I had to guess. [[User:Babysharkboss2|<span style="color: red">Babysharkboss2 was here!!</span>]] ([[User talk:Babysharkboss2|<span style="color:black">Talking Heads</span>]]) ([[Special:Contributions/Babysharkboss2|<span style="color:blue">Buddy Holly</span>]]) 14:07, 19 January 2024 (UTC) |
|||
:@[[User:20 upper|20 upper]]: Recently {{U|Mathglot}} reminded me: "If it really was copied from a copyrighted source, that is one of the most serious violations at Wikipedia, as it violates both our WP:Terms of use as well as copyright law, and as such, it is one of a very small number of violations with legal implications (others include libel, legal threats, and there are a couple others) that require immediate attention and that are not subject to amendment by policy, consensus, ArbCom, or WP:IAR. Content in violation of copyright cannot be left in the article, and not only that, it must be expunged from the page history by an admin, so that no trace of it is left in Wikipedia. As far as what to advise a user, probably the simplest and best response is to tell the user to place a {{tl|Copyvio}} template on the article page itself; this will both immediately hide the suspected offending material, as well as signal an admin to have a look at it and assess the situation for further action. Users should not be told to simply remove the content themselves, or to rewrite or summarize copied content, because that masks the continuing problem of copyrighted content which remains accessible in the page history; it's best just to flag it for admin action with the template." [[User:GoingBatty|GoingBatty]] ([[User talk:GoingBatty|talk]]) 14:27, 19 January 2024 (UTC) |
|||
::{{Re|Babysharkboss2|GoingBatty}} The issue is that the article is a "vital article." I attempted to rephrase a few sentences, but a large portion of the article is directly copied from one source and it uses citations utilized by this particular source. [[User:20 upper|20 upper]] ([[User talk:20 upper|talk]]) 17:19, 19 January 2024 (UTC) |
|||
:::Being a vital article doesn't offer any kind of protection, unfortunately. If the source is incompatibly licensed, the material copied over needs to be removed. I've occasionally seen revdel requests declined because the violation is so old it would require deleting an unholy number of intermediate revisions, but you can just leave that up to the reviewing admin. [[Special:Contributions/57.140.16.1|57.140.16.1]] ([[User talk:57.140.16.1|talk]]) 17:24, 19 January 2024 (UTC) |
|||
:::: IP 57: Can you link the discussions with the declined revdel requests? [[User:Mathglot|Mathglot]] ([[User talk:Mathglot|talk]]) 20:26, 19 January 2024 (UTC) |
|||
:::::@[[User:Mathglot|Mathglot]], I've not kept any track of them, but I think there have been several discussions at [[WP:AN]] - you might turn up some if you search the archives. I've had a few such requests turned down myself over the years, all involving small amounts of text which were added decades earlier, with many, many revisions since; I can't remember, now, which specific articles were involved. [[Special:Contributions/57.140.16.1|57.140.16.1]] ([[User talk:57.140.16.1|talk]]) 20:46, 19 January 2024 (UTC) |
|||
:::::@[[User:Mathglot|Mathglot]], here's a long and involved discussion from 2022 you might find interesting, though it's about an accepted rather then declined request: [[Wikipedia:Administrators%27 noticeboard/Archive340#Revdel on Himachal Pradesh]]. [[Special:Contributions/57.140.16.1|57.140.16.1]] ([[User talk:57.140.16.1|talk]]) 21:03, 19 January 2024 (UTC) |
|||
:::::: IP 57, thanks, that is indeed a very interesting discussion. There is an inherent tension between the legal requirement to remove copyright violations and the licensing requirement in ToU § 7 to preserve attribution, and it seems to me that most of the comments there were trying to square that circle. The response {{u|Moneytrees}} got back from WMF as I understood it resolved that in favor of the long-revdels-are-ok side, especially since contributors' names are still visible in history (through the strikeout font) which is the attribution requirement, and diffability is not. Struck revisions are non-diffable for non-admins, so traceability takes a hit for us peons, but that's short of the requirement (though sometimes irksome to me, personally). {{u|Flatscan}}'s contribution towards the end and {{their|Flatscan}} {{slink|Wikipedia talk:Revision deletion/Archive 5|"Attribution" in RD1|nopage=yes}} section at WP:Revdel and follow-up Rfc were helpful. I'm sorry I missed those, as § 7 is clear about the fact that just a list of names is sufficient therefore diffability is not required, and [[WP:CWW#List of authors]] does mention it. |
|||
:::::: I have an idea about how full traceability for everyone might be kept as well (hint: think Earth Prime with articles having a parallel history only through the revdel era, but where the copyvio never happened at the beginning of it) and I can picture how to do it, I just don't know if it's worth it. But this is getting well afield of the OP question, and if we take this up, it should probably be at [[WP:VPI]]. Thanks again, [[User:Mathglot|Mathglot]] ([[User talk:Mathglot|talk]]) 22:14, 19 January 2024 (UTC) |
|||
:::::::I confess that my eyes glaze over quickly when copyright issues arise - when my requests were declined, I just shrugged and moved on. I think all my requests pre-date that discussion, though, so maybe admins nowadays are handling things differently. Might be an interesting discussion indeed for the Pump, @[[User:Mathglot|Mathglot]]. [[Special:Contributions/57.140.16.1|57.140.16.1]] ([[User talk:57.140.16.1|talk]]) 22:28, 19 January 2024 (UTC) |
|||
Hello ! I'd like to add a book on the article about "[[David Murphy (CIA)]]". <br /> <br /> |
|||
== template ref list == |
|||
This is a book wrote by him not mentionned in the article. <br /> |
|||
I am having trouble correctly citing a book that has numerous references. I have tried giving the book a refrence name and then using code <code><nowiki>{{r|Beacon1997|p=50}}</nowiki></code>. But the article I am editing uses a template ref list and I don't understand how to edit that as the references are only visible in visual editing. I end up with an error code that sends me to the same page repeatedly and nothing I try seems to work. Appreciate any help. [[User:LauraONagel|LauraONagel]] ([[User talk:LauraONagel|talk]]) 16:28, 19 January 2024 (UTC) |
|||
I don't find the ressource explaining how to add the bibliography of someone. [[User:Anatole-berthe|Anatole-berthe]] ([[User talk:Anatole-berthe|talk]]) 16:17, 29 December 2024 (UTC) |
|||
:I recommend using one reference per </ref> code, I’m still new to Wikipedia, so take it with a grain of salt, if it keeps sending you to the page you are editing, I’ll try to research why the problems occur. [[User:Cometkeiko|Cometkeiko]] 16:47, 19 January 2024 (UTC) |
|||
::Thanks for your help. I'll keep trying. [[User:LauraONagel|LauraONagel]] ([[User talk:LauraONagel|talk]]) 17:55, 20 January 2024 (UTC) |
|||
:If you want to indicate the page number or location in a book I recommend using [[Template:Rp]], which doesn't need to be included inside the citation. [[User:Reconrabbit|<span style="color:#6BAD2D">Recon</span>]][[User talk:Reconrabbit|<span style="color:#2F3833">rabbit</span>]] 16:53, 19 January 2024 (UTC) |
|||
::Thank you! Very helpful. I'll try it. [[User:LauraONagel|LauraONagel]] ([[User talk:LauraONagel|talk]]) 17:58, 20 January 2024 (UTC) |
|||
:The template {{tl|r}} can get confusing to use. Iterating on Reconrabbit's advice, the template {{tl|rp}} could be used like this: |
|||
:<code><nowiki><ref name="Beacon">example book<ref>{{rp|23}}</nowiki></code> |
|||
:which renders as: |
|||
:<ref name="Beacon">example book</ref>{{rp|23}}. |
|||
:This way, you can add the citations intuitively in VisualEditor, and simply tack on the RP template to indicate the page number. [[User:Ca|Ca]] <i><sup style="display:inline-flex;rotate:7deg;">[[User talk:Ca|talk to me!]]</sup></i> 17:02, 19 January 2024 (UTC) |
|||
::You can see my article [[Charles Brenton Fisk]] for further examples of this. [[User:Ca|Ca]] <i><sup style="display:inline-flex;rotate:7deg;">[[User talk:Ca|talk to me!]]</sup></i> 17:10, 19 January 2024 (UTC) |
|||
:@[[User:Anatole-berthe|Anatole-berthe]] The standard template to use for book citations is {{t|cite book}}. It is usual, but not essential, to use its |URL= parameter to link to Google Books for the convenience of our readers. In this case that would be [https://www.google.co.uk/books/edition/Battleground_Berlin/3USTBk2dulQC?hl=en&gbpv=1&dq=david%20murphy%20battleground&pg=PR4&printsec=frontcover this link], from which you can also find the ISBN and full list of authors. [[User:Michael D. Turnbull|Mike Turnbull]] ([[User talk:Michael D. Turnbull|talk]]) 16:27, 29 December 2024 (UTC) |
|||
{{reftalk}} |
|||
:Hello, @[[User:Anatole-berthe|Anatole-berthe]]. Why do you want to add that book to that article? Has the book been discussed by independent sources? If not, why is it significant ednough to feature in a Wikipedia article? |
|||
:More to the point, that article is woefully short of sources, and does not establish that Murphy meets the criteria for [[WP:NBIO|notability]]. [[User:ColinFine|ColinFine]] ([[User talk:ColinFine|talk]]) 17:42, 29 December 2024 (UTC) |
|||
::The book was wrote by himself as I said. <br /> <br /> |
|||
::The ISBN-10 is '''"0300107803"''' and the ISBN-13 is '''"978-0300107807"''' for the first edition. |
|||
::The title is '''"What Stalin Knew: The Enigma of Barbarossa"''' , it was published by "[[Yale University Press]]" in 2005. <br /> <br /> |
|||
::The ISBN-10 is '''"030011981X"''' and the ISBN-13 is '''"978-0300119817"''' for the second edition. |
|||
::The second edition was published in 2006 by the same publisher. <br /> <br /> |
|||
::In this edition , it's mentionned the review of the first edition by "[[Niall Ferguson]]" for '''"New York Time Book Review"'''. |
|||
::The first edition was also reviewed by "[[Andrew Nagorski]]" for "[[The Weekly Standard]]". <br /> <br /> |
|||
::There are also a review by "[[Henry Kissinger]]" but I consider it doesn't count to add this book on Wikipedia. |
|||
::This person have to be considered as a reviewer among others even if he's notable accorded to criterias. <br /> <br /> |
|||
::He didn't wrote the review for a magazine or a journal and therefore the fact '''"Kissinger"''' wrote a review should not be taken into account. This is what I think. <br /> <br /> |
|||
::The same for the review by "[[Donald Kagan]]" for "[[Yale University]]" for a particular reason. This person worked for '''"Yale"''' and the book was published by "[[Yale University Press]]". Therefore , I consider it's like a review by the publisher itself. <br /> <br /> <br /> |
|||
::Conclusion : I think this book met the criteria n°1 for "[[Wikipedia:Notability_(books)]]" to create an article on this book. |
|||
::I don't want to create an article on the book itself. <br /> <br /> |
|||
::The '''"Threshold standards"''' is met because "[[Library of Congress]]" catalogued this book. |
|||
::[[LCCN]] is '''"2004065916"'''. <br /> <br /> |
|||
::If it is considered as notable to create an article , I consider it is notable enough to mention this book on the article "[[David Murphy (CIA)]]". [[User:Anatole-berthe|Anatole-berthe]] ([[User talk:Anatole-berthe|talk]]) 12:03, 1 January 2025 (UTC) |
|||
== |
== can you give me a lnk of the Roblox page == |
||
i need Roblox link to sign in [[Special:Contributions/24.192.134.19|24.192.134.19]] ([[User talk:24.192.134.19|talk]]) 01:57, 30 December 2024 (UTC) |
|||
The article [[Ethiopian Manifesto]] was flagged for copyright violation. I have rewritten the section in question. Please note that what I have newly written does not show up unless you enter edit mode. I'd like to know if the rewriting I did has solved the problem. [[User:Deisenbe|deisenbe]] ([[User talk:Deisenbe|talk]]) 18:25, 19 January 2024 (UTC) |
|||
:{{welcometea}} Did you have a question about editing or using Wikipedia? —[[User:Tenryuu|<span style="color:#556B2F">Tenryuu 🐲</span>]] ( [[User talk:Tenryuu|💬]] • [[Special:Contributions/Tenryuu|📝]] ) 02:07, 30 December 2024 (UTC) |
|||
::@[[User:Tenryuu|Tenryuu]] I think hes trolling. [[User:SimpleSubCubicGraph|SimpleSubCubicGraph]] ([[User talk:SimpleSubCubicGraph|talk]]) 02:50, 30 December 2024 (UTC) |
|||
:::That's my standard reply to anyone who doesn't ask questions about Wikipedia here. —[[User:Tenryuu|<span style="color:#556B2F">Tenryuu 🐲</span>]] ( [[User talk:Tenryuu|💬]] • [[Special:Contributions/Tenryuu|📝]] ) 02:58, 30 December 2024 (UTC) |
|||
:Does [[Roblox#External links|this link]] help at all? If you have general computing questions that aren't Wikipedia-specific, go to the [[WP:RD/Computing|Computing reference desk]]. --[[User:Slowking Man|Slowking Man]] ([[User talk:Slowking Man|talk]]) 18:46, 31 December 2024 (UTC) |
|||
== Good Sources for Articles == |
|||
:No, not in preview. If you go there and click Edit you can see what I wrote and saved there. [[User:Deisenbe|deisenbe]] ([[User talk:Deisenbe|talk]]) 18:56, 19 January 2024 (UTC) |
|||
G-Day |
|||
::I'm goingto post this at Help desk. [[User:Deisenbe|deisenbe]] ([[User talk:Deisenbe|talk]]) 20:43, 19 January 2024 (UTC) |
|||
::: {{re|Deisenbe}} I answered at the help desk. Next time, just be patient and don't post in more than one place. [[User:RudolfRed|RudolfRed]] ([[User talk:RudolfRed|talk]]) 21:22, 19 January 2024 (UTC) |
|||
I wanted to ask if there is any reliable source for Articles. I cannot afford my personal favourite, The "Britannica" Encyclopedia, since they seem to be rare and expensive. Thank you. (I know this isn't about a specific Article but I need to know this) [[User:PizzaFrank|PizzaFrank]] ([[User talk:PizzaFrank|talk]]) 13:51, 30 December 2024 (UTC) |
|||
== sandbox issue == |
|||
:@[[User:PizzaFrank|PizzaFrank]] there are all sorts of reliable sources, not just Encyclopædia Britannica, and most don't require a purchase of any kind. See [[Wikipedia:Reliable sources]] for a detailed overview of RSes here on Wikipedia. <span style="white-space:nowrap"><span style="font-family:monospace">'''<nowiki>'''[[</nowiki>[[User:CanonNi]]<nowiki>]]'''</nowiki>'''</span> ([[User talk:CanonNi|talk]] • [[Special:Contributions/CanonNi|contribs]])</span> 13:55, 30 December 2024 (UTC) |
|||
::Thank you, I will check that out. [[User:PizzaFrank|PizzaFrank]] ([[User talk:PizzaFrank|talk]]) 14:03, 30 December 2024 (UTC) |
|||
:{{u|PizzaFrank}} Note that you don't have to personally own the source- it just needs to be publicly accessible, like online, or in a library. [[User:331dot|331dot]] ([[User talk:331dot|talk]]) 14:01, 30 December 2024 (UTC) |
|||
::Understood Thank you for the help. [[User:PizzaFrank|PizzaFrank]] ([[User talk:PizzaFrank|talk]]) 14:04, 30 December 2024 (UTC) |
|||
:::[[User:PizzaFrank|PizzaFrank]] I have used https://www.britannica.com a few times, and there is no fee to use the online version of Encyclopedia Britannica. [[User:Karenthewriter|Karenthewriter]] ([[User talk:Karenthewriter|talk]]) 18:27, 30 December 2024 (UTC) |
|||
:And besides [[public libraries]] which are great, Wikipedia editors with some experience can access [[WP:The Wikipedia Library|The Wikipedia Library]] which grants ''free'' access to tons of "paywalled" "digital content"! Pretty neat! Also see there: even if your account is not "seasoned" enough yet, you can request experienced editors assist you with researching sources through it. --[[User:Slowking Man|Slowking Man]] ([[User talk:Slowking Man|talk]]) 17:45, 1 January 2025 (UTC) |
|||
== Photograph attribution == |
|||
in snadbox rules it didnt say you cant do keffir or other slurs and a person removed my edits there. [[Special:Contributions/81.97.224.185|81.97.224.185]] ([[User talk:81.97.224.185|talk]]) 18:46, 19 January 2024 (UTC) |
|||
I want to use a portrait of my grandfather taken in 1915 by a company that no longer exists. I can show an attribution in the caption, but there is nobody I can seek permission from. How do I proceed, please? [[User:Gangnam Woodford|Gangnam Woodford]] ([[User talk:Gangnam Woodford|talk]]) 16:52, 30 December 2024 (UTC) |
|||
:I don't think that you should need to read a rule to [[Wikipedia:What "Ignore all rules" means#Use common sense|know that you shouldn't post slurs]] [[User:Babysharkboss2|<span style="color: red">Babysharkboss2 was here!!</span>]] ([[User talk:Babysharkboss2|<span style="color:black">Talking Heads</span>]]) ([[Special:Contributions/Babysharkboss2|<span style="color:blue">Buddy Holly</span>]]) 18:59, 19 January 2024 (UTC) |
|||
::I have a question, my imagination is way too wide, but is it possible to create another sandbox? I promise I totally won’t make an entire whole different universe. |
|||
::(for Nick, a little laugh is sometimes just good, we all need an appropriate joke for some collaborative projects) [[User:Cometkeiko|Cometkeiko]] ([[User talk:Cometkeiko|talk]]) 19:43, 19 January 2024 (UTC) |
|||
:::to answer the "second sandbox" question, I don't really see why you would want to, but yes, you could make a sub-page for a second sandbox. [[User:Babysharkboss2|<span style="color: red">Babysharkboss2 was here!!</span>]] ([[User talk:Babysharkboss2|<span style="color:black">Talking Heads</span>]]) ([[Special:Contributions/Babysharkboss2|<span style="color:blue">Buddy Holly</span>]]) 14:54, 22 January 2024 (UTC) |
|||
:To be blunt: if you ever post offensive slurs anywhere on Wikipedia again, your IP address will be blocked from editing. This is an adult project; try not to act like a stupid kid. [[User:Nick Moyes|Nick Moyes]] ([[User talk:Nick Moyes|talk]]) 19:08, 19 January 2024 (UTC) |
|||
::I read it like that dude in the movies who acts smug and is black. [[User:Cometkeiko|Cometkeiko]] ([[User talk:Cometkeiko|talk]]) 20:28, 19 January 2024 (UTC) |
|||
:Hello. It depends on the laws of your country, but a photo taken in 1915 is likely in the public domain. According to https://commons.wikimedia.org/wiki/Commons:Copyright_rules_by_territory/United_States in the US anything published before 1929 is public domain(other than sound recordings), so it would just depend on your own country. [[User:331dot|331dot]] ([[User talk:331dot|talk]]) 16:56, 30 December 2024 (UTC) |
|||
== Time zones on infobox settlement == |
|||
::Yes '''BUT''' (big but!), "publication" in the context of US copyright law has a specifical technical, legal definition. It doesn't mean "creation" or even "giving a copy to a particular person". [[commons:COM:Hirtle chart|Hirtle chart]]. A photograph of a private individual is almost certainly an "unpublished work" under the US copyright meaning of that term, which matters significantly for the length of its copyright term especially if an "older" work. Copyright is a subject that gets quite technical, so if not familiar and experienced with copyright issues, it is best to hesitate to advise others before researching things in detail and double-checking, and if in doubt, direct others to forums like [[commons:COM:VP/C|Commons]] where experts can be found. --[[User:Slowking Man|Slowking Man]] ([[User talk:Slowking Man|talk]]) 20:02, 31 December 2024 (UTC) |
|||
{{Resolved| [[User:Kk.urban|Kk.urban]] ([[User talk:Kk.urban|talk]]) 19:49, 19 January 2024 (UTC)}} |
|||
:@[[User:Gangnam Woodford|Gangnam Woodford]] Is this an American picture? If so, you can upload it as public domain, see [https://commons.wikimedia.org/wiki/Commons:Hirtle_chart]. [[User:Gråbergs Gråa Sång|Gråbergs Gråa Sång]] ([[User talk:Gråbergs Gråa Sång|talk]]) 16:57, 30 December 2024 (UTC) |
|||
Does anyone know where the time zone data comes from on {{tl|Infobox settlement}}? |
|||
::Sorry, should have said: UK. [[User:Gangnam Woodford|Gangnam Woodford]] ([[User talk:Gangnam Woodford|talk]]) 16:58, 30 December 2024 (UTC) |
|||
:::@[[User:Gangnam Woodford|Gangnam Woodford]] Still good, see this example: [https://commons.wikimedia.org/wiki/File:Archibald_Joyce.jpg File:Archibald Joyce.jpg]. [[User:Gråbergs Gråa Sång|Gråbergs Gråa Sång]] ([[User talk:Gråbergs Gråa Sång|talk]]) 17:02, 30 December 2024 (UTC) |
|||
::::Thank you! [[User:Gangnam Woodford|Gangnam Woodford]] ([[User talk:Gangnam Woodford|talk]]) 17:06, 30 December 2024 (UTC) |
|||
:::::Please also see [[MOS:CREDITS]] ".... do not credit the image author or copyright holder in the article. .... as long as the appropriate credit is on the image description page." - [[User:Arjayay|Arjayay]] ([[User talk:Arjayay|talk]]) 17:16, 30 December 2024 (UTC) |
|||
::::::Thanks. [[User:Gangnam Woodford|Gangnam Woodford]] ([[User talk:Gangnam Woodford|talk]]) 17:29, 30 December 2024 (UTC) |
|||
:{{to|Gangnam Woodford}} "Congratulations", you've just tripped onto one of the fun little copyright law landmines that are out there (alternately: "job security for lawyers"). Everything on [[commons:|Wikimedia Commons]] must be public domain in '''''both''''' the US, '''and''' origin country. This means you have to care about US copyright law and all its nuances: see [[commons:COM:Hirtle chart|Hirtle chart]]. Unless the picture was like put on display in an art gallery or something, it is an "unpublished" work and therefore ''still under copyright in the US until 2035!'' (120 yrs from creation date) Really! Welcome to copyright law! |
|||
:{{br}} |
|||
:Here are the practical implications for you in this case: Is this image going to be used in an article? If so, upload it ''here on enwp'' (not Commons) by [[Special:Upload|following this link]], and tag it {{tl|PD-UK-unknown}}, as well as adding a [[WP:fair use rationale|fair use rationale]] since it's still under US copyright. |
|||
:Add it to the article(s), nothing more needed—some bot should automatically transport it over to Commons when it finally lapses into PD-US. However, if you just, say, want to use the image on your [[WP:user page|user page]], I'm afraid you aren't going to be able to for 10 more years. So stick a pin in the calendar for that. --[[User:Slowking Man|Slowking Man]] ([[User talk:Slowking Man|talk]]) 19:54, 31 December 2024 (UTC) |
|||
== An Article About CK == |
|||
For example, if you type in "Pacific" for the time zone parameter, it automatically links to Pacific Time Zone and the UTC offsets, and the daylight saving time. I tried to do the same with "[[Hawaii–Aleutian Time Zone|Hawaii–Aleutian]]" but it's not working. [[User:Kk.urban|Kk.urban]] ([[User talk:Kk.urban|talk]]) 19:26, 19 January 2024 (UTC) |
|||
{{resolved|1=Young person goofing off --[[User:Slowking Man|Slowking Man]] ([[User talk:Slowking Man|talk]]) 20:14, 31 December 2024 (UTC)}} |
|||
: I don't think the data comes from anywhere, you manually add it when you add the template. The examples in the template and in article's using it I see that it is just a Wikilink. What error are you seeing when you do <nowiki>|timezone = [[Hawaii–Aleutian Time Zone|Hawaii–Aleutian]]</nowiki>? [[User:RudolfRed|RudolfRed]] ([[User talk:RudolfRed|talk]]) 19:43, 19 January 2024 (UTC) |
|||
{{collapse top}} |
|||
::@[[User:RudolfRed|RudolfRed]] Thanks, you're right. It's actually only {{tl|Infobox U.S. county}} that does this automatically, and the way to put a county in this time zone is to use |timezone = Hawaii (without a wikilink). [[User:Kk.urban|Kk.urban]] ([[User talk:Kk.urban|talk]]) 19:48, 19 January 2024 (UTC) |
|||
I was asked by christen kuikoua representation to write an article about him and it was declined saying I didn't correctly reference it. Please if anyone wishes to jump in and help me with Will appreciate it [[Draft:Christen Kuikoua]] |
|||
[[User:Silvernet123|Silvernet123]] ([[User talk:Silvernet123|talk]]) 18:27, 30 December 2024 (UTC) |
|||
== Relevance to school articles of non-school activity by students == |
|||
:Hello {{u|Silvernet123}}, I've left some instructions on your talk page about the [[WP:DISCLOSEPAY|mandatory paid editing disclosure]] that is required by Wikimedia's terms of service. Please follow those instructions before making any other edits. -- [[User:DandelionAndBurdock|D'n'B]]-''[[User_talk:DandelionAndBurdock|📞]]'' -- 19:03, 30 December 2024 (UTC) |
|||
Is there an essay or guidance on Wikipedia that discusses the relevance of non-school activity to articles on schools, when it is a current or former student that's involved? The cases I've come across are crimes reported in reliable sources, where the crimes are not school related, yet some think it's relevant to put that in the school's article because the indicted attend(ed) the school. Thanks. <small><sub>''signed'', </sub></small>[[User:Willondon|Willondon]] ([[User Talk:Willondon|talk]]) 20:02, 19 January 2024 (UTC) |
|||
::Okay thank you [[User:Silvernet123|Silvernet123]] ([[User talk:Silvernet123|talk]]) 19:05, 30 December 2024 (UTC) |
|||
:Many colleges/universities have a section for Notable alumni, meaning that those people are the subjects of Wikipedia articles. Notability can be for criminal activity. However, if a person is not Wikipedia-notable, I see no reason to mention that person (and their crime(s)) in a school article. [[User:David notMD|David notMD]] ([[User talk:David notMD|talk]]) 21:04, 19 January 2024 (UTC) |
|||
::{{u| |
:::{{u|Silvernet123}}, are you actually being paid to write complete garbage like {{tpq|He is known as one of the most philosophical minds in the 21st century with a record of 200+ Quotations and 89+ Poetry highlighting themes of love, patience, self-worth, discipline, and Christ-like values}} about an unknown 17 year old? That is the complete opposite of how an encyclopedia article should be written. Would you take a job as a chef if you don't know how to cook? [[User:Cullen328|Cullen328]] ([[User talk:Cullen328|talk]]) |
||
::::{{u|Silvernet123}} I would add that our incentive to help you is low......if you're being paid to be here, it's up to you to learn our standards and what we're looking for. We're here for free. [[User:331dot|331dot]] ([[User talk:331dot|talk]]) 19:18, 30 December 2024 (UTC) |
|||
:::Certainly, I'm on board with that. In the current case [https://en.wikipedia.org/enwiki/w/index.php?title=Hartshill_Academy&diff=prev&oldid=1197147857], these are hitherto unnotable people, indicted for murder; they just happen to attend that school, and the assault took place in a park area beside the school. (BTW Thanks for all the input here.) <!-- Template:Unsigned --><small class="autosigned">— Preceding [[Wikipedia:Signatures|unsigned]] comment added by [[User:Willondon|Willondon]] ([[User talk:Willondon#top|talk]] • [[Special:Contributions/Willondon|contribs]]) 23:10, 19 January 2024 (UTC)</small> |
|||
::::That's fair. I appreciate your feedback and will take the time to learn and meet Wikipedia's standards. Thank you for pointing me in the right direction. but still if you can assist I will appreciate [[User:Silvernet123|Silvernet123]] ([[User talk:Silvernet123|talk]]) 19:25, 30 December 2024 (UTC) |
|||
::::Without a Wikipedia article to link to, it's tough (not always impossible) to have alumni in the school article ([[WP:ALUMNI]] has a bit of info about some guidelines). However as you've pointed out on the talk page, the reference doesn't say anything about the school. With references comes some indication whether the two are related or not, and then there can be a debate about whether it's of significance in the school article. Without any reliable source making the connection it's a non-starter. I note there's a murder victim already in the article, with a linked Wikipedia article. Comparisons will be drawn. -- [[user:zzuuzz|zzuuzz]] <sup>[[user_talk:zzuuzz|(talk)]]</sup> 00:13, 20 January 2024 (UTC) |
|||
:::::Hello, @[[User:Silvernet123|Silvernet123]]. This is probably not what you want to hear, but: {{User:ColinFine/PractiseFirst}} [[User:ColinFine|ColinFine]] ([[User talk:ColinFine|talk]]) 20:59, 30 December 2024 (UTC) |
|||
:::::The entire article is written as self-promotion. One of your references is to fan writing on Medium claiming "(Account Not Own By Christen Kuikoua)", but appears to be copy/paste of material written by the subject. Promotional material does not belong in Wikipedia. Trying to create some fake noteriety by sneaking badly ghost-written articles into Wikipedia and elsewhere is unethical. If you wish to fake fame, then low-accountability social meda sites are probably more appropriate. [[User:Just Al|Just Al]] ([[User talk:Just Al|talk]]) 21:15, 30 December 2024 (UTC) |
|||
:Yo, I got here ‘cause I’ve been hearing you say I paid or my "representation" paid you to write an article. Like, not trying to be rude, but you gotta chill. I didn’t send you anywhere, and no one representing me sent you either. Bro, if you’re one of my friends messing around, you gotta stop. |
|||
:I looked at the draft, and while it’s kind of you to try writing something for me, you gotta do it the right way. Don’t go around saying I sent you somewhere—just saying, it’s not a good look. I’d like Wikipedia to delete the draft because I didn’t pay anyone, and I’m not planning on paying anyone either. [[User:Christenkofficial|Christenkofficial]] ([[User talk:Christenkofficial|talk]]) 23:54, 30 December 2024 (UTC) |
|||
::Hi Christenkofficial, |
|||
::Thank you for reaching out and clarifying your position. First, I want to say that I genuinely admire your work, which is why I was inspired to write about it. However, I now realize that my actions may have caused confusion or unintended trouble for you. I’m really sorry for using your name like that without your knowledge or permission it wasn’t my intention to create any issues. |
|||
::If you’d like the draft to be deleted, I completely understand and will fully support that. Thank you for bringing this to my attention, and I’ll be more careful moving forward. |
|||
::Best regards, |
|||
::Silvernet123 [[User:Silvernet123|Silvernet123]] ([[User talk:Silvernet123|talk]]) 00:09, 31 December 2024 (UTC) |
|||
{{collapse bottom}} |
|||
:Note that the account {{u|Christenkofficial}} has (correctly or not, I'm just reporting) now been blocked as a sockpuppet of {{u|Silvernet123}}, per [[Wikipedia:Sockpuppet investigations/Silvernet123]]. {The poster formerly known as 87.81.230.195} [[Special:Contributions/94.1.223.204|94.1.223.204]] ([[User talk:94.1.223.204|talk]]) 03:35, 31 December 2024 (UTC) |
|||
== Temporary Password == |
|||
== How to add sources == |
|||
I |
I need a simplified tutorial in how to add the number reference and the cite the source. [[User:M. Chris Tucker|M. Chris Tucker]] ([[User talk:M. Chris Tucker|talk]]) 19:02, 30 December 2024 (UTC) |
||
: Use [[Special:PasswordReset]] [[User:RudolfRed|RudolfRed]] ([[User talk:RudolfRed|talk]]) 20:10, 19 January 2024 (UTC) |
|||
:Hello {{user|M. Chris Tucker}}, might I suggest ''[[Help:Referencing for beginners|Referencing For Beginners]]''. -- [[User:DandelionAndBurdock|D'n'B]]-''[[User_talk:DandelionAndBurdock|📞]]'' -- 19:06, 30 December 2024 (UTC) |
|||
== Question == |
|||
:Hi @[[User:M. Chris Tucker|M. Chris Tucker]] and welcome to Wikipedia! The simplest way to add references is, directly after the sentence or paragraph that your source supports, add <nowiki><ref></nowiki>, followed by the text of your reference, followed by <nowiki></ref></nowiki>. The software will sort out the numbering for you. Others here should be able to give you more detailed advice, and the guide that D'n'B linked above looks like a great place to learn more. Thanks for your additions to [[:Edward Dickson (Canadian politician)]]! Best, [[User:Wham2001|Wham2001]] ([[User talk:Wham2001|talk]]) 19:07, 30 December 2024 (UTC) |
|||
:: [[Special:Contributions/2607:FEA8:7D00:95A0:FDD3:6EB9:68D5:D6D7|2607:FEA8:7D00:95A0:FDD3:6EB9:68D5:D6D7]] ([[User talk:2607:FEA8:7D00:95A0:FDD3:6EB9:68D5:D6D7|talk]]) 00:38, 31 December 2024 (UTC) |
|||
:::Thanks very much! I will follow up on your suggestion and wish you a Happy New Year. [[User:M. Chris Tucker|M. Chris Tucker]] ([[User talk:M. Chris Tucker|talk]]) 00:42, 31 December 2024 (UTC) |
|||
== How do I add a "Main Article" to Wikipedia == |
|||
Authority control, Defaultsort and Reflist template are minor edit? [[User:Youknowwhoistheman|Youknowwhoistheman]] ([[User talk:Youknowwhoistheman|talk]]) 20:20, 19 January 2024 (UTC) |
|||
I updated our "Cornhusker Council" section under Boy Scouts of America, Nebraska Scouting. and would like to create a main article for our council. how do I do that? |
|||
:@[[User:Youknowwhoistheman|Youknowwhoistheman]]: Welcome to the Teahouse! [[Help:Minor edit]] says that adding templates is ''not'' a minor edit. Happy editing! [[User:GoingBatty|GoingBatty]] ([[User talk:GoingBatty|talk]]) 03:17, 20 January 2024 (UTC) |
|||
[[User:Cornhusker324|Cornhusker324]] ([[User talk:Cornhusker324|talk]]) 21:15, 30 December 2024 (UTC) |
|||
== No fair!! == |
|||
:{{u|Cornhusker324}} Hello and welcome to the Teahouse. First, please see [[WP:COI|conflict of interest]], as that needs to be formally disclosed. For a standalone article about your specific council, you would need to show with significant coverage in independent [[WP:RS|reliable sources]] that your council meets [[WP:ORG|the special Wikipedia definition of a notable organization]]. That coverage can't just be the reporting of its routine activities, but in depth coverage as to what makes your council important/significant/influential. Be aware that writing a new article is the most difficult task to attempt on Wikipedia, and it's even harder with a conflict of interest. Also be aware that an article is [[WP:PROUD|not necessarily desirable]]. [[User:331dot|331dot]] ([[User talk:331dot|talk]]) 21:23, 30 December 2024 (UTC) |
|||
::Hello @[[User:Cornhusker324|Cornhusker324]]. I have just reverted your edits in accordance with our conflict of interest policy. Please request specific edits on the article's respective [[help:Talk pages|talk page]]. Thanks. [[User:Tarlby|<span style="color:cyan;font-family:Comic Sans MS;">''Tarl''</span><span style="color:orange;font-family:Comic Sans MS;">''by''</span>]] <sup>([[User talk:Tarlby|''t'']]) ([[Special:Contributions/Tarlby|''c'']])</sup> 21:25, 30 December 2024 (UTC) |
|||
:::And I'm not sure why the council itself would merit a standalone article from the article about scouting in Nebraska. [[User:331dot|331dot]] ([[User talk:331dot|talk]]) 21:31, 30 December 2024 (UTC) |
|||
::::Many other Councils already have a standalone article. I would think this would be supported for consistency sake. [[User:Cornhusker324|Cornhusker324]] ([[User talk:Cornhusker324|talk]]) 21:35, 30 December 2024 (UTC) |
|||
:::::@[[User:Cornhusker324|Cornhusker324]], please see [[WP:42]]. Some council have articles. Others don't, and maybe they shouldn't. It all depends on the [[wp:rs|reliable sources]] that we need to use so that an article can be made. An article on Cornhusker Council won't be made if there are no such sources. [[User:Tarlby|<span style="color:cyan;font-family:Comic Sans MS;">''Tarl''</span><span style="color:orange;font-family:Comic Sans MS;">''by''</span>]] <sup>([[User talk:Tarlby|''t'']]) ([[Special:Contributions/Tarlby|''c'']])</sup> 21:40, 30 December 2024 (UTC) |
|||
::::::Noted [[User:Cornhusker324|Cornhusker324]] ([[User talk:Cornhusker324|talk]]) 21:43, 30 December 2024 (UTC) |
|||
:::I have changed my user name in hopes of complying with your COI policy. The stand-alone article seems too difficult at this point. Perhaps in the future. |
|||
:::As such, I would simply like to update our section, "Cornhusker Council," in "Scouting in Nebraska." and will attempt that in the 'talk page,' as you suggested. [[User:Cornhusker324|Cornhusker324]] ([[User talk:Cornhusker324|talk]]) 21:42, 30 December 2024 (UTC) |
|||
Your name change does not show up yet, and regardless of a name change, you still have a COI. I see that you are proposing changes on the Talk page of the article, but much of what you wrote is not referenced. See [[Help:Referencing for beginners]] to understand how to insert inline refs. [[User:David notMD|David notMD]] ([[User talk:David notMD|talk]]) 05:30, 31 December 2024 (UTC) |
|||
I wanted to make a page called: "Republic Of Kit Kat!" on Wikipedia, (a micronation I made up.) but, when clicking on a red link, it did not even let me even create it. Even when I clicked on: "Create a draft and submit it for review." That will not make me create a page!! Even MicroWiki allows you to do this. Please tell me the proper way. |
|||
:I provided four references which is more than what is currently listed. Those currently used are not authoritative and factually incorrect. [[Special:Contributions/209.92.187.50|209.92.187.50]] ([[User talk:209.92.187.50|talk]]) 15:34, 31 December 2024 (UTC) |
|||
From your biggest fan: PolskiSlaskiego! [[User:PolskiSlaskiego!|PolskiSlaskiego!]] ([[User talk:PolskiSlaskiego!|talk]]) 20:23, 19 January 2024 (UTC) |
|||
::How exactly did you change your username? It should be done via either [[Special:GlobalRenameRequest]] or [[WP:CHUS]]. You should continue to use your original username until it is changed, at which time you can begin using the new name. [[User:331dot|331dot]] ([[User talk:331dot|talk]]) 15:48, 31 December 2024 (UTC) |
|||
::Never mind, I see it is changed now. [[User:331dot|331dot]] ([[User talk:331dot|talk]]) 15:49, 31 December 2024 (UTC) |
|||
:::That is not how references are created. Also, you now appear to be editing as an IP address, i.e., not logged into an account versus a name-change. [[User:David notMD|David notMD]] ([[User talk:David notMD|talk]]) 16:54, 31 December 2024 (UTC) |
|||
::::I've used two different means to source the information I provided, both of which were recommended by this site and/or a moderator. [[User:CCJLJ|CCJLJ]] ([[User talk:CCJLJ|talk]]) 17:04, 31 December 2024 (UTC) |
|||
:::::Again, see Help:Referencing for beginners to understand how to insert inline refs. What you posted on the Talk page of the article (below) are not properly formatted references. [[User:David notMD|David notMD]] ([[User talk:David notMD|talk]]) 21:22, 31 December 2024 (UTC) |
|||
References supporting change: |
|||
*Salistean, John, "A History of the Cornhusker Council 1940-1975," Houchen Bindery LTD of Utica, Nebraska, 1st ed., 2011. |
|||
*Golden Sun Lodge Website, www.goldensunlodge.org |
|||
*Cornhusker Council Website, www.cornhuskercouncil.org |
|||
*Brown, Elinor L., "History of Lancaster County, Then and Now," ASIN B0006CJTC2, Jan 1971. |
|||
== Notability and Independence. == |
|||
:{{ping|PolskiSlaskiego!}} [[WP:MADEUP|Wikipedia is not for things you made up]] – please do not create such an article. [[User:Tollens|Tollens]] ([[User talk:Tollens|talk]]) 20:25, 19 January 2024 (UTC) |
|||
:{{ping|PolskiSlaskiego!}} Hello Polski! Unfortunately [[WP:MADEUP|Wikipedia is not the place to create articles on things you made up one day]]. We create articles based on what reliable sources say, and something you just made up most definitely does not have any reliable sources. ― [[User:Blaze Wolf|<b style="background:#0d1125;color:#51aeff;padding:1q;border-radius:5q;">Blaze Wolf</b>]][[User talk:Blaze Wolf|<sup>Talk</sup>]]<sub title="Discord Username" style="margin-left:-22q;">blaze__wolf</sub> 20:25, 19 January 2024 (UTC) |
|||
::I am SO sorry, but that was not my actual problem. (I did that by complete accident.) My actual question is what I stated at the very end: "How do you properly create a page?" I tried clicking on a red link, no luck. I tried creating a draft, did not work. I am even 100% logged in, so this makes no sense to me. I am just stuck here, not able to find the correct way. So please, if you can, reply to this question telling me how to create a page PROPERLY. I am SO sorry to Wikipedia, @[[User:Tollens|Tollens]], and @[[User:Blaze Wolf|Blaze Wolf]] for being so harsh. My apologies. |
|||
::From your biggest fan: PolskiSlaskiego! [[User:PolskiSlaskiego!|PolskiSlaskiego!]] ([[User talk:PolskiSlaskiego!|talk]]) 20:33, 19 January 2024 (UTC) |
|||
:::{{ping|PolskiSlaskiego!}} Well assuming you aren't wanting to create an article (Not a page, tho all articles are technically pages but not all pages are articles) on your made up micronation, I suggest you follow [[WP:YFA]]. What are you seeing when you click on a red link? ― [[User:Blaze Wolf|<b style="background:#0d1125;color:#51aeff;padding:1q;border-radius:5q;">Blaze Wolf</b>]][[User talk:Blaze Wolf|<sup>Talk</sup>]]<sub title="Discord Username" style="margin-left:-22q;">blaze__wolf</sub> 20:38, 19 January 2024 (UTC) |
|||
::::When clicking on a red link, it reads: "The article that you're looking for doesn't exist." And then, it says: "......create it as a draft." and when clicked on, it sends to, indeed a draft, but that is not what I want. But, are you <u>supposed</u> to start with a draft? Because I want an already official article when I hit done, or whatever the finish button is. I think I might have solved the problem, but if there are any alternative ways, then please let me know. |
|||
::::From your biggest fan: PolskiSlaskiego! [[User:PolskiSlaskiego!|PolskiSlaskiego!]] ([[User talk:PolskiSlaskiego!|talk]]) 20:58, 19 January 2024 (UTC) |
|||
:::::New accounts cannot directly create articles, you may use the [[WP:WIZARD|Article Wizard]] to do so- though creating a new article is challenging, it's best to get some editing experience first. [[User:331dot|331dot]] ([[User talk:331dot|talk]]) 21:01, 19 January 2024 (UTC) |
|||
::::::I forgot about the fact that new accounts can't directly create articles. {{u|PolskiSlaskiego!}} I would recommend following 331dot's advice and get some editing experience in first before creating a new article. If you really think you can't wait and want to create an article now, then you may use the Article Wizard to do so. ― [[User:Blaze Wolf|<b style="background:#0d1125;color:#51aeff;padding:1q;border-radius:5q;">Blaze Wolf</b>]][[User talk:Blaze Wolf|<sup>Talk</sup>]]<sub title="Discord Username" style="margin-left:-22q;">blaze__wolf</sub> 21:03, 19 January 2024 (UTC) |
|||
:::::Hello, PolskiSlaskiego, and welcome to the Teahouse and to Wikipedia. To echo what DavidMD says: would you build a car as your first ever engineering project? Or enter a tournament the first day you started a new sport? ''Please'' get some experience editing - and learn about Wikipedia's policies, such as [[WP:verifiability|verifiability]], [[WP:reliable sources|reliable sources]], [[WP:neutral point of view|neutral point of view]], and [[WP:notability|notability]], before you try to create a new article. You will save yourself and other people a lot of frustration and effort if you do this. [[User:ColinFine|ColinFine]] ([[User talk:ColinFine|talk]]) 22:30, 19 January 2024 (UTC) |
|||
My grandmother, Ethel Margaret Streit Harrison, was the first woman elected as Clerk of the Montana Supreme Court, one of the founders of the Montana Association of Female Executives and one of the original board members of the Holter Museum of Art. There are printed sources that talk about her achievements but according to wikipedia the only thing notable about her was that she was married to [[John C. Harrison (judge)]]. I understand that, as a relative, there is a potential conflict here, but I think it is important her contributions are documented on something other than microfiche. Any thoughts? / Thank you :) [[User:Mehap dwhx|Mehap dwhx]] ([[User talk:Mehap dwhx|talk]]) 22:40, 30 December 2024 (UTC) |
|||
Baby steps: Think of a topic. Check to see if an article already exists. If not, find reliable source published references about the topic. See [[WP:42]] to learn what that is about. If no existing article, and references do exist, use Article Wizard to create and then submit a draft, with properly formatted references. Only include in the draft the facts that are provided by the references. Once submitted, an experienced reviewer can Accept, Decline, Reject or Speedy delete your draft. P.S. "Publish changes" means save, it does not mean publish in Wikipedia's mainspace. [[User:David notMD|David notMD]] ([[User talk:David notMD|talk]]) 21:24, 19 January 2024 (UTC) |
|||
:[[User:Mehap dwhx|Mehap dwhx]], none of those three roles that you describe automatically confers [[WP:N|Wikipedia-defined "notability"]] on a person. And the sum of the three doesn't either. But it's possible that she's "notable" all the same. Regardless of your conflict of interest, you're free to create [[Draft:Ethel Margaret Streit Harrison]]. If this demonstrates her notability and follows Wikipedia's other policies, it will be promoted to an article (possibly with a slightly different title), whereupon you shouldn't continue to edit it but would be free to make suggestions and requests on its talk page. -- [[User:Hoary|Hoary]] ([[User talk:Hoary|talk]]) 23:12, 30 December 2024 (UTC) |
|||
:To expand an aspect of Hoary's excellent advice above: in Wikipedia, '[[WP:Notability|Notability]]' boils down to "is there ''enough'' substantial material, published [[Wikipedia:Independent sources|independently]] of the subject, in multiple [[WP:Reliable sources|Reliable sources]], to form the basis of an article about the subject. It doesn't necessarily require that the subject is 'famous', or in one of many possible senses 'important'. |
|||
:@[[User:Blaze Wolf|Blaze Wolf]] Thank you so much! [[User:PolskiSlaskiego!|PolskiSlaskiego!]] ([[User talk:PolskiSlaskiego!|talk]]) 22:12, 19 January 2024 (UTC) |
|||
:Also, the fact that your grandmother's achievements are ''not yet'' in Wikipedia doesn't mean Wikipedia asserts they ''aren't'', or she as a whole ''isn't'', notable; merely that no volunteer editor has yet gathered the necessary sources and added the information (with [[WP:citations|citations]]), whether in her husband's article or in a draft for her own. I encourage you to try. Good luck! |
|||
:While we're here, I notice that [[John C. Harrison (judge)|John C. Harrison's]] article is on shaky ground, because it's entirely cited to a single (though reliable) source: we usually prefer a minimum of three, so if you could flesh out ''that'' article with further referenced material, it would be a good thing. {The poster formerly known as 87.81.230.195} [[Special:Contributions/94.1.223.204|94.1.223.204]] ([[User talk:94.1.223.204|talk]]) 03:56, 31 December 2024 (UTC) |
|||
::Add '''Ethel Margaret Streit Harrison''' to: [[John_C._Harrison_(judge)#Personal_life]] |
|||
::[[Special:Contributions/69.181.17.113|69.181.17.113]] ([[User talk:69.181.17.113|talk]]) 04:45, 31 December 2024 (UTC) |
|||
:::can you find at least 4 [[WP:RS]] sources ? [[Special:Contributions/69.181.17.113|69.181.17.113]] ([[User talk:69.181.17.113|talk]]) 04:47, 31 December 2024 (UTC) |
|||
:::She's already mentioned in that section as Ethel Harrison, cited to the article's (only) reference. Even to add her middle names would require a further citation to a ''published reliable source'' that mentions them (the existing one does not: I've downloaded and searched it). {The poster formerly known as 87.81.230.195} [[Special:Contributions/94.1.223.204|94.1.223.204]] ([[User talk:94.1.223.204|talk]]) 06:56, 31 December 2024 (UTC) |
|||
::::Being married to JCH can be mentioned in her Personal life section, but does not contribute to establishing her notability. [[User:David notMD|David notMD]] ([[User talk:David notMD|talk]]) 05:34, 31 December 2024 (UTC) |
|||
== Restoring previously unmerged articles == |
|||
== Translating a page == |
|||
How can I translate a page from english to bangla [[User:Tausif23|Tausif23]] ([[User talk:Tausif23|talk]]) 21:23, 19 January 2024 (UTC) |
|||
: {{re|Tausif23}} See [[WP:TRANSLATEUS]] for the guidance. [[User:RudolfRed|RudolfRed]] ([[User talk:RudolfRed|talk]]) 21:26, 19 January 2024 (UTC) |
|||
I would like to create a new article but it [https://en.wikipedia.org/enwiki/w/index.php?title=Cyprus%E2%80%93Jordan%20relations&redirect=no once existed back in 2009] and got merged without consensus. Is it okay if I restore the article but with more and better sources from [[User:Underdwarf58/sandbox|my sandbox]] later on? Because one user tried to restore a merged article and it got immediately undone because a consensus existed to merge it. I'm not saying that I'll do it right now as it's still unfinished but I need to ask because if I can't then I'll just cancel. [[User:Underdwarf58|Underdwarf58]] ([[User talk:Underdwarf58|talk]]) 00:29, 31 December 2024 (UTC) |
|||
== Disabling military time == |
|||
:There's very little about the original subject 'Cyprus–Jordan relations' in [[Foreign relations of Cyprus]] that it was merged into, so ''I'' think that if ''you'' think you have enough [[Wikipedia:Reliable sources|Reliably sourced]] material (bearing in mind there's also been another 15 years of history) you should go ahead and create a new [[Wikipedia:Drafts|Draft]] on the subject, via the normal [[Wikipedia:Articles for creation]] process, which can be submitted for assessment. Good luck! {The poster formerly known as 87.81.230.195} [[Special:Contributions/94.1.223.204|94.1.223.204]] ([[User talk:94.1.223.204|talk]]) 04:11, 31 December 2024 (UTC) |
|||
Hullo friends. I would like to disable military time in signatures and the like such that it says something like "[[User:Example|Example]] ([[User talk:Example|talk]]) '''3:00 PM''', 19 January 2024 (UTC)" rather than "[[User:Example|Example]] ([[User talk:Example|talk]]) '''15:00''', 19 January 2024 (UTC)". I was wondering where I would find a setting, gadget, or other such thing to do that. That's all. Thanks! [[User:Antrotherkus|Antrotherkus]] ([[User talk:Antrotherkus|Talk to me!]]) 23:58, 19 January 2024 (UTC) |
|||
== template == |
|||
:@[[User:Antrotherkus|Antrotherkus]] Click [[special:preferences|"Preferences"]] (which you can get to by looking at the top of your display or clicking what I just linked there.) Then, click the "Appearance" tab there. There should be some options for you to adjust the time display there. Cheers [[User talk:Relativity|<b style="border-radius:3em;padding:6px;background:#e82c52;color:white;"> Relativity </b>]]<span style="display:inline-block;margin-bottom:-0.3em;vertical-align:-0.4em;line-height:1.2em;font-size:80%;text-align:left"></span> 00:50, 20 January 2024 (UTC) |
|||
::When I go to the "Appearance" tab, there are only two time-related options: "Date format" and "Time offset". Neither of them seem to be related to 24-hour time. If you're curious, I use the Vector 2010 skin. [[User:Antrotherkus|Antrotherkus]] ([[User talk:Antrotherkus|Talk to me!]]) 01:19, 20 January 2024 (UTC) |
|||
:@[[User:Antrotherkus|Antrotherkus]] I would use [[Wikipedia:Comments in Local Time]]. [[User:Galobtter|Galobtter]] ([[User talk:Galobtter|talk]]) 02:05, 20 January 2024 (UTC) |
|||
::Perfect! Thanks! [[User:Antrotherkus|Antrotherkus]] ([[User talk:Antrotherkus|Talk to me!]]) 02:15, 20 January 2024 (UTC) |
|||
Hello, I'm need of some assistance with creating a Wikipedia page about a music artist. Can anyone advise which template to use? [[User:RATHOMP|RATHOMP]] ([[User talk:RATHOMP|talk]]) 02:45, 31 December 2024 (UTC) |
|||
== NIAC == |
|||
:look at other Wikipedia pages about music artists ... [[Special:Contributions/69.181.17.113|69.181.17.113]] ([[User talk:69.181.17.113|talk]]) 04:40, 31 December 2024 (UTC) |
|||
Can somebody take a look at the [[National Iranian American Council]] article? An editor keeps removing edits cited to news articles and replacing it with information sourced only to the organization's website. [[User:Just want to make some quick edits here|jwtmsqeh]] ([[User talk:Just want to make some quick edits here|talk]]) 00:01, 20 January 2024 (UTC) |
|||
::Hello, IP user, and welcome to the Teahouse. From your question, I suspect that you are focussing on the layout of your proposed article (I'm guessing that that's what you mean by a "template" - we use the word a bit differently here). But while the layout of an article is important, it is MUCH less important than the quality of the sources used. Until you have found adequate sources to establish that the artist meets Wikipedia's criteria for [[WP:notability|}], it's pointless spending any time thinking about the content or the layout. To use a house-building analogy, you may have an idea for what you want your house to look like, and even a plan; but until you've surveyed the site to make sure it's fit to build on, and checked that your plans meet local building regulations, it would be a waste of effort to start building. |
|||
::More generally, {{User:ColinFine/PractiseFirst}} [[User:ColinFine|ColinFine]] ([[User talk:ColinFine|talk]]) 10:36, 31 December 2024 (UTC) |
|||
:Is there a specific place you're currently working on the article at? I can't seem to find it on your userpage as a subpage. For advice, I'd recommend looking at other music artist articles (specifically various quality articles from [[The Beatles]] [FA] to [[Sepultura]] [C-class]) for general outlines on how to write it. For infoboxes, use <nowiki>{{infobox musical artist}}</nowiki> and fill out the template using it's template page at [[Template:Infobox musical artist]]. For general advice on writing, see Wikipedia's content policies and guidelines, I'd recommend WP:Nutshell as a starting point. |
|||
:Thanks, [[User: Sparkle & Fade|''Sparkle and Fade'']] <sup>[[User_talk:Sparkle & Fade|talk]]</sup><sub>[[Special:Contributions/Sparkle & Fade|edits]]</sub> 08:23, 31 December 2024 (UTC) |
|||
== how can I rigth a article in wikipedia == |
|||
:I believe that would be some form of blocking good-faith edits, and I’m pretty sure it is a sign of bad-faith. [[Cometkeiko]] <!-- Template:Unsigned --><small class="autosigned">— Preceding [[Wikipedia:Signatures|unsigned]] comment added by [[User:Cometkeiko|Cometkeiko]] ([[User talk:Cometkeiko#top|talk]] • [[Special:Contributions/Cometkeiko|contribs]]) 08:57, 20 January 2024 (UTC)</small> <!--Autosigned by SineBot--> |
|||
how can I rigth [[User:Daniel Muanga|Daniel Muanga]] ([[User talk:Daniel Muanga|talk]]) 03:49, 31 December 2024 (UTC) |
|||
::Edit warring ongoing! The contending editors (including Just want...) are also heatedly debating their disagreements on the article's Talk page. If they cannot solve it there, then advice needed on how to get third parties involved. [[User:David notMD|David notMD]] ([[User talk:David notMD|talk]]) 11:39, 20 January 2024 (UTC) |
|||
:[[Wikipedia:Article wizard]] |
|||
:[[Special:Contributions/69.181.17.113|69.181.17.113]] ([[User talk:69.181.17.113|talk]]) 04:39, 31 December 2024 (UTC) |
|||
::New editors are strongly advised to first gain skills by doing time improving existing articles. References required. [[User:David notMD|David notMD]] ([[User talk:David notMD|talk]]) 05:39, 31 December 2024 (UTC) |
|||
== Find a Grave = WP:RS ? == |
|||
How do I find a draft article? [[User:OLYMPICHAMMER|OLYMPICHAMMER]] ([[User talk:OLYMPICHAMMER|talk]]) 01:28, 20 January 2024 (UTC) |
|||
Is [[Find a Grave]] = [[WP:RS]] ? |
|||
:What are you trying to do? What draft are you looking for? [[User:Galobtter|Galobtter]] ([[User talk:Galobtter|talk]]) 02:03, 20 January 2024 (UTC) |
|||
[[Special:Contributions/69.181.17.113|69.181.17.113]] ([[User talk:69.181.17.113|talk]]) 04:37, 31 December 2024 (UTC) |
|||
:If it's a draft titled, say, "Aave protocol", then you look for "Draft:Aave protocol". If on the other hand you're wondering how to use a search engine such as Google to find a draft about, say, the "Aave protocol", then the answer is that you cannot. -- [[User:Hoary|Hoary]] ([[User talk:Hoary|talk]]) 02:05, 20 January 2024 (UTC) |
|||
:{{welcometea}} Please consult [[Wikipedia:FINDAGRAVE]]. —[[User:Tenryuu|<span style="color:#556B2F">Tenryuu 🐲</span>]] ( [[User talk:Tenryuu|💬]] • [[Special:Contributions/Tenryuu|📝]] ) 05:03, 31 December 2024 (UTC) |
|||
::Please see my other comment [[User:OLYMPICHAMMER|OLYMPICHAMMER]] ([[User talk:OLYMPICHAMMER|talk]]) 02:07, 20 January 2024 (UTC) |
|||
:The answer is "no" because Find a Grave consists of user generated content, and is specifically mentioned in the [[WP:USERGENERATED]] section of the reliable sources guideline as a source that should not be used as a reference on Wikipedia. That does not mean that Find a Grave is of no value to Wikipedia editors. You may be able to find nuggets in those listings that will inform your searches about various people. Some but not all Find a Grave listings include references to reliable sources, and those sources may be useful as Wikipedia references. One thing that Find a Grave can teach editors is that many people down through the years share the same name, and we need to be very careful to avoid including biographical details about one Andrew Wilson in an article about another Andrew Wilson. That's just one of countless examples. I have been working on [[Andrew Stephen Wilson]] today, so that's why I chose that example.. [[User:Cullen328|Cullen328]] ([[User talk:Cullen328|talk]]) 06:22, 31 December 2024 (UTC) |
|||
:I found my draft, and it has been rejected for unreliable sources. I have 18 sources, including LA Times, YouTube video, Track and Field News publication, and major college athletic websites. I dont know what to do to improve this? [[User:OLYMPICHAMMER|OLYMPICHAMMER]] ([[User talk:OLYMPICHAMMER|talk]]) 02:05, 20 January 2024 (UTC) |
|||
::Are either of you willing to look at my article and comment on what is wrong with my sources? It is Drat Art Venegas [[User:OLYMPICHAMMER|OLYMPICHAMMER]] ([[User talk:OLYMPICHAMMER|talk]]) 02:09, 20 January 2024 (UTC) |
|||
:::Draft Art Venegas [[User:OLYMPICHAMMER|OLYMPICHAMMER]] ([[User talk:OLYMPICHAMMER|talk]]) 02:12, 20 January 2024 (UTC) |
|||
::@[[User:OLYMPICHAMMER|OLYMPICHAMMER]] I see you refer to [[Draft:Art Venegas]] are "my article", but it was created by {{U|Eric-Dieter}}, who has also asked questions at [[Wikipedia:Help desk#(moving an article)]]. Are you using multiple accounts? |
|||
::{{U|S0091}} suggested you review [[Wikipedia:INTREFVE|this guide]] for how to cite sources. Did you have specific questions about that? Maybe you'd prefer the video at [[WP:EASYREFBEGIN]]. |
|||
::{{U|Victor Schmidt}} gave you specific feedback about some of the references. Did you have any specific questions about that feedback? [[User:GoingBatty|GoingBatty]] ([[User talk:GoingBatty|talk]]) 03:11, 20 January 2024 (UTC) |
|||
:::I am working with Eric on this project, and he is the one that posted the article. Victor Schmidt and S0091 only sent me educational materials and guides, which I have gone thoroughly over hours and can see nothing wrong with our sources. |
|||
:::We are citing publications like the LA Times, Track and Field News, YouTube interview videos, major college Athletic Dept websites, and other independent articles online. What more do we need to do? [[User:OLYMPICHAMMER|OLYMPICHAMMER]] ([[User talk:OLYMPICHAMMER|talk]]) 03:18, 20 January 2024 (UTC) |
|||
::::@[[User:OLYMPICHAMMER|OLYMPICHAMMER]]: They expressed concerns with the formatting of the references, so I added an inline citation for reference #1 for you as an example. They also noted that the draft is not adequately supported by reliable sources. For example, where is the source for all the information in the "Art Venegas coaching tree"? If you are Venegas or are working with Venegas, then you and Eric-Dieter have a [[WP:COI|conflict of interest]] that you each need to [[WP:DISCLOSECOI|disclose]] on your respective [[WP:User page|user page]]. [[User:GoingBatty|GoingBatty]] ([[User talk:GoingBatty|talk]]) 03:36, 20 January 2024 (UTC) |
|||
::::@[[User:OLYMPICHAMMER|OLYMPICHAMMER]]: Also note that interviews are not independent, so those won't help with notability. Plus, the tone needs to be changed from promotional (e.g. "legendary", "Venegas is regarded to have few peers", "cemented his coaching preeminence") to neutral and encyclopedic. [[User:GoingBatty|GoingBatty]] ([[User talk:GoingBatty|talk]]) 03:40, 20 January 2024 (UTC) |
|||
:::::GoingBatty, I have cleaned up the Draft: Art Venegas article using the input of you and four other editors. Can you review, I want to submit it again? [[User:OLYMPICHAMMER|OLYMPICHAMMER]] ([[User talk:OLYMPICHAMMER|talk]]) 23:14, 20 January 2024 (UTC) |
|||
== Hello -- I would like to start helping, but I'm having trouble finding what to do! == |
|||
:Sample: {{Olive| Venegas coached UCLA throwers to some of the greatest performances in NCAA history, and his athletes include some of the most notable collegiate competitors ever.}} To which is appended "[6]". "[6]" turns out to be a web page published by UCLA. This is not a disinterested source, and thus it is not a [[WP:RS|reliable source]]. -- [[User:Hoary|Hoary]] ([[User talk:Hoary|talk]]) 04:35, 20 January 2024 (UTC) |
|||
::Hoary, I have cleaned up the Draft: Art Venegas article using the input of you and four other editors. Can you review, I want to submit it again? [[User:OLYMPICHAMMER|OLYMPICHAMMER]] ([[User talk:OLYMPICHAMMER|talk]]) 23:20, 20 January 2024 (UTC) |
|||
:Note discussion about this draft are currently in a lot of places (here, [[Wikipedia:Help_desk#(moving_an_article)|WP:HD]], on the draft itself, at [[User_talk:Victor_Schmidt#Draft:_Art_Venegas|my talk page]], and at [[User_talk:Ad_Orientem#Draft:_Art_Venegas|Ad Orientem's]] user talk). |
|||
::{{u|OLYMPICHAMMER}}, please note that on Wikipedia, declined and rejected mean different things. Draft:Art Venegas was declined. When reviewers decline a draft, this means ''sorry, this cannot be promoted to an article at this time. Please improve it further'', whereas rejected means ''Sorry, there is no potential this becomes an article any time soon. Please stop trying.'' [[User:Victor Schmidt|Victor Schmidt]] ([[User talk:Victor Schmidt|talk]]) 07:05, 20 January 2024 (UTC) |
|||
:::You have done a lot of work after [[Draft:Art Venegas]] was declined, but the referencing method you use is not acceptable. See [[Help:Referencing for beginners]] and examples of articles of the athlete articles you Wikilinked to see proper referencing. When refs are done right, the software inserts a superscript number and adds the ref under References. As noted above, refs need to be independent. Continue to fix stuff and submit again. [[User:David notMD|David notMD]] ([[User talk:David notMD|talk]]) 11:56, 20 January 2024 (UTC) |
|||
::::David, I have cleaned up the Draft: Art Venegas article using the input of you and four other editors. Can you review, I want to submit it again? [[User:OLYMPICHAMMER|OLYMPICHAMMER]] ([[User talk:OLYMPICHAMMER|talk]]) 23:13, 20 January 2024 (UTC) |
|||
::::{{u|OLYMPICHAMMER}} I recommend deleting the 'Coaching Tree' section. P.S. I am not an Reviewer. [[User:David notMD|David notMD]] ([[User talk:David notMD|talk]]) 05:34, 21 January 2024 (UTC) |
|||
:::::Each one of those is annotated with online publications? At some point you editors are ridiculous......... [[User:OLYMPICHAMMER|OLYMPICHAMMER]] ([[User talk:OLYMPICHAMMER|talk]]) 06:21, 21 January 2024 (UTC) |
|||
I see lots of support for ultra basics, but I know what the concept is, and how to edit, and how to make italics and hyperlinks, and that I should use a neutral voice, etc. I am trying to find some guidance on WHAT to contribute. I found the Typo Team (or at least, [[Wikipedia:Typo Team/moss/D|this typo team]]), but I haven't found guidance on interacting with it. (Do I delete entries if I resolve them? Yesterday I found many entries to check, but today none of the articles seem to HAVE the potential typo that was listed, or even a fixed version). I have found [[mediawikiwiki:Growth/FAQ#|this Growth page]], but can't get the features working. For example, it says to enable the Help panel in the Editing tab, but I don't see such a thing in the Editing tab. I also can't find "Display newcomer homepage" in my user preferences. Similar with Suggested Edits -- how can I "use <code>Special:NewcomerTasksInfo</code>"? Etc., etc. I must be missing some key piece of advice -- where can I figure out how to get things rolling? [[User:Skmccormick|SKM]] ([[User talk:Skmccormick|talk]]) 05:08, 31 December 2024 (UTC) |
|||
== Help == |
|||
:Hello @[[User:Skmccormick|Skmccormick]]. [[WP:volunteer|As a volunteer project]], you're free to do whatever you wanna do best. Want to copyedit a bunch of articles and bring our grammar up to shape? Join the [[WP:goce|Guild of Copy Editors]] and go wild. Wanna fight vandalism? Go patrol [[Special:RecentChanges]] and stop those dang vandals! Wanna go help out that typo team? Go right ahead. It's your choice. [[User:Tarlby|<span style="color:cyan;font-family:Comic Sans MS;">''Tarl''</span><span style="color:orange;font-family:Comic Sans MS;">''by''</span>]] <sup>([[User talk:Tarlby|''t'']]) ([[Special:Contributions/Tarlby|''c'']])</sup> 05:26, 31 December 2024 (UTC) |
|||
Í ám á new user who made lots of edits on my IP account. Í was wondering if i could have my IP account transfered to my new one. [[User:Socialstviper|Socialstviper]] ([[User talk:Socialstviper|talk]]) 02:17, 20 January 2024 (UTC) |
|||
::@[[User:Skmccormick|Skmccormick]] |
|||
::*You should find display user homepage at the bottom of [[Special:Preferences]], "User profile" tab. |
|||
::*If I wanted to interact with the typo team, my first place to try would be [[Wikipedia talk:Typo Team]]. |
|||
::*Have you found [[WP:TASKS]] and [[WP:REQUEST]]? [[User:Gråbergs Gråa Sång|Gråbergs Gråa Sång]] ([[User talk:Gråbergs Gråa Sång|talk]]) 07:28, 31 December 2024 (UTC) |
|||
:::Thank you! These links are helpful. I found "display user homepage" under User Profile. So maybe a silly question, but: how do I find this user homepage? I don't really go to Wikipedia generically, I usually jump straight to an article. Tasks and Requests seem like what I'm looking for. [[User:Skmccormick|SKM]] ([[User talk:Skmccormick|talk]]) 01:47, 1 January 2025 (UTC) |
|||
::::The link should be right at the top of every page when you're logged in, in the same place/menu as the link to your userpage. -- [[User:Asilvering|asilvering]] ([[User talk:Asilvering|talk]]) 02:05, 1 January 2025 (UTC) |
|||
:@[[User:Skmccormick|Skmccormick]], do you have any particular topics that you'd like to edit about? One way to find a lot of articles that need help is to go through our various maintenance backlogs (Gråbergs Gråa Sång has already linked you to [[WP:TASKS]]). Some people are content to plug away at a particular backlog chronologically, but if you prefer to edit on things you're generally interested in, it's helpful to filter these by wikiproject. Alternatively, do you have any particular skills or outside knowledge that might be helpful here? There's always demand for multilingual editors, the copyright folks are always backlogged, etc. -- [[User:Asilvering|asilvering]] ([[User talk:Asilvering|talk]]) 10:16, 31 December 2024 (UTC) |
|||
::Thank you. I would prefer to get into the groove with smaller contributions before I start throwing any weight around. Sadly, I cannot offer multilingual help; I am American. I'm mostly having trouble navigating the various pages and internal tools like TASKS or RecentChanges (anything labeled "Special:" is still new to me). [[User:Skmccormick|SKM]] ([[User talk:Skmccormick|talk]]) 01:50, 1 January 2025 (UTC) |
|||
:::I'm a fan of [[:Category:Wikipedia introduction cleanup]] as a newbie task - not ''small'', exactly, but hey, no time like the present to learn to [[WP:BEBOLD]]. You don't (or shouldn't) need to do any research to fix these articles - most of them are here because they are tagged with "lead too short". Find one of those, read the article, then rewrite the lead so it summarizes it accurately. Then remove the tag. All this requires is good English literacy, and since the lead is what most people read and what is used in the google knowledge box etc, it's a high-impact change that requires very little wiki-knowledge. -- [[User:Asilvering|asilvering]] ([[User talk:Asilvering|talk]]) 02:16, 1 January 2025 (UTC) |
|||
== Different images for Light/Dark mode == |
|||
:Hi, {{u|Socialstviper}}, and welcome! I’m afraid it’s not possible to assign IP contributions to your account, although if you wish you can indicate on your [[Help:userpage|userpage]] that you edited previously under an IP and can specify there. Happy editing! [[User:Perfect4th|Perfect4th]] ([[User talk:Perfect4th|talk]]) 02:23, 20 January 2024 (UTC) |
|||
Is there any way to tell Wikipedia to display different images / media for Light vs Dark mode users? Due to transparency, some SVG and PNG images have bad contrast when viewed in Dark mode. [[User:CrushedAsian255|CrushedAsian255]] ([[User talk:CrushedAsian255|talk]]) 06:04, 31 December 2024 (UTC) |
|||
== What's the best way to proceed once I have a draft I like? == |
|||
:@[[User:CrushedAsian255|CrushedAsian255]] This question is likely to have a better audience at [[WP:VPT]]. That board has a more technically oriented team than here. 🇺🇦 [[User:Timtrent|<span style="color:#800">Fiddle</span><sup><small>Timtrent</small></sup>]] [[User talk:Timtrent|<span style="color:#070">Faddle</span><sup><small>Talk to me</small></sup>]] 🇺🇦 09:26, 31 December 2024 (UTC) |
|||
I have a draft of a page of a surrealist artist that I have been heavily researching and gathering links on. It looks and feels like a real page, but I don't want to put it out too early and have it deleted, etc. Who should I send it to (and HOW) I have never attempted a page before. [[User:BillE.Vader1963|BillE.Vader1963]] ([[User talk:BillE.Vader1963|talk]]) 06:02, 20 January 2024 (UTC) |
|||
== Wider vs specific consensus == |
|||
:[[User:BillE.Vader1963|BillE.Vader1963]], [[Draft:Raymond A. Whyte]]? You could ask about that either here or at [[Wikipedia:WikiProject Articles for creation/Help desk]] (but please not both). -- [[User:Hoary|Hoary]] ([[User talk:Hoary|talk]]) 07:20, 20 January 2024 (UTC) |
|||
::thank you! good tip! [[User:BillE.Vader1963|BillE.Vader1963]] ([[User talk:BillE.Vader1963|talk]]) 08:39, 20 January 2024 (UTC) |
|||
:@[[User:BillE.Vader1963|BillE.Vader1963]]: Welcome to the Teahouse! I suggest using the Articles for creation process for your draft, and have added a template to your draft for you. When you're ready to have your draft reviewed, click the blue "Submit the draft for review" button. Good luck with the draft! [[User:GoingBatty|GoingBatty]] ([[User talk:GoingBatty|talk]]) 07:59, 20 January 2024 (UTC) |
|||
::Thank you so much! [[User:BillE.Vader1963|BillE.Vader1963]] ([[User talk:BillE.Vader1963|talk]]) 08:39, 20 January 2024 (UTC) |
|||
:{{u|BillE.Vader1963}}, should "mystical animals" be "mythical animals"? [[User:Maproom|Maproom]] ([[User talk:Maproom|talk]]) 08:35, 20 January 2024 (UTC) |
|||
::ooh. Good catch. Thanks! [[User:BillE.Vader1963|BillE.Vader1963]] ([[User talk:BillE.Vader1963|talk]]) 08:38, 20 January 2024 (UTC) |
|||
:::Once submitted to AfC there is a waiting period than can be as short as hours and as long as weeks - even months if the draft backlog gets huge. You can continue to work on a submitted draft. The reviewer will either accept, decline with reasons given, reject if opinion is no potential, or speedy delete (hopefully not!!) for major transgressions. [[User:David notMD|David notMD]] ([[User talk:David notMD|talk]]) 12:05, 20 January 2024 (UTC) |
|||
Can a case-specific consensus triumph a wider topic consensus already established? Not that consensus over wider topic is changed but maybe because that case is viewed from a different perspective. [[User:ExclusiveEditor|<span style="background:linear-gradient(to right,#FF7043,#FFCC80);color:#fff;padding:4px 8px;border-top-left-radius:6px;border-bottom-left-radius:6px;font-size:12px;font-weight:bold;box-shadow:0 2px 4px rgba(0,0,0,.2);text-transform:uppercase;">Exclusive</span><span style="background:linear-gradient(to right,#263238,#37474F);color:#fff;padding:4px 8px;border-top-right-radius:6px;border-bottom-right-radius:6px;font-size:12px;font-weight:bold;box-shadow:0 2px 4px rgba(0,0,0,.2);text-transform:uppercase;margin-left:-1px;">Editor</span>]] [[User talk:ExclusiveEditor|<sub style="font-size:11px;color:#757575;font-style:italic;letter-spacing:.5px;">🔔 Ping Me!</sub>]] 07:33, 31 December 2024 (UTC) |
|||
== Userbox alignment == |
|||
:@[[User:ExclusiveEditor|ExclusiveEditor]] I doubt it. Were that to be the case we would not have consistency of operation. This question is likely to have a better audience at [[WP:VPP]]. That board has a more policy oriented team than here 🇺🇦 [[User:Timtrent|<span style="color:#800">Fiddle</span><sup><small>Timtrent</small></sup>]] [[User talk:Timtrent|<span style="color:#070">Faddle</span><sup><small>Talk to me</small></sup>]] 🇺🇦 09:28, 31 December 2024 (UTC) |
|||
Hi. I added some userboxes to [[User:CanonNi|my user page]] but no matter what parameter I use, they won't align. Thanks. [[User:CanonNi|CanonNi]] ([[User talk:CanonNi|talk]]) 07:50, 20 January 2024 (UTC) |
|||
:Hello, @[[User:ExclusiveEditor|ExclusiveEditor]], and welcome to the Teahouse. While I agree with FiddleFaddle's advice, I would also remark that general questions like yours are very frustrating for people who attempt to answer questions here. If you explain the specific issue you want guidance on, you are much more likely to get a useful answer (and also be less likely to be suspected of [[WP:wikilawyering|wikilawyering]]). |
|||
:I am aware of the possible irony in my answer, given your question. [[User:ColinFine|ColinFine]] ([[User talk:ColinFine|talk]]) 10:43, 31 December 2024 (UTC) |
|||
::Thanks for replying! Being a half-host on Teahouse myself I get you. I was inspired to ask this question by the discussion happening [[Wikipedia:In_the_news/Candidates#(Posted_blurb)_RD/Blurb_Jimmy_Carter|here]]. The latter part of the discussion specifically focuses on if 'Nobel prize' should be added or not in Jimmy Carter's death blurb on Main page's ITN section. It was initially proposed by the nominator and many supported it (albeit not mentioning specifically the Nobel prize) and it got posted without the mention of the prize. However later there was some more scrambling, this time with more regard to the Nobel prize and so it currently updated to include the prize in the blurb. The opposers are generally arguing that it is editorializing and other things. I may not be very good in summarizing discussions, so I left it in the question. --{{User:ExclusiveEditor/Signature}} 10:58, 31 December 2024 (UTC) |
|||
== How to upload a >100 MB file to Wikipedia? == |
|||
:@[[User:CanonNi|CanonNi]]: Welcome to the Teahouse! I added {{tl|userboxtop}} and {{tl|userboxbottom}} to your user page, which helped a bit, but didn't completely solve the issue. [[User:GoingBatty|GoingBatty]] ([[User talk:GoingBatty|talk]]) 08:03, 20 January 2024 (UTC) |
|||
::'''Thank you''' but @[[User:Sirdog|Sirdog]] has provided a better solution. [[User:CanonNi|CanonNi]] ([[User talk:CanonNi|talk]]) 08:15, 20 January 2024 (UTC) |
|||
:{{u|CanonNi}}, it appears that {{tl|Babel}} is the issue. I think it's because it's literally just a raw module and the module may be conflicting with {{tl|Infobox Wikipedia user}}. There is a <code>languages</code> parameter for {{tl|Infobox Wikipedia user}} you can make use of, or you could use the various [[:Category:Wikipedians by language|specific language templates]] (example: {{tl|User en}}). Perhaps someone with more technical knowledge may be able to assist you in getting it to work if those solutions aren't what you are looking for. Cheers! —[[User:Sirdog|<span style="color:#058700">'''Sirdog'''</span> ]]([[User talk:Sirdog|talk]]) 08:05, 20 January 2024 (UTC) |
|||
::@[[User:Sirdog|Sirdog]] '''Thank you!''' Used the specific language templates in |languages instead of Babel and that fixed the issue. |
|||
::Quick question, what does ''raw module'' mean? Does it just mean a module is badly coded? [[User:CanonNi|CanonNi]] ([[User talk:CanonNi|talk]]) 08:14, 20 January 2024 (UTC) |
|||
:::Badly coded? I'm certainly not qualified to make that assessment! I just mean that if you look at the source for [[Template:Babel]] all it does is invoke a module, rather than having wikitext with perhaps modules invoked inside of that. {{smiley}} —[[User:Sirdog|<span style="color:#058700">'''Sirdog'''</span> ]]([[User talk:Sirdog|talk]]) 08:17, 20 January 2024 (UTC) |
|||
::::I understand. Thank you! [[User:CanonNi|CanonNi]] ([[User talk:CanonNi|talk]]) 08:18, 20 January 2024 (UTC) |
|||
I've uploaded plenty of files to Commons before, but I've only uploaded a few (non-free) files to WP. I would like to upload a short film that will become public domain in the US at the start of 2025 (won't be PD in its country of origin for a few years, so no uploading it to commons), but the file is over 100 MB, the maximum file size listed on the [[Special:Upload|upload page]]. I don't want to compress it any more than it already is, so how to I get around this? I've seen several films large than 100 MB on WP already ([[:File:The Adventures of Prince Achmed (1926).webm|1]], [[:File:The Cabinet of Dr. Caligari (1920).webm|2]]), so it must be possible. |
|||
== link on my name == |
|||
Any help with this is greatly appreciated. Thanks. — [[User:Toast for Teddy|Toast for Teddy]] ([[User talk:Toast for Teddy|talk]]) 07:48, 31 December 2024 (UTC) |
|||
I would like to add a link to my name , mentioned at So you think you can dance info. |
|||
Thanks a lot! |
|||
Miriam Larici [[Special:Contributions/47.157.5.19|47.157.5.19]] ([[User talk:47.157.5.19|talk]]) 08:39, 20 January 2024 (UTC) |
|||
:@[[User:Toast for Teddy|Toast for Teddy]] This question is likely to have a better audience at [[WP:VPT]]. That board has a more technically oriented team than here 🇺🇦 [[User:Timtrent|<span style="color:#800">Fiddle</span><sup><small>Timtrent</small></sup>]] [[User talk:Timtrent|<span style="color:#070">Faddle</span><sup><small>Talk to me</small></sup>]] 🇺🇦 09:26, 31 December 2024 (UTC) |
|||
:{{courtesy link|WP:Help desk#link on my name}}; and referring to [[So You Think You Can Dance]] |
|||
::@[[User:Toast for Teddy|Toast for Teddy]] That first file you linked was uploaded with the aid of a userscript discussed at [[:Commons:User talk:Rillke/bigChunkedUpload.js]]. I don't pretend to understand the details, but you may ;-) [[User:Michael D. Turnbull|Mike Turnbull]] ([[User talk:Michael D. Turnbull|talk]]) 14:27, 31 December 2024 (UTC) |
|||
:hi ip user! 331dot has already answered your question in the Help Desk. for next time, please don't post your question to both the Teahouse and Help Desk to help keep responses to your question in one central place (and besides, most people answering questions in one probably also answer questions in the other as well). happy editing! 💜 <span style="border-radius:4px;background:#edf"> [[User:Melecie|<span style="color:#471a7a">'''mel'''ecie</span>]] </span> [[User talk:Melecie|<span style="color:#471a7a">talk</span>]] - 09:00, 20 January 2024 (UTC) |
|||
:Even if you're uploading "locally", [[commons:Commons:Upload tools|Commons:Upload tools]] is likely helpful. All the projects use the same [[MediaWiki]] software, all that needs adjustment is the destination for the upload. --[[User:Slowking Man|Slowking Man]] ([[User talk:Slowking Man|talk]]) 20:54, 31 December 2024 (UTC) |
|||
== Editing a sandbox, references number is doubling == |
|||
== Am I doing anything wrong? == |
|||
Hi there! I was editing my [https://en.wikipedia.org/enwiki/w/index.php?title=User:Andrea_Biographer/sandbox&action=submit#Gridspertise sandbox] and for some reasons all the different references I am adding are doubling the number of them and the previous ones are not disappeared. I was following your suggestions to add COI edit to my text, eliminating the internal sources and the bold words. Can you help me or it is just a matter of viewing and once I publish the sand box they will all disappear? [[User:Andrea Biographer|Andrea Biographer]] ([[User talk:Andrea Biographer|talk]]) 09:10, 31 December 2024 (UTC) |
|||
I have recently being accused of being biased for reverting the unsourced ccontent additions. I am trying to be very gentle and let the other person uderstand why I reverted their information. But since now I lack the Wikipedia norms handy I started doubting myself. Should I mend my ways or what all I am doing is right? Should I be referring any extra guideliness or am I lacking any Wikipedia rules? I am unsure and please help me out in assessing my activity if it's going fine or not. Thank you |
|||
:Hi @[[User:Andrea Biographer|Andrea Biographer]]: it seems that with [https://en.wikipedia.org/enwiki/w/index.php?title=User:Andrea_Biographer/sandbox&diff=prev&oldid=1266218280 this edit] you've duplicated the contents by pasting an earlier edit into the page, thus embedding a copy of the entire page within the page (if that makes sense). You should undo your most recent edits up to and including that one. Or if you'd like me to do it, let me know. -- [[User:DoubleGrazing|DoubleGrazing]] ([[User talk:DoubleGrazing|talk]]) 09:16, 31 December 2024 (UTC) |
|||
Please refer to [[Talk:N. T. Rama Rao#Reply to ip user 136.54.56.86]] and [[Talk:2024 Andhra Pradesh Legislative Assembly election#Latest edit - JSP infobox]]. [[User:456legend|<span style="background:#ADD8E6; color:black; padding:5px; box-shadow:darkgray 2px 2px 2px;">456legend</span>]][[User talk:456legend|<span style="background:#FF474C;color:black; padding:2px; box-shadow:darkgray 2px 2px 2px;">talk</span>]] 09:43, 20 January 2024 (UTC) |
|||
::Hi @[[User:DoubleGrazing|DoubleGrazing]], thanks for your prompt answer! I think you made sense, could you check and fix this for me? I don't want to commit any further mistakes in the editing process or in the COI different templates, thanks in advance! Andrea [[User:Andrea Biographer|Andrea Biographer]] ([[User talk:Andrea Biographer|talk]]) 09:31, 31 December 2024 (UTC) |
|||
:::User:Theroadislong has already sorted this out. Also, your draft is now located at [[Draft:Gridspertise]]. -- [[User:DoubleGrazing|DoubleGrazing]] ([[User talk:DoubleGrazing|talk]]) 10:30, 31 December 2024 (UTC) |
|||
== how to give feedback == |
|||
:Althought people who accused the same later clarified that they misundertood but still I need a through check on what I am lacking. Can I get more educated before I continue editing here? [[User:456legend|<span style="background:#ADD8E6; color:black; padding:5px; box-shadow:darkgray 2px 2px 2px;">456legend</span>]][[User talk:456legend|<span style="background:#FF474C;color:black; padding:2px; box-shadow:darkgray 2px 2px 2px;">talk</span>]] 09:44, 20 January 2024 (UTC) |
|||
:Welcome to the Teahouse, {{u|456legend}}! I'm not seeing anything egregious. I see an innocent interaction at the 1st link and a good ole' civil content dispute at the 2nd (at time of writing). Noting that I have not checked any page history, just read the talk pages. My one comment would be that "{{tq|First thing you must understand that any content addition that is contested shall be removed at the first place and will be discussed whether to add it or not}}" is not strictly true. Aside from fairly obvious exceptions, such as enforcing [[WP:BLP]]/[[WP:V]] policies, removing [[WP:COPYVIO]], or fighting [[WP:VANDALISM]], there is no policy I'm aware of that demands an article be kept at a certain version until discussed. Regardless of the article state, it's better that once 2 or more editors are aware others disagree that no one continues editing that part until a discussion happens. What you said tends to occur in practice, however, as a reversion of an edit tends to be the way editors learn there is disagreement (see: [[WP:BRD]]). Cheers! —[[User:Sirdog|<span style="color:#058700">'''Sirdog'''</span> ]]([[User talk:Sirdog|talk]]) 10:20, 20 January 2024 (UTC) |
|||
::@[[User:Sirdog|Sirdog]] I get that. I should have refrained from removing the addition at the first and it was my fault that I assumed it to be that way and informed the same. Other than that I think I am fine with the rest of the discussion involving the content discussion. Meanwhile, I found the same user to add such similar addition on a different article and I assumed it to be a affliation to a organisation on his part and safely requested him to disclose any such if true. The user got offended and accused me of stalking and challenging my behaviour, thus I decided to recuse myself from the article. I don't intend to complaint anything against the user now but would like to know if looking upon other user contributions here on wikipedia not acceptable and is assuming someone to be paid for editing and asking the same to them to diclose not accepted? [https://en.wikipedia.org/enwiki/w/index.php?title=Talk%3A2024_Andhra_Pradesh_Legislative_Assembly_election&diff=1197402387&oldid=1197402255 here] And since I have recused from the article, do I need to bother about anything else on that particular talk page and do I need to further get involved in the discussion there if the other users show up? Please guide about this too. [[User:456legend|<span style="background:#ADD8E6; color:black; padding:5px; box-shadow:darkgray 2px 2px 2px;">456legend</span>]][[User talk:456legend|<span style="background:#FF474C;color:black; padding:2px; box-shadow:darkgray 2px 2px 2px;">talk</span>]] 10:40, 20 January 2024 (UTC) |
|||
:::It is not uncommon for editors who see what may be a flaw in an editor's edit in one article - for example updating a fact on a place or person without providing a reference - to see if the same flaw was committed at other articles. This may also happen when an editor posts a question here at Teahouse. To the recipient, can it feel like stalking? Yes. Edit summaries at the articles should be about correcting the errors. Separately, advice can be posted to the editor's Talk page. And absolutely, asking an editor if a COI or PAID situation exists is valid, and very common if there is reason to suspect such. [[User:David notMD|David notMD]] ([[User talk:David notMD|talk]]) 12:19, 20 January 2024 (UTC) |
|||
::::@[[User:David notMD|David notMD]] Thank you for the detailed analysis. Also thank you @[[User:Sirdog|Sirdog]] for the information. [[User:456legend|<span style="background:#ADD8E6; color:black; padding:5px; box-shadow:darkgray 2px 2px 2px;">456legend</span>]][[User talk:456legend|<span style="background:#FF474C;color:black; padding:2px; box-shadow:darkgray 2px 2px 2px;">talk</span>]] 13:08, 20 January 2024 (UTC) |
|||
i had a question on why an article was written and sent it to info@ wikipedia. they told me that I needed to engage in the "talk" feature and ask the editors, since wikipedia is only a platform. I did that and not only was the answer not given I was ridiculed, because I am not actually sure why. I am not interested in editing wikipedia, I am only interested in engaging with the editors, to understand inconsistencies. How exactly would I do that, if I should not be using the talk feature. |
|||
== Image preview on hover == |
|||
thanks [[User:Mommer264|Mommer264]] ([[User talk:Mommer264|talk]]) 12:34, 31 December 2024 (UTC) |
|||
:Hello. Discussion about an article usually takes place on its talk page. Discussion with an editor directly can occur on their user talk page. |
|||
When I hover over link to some articles, infobox image is shown along with lede text. But in some article hovers image is not shown, even when infobox has an image ([[Andrea Kevichüsa|this]] and [[Mahima Makwana|this]]). Why? How to make sure that image is visible in preview. Also, what should be ratios of infobox image which make sure that images are not cutoff in preview (cut from top like [[Ananya Panday|this]]). -- [[User:Parnaval|Parnaval]] ([[User talk:Parnaval|talk]]) 12:43, 20 January 2024 (UTC) |
|||
:Note that your only other edit was about the Israeli-Arab/Palestinian conflict, which is [[WP:CTOP|a topic area with special rules]] that I will notify you of on your user talk page. One of those is that you must be an experienced user in order to make edits to any type of page about it. Your account must be 30 days old with 500 edits. [[User:331dot|331dot]] ([[User talk:331dot|talk]]) 12:53, 31 December 2024 (UTC) |
|||
:Hi [[User:Parnaval|Parnaval]]. The preview feature is [[:mw:Extension:Popups]]. At [[:mw:Topic:X24ym9nooumpgr1h]] I wrote: |
|||
::Also answering your query: :You asked a question that is now at [[Talk:State of Palestine/Archive 21]]. Talk pages of articles are not for 'open' questions. Instead, the proper method is to propose a specific change of text, as in replace A with B, and include a reference to support your proposal. Given that [[State of Palestine]] is a very controversial article, editors who participate there - at both article and talk - can be short on tact. [[User:David notMD|David notMD]] ([[User talk:David notMD|talk]]) 12:54, 31 December 2024 (UTC) |
|||
::I suspect the actual rule is simply something like this: |
|||
:{{tpq|I am not interested in editing wikipedia}} If you're not interested in contributing to the project, then you probably ought to go elsewhere. [[WP:PURPOSE|The purpose of Wikipedia]] is to produce an encyclopedia; notably, [[WP:TP|talk pages]] are for constructively discussing the project and its content, not general [[Internet forums]] for discussion. There are many many discussion forums elsewhere on the Internet, and Wikipedia can even [[:Category:Internet forums|help direct you to some of them]]. If you do wish to contribute to the encyclopedia, take a look at [[WP:Welcome]]. Thank you and I hope you have a good day. --[[User:Slowking Man|Slowking Man]] ([[User talk:Slowking Man|talk]]) 21:03, 31 December 2024 (UTC) |
|||
::width × height must be either at least 320 × 200 px or 203 × 250 px. |
|||
:The images in your examples [[:File:Andrea-Kevichusa-BH (cropped).jpg]] and [[:File:Mahima Makwana snapped at an airport (cropped).jpg]] are too small. We don't pick images based on how they are currently processed in previews. [[User:PrimeHunter|PrimeHunter]] ([[User talk:PrimeHunter|talk]]) 23:38, 20 January 2024 (UTC) |
|||
::Thank you [[User:PrimeHunter|PrimeHunter]]. I prefer that when I hover over links, then image is visible. So I would try to add portraits with min 210 x 250px. One more question. Why on hovering over [[Manushi Chhillar]] show her signature, and not the portrait image? Portrait is the first image in infobox, and clearly larger than min required size -- [[User:Parnaval|Parnaval]] ([[User talk:Parnaval|talk]]) 15:34, 22 January 2024 (UTC) |
|||
:::{{ping|Parnaval}} The preview feature can only display an image if it has been selected as the page image by another feature [[:mw:Extension:PageImages]]. A page can only have one page image and many pages have none. [[:mw:Extension:PageImages#How are images scored?]] says: |
|||
:::* The ratio of the image's width to height is considered <code>$wgPageImagesScores['ratio']</code> |
|||
:::** On Wikimedia wikis a ratio of 0.4 to 3.1 is allowed, with 0.6 to 2.1 preferred. |
|||
:::[[:File:A Presidente do FUSSESP Lucia França recebe a Miss Mundo, Manushi Chhilar (41430422792) (cropped).jpg]] has ratio 445/780 = 0.57, so it's allowed but not preferred. The signature is in the preferred interval. [[User:PrimeHunter|PrimeHunter]] ([[User talk:PrimeHunter|talk]]) 17:23, 22 January 2024 (UTC) |
|||
::::I have disallowed the signature.[https://en.wikipedia.org/enwiki/w/index.php?title=Manushi_Chhillar&diff=prev&oldid=1197983548] The portrait is now the page image and displayed in the preview. [[User:PrimeHunter|PrimeHunter]] ([[User talk:PrimeHunter|talk]]) 17:35, 22 January 2024 (UTC) |
|||
:::::Ok understood. Thank You! -- [[User:Parnaval|Parnaval]] ([[User talk:Parnaval|talk]]) 06:19, 23 January 2024 (UTC) |
|||
== VisualEditor == |
|||
== How do I correctly start a GA Reassesment? == |
|||
I've just changed my Preferences > Editing |
|||
I know sometimes but sometimes don't [[User:GabrielPenn4223|GabrielPenn4223]] ([[User talk:GabrielPenn4223|talk]]) 14:28, 20 January 2024 (UTC) |
|||
:See [[Wikipedia:Good article reassessment]] (with four Ss). [[User:Maproom|Maproom]] ([[User talk:Maproom|talk]]) 15:31, 20 January 2024 (UTC) |
|||
Turned on "Enable the visual editor" <small>(which, I think, it's on by default in Wikipedia in Italian language)</small> |
|||
== Disambiguation pages allowed if no article? == |
|||
But i can not find out how to edit with VisualEditor. |
|||
Idk how to put the title, but I will try my best to explain it. |
|||
I've looked at [[Help:VisualEditor]] and it says ([[Help:VisualEditor#Opening VisualEditor]] to click on the "Edit" tab (in the picture I can see a "drop down" menu to choose the editor). |
|||
Basically, can disambiguation (or dab) pages display something if no article exists. For example if you look at the dab page for [[Cross country]], there is a section that does not have an article. |
|||
Copied from the article above |
|||
* ''Cross Country'' (film), a 1983 drama film starring [[Nina Axelrod]] |
|||
:I've removed a couple of items from [[Cross country]], including that one. |
|||
But I've not this choice. My tab is named "Edit source". [[User:Centrodiurnomilano|Centrodiurnomilano]] ([[User talk:Centrodiurnomilano|talk]]) 14:47, 31 December 2024 (UTC) |
|||
:My guess is that it's to disambiguate. There's an article on [[Great Western Railway]] as well as a dab page, but for [[Cross country]] there's only a dab page. [[User:Maproom|Maproom]] ([[User talk:Maproom|talk]]) 15:28, 20 January 2024 (UTC) |
|||
::Basically it is unclear whether to include something in a dab page if it does not have an article, like the example above. Even the [[WP:DAB]] article is unclear or I just couldnt find it, although it does have a 'what not to include' section but not the inverse? |
|||
::And so that explains why GWR does and does not have the dab suffix? I mean we also have [[CrossCountry]], a UK railway company without any additional suffixes, although the only difference is that it is spelt as one word. [[User:Breathinkeeps32|Breathinkeeps32]] ([[User talk:Breathinkeeps32|talk]]) 15:41, 20 January 2024 (UTC) |
|||
Edit 16:53 UTC - see also and copies from: [[MOS:DABMENTION]]. Idk how to use green text below so im using quotes instead |
|||
* "If a topic does not have an article of its own, but is discussed within another article, then a link to that article may be included if it would provide value to the reader". [[User:Breathinkeeps32|Breathinkeeps32]] ([[User talk:Breathinkeeps32|talk]]) 16:59, 20 January 2024 (UTC) |
|||
:::It seems quite clear to me. [[Wikipedia:Disambiguation dos and don'ts]] says {{tq|Don't include entries without a blue link.}} and {{tq|Don't include red links unless used in articles}}. [[User:Shantavira|Shantavira]]|[[User talk:Shantavira|<sup>feed me</sup>]] 16:22, 20 January 2024 (UTC) |
|||
:::If there is a [[WP:primary topic|primary topic]] for the name, then that article gets the bare title, and a DAB page is labelled as such in its title. If there is no primary topic, then the DAB page gets the bare title. [[User:ColinFine|ColinFine]] ([[User talk:ColinFine|talk]]) 16:42, 20 January 2024 (UTC) |
|||
:I've found out, reading [[Help:VisualEditor#First step: enabling VE]] (which seems a bit outdated, by the way). |
|||
== Article submission declined - help, don't know how to fix it :( == |
|||
:After I've changed my Preferences > Editing > "Enable the visual editor" = ON |
|||
:and the press "save" button at the bottom. |
|||
:Only then, a "drop down menu" show up in the same section: "Editing mode", which is setted by default "Remember the last editor". I've changed it to "Show both editor tabs". |
|||
:It's not easy to find out. [[User:Centrodiurnomilano|Centrodiurnomilano]] ([[User talk:Centrodiurnomilano|talk]]) 15:04, 31 December 2024 (UTC) |
|||
::By the way, I've looked at the settings on Wikipedia in Italian language and "Enable the visual editor" it's enough there (there is no "Editing mode" option at all) [[User:Centrodiurnomilano|Centrodiurnomilano]] ([[User talk:Centrodiurnomilano|talk]]) 15:08, 31 December 2024 (UTC) |
|||
:Italian Wikipedia and English Wikipedia run on the same system, MediaWiki, but are two different Wikipedias. So if this is a question about itwiki, then you should go to a help board on itwiki. If this a question about enwiki, you should be able to switch between the source editor and visual editor by pressing the pencil button next to the preview button in the source editor. <span style="font-family:Arial;background-color:#fff;border:2px dashed#69c73e">[[User:Cowboygilbert|<span style="color:#3f6b39">'''Cowboygilbert'''</span>]] - [[User talk:Cowboygilbert|<span style="color:#d12667"> (talk) ♥</span>]]</span> 16:07, 31 December 2024 (UTC) |
|||
::But, @[[User:Centrodiurnomilano|Centrodiurnomilano]], do note that you can only use the VisualEditor is some namespaces but not all. You can in mainspace, userspace, draftspace but not in wikispace which is the space with the Wikipedia: prefix. <span style="font-family:Arial;background-color:#fff;border:2px dashed#69c73e">[[User:Cowboygilbert|<span style="color:#3f6b39">'''Cowboygilbert'''</span>]] - [[User talk:Cowboygilbert|<span style="color:#d12667"> (talk) ♥</span>]]</span> 16:09, 31 December 2024 (UTC) |
|||
:::Thank you, @[[User:Cowboygilbert|Cowboygilbert]] . I've tried in main space and the in user space. [[User:Centrodiurnomilano|Centrodiurnomilano]] ([[User talk:Centrodiurnomilano|talk]]) 16:29, 31 December 2024 (UTC) |
|||
::::Were you able to fix it using the instructions that I put, {{tq|you should be able to switch between the source editor and visual editor by pressing the pencil button next to the preview button in the source editor.}}? <span style="font-family:Arial;background-color:#fff;border:2px dashed#69c73e">[[User:Cowboygilbert|<span style="color:#3f6b39">'''Cowboygilbert'''</span>]] - [[User talk:Cowboygilbert|<span style="color:#d12667"> (talk) ♥</span>]]</span> 16:30, 31 December 2024 (UTC) |
|||
== "No Such Number tone" / "crybaby tone" (type of intercept error) == |
|||
[[Draft:Together UK Foundation]] |
|||
Hello everyone. This is one of my first times writing a brand-new article, and I'm not sure if this is a notable enough topic for its own page or a subsection in the intercept message. |
|||
Hi I'd appreciate someone's help please in showing me how to fix the issues a reviewer outlined in my article submission. |
|||
This technically isn't a message, but before the intercept message was used, a different type of tone could also be returned instead of connecting to an operator. This is known under many different names, such as the "no such number tone" or even the "crybaby tone." This tone would be returned if a caller attempts to dial a number that can't possibly exist according to the numbering plan. The call would not be allowed to go through because some of these numbers could be reserved for private use. The tone itself is continuous, sweeping from 200-400Hz back to 200Hz again over the course of one second. |
|||
I don't feel they're fair considering The News Letter (oldest newspaper in English speaking world) and The Irish News are both well-respected news papers. Second, this organisation's opponent (Ireland's Future) has a wiki page, so it's unbalanced if TUKF does not also have one. [[User:TheHandsomeHistorian|TheHandsomeHistorian]] ([[User talk:TheHandsomeHistorian|talk]]) 16:06, 20 January 2024 (UTC) |
|||
The issue is that it's almost exclusively used in North America, if not, completely exclusively. It was introduced by the Bell System in 1941, but I've also heard of it in use by the 3CX Asterisk system. However, my intent is to preserve information about this tone, as next to no information seems to exist about it. An excerpt from the Bell Labs Record describes it here. It also seems to be exclusive to crossbar systems. |
|||
:Hello, TheHandsomeHistorial, and welcome to the Teahouse. There are three separate requirements on a source in order to count towards notability: reliability is the most obvious one, but independence, and significant coverage, are also important: see [[WP:golden rule|golden rule]]. |
|||
:{{HD/WINI}} |
|||
:Looking over your reference list, almost all of them are clearly not independent; the Irish News might be, but does it contain [[WP:significant coverage|significant coverage]] of the Foundation? (It might do, but I haven't looked). If it does, it will be one acceptable source, but we need more than one. |
|||
:As for Ireland's Future: see [[WP:OTHERSTUFFEXISTS]]. It is possible that the article [[Ireland's Future]] (which is not ''their'' article, by the way), is inadequately sourced: I haven't looked at it closely. If you think it is, you are welcome to improve it, or propose it for deletion if you think the organisation does not meet Wikipedia's criteria for [[WP:NORG|notability]]. [[User:ColinFine|ColinFine]] ([[User talk:ColinFine|talk]]) 16:55, 20 January 2024 (UTC) |
|||
[https://www.telephonecollectors.info/index.php/browse/document-repository/catalogs-manuals/bell-system-we/bell-labs-record/4432-41apr-blr-no-such-number-tone-for-dial-systems] Bell Labs record |
|||
== Value of the TeaHouse == |
|||
[https://www.nature.com/articles/148193b0] Article about this record |
|||
I just want to say thanks to the team running the Tea House. I have told my professor how prompt your responses are and how generous too. I was running into some editors who were…. not so much, and it was becoming discouraging to try to manage the steep learning curve. Sometimes I come here just to read. Thanks again, [[User:WikiTikiTavi63|WikiTikiTavi63]] ([[User talk:WikiTikiTavi63|talk]]) 16:22, 20 January 2024 (UTC) |
|||
[https://www.youtube.com/watch?v=Ipuruc9Ur_4] Sample of this sound |
|||
:Hi {{u|WikiTikiTavi63}}, you're welcome, glad you like it. [[User:Alextejthompson|''Alextejthompson'']] (''Ping me or leave a message on my [[User talk:Alextejthompson|talk page]]'') 18:34, 20 January 2024 (UTC) |
|||
: You're very welcome here! Thanks for the feedback; it's nice to hear. [[User:Mathglot|Mathglot]] ([[User talk:Mathglot|talk]]) 11:11, 21 January 2024 (UTC) |
|||
What do you think of this? [[User:ZetaformGames|ZetaformGames]] ([[User talk:ZetaformGames|talk]]) 19:01, 31 December 2024 (UTC) |
|||
== nurulhuda.kssg == |
|||
:Hi @[[User:ZetaformGames|ZetaformGames]], welcome to the teahouse and welcome to Wikipedia! I think that this could be a good idea as a subsection in the article for [[Intercept message]] or some other related article. I don't think that there is enough for a full article with the sources you've posted but could definitely be an interesting paragraph! [[User:Justiyaya|'''<span style="color:#1d556d">Just</span>''']][[Special:Contributions/Justiyaya|'''<span style="color:#000000">i</span>''']][[User talk:Justiyaya#top|'''<span style="color:#6d351d">yaya</span>''']] 07:12, 1 January 2025 (UTC) |
|||
help me please recover my Facebook account [[Special:Contributions/176.16.231.187|176.16.231.187]] ([[User talk:176.16.231.187|talk]]) 17:25, 20 January 2024 (UTC) |
|||
: |
::Alright, thank you! And thanks for the welcome. I made this account a while ago, but haven't felt confident enough in my editing skills until now to contribute. [[User:ZetaformGames|ZetaformGames]] ([[User talk:ZetaformGames|talk]]) 17:46, 1 January 2025 (UTC) |
||
== Maryam Mirzakhani == |
|||
== feedback on draft f0r musical artist == |
|||
Can someone fix the error in the [[Maryam Mirzakhani]] article? |
|||
Hi all - happy new year! I've re-edited a draft wiki page for a musical band and would be grateful if an editor could let me know if it now qualifies for publication. https://en.wikipedia.org/wiki/Draft:Say_She_She_(band) |
|||
Mirzakhani solved this counting problem by relating it to the problem of computing volumes in moduli space—a space whose points correspond to different complex structures on a surface genus '''<span style="color:red">Failed to parse (SVG (MathML can be enabled via browser plugin): Invalid response ("Math extension cannot connect to Restbase.") from server "http://localhost:6011/en.wikipedia.org/v1/":): {\displaystyle g}</span> .''' In her thesis, Mirzakhani found a volume formula for the moduli space of bordered Riemann surfaces of genus g {\displaystyle g} with n {\displaystyle n} geodesic boundary components. From this formula followed the counting for simple closed geodesics mentioned above, as well as a number of other results. This led her to obtain a new proof for the formula discovered by Edward Witten and Maxim Kontsevich on the intersection numbers of tautological classes on moduli space. |
|||
Thanks! |
|||
georges_mille [[User:Georges1K|Georges1K]] ([[User talk:Georges1K|talk]]) 17:46, 20 January 2024 (UTC) |
|||
Thanks. [[Special:Contributions/76.14.122.5|76.14.122.5]] ([[User talk:76.14.122.5|talk]]) 19:28, 31 December 2024 (UTC) |
|||
:@[[User:Georges1K|Georges1K]]: Welcome to the Teahouse! I made some tweaks and restored the {{tl|AFC submission}} templates (the ones with comments stating "Do not remove this line!") I see you added some references since the last review. When you're ready for the draft to be reviewed again, click the blue "Resubmit" button. Good luck with your draft! [[User:GoingBatty|GoingBatty]] ([[User talk:GoingBatty|talk]]) 18:34, 20 January 2024 (UTC) |
|||
:<s>Hi IP 76.14.122.5. Generally, the best place to discuss something like this is on the talk page for the article in question, which in this case would be [[:Talk:Maryam Mirzakhani]]; however, if you truly believe there's an error in the article, you can be [[:WP:BOLD]] and fix it yourself if you think you can. Please understand though that "fix it" in this context means to correct the article in accordance with relevant Wikipedia policies and guidelines, and the policies/guidelines most likely applicable in this case are going to be [[:Wikipedia:Verifiability]] and [[:Wikipedia:No original research]]. Ideally, you're going to need to find [[:WP:SECONDARY]] [[:WP:RS|reliable sources]] (as defined by Wikipedia) to cite in support of the change you want to make; [[:WP:VNT|even if you know such changes to be true]], you're still going to need to cite reliable sources in support to allow others to verify the changes. If you just make a change without providing any citations to a reliable source in support, there's a good chance the change will be undone by another user. Given that this seems to be related to mathematics, you might argue that "proving" the information to be incorrect based on Mirakhani's academic thesis is more than sufficient in and of itself, but Wikipedia generally requires something more and a thesis is going to be, for the most part, considered a [[:WP:PRIMARY]] source and could have other issues as explained in [[:WP:SCHOLARSHIP]]. You could also try asking about this at [[:Wikipedia:WikiProject Mathematics]] since that's where you're going to have a better chance of finding someone sufficiently versed in mathematics who might be able to help sort this out. -- [[User:Marchjuly|Marchjuly]] ([[User talk:Marchjuly|talk]]) 19:52, 31 December 2024 (UTC)</s>;<ins>post edited. -- 22:29, 31 December 2024 (UTC)</ins> |
|||
::Please disregard my reply IP 76.14.122.5. I misunderstood what you were asking about. ColinFine's suggestion below seems to be the best course of action here. -- [[User:Marchjuly|Marchjuly]] ([[User talk:Marchjuly|talk]]) 22:29, 31 December 2024 (UTC) |
|||
:Hello, IP user. What you are reporting is a problem in the software, the network, or the user interface, and not in the content of the article. Generally, technical problems of this sort are better handled at [[WP:VPT]] than here. However, I'm not seeing that problem, either on the browser on my laptop, or on the Android app. Is it repeatable, or might it have been a temporary glitch? |
|||
:Actually, now I look at it, the URL above appears to be a local proxy, so it may be that whoever manages your local network has not configured the proxy in a way that Wikipedia requires. Again, [[WP:VPT]] is a better place to ask about this. |
|||
:@[[User:Marchjuly|Marchjuly]]. The problem that the IP is reporting is obviously a technical one, so your answer is entirely off the point. [[User:ColinFine|ColinFine]] ([[User talk:ColinFine|talk]]) 20:21, 31 December 2024 (UTC) |
|||
::Thanks for catching my mistake and pointing the OP in the right direction {{u|ColinFine}}. I've stricken my reply so as to not confuse the OP or anyone else. -- [[User:Marchjuly|Marchjuly]] ([[User talk:Marchjuly|talk]]) 22:29, 31 December 2024 (UTC) |
|||
== Stuck on Puerto Rico outages notability. == |
|||
== Posting articles written on linkedin == |
|||
Hi, new to Wikipedia here and probably gonna create a draft on the Puerto Rico power outage crisis but wanted to ask: |
|||
i am a seasoned digital marketer of global repute, written many articles on LinkedIn etc, i dont want to promote myself on Wikipedia, but want to articles written already on linked by me only. It will definitely benefit users as these are related to field of Digital Marketing and SEO and solves many issues of users. [[User:Anoopsrivastava784|Anoopsrivastava784]] ([[User talk:Anoopsrivastava784|talk]]) 18:31, 20 January 2024 (UTC) |
|||
I’m stuck in the notability of this topic, so, is the Puerto Rico power outage crisis notable enough for Wikipedia? By power outage crisis I mean the beginning of the Puerto Rican outages from [[Hurricane Maria]] to [https://amp.cnn.com/cnn/2024/12/31/us/puerto-rico-power-outage now] since it has lasted multiple years with sustained media coverage when an outage does occur. |
|||
Cheers! [[Special:Contributions/66.50.50.222|66.50.50.222]] ([[User talk:66.50.50.222|talk]]) 20:29, 31 December 2024 (UTC) |
|||
:Rather than a new article, you could expand on the outages already documented in [[:Puerto Rico Electric Power Authority]] and in [[:LUMA Energy]]. [[User:Schazjmd|<span style="color:#066293;">'''Schazjmd'''</span>]] [[User talk:Schazjmd|<span style="color:#738276;">''(talk)''</span>]] 20:44, 31 December 2024 (UTC) |
|||
::A search within Wikipedia on "Puerto Rico power outage" yields a list of several articles, including [[:Puerto Rico Electric Power Authority]] and in [[:LUMA Energy]] [[User:David notMD|David notMD]] ([[User talk:David notMD|talk]]) 21:15, 31 December 2024 (UTC) |
|||
::Noted. Will expand those articles then instead. Thanks! [[Special:Contributions/66.50.50.222|66.50.50.222]] ([[User talk:66.50.50.222|talk]]) 21:40, 31 December 2024 (UTC) |
|||
:My first impression would be that an article is likely warranted. PR is an island of ~3 million people, in the same neighborhood as Los Angeles. If Los Angeles were having sustained multi-year-long Issues with electrical service, there would be an article about it. [[WP:NONENG|Non-English language sources are perfectly acceptable]] for citing in articles, as long as a little care is observed. |
|||
:Note, to the anonymous editor: if you [[WP:create an account|create an account]] you get your own shiny neato [[WP:userspace|userspace]] to use mostly at your leisure, where for instance you can work on draft articles with no hurry. I have one underway in mine in fact. And thank you again for being interested in contributing to Wikipedia! |
|||
:(Regarding LA: some may have had come to mind the [[California energy crisis]], but, neat fact, LA actually escaped impact from that because of having [[Los Angeles DWP|its own municipal utility]] with its own generation capacity!) --[[User:Slowking Man|Slowking Man]] ([[User talk:Slowking Man|talk]]) 01:38, 1 January 2025 (UTC) |
|||
::<small>Note to [[User:Slowking Man]]: The userspace is the wrong place to draft an article. Either use your Sandbox or else follow instructions at [[WP:YFA]] to create a draft. [[User:David notMD|David notMD]] ([[User talk:David notMD|talk]]) 03:33, 1 January 2025 (UTC)</small> |
|||
:::Is "userspace" not the colloquial term for "your user page and any and all subpages of it"? The "official" user sandbox link is [[Special:MyPage/sandbox]], which takes you to the /sandbox subpage of your user page. --[[User:Slowking Man|Slowking Man]] ([[User talk:Slowking Man|talk]]) 05:29, 1 January 2025 (UTC) |
|||
::Hello, the IP editor (66.xx) here, just created my account, and I mainly refrained from creating an article due to being worried about a potential conflict-of-interest (I live in Puerto Rico myself). I might work on a draft later today and collect sources (as @[[User:BusterD|BusterD]] suggested), thank you all btw for helping me clear up this question I’ve had for some time now! [[User:Atheions|Atheions]] ([[User talk:Atheions|talk]]) 04:11, 1 January 2025 (UTC) |
|||
:On the other hand, [[User:Slowking Man]] is correct that an article might be created on this newsworthy subject and that one shouldn't necessarily rely strictly on English-language sources. Looking at the existing material, it certainly seems a sequence of outages could be established from some of the reliable sources already applied to pagespace. I agree with [[User:David notMD]] that [[WP:YFA]] is a place to consider how to start a new page. I would start collecting sources, online and in print. [[User:BusterD|BusterD]] ([[User talk:BusterD|talk]]) 03:58, 1 January 2025 (UTC) |
|||
:Apologies if jumping the gun or creating a draft too early, but I have [https://en.m.wikipedia.org/wiki/Draft:2017-25_Puerto_Rico_power_outages created one]. Won’t be able to do much progress today but will def collect sources to use. [[User:Atheions|Atheions]] ([[User talk:Atheions|talk]]) 07:32, 1 January 2025 (UTC) |
|||
== Am I allowed to post something about a game I am making on Wikipedia? == |
|||
:@[[User:Anoopsrivastava784|Anoopsrivastava784]]: Welcome to the Teahouse! We already have articles on [[Digital marketing]] and [[Search engine optimization]]. You may improve those articles using published [[WP:RS|reliable sources]] that you didn't write, but adding links to your own work would be considered promotion, and [[WP:NOPROMO|Wikipedia is not the place to promote your work]]. [[User:GoingBatty|GoingBatty]] ([[User talk:GoingBatty|talk]]) 18:38, 20 January 2024 (UTC) |
|||
::If what you have posted at LinkedIn is copyright protected (I am not familiar with LI policy), then you cannot use that content in an existing or new Wikipedia article. And, as noted above, you may not use your LI articles as references. [[User:David notMD|David notMD]] ([[User talk:David notMD|talk]]) 21:27, 20 January 2024 (UTC) |
|||
:: And if it is not copyright protected, you still should not use it, due to [[WP:SPS]] and [[WP:COI|conflict of interest]] concerns. In brief, don't use it. [[User:Mathglot|Mathglot]] ([[User talk:Mathglot|talk]]) 11:13, 21 January 2024 (UTC) |
|||
Am i allowed to post somthing about a game i am making? On Wikipedia [[User:Aaronfart14|Aaronfart14]] ([[User talk:Aaronfart14|talk]]) 22:09, 31 December 2024 (UTC) |
|||
== creating a page == |
|||
:Allowed? Yes. Good idea? No. Likely to remain on Wikipedia? No. Writing an article is difficult, particularly for new editors. And since this is about '''your''' game, you have a conflict of interest in writing about it. See [[H:YFA]] and [[WP:COI]]. Simply posting information about your game rather than writing an article would be promotion. See [[WP:PROMOTION]] [[User:Meters|Meters]] ([[User talk:Meters|talk]]) 22:20, 31 December 2024 (UTC) |
|||
::{{u|Aaronfart14}}, the relevant content guideline is [[WP:Wikipedia is not for things made up one day]]. [[User:Cullen328|Cullen328]] ([[User talk:Cullen328|talk]]) 02:36, 1 January 2025 (UTC) |
|||
:::If your game goes public, and if it becomes so popular that people are publishing about it, then there is a chance that someone other than you will create an article about it. [[User:David notMD|David notMD]] ([[User talk:David notMD|talk]]) 03:37, 1 January 2025 (UTC) |
|||
== Writing about Childhood and Early Life == |
|||
Hello, I created a userpage yesterday and and watching tutorials on how to edit and build a page. My goal is to create a page for a notable musician. I understand that I need to have an account 4 days and do at least 10 edits before a page I create is considered for approval. |
|||
I am writing an article about Andrea Sheridan Ordin, who is already included in two existing wikipedia articles entitled, "List of first women lawyers and judges in California" and "United States District Court for the Central District of California." I am interviewing her personally and have reputable sources about her career notability, but I'm not sure how to write her "early life" section, since there are not many sources describing her childhood aside from her firsthand account. How to I write about her early life without secondary sources? [[User:Aharten97|Aharten97]] ([[User talk:Aharten97|talk]]) 22:17, 31 December 2024 (UTC) |
|||
I created a userpage in the sandbox as a test. Do I need to have a userpage approved before I create another page? [[User:Holyhootenany|Holyhootenany]] ([[User talk:Holyhootenany|talk]]) 18:36, 20 January 2024 (UTC) |
|||
:{{re|Aharten97}} It's not possible without reliable secondary published sources. For the policies, see [[WP:V]]. Personal notes from an interview are neither published nor reliable, and self-published statements are rarely reliable. If it's noteworthy it will have been published somewhere. If not then it isn't. -- [[user:zzuuzz|zzuuzz]] <sup>[[user_talk:zzuuzz|(talk)]]</sup> 22:45, 31 December 2024 (UTC) |
|||
::https://ethics.lacity.gov/news/murray-and-ordin-re-elected-as-ethics-commission-leaders/ provides some info for an Early life and education section. [[User:David notMD|David notMD]] ([[User talk:David notMD|talk]]) 04:24, 1 January 2025 (UTC) |
|||
== What to write on your talk page? == |
|||
:Hi {{u|Holyhootenany}}, you're free to continue working on the draft, you don't need permission to create a draft or drafts in userspace. [[User:Alextejthompson|''Alextejthompson'']] (''Ping me or leave a message on my [[User talk:Alextejthompson|talk page]]'') 18:40, 20 January 2024 (UTC) |
|||
::Thank you! |
|||
::Could I get an editor to look at the draft page and suggest edits that help with approval? [[User:Holyhootenany|Holyhootenany]] ([[User talk:Holyhootenany|talk]]) 19:30, 21 January 2024 (UTC) |
|||
:@[[User:Holyhootenany|Holyhootenany]]: Welcome to the Teahouse! I see you created [[User:Holyhootenany/sandbox]], which was declined because it did not contain multiple published, reliable, secondary sources that are independent of the subject. Creating a new Wikipedia article can be quite challenging, especially if you do not have a lot of experience editing existing Wikipedia articles. To learn how to edit, I suggest you start at [[Help:Introduction]], and then spend a significant amount of time editing existing articles to hone your skills. Once you're ready to create an article, you would gather multiple [[WP:IS|independent]] [[WP:RS|reliable sources]] that have provided [[WP:SIGCOV|significant coverage]] of the subject, and determine whether it meets Wikipedia's criteria for inclusion, called "[[WP:N|notability]]". If so, you could follow the instructions at [[Help:Your first article]] and summarize what the sources have published, and be prepared for a process that may include waiting for review, declines, and rewrites before an article is accepted. Hope this helps, and happy editing! [[User:GoingBatty|GoingBatty]] ([[User talk:GoingBatty|talk]]) 18:40, 20 January 2024 (UTC) |
|||
Above question [[User:HELSINKI!233|HELSINKI!233]] ([[User talk:HELSINKI!233|talk]]) 22:32, 31 December 2024 (UTC) |
|||
== Do redirects from one user talk page to another work? == |
|||
:Hi and welcome, @[[User:HELSINKI!233|HELSINKI!233]]. Your talk page is where other editors can leave messages for you or begin conversations with you. You can learn more at [[WP:USERTALK|the guidelines for user pages]]. [[User:Schazjmd|<span style="color:#066293;">'''Schazjmd'''</span>]] [[User talk:Schazjmd|<span style="color:#738276;">''(talk)''</span>]] 22:37, 31 December 2024 (UTC) |
|||
::It is not for chat, or your thoughts/opinions about stuff. Also, with a few exceptions, you are allowed to delete content from your Talk page, although some people prefer to archive older content instead. [[User:David notMD|David notMD]] ([[User talk:David notMD|talk]]) 04:19, 1 January 2025 (UTC) |
|||
== Citations about movies == |
|||
Hi! I operate two accounts - this one, and a mobile account, [[User:Phönedinger's jellyfish]]. I checked the list of reasons ''to'' make a redirect, but didn't see anything about talk pages on there. Is it possible to redirect the mobile account's talk page to my main one? If not, I'll watchlist the mobile talk page on both accounts. Would rather be safe than sorry. Thanks! [[User:Schrödinger's jellyfish|<small style="color:#663399;background:Thistle;border:2px solid;border-radius:4px;padding:0 4px">Schrödinger's jellyfish</small>]] [[User talk:Schrödinger's jellyfish|✉]] 19:02, 20 January 2024 (UTC) |
|||
I'm trying to improve [[The Lincoln Lawyer (film)]] |
|||
:Hi, {{u|Schrödinger's jellyfish}}! While I don't see it specifically listed for mobile accounts per se, redirecting an alternate account's talk page to your main account is a practice doumented at [[WP:PUBSOCK|the public sock account policy]], so I'd say go for it and just add the [[Help:redirect|redirect]] template to your mobile talk page. Happy editing! [[User:Perfect4th|Perfect4th]] ([[User talk:Perfect4th|talk]]) 22:37, 20 January 2024 (UTC) |
|||
::Wonderful! Thank you so much! [[User:Schrödinger's jellyfish|<small style="color:#663399;background:Thistle;border:2px solid;border-radius:4px;padding:0 4px">Schrödinger's jellyfish</small>]] [[User talk:Schrödinger's jellyfish|✉]] 22:38, 20 January 2024 (UTC) |
|||
1) Is IMDB considered a good citation for the cast list, producer name, etc? |
|||
== Reference add which is not available online == |
|||
I want to add news to a article but online news is not available print news is available how to add? [[Special:Contributions/45.124.7.168|45.124.7.168]] ([[User talk:45.124.7.168|talk]]) 19:25, 20 January 2024 (UTC) |
|||
2) How do I add a citation to an existing infobox? (Visual or Source editor) |
|||
:Asked and answered on the Help desk. Please don't ask the same question on more than one desk: it can lead to confusion and potentially wastes the limited time of the volunteers who respond on the desks. {The poster formerly known as 87.81.230.195} [[Special:Contributions/176.24.47.60|176.24.47.60]] ([[User talk:176.24.47.60|talk]]) 20:18, 20 January 2024 (UTC) |
|||
Many thanks |
|||
== Nazism == |
|||
{{cot|bg=cornsilk|The ol', "far-left, not far-right" trope. Please see [[Talk:Nazi Party/FAQ]].}} |
|||
Nazism is incorrectly labeled as being far-right, when it should be labeled as far-left. The politics that the party represented (fascism), as well as the National Socialism they represented, are all facets found within the far-left leaning ideologue. Yes, they despised liberals, but Hitler HATED capitalism and also utilized racial ideologues not found in either far-right or far-left of his era. It wasn't until now that the far-left have embraced racial ideologues to replace the class ideologues found within their Marxism (another facet of the Nazi's, by the way). Neo-Nazi's are the current label for any new KKK member and since it WAS the Democrat party (the same that exists today, there was no "flip") that created the KKK, Neo-Nazi's are far-left as well, so you might as well correct that article as well. History is important in the fact that you can't change it, so why are you trying to hide the left's dark history? They have so much more than any group from the right.... [[Special:Contributions/70.122.110.127|70.122.110.127]] ([[User talk:70.122.110.127|talk]]) 20:58, 20 January 2024 (UTC) |
|||
:The Teahouse is not the place for this, I suggest you post on the relevant article talk pages, don't expect a kind reaction though. [[User:Theroadislong|Theroadislong]] ([[User talk:Theroadislong|talk]]) 21:02, 20 January 2024 (UTC) |
|||
:See the boxes "Frequently asked questions" at [[Talk:Nazism]], and "Fascism is a right-wing ideology" at [[Talk:Fascism]]. See also [[Talk:Fascism/FAQ]]. [[User:PrimeHunter|PrimeHunter]] ([[User talk:PrimeHunter|talk]]) 21:22, 20 January 2024 (UTC) |
|||
::The OPs remarks are fringe views that are complete and utter baloney, and these views are rejected by all competent political scientists of the last 90 years. [[User:Cullen328|Cullen328]] ([[User talk:Cullen328|talk]]) 23:53, 20 January 2024 (UTC) |
|||
<small>Collapsed; wrong venue; [[WP:NOTFORUM]]; perennial comment already answered in the [[Talk:Nazi Party/FAQ|Nazi party FAQ]]. [[User:Mathglot|Mathglot]] ([[User talk:Mathglot|talk]]) 11:18, 21 January 2024 (UTC)</small> |
|||
{{cob}} |
|||
[[User:Littenberg|Ben]] ([[User talk:Littenberg|talk]]) 22:36, 31 December 2024 (UTC) |
|||
== Can I delete everything, then rewrite the whole article? == |
|||
:Hi and welcome, @[[User:Littenberg|Littenberg]]. Imdb is not considered a reliable source because much of its information is [[WP:UGC|user-generated]]. You can learn how to add citations at [[User:Nick Moyes/Easier Referencing for Beginners|Easier Referencing for Beginners]]. [[User:Schazjmd|<span style="color:#066293;">'''Schazjmd'''</span>]] [[User talk:Schazjmd|<span style="color:#738276;">''(talk)''</span>]] 22:39, 31 December 2024 (UTC) |
|||
::If you click on Edit at the top menu it allows you to edit the entire article, including the Infobox. [[User:David notMD|David notMD]] ([[User talk:David notMD|talk]]) 04:15, 1 January 2025 (UTC) |
|||
== How do I create a Wikipedia article == |
|||
Article: ''[[Tumblestone]]'' <br> |
|||
So here's the thing... I want to improve this article, but I'm kind of cringing how bad I was writing it back when I had less experience. I was thinking to rewrite it from scratch. When is it okay to blow everything up? [[User:TrademarkedTWOrantula|<span style="color: darkgreen"><span style="font-family: Georgia">TWOrantula</span></span>]]<sup><small>TM</small></sup> <small>([[User talk:TrademarkedTWOrantula|enter the web]])</small> 22:02, 20 January 2024 (UTC) |
|||
::{{u|TrademarkedTWOrantula}}, although you wrote most of that article, several other editors contributed significantly as well. Suddenly eliminating their work seems harsh to me. Instead, I recommend a series of smaller, incremental edits to gradually improve the article. [[User:Cullen328|Cullen328]] ([[User talk:Cullen328|talk]]) 22:10, 20 January 2024 (UTC) |
|||
How do I create an article on Wikipedia? [[User:Красный Октябрь|Красный Октябрь]] ([[User talk:Красный Октябрь|talk]]) 23:12, 31 December 2024 (UTC) |
|||
:@[[User:TrademarkedTWOrantula|TrademarkedTWOrantula]]: Hello! The article right now seems short enough, so I think it would be OK to just rewrite everything. [[User:Deltaspace42|'''<span style="color:orange">Delta</span>'''<span style="color:green">space</span><sup style="color:#013220">42</sup>]] ([[User talk:Deltaspace42|talk]] • [[Special:Contributions/Deltaspace42|contribs]]) 22:07, 20 January 2024 (UTC) |
|||
::I agree with Cullen328 - copy a section into your Sandbox, improve it there, then paste back - would be preferred to blanking content and starting over. [[User:David notMD|David notMD]] ([[User talk:David notMD|talk]]) 00:38, 21 January 2024 (UTC) |
|||
:: Agree with David notMD and Cullen328. Flip the situation around: an article created by someone else, inexperienced, which you subsequently improved in numerous edits back in the day. Original user pops back up, decides to throw everything out, and start over. How does that make you feel about all the contributions you made, now dumped in the trash? See also [[WP:OWN]]. Not saying that [[WP:TNT]] is never a valid option, but you should always consider the contributions of other users. [[User:Mathglot|Mathglot]] ([[User talk:Mathglot|talk]]) 11:33, 21 January 2024 (UTC) |
|||
:Здравствуйте, @[[User:Красный Октябрь|Красный Октябрь]], and welcome to the Teahouse. {{User:ColinFine/PractiseFirst}} [[User:ColinFine|ColinFine]] ([[User talk:ColinFine|talk]]) 23:33, 31 December 2024 (UTC) |
|||
== math == |
|||
::Can you help me learn how to make one? I want to make an article on the Kazan bombing that happened about a 1.5 week ago. [[User:Красный Октябрь|Красный Октябрь]] ([[User talk:Красный Октябрь|talk]]) 23:50, 31 December 2024 (UTC) |
|||
:::{{u|Красный Октябрь}}, please be aware that the [[Russo-Ukrainian War]] is a designated contentious topic. You cannot write new content about that war until your account is [[WP:XC|Extended confirmed]], which means the account is over a month old (it is) ''and'' has made over 500 constractive edits (you have a long way to go). [[User:Cullen328|Cullen328]] ([[User talk:Cullen328|talk]]) 02:56, 1 January 2025 (UTC) |
|||
::::Then can someone else make an article about it? [[User:Красный Октябрь|Красный Октябрь]] ([[User talk:Красный Октябрь|talk]]) 04:08, 1 January 2025 (UTC) |
|||
:::::Teahouse Hosts are here to advise, but not to be authors or co-authors. [[User:David notMD|David notMD]] ([[User talk:David notMD|talk]]) 04:13, 1 January 2025 (UTC) |
|||
::::::Who do I ask about it? [[User:Красный Октябрь|Красный Октябрь]] ([[User talk:Красный Октябрь|talk]]) 08:59, 1 January 2025 (UTC) |
|||
:::::::{{u|Красный Октябрь}}, if you persist in discussing a designated contentious topic such as the [[Russo-Ukrainian War]] before you are extended confirmed, you may be blocked. So, please edit other topic areas until then. [[User:Cullen328|Cullen328]] ([[User talk:Cullen328|talk]]) 09:16, 1 January 2025 (UTC) |
|||
:::::::You could ask on [[Talk:Russo-Ukrainian War]]. Normally such talk pages are protected, but that one doesn't seem to be at this time. What might happen is that someone adds a paragraph about the incident to the existing article rather than create a new article. |
|||
:::::::I'll add that editors who aren't extended-confirmed generally aren't permitted to use the talk pages of such articles either, and I have always disagreed with this, mainly because constructive edit requests end up in [[WP:RFED]], making extra work for administrators. ~[[User:Anachronist|Anachronist]] <small>([[User talk:Anachronist|talk]])</small> 16:34, 1 January 2025 (UTC) |
|||
== Why are padlocks not automatically added when an article is protected? == |
|||
whitch number is closest to 0 1 fourth or 1 sixth [[Special:Contributions/67.81.11.50|67.81.11.50]] ([[User talk:67.81.11.50|talk]]) 22:47, 20 January 2024 (UTC) |
|||
:It is 1/6, but please ask these questions at [[Mathematics reference desk| the mathematics reference desk]] in the future. <span style="border: 1px solid #0000FF ;color:#0000FF; padding:0px 7px;border-radius:10px"><b>[[User:NightWolf1223|NW1223]]<[[User talk:NightWolf1223|Howl at me]]•[[Special:Contributions/NightWolf1223|My hunts]]></b></span> 22:51, 20 January 2024 (UTC) |
|||
Sorry if this is the wrong place. [[User:Heyaaaaalol|Heyaaaaalol]] ([[User talk:Heyaaaaalol|talk]]) 02:38, 1 January 2025 (UTC) |
|||
== Can I join any Wikipedia:Articles for deletion process? == |
|||
:@[[User:Heyaaaaalol|Heyaaaaalol]] Because an administrator needs to add a protection notice (the padlocks) to a page in order to show the padlock on pages. Sometimes the padlocks do not get placed on a page, especially if it's a user page, unless if the protection notice is placed by an administrator. [[User:NicePrettyFlower|NicePrettyFlower]] ([[User talk:NicePrettyFlower|talk]]) 03:35, 1 January 2025 (UTC) |
|||
Hi! I know I'm not a Wikipedia expert, but after having created several Wikipedia pages in the past few weeks, I have gained a better understanding of the rules, particularly regarding the Notability Guidelines. While I enjoy creating biographies of random notable people, I am interested in participating in the deletion process and casting my vote. Is there a specific rule for this? Do I need to be an expert to do so? Additionally, where can I find a list of articles that have been tagged for deletion? I hope you can answer my questions. If not, I guess I will stick to editing or creating pages.[[User:Aona1212|Aona1212]] ([[User talk:Aona1212|talk]]) 22:48, 20 January 2024 (UTC) |
|||
:Hello, {{u|Aona1212}}. Any editor in good standing can participate in AfD debates. Just be sure that your recommendation is based on our [[WP:PAG|policies and guidelines]]. As for finding deletion debates, [[WP:AFDT]] is the shortcut to today's deletion debates. "Today" is based on [[UTC]], so depending on where you live, the day may not end at midnight. That page has a back arrow, allowing you to see a list of yesterday's debates, and so on. [[User:Cullen328|Cullen328]] ([[User talk:Cullen328|talk]]) 23:19, 20 January 2024 (UTC) |
|||
::@[[User:Cullen328|Cullen328]], thank you. [[User:Aona1212|Aona1212]] ([[User talk:Aona1212|talk]]) 12:11, 21 January 2024 (UTC) |
|||
:::@[[User:Aona1212|Aona1212]] Just to mention that deletion discussions are not based on votes. Editors express their views about the compliance of the article according to policy/guidelines and an administrator (usually) who is technically equipped to make the deletion will do so (or not) based on the arguments made. Most often, it comes down to deciding whether the subject of the article is [[WP:N|wikinotable]], which can be a judgement call. See also [[WP:DEL]] for a full discussion of the policy and process. [[User:Michael D. Turnbull|Mike Turnbull]] ([[User talk:Michael D. Turnbull|talk]]) 16:06, 21 January 2024 (UTC) |
|||
:[[User:Heyaaaaalol|Heyaaaaalol]], I have done some page protection myself. Usually when I apply protection, I wish to see the "padlock" icon applied. Sometimes when I use the protection script, I forget to click the toggle which leaves the padlock icon. In that case, there's a bot which usually fixes that mistake automatically. If you see a protected page without an icon, you might tell someone. Do our answers help? [[User:BusterD|BusterD]] ([[User talk:BusterD|talk]]) 03:44, 1 January 2025 (UTC) |
|||
== How can I restore edits that were deleted by Wikipedia? == |
|||
:Hello, @[[User:Heyaaaaalol|Heyaaaaalol]]. To actually answer your question: because nobody has implemented that function in the software. I don't know whether there is a technical reason for that, or whether it's just that nobody's got round to it. Questions about the software and user interface are better asked at the Village pump: either [[WP:VPT]] or [[WP:VPR]] [[User:ColinFine|ColinFine]] ([[User talk:ColinFine|talk]]) 13:34, 1 January 2025 (UTC) |
|||
:{{ping|Heyaaaaalol}} Protecting a page is a log action in the MediaWiki software which powers Wikipedia and thousands of other wikis. It does not make an edit to the page. Logs are separate from edits. Displaying a padlock with a link is a Wikipedia practice. We do it by editing the page and adding special code which places a padlock in the corner instead of the normal text area. Other wikis may use no or other symbols for protected pages, place them in other places, and make no or other links on them. A MediaWiki feature to automatically display a symbol on protected pages was recently added at [[phab:T12347]] but it's disabled by default. I haven't examined how flexible it is but I guess it would be non-trivial for us to convert to using it when we already have a well-functioning system. There is so far only a single Wikimedia wiki which has set <code>wgEnableProtectionIndicators</code> to true in https://noc.wikimedia.org/conf/highlight.php?file=InitialiseSettings.php. [[User:PrimeHunter|PrimeHunter]] ([[User talk:PrimeHunter|talk]]) 15:08, 1 January 2025 (UTC) |
|||
== Multiple non-free album covers on one page == |
|||
I wrote neutral and accurate information about the services provided by the Arizona Department of Transportation, a government agency. The current information is outdated so major editing is needed. Most of the content was taken from the Arizona Department of Transportation's website, azdot.gov. |
|||
For the album ''[[Breakfast with Girls]]'', there's an associated EP located at the section ''[[Breakfast with Girls#Brunch|Brunch]]''. The EP has an infobox but is currently without cover artwork, so I'm wondering if it's appropriate for the article to have another non-free image when one is already in use for the album's artwork. If not within free use, ''Brunch''{{'}}s artwork (seen [https://self.is/brunch here]) is mostly text on a black background, so would I be allowed to crop out the non-text part and use that as a [[Template:PD-textlogo|public domain text logo]] image in an infobox? [[User:Koopastar|Koopastar]] ([[User talk:Koopastar|talk]]) 04:50, 1 January 2025 (UTC) |
|||
I disclosed that I am an employee of the Arizona Department of Transportation and I received a message saying that I have a conflict of interest and the content I provided was rejected. |
|||
:Hi [[User:Koopastar|Koopastar]], I do think that another cover would be within fair use. There's also been instances of this in the past like [[In Rainbows#In Rainbows Disk 2]]. Happy editing! [[User:Justiyaya|'''<span style="color:#1d556d">Just</span>''']][[Special:Contributions/Justiyaya|'''<span style="color:#000000">i</span>''']][[User talk:Justiyaya#top|'''<span style="color:#6d351d">yaya</span>''']] 06:59, 1 January 2025 (UTC) |
|||
How can I restore the edits that I had made? [[User:Loribaker2757|Loribaker2757]] ([[User talk:Loribaker2757|talk]]) 22:03, 20 January 2024 (UTC) |
|||
:{{u|Loribaker2757}}, you do have a [[WP:COI|conflict of interest]]. Thank you for disclosing it. An acceptable Wikipedia article primarily summarizes what reliable sources ''independent'' of the topic say about the topic. Accordingly, use of azdot.gov should be limited. The policy language at [[WP:ABOUTSELF]] makes it clear that the article should not be based primarily on sources like azdot.gov. Any reader who wants more information from your employer's website can simply visit that website. The best practice for you as a paid editor is to make well-referenced formal [[WP:ER|Edit requests]] at [[Talk: Arizona Department of Transportation]], where you must gain consensus for your proposed changes. [[User:Cullen328|Cullen328]] ([[User talk:Cullen328|talk]]) 22:24, 20 January 2024 (UTC) |
|||
::This is a state government agency not a business. [[Special:Contributions/2600:8800:5662:C0:F456:239B:6DC5:E2EB|2600:8800:5662:C0:F456:239B:6DC5:E2EB]] ([[User talk:2600:8800:5662:C0:F456:239B:6DC5:E2EB|talk]]) 22:38, 20 January 2024 (UTC) |
|||
:::From a policy point of view, government agencies are treated the same as private businesses. [[User:Cullen328|Cullen328]] ([[User talk:Cullen328|talk]]) 23:27, 20 January 2024 (UTC) |
|||
== afd == |
|||
== Can you do those citation things on mobile? == |
|||
I really like a certain article, but it's afd, if it is deleted, is there a way to still view it [[User:Saarabout|🐢]] ([[User talk:Saarabout|talk]]) 05:24, 1 January 2025 (UTC) |
|||
:Hello, {{u|RoboDoggo9123}}. My suggestion is to use the fully functional desktop site, which works just fine on mobile devices. I edit using a smartphone 99% of the time, and have added thousands of references to Wikipedia that way. Just scroll to the bottom of any mobile Wikipedia page, and there is a "Desktop" link and if you click it, you can edit using the desktop site. [[User:Cullen328|Cullen328]] ([[User talk:Cullen328|talk]]) 23:47, 20 January 2024 (UTC) |
|||
:History checker [[User:SimpleSubCubicGraph|SimpleSubCubicGraph]] ([[User talk:SimpleSubCubicGraph|talk]]) 05:26, 1 January 2025 (UTC) |
|||
== Shumpei Fukahori == |
|||
:If the article is still "public" currently, you can save a copy for yourself: [[Wikipedia:Download as PDF|Download as PDF]] (maybe see also [[mw:Help:Export|Help:Export]]). Otherwise go to: [[CAT:RESTORE]]. --[[User:Slowking Man|Slowking Man]] ([[User talk:Slowking Man|talk]]) 06:14, 1 January 2025 (UTC) |
|||
::This appears to be about [[X11 color names]], which has been in existance since 2003 and has been edited more than 700 times since then. While it is unlikely that it will be deleted, as noted, you can save a copy to your computer. [[User:David notMD|David notMD]] ([[User talk:David notMD|talk]]) 13:55, 1 January 2025 (UTC) |
|||
== Hypothetically, could it be possible to write a netural autobiography? == |
|||
I honestly have no idea what I did to [[Shumpei Fukahori]]'s article. Please can someone check it out and fix it? Thanks :) [[User:RossEvans18|RossEvans18]] ([[User talk:RossEvans18|talk]]) 23:58, 20 January 2024 (UTC) |
|||
Hypothetically, would it be possible for someone on Wikipedia to write a neutral autobiography of themselves? Showing the good and bad and making no favor to either side, providing proof and checking all of the boxes. How would that go and would it be accepted? What if say a Wikipedia administrator that is not really well known becomes for example the president of the US? Can they have oversight over their own article? Are they removed from their position? Can they no longer edit anything involving US politics due to their inherent bias? And finally has there been any real examples of this over the past 20+ years? [[User:SimpleSubCubicGraph|SimpleSubCubicGraph]] ([[User talk:SimpleSubCubicGraph|talk]]) 05:25, 1 January 2025 (UTC) |
|||
:I have no idea why it looked like that but I fixed it eitherway, haha. [[User:Enlightened Southerner|Enlightened Southerner]] ([[User talk:Enlightened Southerner|talk]]) 00:02, 21 January 2024 (UTC) |
|||
:Hello, {{u|SimpleSubCubicGraph}}. Hypothetically, what you describe is possible but the president of the US example is implausible because credible candidates for that office would already be the subject of a Wikipedia article. More plausible would be a longtime editor elected to a state or provincial legislature. I see no reason why that person could not submit an autobiography through [[WP:AFC|Articles for Creation]] with full disclosure, have the article accepted and continue as an editor or even as an administrator. They should certainly recuse from the current legislative affairs of their state or province but otherwise I would not see a broader problem. As for whether anything like that has ever happened, I do not know. Maybe another editor does. [[User:Cullen328|Cullen328]] ([[User talk:Cullen328|talk]]) 06:49, 1 January 2025 (UTC) |
|||
::Thanks! :) [[User:RossEvans18|RossEvans18]] ([[User talk:RossEvans18|talk]]) 00:06, 21 January 2024 (UTC) |
|||
:@[[User:SimpleSubCubicGraph|SimpleSubCubicGraph]] welcome to the Teahouse! Interesting questions. I would say that while it is hypothetically possible, guidelines still strongly discourage creations of autobiographies and I haven't seen an instance of it being successful outside of early Wikipedia. This would have to go through [[WP:AFC|articles for creation]] and the editor should [[Wikipedia:DISCLOSE|declare]] their conflict of interest. Subjects do not have oversight of their own article. |
|||
:I would argue that if someone is elected president of the US they would have a financial relationship with the federal government and should stay away from editing those topics. I think they would have potentially less of a conflict of interest with state governments and historical united states politics. I don't think one would be removed as an administrator because they are elected to a public office here. I don't know of any Wikipedian that has been elected. [[CongressEdits]] is probably the closest to an example of this. [[User:Justiyaya|'''<span style="color:#1d556d">Just</span>''']][[Special:Contributions/Justiyaya|'''<span style="color:#000000">i</span>''']][[User talk:Justiyaya#top|'''<span style="color:#6d351d">yaya</span>''']] 06:46, 1 January 2025 (UTC) |
|||
::Succeeding at autobiography is more likely to occur in an area such as [[WP:NACADEMIC]]. A senior professor at a university would have as models articles about other professors at their university. [[User:David notMD|David notMD]] ([[User talk:David notMD|talk]]) 14:02, 1 January 2025 (UTC) |
|||
:The only time I recall a good autobiography being written was when the CEO of a small business wrote one, submitted it for review, and it was accepted after some minor revisions. So it is possible. |
|||
:By now I've gained enough experience on Wikipedia that I could probably write a neutral biography about myself, but because I am not notable, there's no point. ~[[User:Anachronist|Anachronist]] <small>([[User talk:Anachronist|talk]])</small> 16:26, 1 January 2025 (UTC) |
|||
:a) It would probably go over reasonably well, presuming they followed the Correct way to do that kind of thing: don't create your own bio article (meaning here the literal "creating a new page in [[WP:Main namespace|Main namespace]]") and don't edit it directly but put up stuff on the [[WP:Talk page|Talk page]] for others to evaluate, revise, and put into the article if they decide. |
|||
:b) No, [[WP:OWN|no one "owns" articles]] and gets special "powers" over them. Since all Wikipedia content is "[[WP:Wikipedia is free content|free as in freedom]]" you and anyone else can copy it put it up elsewhere and do whatever with the copies, as long as you credit the original creators. |
|||
:c) Why would they get adminship removed if they haven't misused it? |
|||
:d) They probably ought to stay away from US politics content yes, being rather [[WP:INVOLVED]]. Also realistically the POTUS is not going to have ample free time to devote to Wikipedia contributing, or to be inclined to devote what little precious free time they get to, doing more work. |
|||
:e) Have a look at [[WP:List of Wikipedians with articles]]. --[[User:Slowking Man|Slowking Man]] ([[User talk:Slowking Man|talk]]) 17:26, 1 January 2025 (UTC) |
|||
== Wiki page written by subject or friend of subject? == |
|||
== Policy question == |
|||
Hello, I saw a page on Wikipedia that looks like it was, based on the way it's presented, mostly written by the person who is the subject of the page, or a close friend. What is the standard flag or way to raise this on the page's talk page? Thank you for your help. [[User:FireBatV|FireBatV]] ([[User talk:FireBatV|talk]]) 08:05, 1 January 2025 (UTC) |
|||
:Hello, {{u|Enlightened Southerner}}, if the killer was convicted in a court of law, then it is OK to mention the name. If the killer died without being tried, but reliable sources and investigative bodies agree that they were responsible, it is probably OK to mention. We say, of course, that John Wilkes Booth murdered Abraham Lincoln, although Booth was killed when he was a fugitive, and was never tried or convicted. On another point, was the victim notable before the crime in this case? Otherwise, per [[WP:BIO1E]], the article should be about the notable crime rather than a biography of the victim. [[User:Cullen328|Cullen328]] ([[User talk:Cullen328|talk]]) 01:21, 21 January 2024 (UTC) |
|||
::The article is [[Juraj Vankulič]] who was a Slovak drag performer who was murdered in the [[2022 Bratislava shooting]] and the murderer was Juraj Krajčik. I reworded the article in ways and in the death section I wrote “ On October 12, 2022 at around 6 p.m., Vankulič was murdered when shot by a 19-year old teenager, [NAME REDACTED by an administrator]…” however my edit was reverted with the edit summary: “No mention of the murderer, please”, I was unsure if this is just an opinion or a policy regarding murder people, would you say it is appropriate to mention the name? [[User:Enlightened Southerner|Enlightened Southerner]] ([[User talk:Enlightened Southerner|talk]]) 02:33, 21 January 2024 (UTC) |
|||
:::@[[User:Enlightened Southerner|Enlightened Southerner]] Looking at the source you cited, I see no mention of the killer's name. So it would be highly inappropriate to mention it there - or here - without proper citations! [[User:Nick Moyes|Nick Moyes]] ([[User talk:Nick Moyes|talk]]) 16:20, 21 January 2024 (UTC) |
|||
:Hello @[[User:FireBatV|FireBatV]]! Welcome to the Teahouse. The standard way is to tag the article with the [[Template:COI]] on the main article page. Alternatively, you can use [[Wikipedia:Twinkle|Twinkle]] to tag it as well. '''[[User:TNM101|<span style="color:red;">TNM</span><span style="color:black;">101</span>]]''' ([[User talk:TNM101|<span style="color:blue;">chat</span>]]) 12:17, 1 January 2025 (UTC) |
|||
== Can I copy a Simple English Wikipedia page into English Wikipedia? == |
|||
:: Thank you for the help! I've added the {{tlx|Template:COI}} to the page in question ([[Rhett Ayers Butler]]) and as per the instructions on the template page I am have also added{{tlx|Connected contributor}} to the talk page, so editors can take it from there. Thank you again for the help and the friendly welcome, my issue has been resolved. [[User:FireBatV|FireBatV]] ([[User talk:FireBatV|talk]]) 21:13, 1 January 2025 (UTC) |
|||
Hello. I noticed there are some Simple English Wikipedia articles that do not exist on English Wikipedia. Would it be okay to copy them to here, in a similar manner to how [https://simple.m.wikipedia.org/wiki/Wikipedia:How_to_copy_from_another_Wikipedia copying from other Wikipedias works on Simple English Wikipedia?] I couldn’t find any guidelines that say whether this is alright, so I thought I should defer to here. |
|||
:::No problem! Feel free to reach out if you need any more help '''[[User:TNM101|<span style="color:red;">TNM</span><span style="color:black;">101</span>]]''' ([[User talk:TNM101|<span style="color:blue;">chat</span>]]) 06:46, 2 January 2025 (UTC) |
|||
== Editing the Wikipedia page of "The crown" == |
|||
I’d also like to say that, if this is alright to do, I would of course disclose that it was copied over in the talk page, as [[WP:CWW]] recommends. |
|||
i want to edit the Wikipedia page of "the crown" on netflix why i wont add the seventh season of "the crown" if you cant tell the staff at netflix please add aditional information like the royal wedding of prince william and kate Middleton and the queen involvement in the 2012 summer olympics in london england alongside James bond!!! [[Special:Contributions/89.128.137.159|89.128.137.159]] ([[User talk:89.128.137.159|talk]]) 14:08, 1 January 2025 (UTC) |
|||
Thank you! [[User:Slamforeman|Slamforeman]] ([[User talk:Slamforeman|talk]]) 00:47, 21 January 2024 (UTC) |
|||
:Hello, {{u|Slamforeman}}. The answer is "yes" as long as the article is in complete compliance with English Wikipedia's [[WP:PAG|policies and guidelines]]. [[User:Cullen328|Cullen328]] ([[User talk:Cullen328|talk]]) 01:11, 21 January 2024 (UTC) |
|||
::As an aside, editors who are blocked on English Wikipedia are sometimes advised to edit on Simple English Wikipedia to show their ability to edit Wikimedia projects productively. So, keep that in mind when assessing articles that you find there. [[User:Cullen328|Cullen328]] ([[User talk:Cullen328|talk]]) 01:14, 21 January 2024 (UTC) |
|||
:Hi, @[[User:Slamforeman|Slamforeman]]. Yes, but with limitations. You can copy an article from Simple English and provide attribution. We have stiffer requirements on providing references than Simple English does. [[Wikipedia:Verifiability]] is a policy while [[:simple:Wikipedia:Citing sources]] is a guideline. So you may need to find more sources or leave material out, depending on the article. [[User:StarryGrandma|StarryGrandma]] ([[User talk:StarryGrandma|talk]]) 01:19, 21 January 2024 (UTC) |
|||
:Thank you both very much for your help. [[User:Slamforeman|Slamforeman]] ([[User talk:Slamforeman|talk]]) 02:06, 21 January 2024 (UTC) |
|||
:: [[User:Slamforeman|Slamforeman]], Yes, as others have said, but note that [[WP:CWW|copy attribution]] is '''required''', per Wikipedia's Terms of Use. You may use a modified copy of the model attribution statement near the top of [[WP:CWW]], modified to fit your situation. I.e., this: |
|||
::: <code><nowiki>Copied content from [[:simple:<Exact page name>]]; see that page's history for attribution.</nowiki></code> |
|||
:: Add that to your edit summary when you copy the content. [[User:Mathglot|Mathglot]] ([[User talk:Mathglot|talk]]) 11:41, 21 January 2024 (UTC) |
|||
:::Ok, thank you very much. Will do! [[User:Slamforeman|Slamforeman]] ([[User talk:Slamforeman|talk]]) 16:19, 21 January 2024 (UTC) |
|||
{{Resolved}} |
|||
:Hello IP Editor: there was no 7th season of The Crown, so your additions were inappropriate and were removed. Please don't add fake information to Wikipedia. <span style="background-color: RoyalBlue; border-radius: 1em; padding: 3px 3px 3px 3px;">'''[[User:Qcne|<span style="color: GhostWhite">qcne</span>]]''' <small>[[User talk:Qcne|<span style="color: GhostWhite">(talk)</span>]]</small></span> 14:18, 1 January 2025 (UTC) |
|||
== Need info on Patna == |
|||
::False information is vandalism, which if continued can lead to your IP address being blocked and any account your subsequently register being blocked. [[User:David notMD|David notMD]] ([[User talk:David notMD|talk]]) 15:20, 1 January 2025 (UTC) |
|||
::Look for verified information about future seasons. This isn't forums and speculations. [[User:Cwater1|Cwater1]] ([[User talk:Cwater1|talk]]) 20:34, 2 January 2025 (UTC) |
|||
== The show button doesn't work on phone == |
|||
What to visit [[Special:Contributions/223.191.1.204|223.191.1.204]] ([[User talk:223.191.1.204|talk]]) 02:27, 21 January 2024 (UTC) |
|||
:The ''Teahouse'' is not really the place for questions about locating relevant content, that would be more suited to [[WP:Reference Desk]]. Then again, you could just do a simple search, though in this case, it seems that there's an existing article on the content you have requested. Sigh. [[User:Fabrickator|Fabrickator]] ([[User talk:Fabrickator|talk]]) 02:55, 21 January 2024 (UTC) |
|||
:Try https://en.wikivoyage.org/wiki/Patna [[User:Robertsky|– robertsky]] ([[User talk:Robertsky|talk]]) 03:06, 21 January 2024 (UTC) |
|||
Hi. Some articles have [https://s8.uupload.ir/files/screenshot_20250101_182319_chrome_rpkr.jpg this] dialouge box on top of them. But when we touch the show button on phone (Chrome for Android for me), [https://s8.uupload.ir/files/screenshot_20250101_182322_chrome_yxid.jpg nothing] happens. I don't know about desktop. [[User:Aminabzz|Aminabzz]] ([[User talk:Aminabzz|talk]]) 15:38, 1 January 2025 (UTC) |
|||
== List Class? == |
|||
:What is an example of such an article? What happens when you use the Wikipedia app? Does it work then? ~[[User:Anachronist|Anachronist]] <small>([[User talk:Anachronist|talk]])</small> 16:21, 1 January 2025 (UTC) |
|||
Hi. I was looking at the [[C++23]] page, and started thinking what exactly a list-class article is... |
|||
:Hello, @[[User:Aminabzz|Aminabzz]], and welcome to the Teahouse. |
|||
:I agree. If I look at [[Amathus]], I get that box (from template {{tl|expand language}}) at the top, but on a browser on my Android phone, when I pick "Show" it replaces the word "Show" with "Hide", but doesn't expand the box, so I can't see additional information. |
|||
:However, the box has a "Learn more" button, and if I pick that it shows me a little more, but not the full information that I see on my browser on a computer. |
|||
:This looks like a bug in the user interface: [[WP:VPT]] is a better place to ask/report such things than here, and I suggest you post there. Thank you for pointing it out. [[User:ColinFine|ColinFine]] ([[User talk:ColinFine|talk]]) 18:04, 1 January 2025 (UTC) |
|||
::I note that I cannot find any of these problems when I use the Wikipedia app. ~[[User:Anachronist|Anachronist]] <small>([[User talk:Anachronist|talk]])</small> 15:59, 2 January 2025 (UTC) |
|||
== How to find the full form of abbreviations on phone? == |
|||
This article really seems like a long list, so would this be considered a list-class article? [[User:Coulomb1|Coulomb1]] ([[User talk:Coulomb1|talk]]) 02:50, 21 January 2024 (UTC) |
|||
Hi. On Wikipedia, some abbreviation words have dotline underlines. When we hover the mouse cursor on them on PC we can see the full form. For example, TBA reveals to be "to be announced". |
|||
:I don't think this would be considered a list. According to the criteria on [[Wikipedia:Content assessment|WP:CLASSES]], a list needs organized links to other Wikipedia articles. [[C++23]] is mostly a changelog. If you think the article should be classified differently, you can request a assessment [[Wikipedia:WikiProject_Wikipedia/Assessment#Requesting_an_assessment|here]]. [[User:CanonNi|CanonNi]] ([[User talk:CanonNi|talk]]) 03:19, 21 January 2024 (UTC) |
|||
But on mobile phones (Chrome for Android for me) there is no mouse so that we hover the cursor on them! So how can we find the full form of abbreviations on these dotline-underlined words in cellphones? Holding the word doesn't work. |
|||
== Any good sites for finding record chart placements for all/many countries? == |
|||
Look at [https://s8.uupload.ir/files/screenshot_20250101_182537_chrome_dbd2.jpg this] for seeing the dotlines. [[User:Aminabzz|Aminabzz]] ([[User talk:Aminabzz|talk]]) 15:40, 1 January 2025 (UTC) |
|||
Are there any quality, reliable websites that have chart information pertaining to multiple countries (not just the US and UK), particularly one where I can look up a certain artist and find the corresponding album and song chart placements by country for said artist? Thanks - [[User:Elephantranges|Elephantranges]] ([[User talk:Elephantranges|talk]]) 04:22, 21 January 2024 (UTC) |
|||
:That's one of the limitations of using a smartphone as a browser. There is no notion of "hover". I have seen hovering implemented in some Samsung phones, in which holding your finger near to the screen without touching it is sensed as a hover, but this worked only in certain apps and wasn't a universal experience across all apps on the phone. Unless someone has a better answer, I'd say that features reliant on hovering are generally not accessible on smartphones. ~[[User:Anachronist|Anachronist]] <small>([[User talk:Anachronist|talk]])</small> 16:16, 1 January 2025 (UTC) |
|||
:@[[User:Elephantranges|Elephantranges]] That's the sort of question that experts at [[WP:RD/E|this reference desk]] ought to be able to answer. Alternatively, try the Talk Page of a relevant music project, listed at [[WP:PROJDIR/MUS]]. [[User:Michael D. Turnbull|Mike Turnbull]] ([[User talk:Michael D. Turnbull|talk]]) 15:36, 21 January 2024 (UTC) |
|||
::Samsung devices with [[S Pen]] support actually support "hover" by well, hovering the S Pen close to the display. The display has an "active digitizer" that can sense the Pen with [[Near-field comms|NFC]]. Shows a little cursor and pops up stuff and you can hit the Pen's button to do things, very neat actually. ...Buuuut still doesn't work for the abbr stuff in browsers because it's handled differently in the system software (as it's not "really a mouse", it's handled by different code) and so doesn't "pass through" a mouse hover event down to the browser software. Shucks. --[[User:Slowking Man|Slowking Man]] ([[User talk:Slowking Man|talk]]) 17:07, 1 January 2025 (UTC) |
|||
:Oh yeah [[Template:abbr|this is a known problem for years]]. Abbr and friends use the standard [[HTML tags]] for such things, and browsers don't gaf apparently about making them "work" when there's no pointing device (mouse), and the attitude appears to be {{shrug}}. WP could implement some sort of JavaScript "workaround" that pops up a thing, but, that requires deploying something "globally" site-wide and that is a Big Deal so it needs Official approval: ask around at [[WP:VPT]] whether there's any effort in this direction. (FYI templates can't have JavaScript, it needs to be JS to "dynamically" add new page elements and display them) |
|||
:In the meantime the "workaround" is to hit edit and look at the wikitext. Or you could also view the page HTML source and use the browser's "find" to go to the abbreviation which will show its definition. Also also there might be some [[WP:userscript|userscript]] someone has made to make it pop up, which you can "install" to use while logged-in. (You could also always plug in a mouse/trackpad/etc or connect a wireless one {{;)}}) --[[User:Slowking Man|Slowking Man]] ([[User talk:Slowking Man|talk]]) 17:07, 1 January 2025 (UTC) |
|||
::@[[User:Aminabzz|Aminabzz]], [[User:Anachronist|Anachronist]], and [[User:Slowking Man|Slowking Man]]: I couldn't find any such script, so [[User:JJPMaster/abbrMobile.js|I made it]]. [[User:JJPMaster|JJP]]<sub>[[User talk:JJPMaster|Mas]]<sub>[[Special:Contributions/JJPMaster|ter]]</sub></sub> ([[She (pronoun)|she]]/[[Singular they|they]]) 05:08, 2 January 2025 (UTC) |
|||
:::Is this something that needs to be deployed in the Wikipedia software, or is it something that a user can add to vector.js? ~[[User:Anachronist|Anachronist]] <small>([[User talk:Anachronist|talk]])</small> 15:52, 2 January 2025 (UTC) |
|||
::::@[[User:Anachronist|Anachronist]]: The latter. [[User:JJPMaster|JJP]]<sub>[[User talk:JJPMaster|Mas]]<sub>[[Special:Contributions/JJPMaster|ter]]</sub></sub> ([[She (pronoun)|she]]/[[Singular they|they]]) 17:46, 2 January 2025 (UTC) |
|||
:::::{{reply|Aminabzz}} Well, JJP has a solution for you, but it requires that you be logged into Wikipedia to take advantage of it. Me, I use the Wikipedia app for reading Wikipedia on my phone, and I never encountered a "hover" situation in that app. ~[[User:Anachronist|Anachronist]] <small>([[User talk:Anachronist|talk]])</small> 17:58, 2 January 2025 (UTC) |
|||
== |
== GA Spotcheck == |
||
Apparently I might've misunderstood what is needed for a source spotcheck in a good article review. Could someone please explain what is needed? [[User:History6042|<span style="color:darkorange">History6042😊</span>]] '''([[User talk:History6042|<span style="color:blue">Contact me</span>]])''' 15:53, 1 January 2025 (UTC) |
|||
Dear Team |
|||
:Appears that at [[Wikipedia talk:Good article nominations]], date 1/1/25, there has been a question raised about GA nomination reviews conducted by History60432. [[User:David notMD|David notMD]] ([[User talk:David notMD|talk]]) 17:07, 1 January 2025 (UTC) |
|||
::Yes, that is why I am asking. [[User:History6042|<span style="color:darkorange">History6042😊</span>]] '''([[User talk:History6042|<span style="color:blue">Contact me</span>]])''' 17:09, 1 January 2025 (UTC) |
|||
:@[[User:History6042|History6042]], note 2 on the [[Wikipedia:Good article criteria]] page explains source checking. If you're concerned that your interpretation isn't in line with the community's, you might find it helpful to discuss with other editors involved with the good article process at [[Wikipedia talk:Good article nominations]]. [[User:Schazjmd|<span style="color:#066293;">'''Schazjmd'''</span>]] [[User talk:Schazjmd|<span style="color:#738276;">''(talk)''</span>]] 17:33, 1 January 2025 (UTC) |
|||
::So it means Verifiable with no original research: |
|||
::it contains a list of all references (sources of information), presented in accordance with the layout style guideline; |
|||
::reliable sources are cited inline. All content that could reasonably be challenged, except for plot summaries and that which summarizes cited content elsewhere in the article, must be cited no later than the end of the paragraph (or line if the content is not in prose); |
|||
::it contains no original research; and |
|||
::it contains no copyright violations or plagiarism? [[User:History6042|<span style="color:darkorange">History6042😊</span>]] '''([[User talk:History6042|<span style="color:blue">Contact me</span>]])''' 19:25, 1 January 2025 (UTC) |
|||
:::The '''note''' says {{tq| "Ideally, a reviewer will have access to all of the source material, and sufficient expertise to verify that the article reflects the content of the sources; this ideal is not often attained. At a minimum, check that the sources used are reliable (for example, blogs are not usually reliable sources) and that those you can access support the content of the article (for example, inline citations lead to sources that agree with what the article says) and are not plagiarized (for example, close paraphrasing of source material should only be used where appropriate, with in-text attribution if necessary)."}} [[User:Schazjmd|<span style="color:#066293;">'''Schazjmd'''</span>]] [[User talk:Schazjmd|<span style="color:#738276;">''(talk)''</span>]] 20:40, 1 January 2025 (UTC) |
|||
::::Okay I think i did that but do I just need to like write it down? [[User:History6042|<span style="color:darkorange">History6042😊</span>]] '''([[User talk:History6042|<span style="color:blue">Contact me</span>]])''' 21:06, 1 January 2025 (UTC) |
|||
:::::Saying in your review what you checked is helpful. Perhaps read through [[Wikipedia_talk:Good_article_nominations/Archive_30#Spot_checks?|this discussion on spot checks]]. You might also find it useful to read through some GA reviews by experienced reviewers, see what they're doing in their written reviews that you can learn from. [[User:Schazjmd|<span style="color:#066293;">'''Schazjmd'''</span>]] [[User talk:Schazjmd|<span style="color:#738276;">''(talk)''</span>]] 21:47, 1 January 2025 (UTC) |
|||
::::::Thank you. [[User:History6042|<span style="color:darkorange">History6042😊</span>]] '''([[User talk:History6042|<span style="color:blue">Contact me</span>]])''' 00:39, 2 January 2025 (UTC) |
|||
== Creating an article heavily lacking sources :/ == |
|||
i am create the my self biography but after publish our content is delete speedy deletion. how to solve the issues [[User:Dr.AhmedAlSulaiti|Dr.AhmedAlSulaiti]] ([[User talk:Dr.AhmedAlSulaiti|talk]]) 05:15, 21 January 2024 (UTC) |
|||
I've been working on trying create an article ([[Draft:Millennium Force's effects]]) but I've run into some issues; including after initial submission. The only sources I can find are mainly from various YouTube videos (not tied to the subject) and very few separate links; so it's no wonder why it wasn't accepted. |
|||
:Hello [[User:Dr.AhmedAlSulaiti|Dr.AhmedAlSulaiti]] – while I cannot view the deleted draft, the reason given for deletion indicates that your draft was promotional. Using Wikipedia to promote yourself, your business, or anything else is not permitted; Wikipedia is an encyclopedia. Wikipedia articles also must demonstrate that [[WP:RS|reliable sources]] not connected to the subject have chosen to write about them already, and may not include any information not already in other sources. Writing autobiographies in particular is also strongly discouraged for a variety of reasons, which I see has already been mentioned on your user talk page. If you meet Wikipedia's criteria for inclusion someone else will likely write an article about you eventually. [[User:Tollens|Tollens]] ([[User talk:Tollens|talk]]) 05:52, 21 January 2024 (UTC) |
|||
So I know what I've written down is true, but I don't have the secondary sources to prove it and that makes my info [[No original research|Original Research]]. What can I do? I was told I could try another Wiki (ex: Amusement Park Wiki) but is there anything I can do to keep it on Wikipedia? Thanks! [[User:Therguy10|<span style="color:blue;">'''Therguy10'''</span>]] ([[User talk:Therguy10|talk]]) 16:49, 1 January 2025 (UTC) |
|||
::Dear Team |
|||
::we create article for Dr.AhmedAlSulaiti related achievement and occupation ,company we are mentioned in the Articles . In the Infobox we mention he name ,occupation of company website only . kindly share guides create page [[User:Dr.AhmedAlSulaiti|Dr.AhmedAlSulaiti]] ([[User talk:Dr.AhmedAlSulaiti|talk]]) 06:07, 21 January 2024 (UTC) |
|||
:@[[User:Therguy10|Therguy10]] Welcome to theTeahouse. I'm afraid not. [[WP:RS|Reliable sources]] are fundamental to Wikipedia articles. See [[WP:42]]. [[User:Shantavira|Shantavira]]|[[User talk:Shantavira|<sup>feed me</sup>]] 18:01, 1 January 2025 (UTC) |
|||
:::[[User:Dr.AhmedAlSulaiti|Dr.AhmedAlSulaiti]], let me quote: {{Olive|Al Sulaiti Holdings was founded and is chaired by Dr. Ahmed Al Sulaiti, a visionary businessman from Qatar. He has built a diversified business portfolio that is well-known both locally and internationally.}} (And the deleted "[[Dr Ahmed Mohammed Hassan Al Hayya Al Sulaiti]]" continued in the same vein.) If it is well-known both locally and internationally, then those who know it are sure to find disinterested, reliable sources about it and to want to create an article purely based on those sources. Incidentally, the articles I've seen on companies that are well-known both locally and internationally don't bother to say that these companies are well-known both locally and internationally, and one reason for not saying this is that readers will already know it. That matter aside, I note that your user name is the same as the name of your subject. -- [[User:Hoary|Hoary]] ([[User talk:Hoary|talk]]) 07:03, 21 January 2024 (UTC) |
|||
:: |
:Hello, @[[User:Therguy10|Therguy10]]. I agree with Shantavira. Please see also [[WP:No amount of editing can overcome a lack of notability|No amount of editing can overcome a lack of notability]] [[User:ColinFine|ColinFine]] ([[User talk:ColinFine|talk]]) 18:07, 1 January 2025 (UTC) |
||
:@[[User:Shantavira|Shantavira]] @[[User:ColinFine|ColinFine]] Thank you both! I do believe that the article has potential in the future, but I also understand it isn't notable enough as of now. With the article being a history of a subject, there is older information that I fear may not be written about. My hope is that the new changes made this year were important enough to spark some kind of interest for a source; I'll have to wait and see. [[User:Therguy10|<span style="color:blue;">'''Therguy10'''</span>]] ([[User talk:Therguy10|talk]]) 18:13, 1 January 2025 (UTC) |
|||
::well i guess technically the changes were made last year...lol [[User:Therguy10|<span style="color:blue;">'''Therguy10'''</span>]] ([[User talk:Therguy10|talk]]) 18:14, 1 January 2025 (UTC) |
|||
:::If you can find reliable sources, why not improve the '''Ride experience''' section of [[:Millennium Force]]? [[User:Schazjmd|<span style="color:#066293;">'''Schazjmd'''</span>]] [[User talk:Schazjmd|<span style="color:#738276;">''(talk)''</span>]] 18:27, 1 January 2025 (UTC) |
|||
::::That was a strong consideration of mine. I certainly won't be able to fit ''everything'' inside of the main article but it might not hurt to add a little more than what I already have there. Once again my biggest issue would be collecting sources; this seems more doable. Thanks! [[User:Therguy10|<span style="color:blue;">'''Therguy10'''</span>]] ([[User talk:Therguy10|talk]]) 18:34, 1 January 2025 (UTC) |
|||
== How do i delete a Wikepedia page? == |
|||
Indef blocked on 21 January. [[User:David notMD|David notMD]] ([[User talk:David notMD|talk]]) 14:22, 21 January 2024 (UTC) |
|||
a school has a wikepedia page, (carlisle public schools) i have moved the page to inside the town page (carlisle masssachusetts) so how do i delete the old school page? it is useless (i moveed the page because it contains about only 1 paragraph, so why not make it a section) so yeah, if someone knows how to delete a page, please delete Carlisle Public Schools |
|||
== Infobox use == |
|||
how to find page - go to the education part of carlisle, massachusetts, press carlisle public schools, and you will be there. [[User:Theawezomefriend12|Theawezomefriend12]] ([[User talk:Theawezomefriend12|talk]]) 19:39, 1 January 2025 (UTC) |
|||
What would be the right infobox to use for [[Bart's Books|this article]]? [[User:TrademarkedTWOrantula|<span style="color: darkgreen"><span style="font-family: Georgia">TWOrantula</span></span>]]<sup><small>TM</small></sup> <small>([[User talk:TrademarkedTWOrantula|enter the web]])</small> 05:53, 21 January 2024 (UTC) |
|||
:[[Carlisle Public Schools]] is an article. If you believe it should be deleted, the process is to nominate it for deletion via the [[WP:AFD]] process. What you added to [[Carlisle, Massachusetts]] was rightfully deleted because you did not include any references. See [[Maynard, Massachusetts]] for an example of a referenced education section. I was going to point you to Acton or Sudbury, but those have unreferenced content. There is also [[Concord-Carlisle High School]], but that has its own problems. [[User:David notMD|David notMD]] ([[User talk:David notMD|talk]]) 20:02, 1 January 2025 (UTC) |
|||
::Okay, Thanks! [[User:Theawezomefriend12|Theawezomefriend12]] ([[User talk:Theawezomefriend12|talk]]) 20:30, 1 January 2025 (UTC) |
|||
:::{{re|Theawezomefriend12}} I'm going to offer some slightly different advice. For a small article in this situation: First copy the relevant text to the target article (the article about the town). If there are no references then be sure to add one or some. See [[WP:CWW]] for the correct process, if you are moving any text. Once that has been successfully done, turn the former article (the school district) into a redirect to the town as you did previously. There is no need to actually delete anything: redirects are useful for people trying to find things, such as information about Carlisle Public Schools, and can be safely left behind. -- [[user:zzuuzz|zzuuzz]] <sup>[[user_talk:zzuuzz|(talk)]]</sup> 21:12, 1 January 2025 (UTC) |
|||
::::Thanks for the advice, i will remove the deletion (i dont know if thats possible i am new) and make the page redierect to Carlisle, Massachusetts. [[User:Theawezomefriend12|Theawezomefriend12]] ([[User talk:Theawezomefriend12|talk]]) 21:30, 1 January 2025 (UTC) |
|||
:::::{{re|Theawezomefriend12}} I've helped a bit with the deletion part. The best way to learn is to just do things, so I'll leave you to continue the process (and will probably pop back in a bit). I'll just mention that it appears that the only reason your previous attempt was reverted was because you didn't include any references for the content you were adding/moving to the town article. Sufficient references probably exist in the district article so they can just be copied. If not, I'm sure they won't be difficult to find. -- [[user:zzuuzz|zzuuzz]] <sup>[[user_talk:zzuuzz|(talk)]]</sup> 21:38, 1 January 2025 (UTC) |
|||
::::::oh ok, i am propably going to be moving the main description and the new gallery part i added, i probably will not be added refrences and external links. [[User:Theawezomefriend12|Theawezomefriend12]] ([[User talk:Theawezomefriend12|talk]]) 21:45, 1 January 2025 (UTC) |
|||
== Hi! PARIS == |
|||
:Welcome to the Teahouse, {{u|TrademarkedTWOrantula}}! I'd probably say the best fit for a bookstore is {{tl|Infobox company}} as I don't see any other infobox that'd be more specific. You can see if another infobox suites your purposes by scrolling to the bottom of the documentation and looking at the table. Cheers! —[[User:Sirdog|<span style="color:#058700">'''Sirdog'''</span> ]]([[User talk:Sirdog|talk]]) 05:56, 21 January 2024 (UTC) |
|||
Good morning, good afternoon or good evening. Happy new year 2025. Please how to move to the main space the [[Draft:Hi! PARIS]]. This research center in AI is now one of the most important in France : https://www.lesechos.fr/tech-medias/intelligence-artificielle/une-dotation-de-70-millions-pour-la-formation-de-lelite-de-lia-en-france-2096150 and https://www.lemondeinformatique.fr/actualites/lire-hec-et-l-institut-polytechnique-remportent-l-appel-a-projets-cluster-ia-93818.html. Many thanks in advance for your help. Have a great day. [[Special:Contributions/2A01:CB00:B48:9900:1C67:231F:4C2F:9933|2A01:CB00:B48:9900:1C67:231F:4C2F:9933]] ([[User talk:2A01:CB00:B48:9900:1C67:231F:4C2F:9933|talk]]) 21:57, 1 January 2025 (UTC) |
|||
== Chrome browser == |
|||
:On the AoPS Wiki go to [[special:move]]. I don't know bout wikipedia tho [[Special:Contributions/73.31.42.97|73.31.42.97]] ([[User talk:73.31.42.97|talk]]) 22:22, 1 January 2025 (UTC) |
|||
::Sorry its [[Special:MovePage]] [[Special:Contributions/73.31.42.97|73.31.42.97]] ([[User talk:73.31.42.97|talk]]) 22:23, 1 January 2025 (UTC) |
|||
:Bonsoir, IP user. |
|||
:73. is right that the technique for moving pages is to use the [[WP:Move|Move]] function (though most people do it from their user-interface, rather than going to [[Special:MovePage]]. |
|||
:However, not logged in users do not have access to that function. |
|||
:I was going to advise you to submit it for review. However, I see that the draft was created in mainspace user BobVillars (since blocked for sockpuppetry) was then moved to Draft space by @[[User:Rosguill|Rosguill]], was submitted for review by an ipV6 user in the same range as you, then moved to main space again by McSyl, also blocked as a sockpuppet of BobVillars, and moved back to draft space ''again'' by @[[User:Janhrach|Janhrach]]. |
|||
:Neither you nor anybody else has since made any substantial edits to the draft. |
|||
:I am finding it very difficult to [[WP:assume good faith|assume good faith]]. I will ask you directly: are you BobVillars/McSyl? If so, you are evading your block, which is not permitted. [[User:ColinFine|ColinFine]] ([[User talk:ColinFine|talk]]) 22:59, 1 January 2025 (UTC) |
|||
::Hi Colin. Thanks for your answer. I don’t know what you are talking about. I am using 5G access so my Ip range is used by million of users. But if it is too complicated no worries, please cancel my request. This AI cluster is one of the most important in France and one of the biggest in Europe. So soon or later it will be back to mainspace. Have a great day. [[Special:Contributions/2A01:CB06:B064:82B5:2C9D:5034:3AC5:1BD4|2A01:CB06:B064:82B5:2C9D:5034:3AC5:1BD4]] ([[User talk:2A01:CB06:B064:82B5:2C9D:5034:3AC5:1BD4|talk]]) 06:21, 2 January 2025 (UTC) |
|||
== AoPS page == |
|||
Using Chrome browser after a system update, I noticed my Chromebook's not displaying Tex (math formulas, etc.) and hyperlinks have to be double clicked. I don't normally log in. Any suggestions? [[Special:Contributions/73.192.182.36|73.192.182.36]] ([[User talk:73.192.182.36|talk]]) 06:58, 21 January 2024 (UTC) |
|||
: Wrong venue. Try [https://support.google.com/chrome?p=help&ctx=menu#topic=7439538 Google Chrome help]. [[User:Mathglot|Mathglot]] ([[User talk:Mathglot|talk]]) 11:52, 21 January 2024 (UTC) |
|||
::[[Wikipedia:Reference desk/Computing]] is also appropriate for such issues. <span class="nowrap">[[User:Verbarson|-- Verbarson ]] <sup>[[User talk:Verbarson|talk]]</sup><sub>[[Special:Contributions/Verbarson|edits]]</sub></span> 17:24, 21 January 2024 (UTC) |
|||
Why isn't there a page for [[Art of Problem Solving]]? [[Special:Contributions/73.31.42.97|73.31.42.97]] ([[User talk:73.31.42.97|talk]]) 22:15, 1 January 2025 (UTC) |
|||
== GA controversy, how to resolve? == |
|||
: Hi IP 73.31.42.97. There could be a couple reasons why: (1) nobody thought enough about the book/series to try and create one; or (2) somebody did try to create one, but the subject wasn't deemed to meet [[:Wikipedia:Notability (books)]] and was either deleted altogether or added to [[:Richard Rusczyk]]. ''{{no redirect|Art of Problem Solving}}'' current [[:WP:REDIRECT]]s to the "Rusczyk" article and its [[:H:PH|page history]] shows that there once was a stand-alone article about the book/series, but it was "redirected" per [[:Wikipedia:Articles for deletion/Art of Problem Solving (2nd nomination)]]. If you feel something has changed since then, you might want to first explain why to the administrator who closed the Articles for Deletion discussion. That administrator's name is {{u|Liz}}. Perhaps by asking at [[:User talk:Liz]], Liz can tell you what is needed for the article to be recreated or restored. -- [[User:Marchjuly|Marchjuly]] ([[User talk:Marchjuly|talk]]) 22:30, 1 January 2025 (UTC) |
|||
== Where is Wikipedia year in review for iOS? == |
|||
Has had controversy over my practices of GARs and GANs, how to resolve? I don't properly know GAs [[User:GabrielPenn4223|GabrielPenn4223]] ([[User talk:GabrielPenn4223|talk]]) 10:11, 21 January 2024 (UTC) |
|||
:Your user talk page, [[User talk:GabrielPenn4223]], suggests that you have little or no idea of what you are doing. For a start, two points: (i) Do not nominate any article unless you have done a large amount of work on the article. (ii) Do not review any article that you have worked on more than trivially, let alone any article that you have nominated. -- [[User:Hoary|Hoary]] ([[User talk:Hoary|talk]]) 10:46, 21 January 2024 (UTC) |
|||
::I know, but when will this debate controversy stop for once? I am annoyed by this. [[User:GabrielPenn4223|GabrielPenn4223]] ([[User talk:GabrielPenn4223|talk]]) 11:09, 21 January 2024 (UTC) |
|||
:::What debate controversy? Just stop nominating articles for GA and walk away from any discussions that might be happening there. Find something else to do instead, here is a good place to find things to do: [[Wikipedia:Task Center]]. <small>'''''[[User:Polyamorph|Polyamorph]]''''' ([[User talk:Polyamorph#top|talk]])</small> 11:48, 21 January 2024 (UTC) |
|||
:::@[[User:GabrielPenn4223|GabrielPenn4223]] There is no controversy. You';ve simply been told that you do not know what you're doing, and you have only just come to realise people are right. Any 'controversy,'as you perceive it, will stop when you start to appreciate that you don't yet know enough about how Wikipedia works, especially in relation to Good Article status and reviews. You've only made 60 contributions to articles, yet you seem to have caused quite some disruption and chaos by thinking you can storm in and randomly nominate articles, then seek talk page protection on those nomination or review pages just because you don't like being told to stop what you're doing and gain some understanding first. |
|||
:::You may be editing in good faith, but disruption is disruption. Adminstrators have the ability to put partial blocks on areas where good faith editors are nevertheless causing undue disruption and time-wastage. If you continue as you have done, I fear that could happen to your account. So, simply keep away from article assessments, and work in other areas which will improve your article-editing skills and understanding. |
|||
:::If you've recently made other inappropriate nominations, please go back and undo them all to avoid wasting other editors' time. [[User:Nick Moyes|Nick Moyes]] ([[User talk:Nick Moyes|talk]]) 11:57, 21 January 2024 (UTC) |
|||
:::Wherever you're asked, just promise that you'll stop nominating and reviewing GAs until you've gained more experience or that you've already stopped, then follow-through. Whenever someone comes to you about a mess you've made, tell them you're sorry, and tell them to help you fix it if you need to be the one to fix it, or fix it themselves if they can. If you've made a big mess, you could post to [[WT:GAN]] apologising, and asking them to fix it or help you fix it. Best, '''[[User:Usedtobecool|Usedtobecool]]''' [[User talk:Usedtobecool|☎️]] 11:58, 21 January 2024 (UTC) |
|||
::::@[[User:Nick Moyes|Nick Moyes]] and @[[User:Usedtobecool|Usedtobecool]], I've already apollogized on [[WT:GAN]]. [[User:GabrielPenn4223|GabrielPenn4223]] ([[User talk:GabrielPenn4223|talk]]) 13:28, 21 January 2024 (UTC) |
|||
:::::@[[User:GabrielPenn4223|GabrielPenn4223]], that's it then. You don't need to run; it's better to hang around in case other people show up with questions. As the mess clears up, it will be forgotten. Meanwhile, you can do other things, as was suggested above. You are not in any trouble, as long as you stop and engage with others politely. Just take it slow, whatever you do next. '''[[User:Usedtobecool|Usedtobecool]]''' [[User talk:Usedtobecool|☎️]] 13:38, 21 January 2024 (UTC) |
|||
:::::@[[User:GabrielPenn4223|GabrielPenn4223]] Thank you - that's the best way to deal with such issues. I've just deleted your GAR proposals for [[Nature]] and for [[National Register of Historic Places]] so that's two less thing to clear up. Regards. [[User:Nick Moyes|Nick Moyes]] ([[User talk:Nick Moyes|talk]]) 13:40, 21 January 2024 (UTC) |
|||
I am looking for Wikipedia year in review in iOS where is it? [[Special:Contributions/172.59.25.192|172.59.25.192]] ([[User talk:172.59.25.192|talk]]) 22:18, 1 January 2025 (UTC) |
|||
== Article name for YouTube channels == |
|||
:Welcome to the Teahouse, IP user. This isn't something I'm familiar with, but a Google search yields this link which I hope gives you what you seek: https://www.mediawiki.org/wiki/New_Engagement_Experiments/PES_1.3.1:_Wikipedia_insights. Regards, [[User:Nick Moyes|Nick Moyes]] ([[User talk:Nick Moyes|talk]]) 01:56, 2 January 2025 (UTC) |
|||
== How to find vandalism on Wikipedia? == |
|||
:I think the name should be the YouTube channel's name because the article is about the YouTube channel not the person. [[User:Tusharhero|Tusharhero]] ([[User talk:Tusharhero|talk]]) 12:35, 21 January 2024 (UTC) |
|||
:{{re|CanonNi}} Hello, and welcome to the Teahouse. Looking through [[:Category:YouTube channels]] and , it depends under which name they are more known. Articles about YouTuber who are mostly known under their channel name use the channel name, if they are mostly known under a different name (real name or not) use that, and in Edge cases use editorial judgement to decide on one. A redirect or disambiguation entry can be considered from the other one. [[User:Victor Schmidt|Victor Schmidt]] ([[User talk:Victor Schmidt|talk]]) 12:45, 21 January 2024 (UTC) |
|||
::Thank you. I will use the name of the channel for the article then. [[User:CanonNi|CanonNi]] ([[User talk:CanonNi|talk]]) 02:41, 22 January 2024 (UTC) |
|||
The closest thing I have found to vandalism is on the page for the number 3, where someone changed references to “3” with “2.” I’m wondering how to find vandalism, so I can revert it. Most of my edits are fixing grammar. Thank you. [[User:Heyaaaaalol|Heyaaaaalol]] ([[User talk:Heyaaaaalol|talk]]) 00:18, 2 January 2025 (UTC) |
|||
== Edit quality == |
|||
:@[[User:Heyaaaaalol|Heyaaaaalol]] Welcome to the Teahouse. If you'd care to read [[WP:VANDALISM]], and especially the section linked from this shortcut: ([[WP:SPOTVAN]]), you should have all the answers you need. Monitoring 'Recent Changes' is the best way to spot ongoing vandalism, and you can choose to select only certain types of edits which highlight the most likely problematic edits for you to assess and respond to, and ignore all the honest ones. Thank you for your interest in helping out in this field. Regards, [[User:Nick Moyes|Nick Moyes]] ([[User talk:Nick Moyes|talk]]) 01:50, 2 January 2025 (UTC) |
|||
How is edit quality measured in the mobile app and why isn't it shown in the website? [[User:Tusharhero|Tusharhero]] ([[User talk:Tusharhero|talk]]) 13:19, 21 January 2024 (UTC) |
|||
:Go to the “recent changes” link and there are filter options you can select. What you’ll want to do is choose “likely bad faith”, “very likely bad faith”, and “latest revision”. Then save that filter as your default. [[User:Grimaceshakeohio|Grimaceshakeohio]] ([[User talk:Grimaceshakeohio|talk]]) 04:35, 3 January 2025 (UTC) |
|||
== "Series of small edits" == |
|||
:I am not a mobile user, but it's possible the Wikimedia Foundation is adding features, trying things, hoping to attract more people, encourage them to become editors. As for the website, the quality of your edits will become clear to you through feedback from others. People will revert you and warn you if you do things wrong, you may get no feedback if you're doing well. People may occassionally show up to thank you and encourage you if you keep on with good work. Best, '''[[User:Usedtobecool|Usedtobecool]]''' [[User talk:Usedtobecool|☎️]] 13:42, 21 January 2024 (UTC) |
|||
:{{ping|Tusharhero}} If you refer to a specific feature which isn't shown somewhere then please describe the feature and where you see it. If it's red and green numbers in parentheses then it's not about quality but size. See [[Wikipedia:Added or removed characters]]. [[User:PrimeHunter|PrimeHunter]] ([[User talk:PrimeHunter|talk]]) 16:01, 21 January 2024 (UTC) |
|||
::No, it just says "Edit quality perfect". Its in the edit tab of the mobile app. [[User:Tusharhero|Tusharhero]] ([[User talk:Tusharhero|talk]]) 01:39, 22 January 2024 (UTC) |
|||
:::@[[User:Tusharhero|Tusharhero]], I also see this statistic. If I tap and hold on the indicator, an explanation pops up saying, {{tq|"Based on how many times one of your contriburions was reverted (undone by another editor). Reverted edits: 0."}} |
|||
:::I think it must be only looking at your most recent edits for the count. The Wikipedia app is [[FLOSS]], so you could look at the source code to confirm. --[[User:Habst|Habst]] ([[User talk:Habst|talk]]) 10:25, 22 January 2024 (UTC) |
|||
:::--[[User:Habst|Habst]] ([[User talk:Habst|talk]]) 10:25, 22 January 2024 (UTC) |
|||
::::Oh I see. Thanks. [[User:Tusharhero|Tusharhero]] ([[User talk:Tusharhero|talk]]) 13:50, 22 January 2024 (UTC) |
|||
I find it difficult to follow and review edits for valid constructive contributions when an editor makes a "series of small edits" (example: [https://en.wikipedia.org/enwiki/w/index.php?title=Marty_Makary&diff=prev&oldid=1266013859 1], [https://en.wikipedia.org/enwiki/w/index.php?title=Marty_Makary&diff=prev&oldid=1265997397 2], [https://en.wikipedia.org/enwiki/w/index.php?title=Marty_Makary&diff=prev&oldid=1265999775 3]) where several of the edits could, IMHO, easily and conceivably be made in a single edit with a single edit summary encompassing all of what is edited. Is there policy or guidelines about this practice that could help me in a [[User talk:Llll5032#Marty Makary|talk page discussion]] I am currently engaged in? [[User:Iljhgtn|Iljhgtn]] ([[User talk:Iljhgtn|talk]]) 00:23, 2 January 2025 (UTC) |
|||
== Book cover and page referencing == |
|||
:Well, there's [[Wikipedia:CAUTIOUS|WP:Cautious]], which touches on not making too large edits. There is probably better material than this, but I can't find it. |
|||
Hello, how would I go about making a book reference where a page and the back of the book back up 2 separate claims on an article? Should I make 2 different references with one specifying a page, and then the other specifying the back of the book? [[User:Goldclock|Goldclock]] ([[User talk:Goldclock|talk]]) 14:27, 21 January 2024 (UTC) |
|||
:The point has been discussed on the Help/Teahouse desks before, and various editors have said that more smaller edits are better than one all-encompassing one, because if only one or two of several small edits are objected to, they can be reverted individually, but if they're part of a larger edit the whole thing has to be reverted and then it becomes difficult to sort out which details are accepted and which disputed. |
|||
:I myself, when reading an article for its content, sometimes make small edits as I go along on noticing typos, misspellings, incorrect grammar, etc. If I were to be deliberately setting out to copyedit the article (I used to be a professional editor) I would likely make them in larger batches, perhaps one per Section or Subsection (basically, as often as [edit] is present in the text), because that makes for a smaller and easier chunk (with all its confusing reference codings) to navigate in the edit box. |
|||
:In general, I don't think an editor making good-faith edits need think primarily of the convenience of some hypothetical would-be reverter, and I wouldn't criticise anyone for making successive small edits. Were the edits to smell of some deliberate obfustication as a cover for vandalism, or edit-count inflation, it would be another matter. {The poster formerly known as 87.81.230.195} [[Special:Contributions/94.6.84.253|94.6.84.253]] ([[User talk:94.6.84.253|talk]]) 02:11, 2 January 2025 (UTC) |
|||
::Small or a series isn't the issue really. It is that several of the edits could fall under a single edit summary, and therefore could be made as one. I am curious about this comment though, "{{tq|Were the edits to smell of some deliberate obfustication as a cover for vandalism, or edit-count inflation, it would be another matter}}", because that is kind of what I think is the heart of the matter. How can you differentiate between the two if we are not allowed to scrutinize editing behavior which is a mix perhaps of both lots of small, with some more impactful edits laced in there? [[User:Iljhgtn|Iljhgtn]] ([[User talk:Iljhgtn|talk]]) 14:33, 2 January 2025 (UTC) |
|||
:::The key being the "lots of small" could just as easily be one edit under one edit summary, and then more larger or more substantive edits separately under another. If the "lots of small edits" are all broken into separate edits, for me at least, it makes it harder to check on someone's editing in the first place to determine IF there is in fact a case of real vandalism, deliberate edit-count inflation, or some other intended obfuscation. [[User:Iljhgtn|Iljhgtn]] ([[User talk:Iljhgtn|talk]]) 14:35, 2 January 2025 (UTC) |
|||
::::You can differentiate because if it is a cover for vandalism, you can point out the vandalism. Edit-count inflation these days is mostly people trying to game their way to extended confirmed status - the edits you point out here are from an editor who already has that status. There's no policy against editing this way - this editor isn't doing anything wrong. |
|||
::::If you want to examine a set of contiguous edits at once, you can get a multi-edit diff by using the radio buttons on the history page of the article in question. [[User:MrOllie|MrOllie]] ([[User talk:MrOllie|talk]]) 14:40, 2 January 2025 (UTC) |
|||
:::::Understood, and no accusation of vandalism has been levied, but if several edits could be made under a single edit summary, is there any policy that they should be? That is really the essence of my question. [[User:Iljhgtn|Iljhgtn]] ([[User talk:Iljhgtn|talk]]) 14:43, 2 January 2025 (UTC) |
|||
::::::There is not. [[User:Schazjmd|<span style="color:#066293;">'''Schazjmd'''</span>]] [[User talk:Schazjmd|<span style="color:#738276;">''(talk)''</span>]] 14:46, 2 January 2025 (UTC) |
|||
:::::::Ok. Thank you for answering my question then. [[User:Iljhgtn|Iljhgtn]] ([[User talk:Iljhgtn|talk]]) 14:56, 2 January 2025 (UTC) |
|||
:How isn't that benign? Is there an issue with the individual edits? My editing tends to match my own writing (on non-Wikipedia, Word, etc.): dump a bunch of text, and make then increasingly granular refining edits. -- [[User:Very Polite Person|Very Polite Person]] ([[User talk:Very Polite Person|talk]]) 00:37, 3 January 2025 (UTC) |
|||
== Political Party Relations == |
|||
:@[[User:Goldclock|Goldclock]] Welcome to the Teahouse. Two separate references are not necessary. Both of our editing tools allow for one reference to be named and used again and again. See [[WP:REFNAME]] if using [[WP:Source Editor|Source Editor]]. Personally, I would use the {{tlx|rp}} template after each of the citation, such as [citation]{{rp|9}} and [same citation]{{rp|rear cover}} Hopefully this helps. But do bear in mind that if you're basing a factual statement about, say, the author of a book by what they have written about themselves on the back cover, then that source is not deemed as 'reliable' as it's written by the subject. [[User:Nick Moyes|Nick Moyes]] ([[User talk:Nick Moyes|talk]]) 14:42, 21 January 2024 (UTC) |
|||
::Thanks for your reply, I use visual editor. If I use the <nowiki>{{rp}}</nowiki> template after the inline citations should I remove the "page cited/pages cited/at" field from the reference since its specified by the <nowiki>{{rp}}</nowiki> template? If not, then how do I add both the back cover and page number to the reference, because if I try to use both the "page cited" and "at" fields then it rejects it. [[User:Goldclock|Goldclock]] ([[User talk:Goldclock|talk]]) 15:04, 21 January 2024 (UTC) |
|||
:::@[[User:Goldclock|Goldclock]] Yes, I would remove the specific page number from within the original reference so that it simply points to the book as a whole, then I would add the respective page number/frontispiece/back cover mention to the rp template for each specific use. (Note: I have seen others here recommending the {{tlx|sfn}} template, but I find {{tlx|rp}} works fine for my needs, so I've never investigated any alternatives.) [[User:Nick Moyes|Nick Moyes]] ([[User talk:Nick Moyes|talk]]) 15:59, 21 January 2024 (UTC) |
|||
::::Thank you very much. [[User:Goldclock|Goldclock]] ([[User talk:Goldclock|talk]]) 16:57, 21 January 2024 (UTC) |
|||
Just like how we have [[:Category:Sign language family tree templates]], should we have that for political parties kind of like below but for whole countries? These would be based on the infobox political party preceded by/succeeded by/merger of/merged into. If so would these be their own article or would they be in the many "List of political parties in ___"? I am curious and want to know if these are wanted/needed. |
|||
== When is it OK to re-write a page from scratch? == |
|||
{{Tree chart/start|align=center}} |
|||
{{Tree chart| | | | | | | | | | |SPD ||SPD=[[Social Democratic Party of Germany|SPD]]}} |
|||
{{Tree chart| |,|- |- |- |- |- |- |- |-|-|(|SL=[[Spartacus League]]}} |
|||
{{Tree chart|SL| | | | | | | | |!|SL=[[Spartacus League]]}} |
|||
{{Tree chart| |!| | | | | | | | | |!|SPD=[[Social Democratic Party of Germany|SPD]]}} |
|||
{{Tree chart| KPD |v|-|-|-|-|-|-|-|(| |KPD=[[Communist Party of Germany|KPD]]}} |
|||
{{Tree chart| | | |!| | | | | | | |!| |}} |
|||
{{Tree chart| | | SED | | | | | | |!| |SED=[[Socialist Unity Party of Germany|SED]]}} |
|||
{{Tree chart| | | |!| | | | | | | |!| |}} |
|||
{{Tree chart| | | PDS |-|v|-| WAS |(|PDS=[[Party of Democratic Socialism (Germany)|PDS]]|WAS=[[Labour and Social Justice – The Electoral Alternative|WASG]]}} |
|||
{{Tree chart| | | | | | |!| | | | |!| |}} |
|||
{{Tree chart| | | | | | LIN | | | |!| |LIN=Die Linke}} |
|||
{{Tree chart| | | | | | |!| | | | |!| |}} |
|||
{{Tree chart| | | | BSW |(| | | | |!| |BSW=[[Bündnis Sahra Wagenknecht|BSW]]}} |
|||
{{Tree chart| | | | |!| |!| | | | |!| |}} |
|||
{{Tree chart/end}} [[User:History6042|<span style="color:darkorange">History6042😊</span>]] '''([[User talk:History6042|<span style="color:blue">Contact me</span>]])''' 00:39, 2 January 2025 (UTC) |
|||
:Hi [[User:History6042|History6042]], that's an interesting idea, you might want to pitch this at [[Wikipedia talk:WikiProject Politics]]. [[User:Justiyaya|'''<span style="color:#1d556d">Just</span>''']][[Special:Contributions/Justiyaya|'''<span style="color:#000000">i</span>''']][[User talk:Justiyaya#top|'''<span style="color:#6d351d">yaya</span>''']] 03:23, 2 January 2025 (UTC) |
|||
Hello... what do you do when a page is so badly written, with lots of mis-information and few proper references, that it would be much easier to start again than try and correct / edit it? Is it Ok to do that?! In this case I'm talking specifically about [[Antony Gibbs & Sons]] - I have read the key texts about the company and his family recently, so am aware of all the mistakes in the Wiki page. It's ages since I've edited anything, so can't remember the etiquett about signing off... I'm @[[User:Ruthhenrietta|Ruthhenrietta]] ;-) [[User:Ruthhenrietta|Ruthhenrietta]] ([[User talk:Ruthhenrietta|talk]]) 15:32, 21 January 2024 (UTC) |
|||
::Okay, I'll do that, thanks. [[User:History6042|<span style="color:darkorange">History6042😊</span>]] '''([[User talk:History6042|<span style="color:blue">Contact me</span>]])''' 03:27, 2 January 2025 (UTC) |
|||
== Freedom of Panorama Inquiries == |
|||
:Hi, [[User:Ruthhenrietta|Ruthhenrietta]]. Welcome to the Teahouse. Nowadays, your signature is automatically added to your post, or to any response if you click 'reply'. But if you respond the old way by clicking 'edit source' then you still need to use four tilde characters (like this: {{#ifeq: |yes|<code>~~~~</code>|{{#ifeq: |on|<code>~~~~</code>|~~~~}}}}) to insert your signature. |
|||
:Now, regarding a 100% rewrite: personally I would strongly advise against it. It would only take one error in your version for someone to completely revert what changes you'd made and take you back to square one. A better way would be to first add a note to the article's talk page to explain your concerns and highlighting major flaws, and saying what you'd like to do, and inviting any feedback or cooperation on the project. Taking all the sources and starting from the beginning, I might work on rewriting each section, one at a time. Making clear edit summaries when you save changes means others can see what you've done, and relatively small edits are easier to understand and less soul-destroying to have reverted and to re-fix than massive edits and massive reverts. |
|||
:If you insisted on doing a complete rewrite, you should prepare an alternative version in your sandbox and then link to that on the article talk page along with your concerns and seek comments and feedback. But there are probably less than 10 people 'watching' that article, which gets about ten views a day, so the likelihood of many people seeing your 'call to arms' is quite low. |
|||
:So, whilst I would encourage you to [[WP:BEBOLD]], I'd also recommend you make changes in a piecemeal fashion, rather than one quantum jump. Does that help at all? [[User:Nick Moyes|Nick Moyes]] ([[User talk:Nick Moyes|talk]]) 16:12, 21 January 2024 (UTC) |
|||
::Yes - it really helps - thank you... I'll take your advice... I realise it's a very niche subject! But it bugs me when information is so poor, so I think it's worth doing [[User:Ruthhenrietta|Ruthhenrietta]] ([[User talk:Ruthhenrietta|talk]]) 16:41, 21 January 2024 (UTC) |
|||
:::@[[User:Ruthhenrietta|Ruthhenrietta]] Yes, I've just looked at it a bit closer and it does rather cover a broad range of topics in one article, so it could well be that a rewrite in your sandbox is a good way forward. Feel free to bring anything back to the Teahouse if you want further input. |
|||
:::...And good luck! [[User:Nick Moyes|Nick Moyes]] ([[User talk:Nick Moyes|talk]]) 16:49, 21 January 2024 (UTC) |
|||
::::I will definitely get back to you if I need help - thanks [[User:Ruthhenrietta|Ruthhenrietta]] ([[User talk:Ruthhenrietta|talk]]) 17:14, 21 January 2024 (UTC) |
|||
:{{reply|Ruthhenrietta}} {{reply|Nick Moyes}} We also have the essay [[Wikipedia:Blow it up and start over]] but in my experience that typically involves a valid reason to delete the article first. ~[[User:Anachronist|Anachronist]] <small>([[User talk:Anachronist|talk]])</small> 21:11, 21 January 2024 (UTC) |
|||
This is regarding the article [[Liliget Feast House]]. The site where the restaurant once stood was torn down 2008, and I wanted to inquire regarding the best way to navigate [[Fair Use]] and [[Freedom of Panorama]] rules on Wikipedia in order to get photographs of the site on the page. This is probably best asked on Commons, but I'm not familiar with where I might be able to ask on there either. [[User:Ornithoptera|Ornithoptera]] ([[User talk:Ornithoptera|talk]]) 02:19, 2 January 2025 (UTC) |
|||
== most References == |
|||
:Hi [[User:Ornithoptera|Ornithoptera]]. I think you can take a picture of the where the restaurant once stood and upload it here, which should be fine under [[Freedom of panorama#Canada]] which also allows incidental inclusion of copyrighted, non-architectural work (also [[commons:COM:FOP Canada]]). You would still be able to release a photograph that you took of publicly viewable buildings under a free license. |
|||
is Vic Damone You Were Only Fooling the most References i ever in article i ever did? [[User:Samchristie05|Samchristie05]] ([[User talk:Samchristie05|talk]]) 17:09, 21 January 2024 (UTC) |
|||
:If you are asking about whether an inclusion of a [[WP:NFCC|non free file]] is allowed, I'm not sure but I think that it meets all the criteria seeing that the building was demolished and if a suitable free photo cannot be located. [[User:Justiyaya|'''<span style="color:#1d556d">Just</span>''']][[Special:Contributions/Justiyaya|'''<span style="color:#000000">i</span>''']][[User talk:Justiyaya#top|'''<span style="color:#6d351d">yaya</span>''']] 05:48, 2 January 2025 (UTC) |
|||
::Hi {{u|Justiyaya}}, thank you so much for your time! Truthfully I'm uncertain about using a photograph of where the restaurant once stood because it would be irrelevant to the content of the article (or at least for the intended use within the infobox). I am more asking about if I am able to use a non free file, yes, is there a venue where I can have a conversation with individuals who are more familiar with the matter? I'm not entirely sure if there is a relevant noticeboard/talk page/etc where I can bring this matter up with, if you or one of the editors here know one do let me know! [[User:Ornithoptera|Ornithoptera]] ([[User talk:Ornithoptera|talk]]) 05:52, 2 January 2025 (UTC) |
|||
:::@[[User:Ornithoptera|Ornithoptera]] I think it would be a good use of an non-free file. [[Wikipedia:Media copyright questions]] would be a relevant noticeboard for asking these types of questions. Commons won't be the place to go because they don't deal with non-free stuff. Hope this helps and happy editing! :D [[User:Justiyaya|'''<span style="color:#1d556d">Just</span>''']][[Special:Contributions/Justiyaya|'''<span style="color:#000000">i</span>''']][[User talk:Justiyaya#top|'''<span style="color:#6d351d">yaya</span>''']] 06:06, 2 January 2025 (UTC) |
|||
::::Wonderful! Thank you so much for the assistance {{u|Justiyaya}}, didn't know there was a place like that! I'll go and send them a message hopefully within the week, I hope you have a wonderful day! [[User:Ornithoptera|Ornithoptera]] ([[User talk:Ornithoptera|talk]]) 07:03, 2 January 2025 (UTC) |
|||
== Why am I blocked from doing stuff on Russian Wikipedia? == |
|||
:Hi @[[User:Samchristie05|Samchristie05]], could you please re-phrase your question so it makes sense? <span style="background-color: RoyalBlue; border-radius: 1em; padding: 3px 3px 3px 3px;">'''[[User:Qcne|<span style="color: GhostWhite">Qcne</span>]]''' <small>[[User talk:Qcne|<span style="color: GhostWhite">(talk)</span>]]</small></span> 18:33, 21 January 2024 (UTC) |
|||
::Vic Damone You Were Only Fooling References [[User:Samchristie05|Samchristie05]] ([[User talk:Samchristie05|talk]]) 18:57, 21 January 2024 (UTC) |
|||
:::That's not really a question @[[User:Samchristie05|Samchristie05]]..? <span style="background-color: RoyalBlue; border-radius: 1em; padding: 3px 3px 3px 3px;">'''[[User:Qcne|<span style="color: GhostWhite">Qcne</span>]]''' <small>[[User talk:Qcne|<span style="color: GhostWhite">(talk)</span>]]</small></span> 19:20, 21 January 2024 (UTC) |
|||
::::anyway have you heard the story of me at the beginning at website, that I acted like a reviewer approved my own article!? [[User:Samchristie05|Samchristie05]] ([[User talk:Samchristie05|talk]]) 19:41, 21 January 2024 (UTC) |
|||
:::::Again, @[[User:Samchristie05|Samchristie05]], I have no idea what you are asking. Is English perhaps not your first language? <span style="background-color: RoyalBlue; border-radius: 1em; padding: 3px 3px 3px 3px;">'''[[User:Qcne|<span style="color: GhostWhite">Qcne</span>]]''' <small>[[User talk:Qcne|<span style="color: GhostWhite">(talk)</span>]]</small></span> 19:42, 21 January 2024 (UTC) |
|||
:@[[User:Samchristie05|Samchristie05]]: Welcome to the Teahouse! I believe you're asking us whether the number of references in the article ''[[You Were Only Fooling]]'' (an album by [[Vic Damone]]) is greater than those in other articles you have created. According to [https://xtools.wmcloud.org/pages/en.wikipedia.org/Samchristie05 this report] you have created 42 articles - congratulations! You can use the report to compare number of references in each article. |
|||
:In the future, providing a link to the article you're asking about will help other editors understand your question and provide a quicker answer. Thanks, and happy editing! [[User:GoingBatty|GoingBatty]] ([[User talk:GoingBatty|talk]]) 02:46, 22 January 2024 (UTC) |
|||
::anyway have you heard the story of me at the beginning at website, that I acted like a reviewer approved my own article!? [[User:Samchristie05|Samchristie05]] ([[User talk:Samchristie05|talk]]) 02:49, 22 January 2024 (UTC) |
|||
According to my global account information I've been indefinitely blocked from doing anything other than viewing the Russian Wikipedia because I'm "using multiple accounts or having others promote me (so what I say is seen more correct)", but I'm not? I don't know anyone else with a Wikipedia account and as far as I know I only have one. It's just on the Russian Wikipedia, none of the others. I haven't really done anything on the Russian Wikipedia anyways. I've only edited my own user page. Why did this happen and how do I fix it? |
|||
== Amir Tsarfati == |
|||
The Amir Tsarfati article has not been published although he is a famous author and speaker. Can anyone shed some light on this? [[User:Aiinceku|Aiinceku]] ([[User talk:Aiinceku|talk]]) 18:38, 21 January 2024 (UTC) |
|||
More information: |
|||
:I have rejected the draft @[[User:Aiinceku|Aiinceku]], so it will not be published. <span style="background-color: RoyalBlue; border-radius: 1em; padding: 3px 3px 3px 3px;">'''[[User:Qcne|<span style="color: GhostWhite">Qcne</span>]]''' <small>[[User talk:Qcne|<span style="color: GhostWhite">(talk)</span>]]</small></span> 18:39, 21 January 2024 (UTC) |
|||
The exact stated reason was обход блокировки (bypass blocking) and it linked [https://ru.m.wikipedia.org/wiki/%D0%92%D0%B8%D0%BA%D0%B8%D0%BF%D0%B5%D0%B4%D0%B8%D1%8F:%D0%9C%D0%BD%D0%BE%D0%B3%D0%BE%D0%BB%D0%B8%D0%BA%D0%BE%D1%81%D1%82%D1%8C here]. It also gave me a link to [https://ru.m.wikipedia.org/enwiki/w/index.php?title=%D0%A1%D0%BB%D1%83%D0%B6%D0%B5%D0%B1%D0%BD%D0%B0%D1%8F:%D0%96%D1%83%D1%80%D0%BD%D0%B0%D0%BB%D1%8B/block&page=User%3A%D0%9A%D1%80%D0%B0%D1%81%D0%BD%D1%8B%D0%B9+%D0%9E%D0%BA%D1%82%D1%8F%D0%B1%D1%80%D1%8C here]. [[User:Красный Октябрь|Красный Октябрь]] ([[User talk:Красный Октябрь|talk]]) 04:25, 2 January 2025 (UTC) |
|||
::Is Qcne (talk) a bot? [[User:Aiinceku|Aiinceku]] ([[User talk:Aiinceku|talk]]) 18:41, 21 January 2024 (UTC) |
|||
:::No? <span style="background-color: RoyalBlue; border-radius: 1em; padding: 3px 3px 3px 3px;">'''[[User:Qcne|<span style="color: GhostWhite">Qcne</span>]]''' <small>[[User talk:Qcne|<span style="color: GhostWhite">(talk)</span>]]</small></span> 18:46, 21 January 2024 (UTC) |
|||
::::Thank you for your fast response. |
|||
::::You clearly are a very dedicated moderator to be responding during a weekend. |
|||
::::Would it be possible to give reason for your rejection of the Amir Tsarfati article? |
|||
::::Thank you. [[User:Aiinceku|Aiinceku]] ([[User talk:Aiinceku|talk]]) 18:51, 21 January 2024 (UTC) |
|||
:::::There are no "moderators" on Wikipedia- we are all volunteers. |
|||
:::::I rejected the draft as there is zero evidence this person meets our special definition of a notable person, which can be found at [[WP:NPEOPLE]]. |
|||
:::::I would also recommend reading [[Wikipedia:Articles_for_deletion/Amir_Tsarfati]] which sets out why the article was deleted in December. <span style="background-color: RoyalBlue; border-radius: 1em; padding: 3px 3px 3px 3px;">'''[[User:Qcne|<span style="color: GhostWhite">Qcne</span>]]''' <small>[[User talk:Qcne|<span style="color: GhostWhite">(talk)</span>]]</small></span> 18:53, 21 January 2024 (UTC) |
|||
::::::Thanks for volunteering and thanks for replying. |
|||
::::::BR [[User:Aiinceku|Aiinceku]] ([[User talk:Aiinceku|talk]]) 18:56, 21 January 2024 (UTC) |
|||
:Hi @[[User:Красный Октябрь|Красный Октябрь]], welcome to the teahouse. English and Russian Wikipedia are moderated differently, so a block there should not affect editing here nor could the community here fix anything there. From what I've been able to see with google translate and all, it appears that you've been checkuser blocked by @{{u|Q-bit array}} without email or talk page access. The administrator appears to monitor cross wiki notifications so hopefully they would respond here. They also appear to monitor [[:meta:User talk:Q-bit array]]. Usually appeals without talk page access go through [[WP:UTRS]] but I can't seem to find a similar system in ru-wiki. [[User:Justiyaya|'''<span style="color:#1d556d">Just</span>''']][[Special:Contributions/Justiyaya|'''<span style="color:#000000">i</span>''']][[User talk:Justiyaya#top|'''<span style="color:#6d351d">yaya</span>''']] 05:34, 2 January 2025 (UTC) |
|||
::{{u|Красный Октябрь}}, the English Wikipedia and the Russian Wikipedia are entirely separate projects. Each has its own policies and guidelines, and its own separate teams of administrators. Here at the Teahouse, we can do our best to help you with problems you may have on the English Wikipedia. But we have no knowledge, power nor influence over the Russian Wikipedia, which is an autonomous project. [[User:Cullen328|Cullen328]] ([[User talk:Cullen328|talk]]) 08:38, 2 January 2025 (UTC) |
|||
::Do you know where I can go on the Russian Wikipedia to get it fixed? [[User:Красный Октябрь|Красный Октябрь]] ([[User talk:Красный Октябрь|talk]]) 14:22, 2 January 2025 (UTC) |
|||
:::Hello again, @[[User:Красный Октябрь|Красный Октябрь]] I think the relevant section is [[:ru:ВП:РАЗБЛОК]]. [[User:ColinFine|ColinFine]] ([[User talk:ColinFine|talk]]) 15:24, 2 January 2025 (UTC) |
|||
::::I went through the whole section you linked and it gives me instructions to request an unblock but I can't because I can't access my talk page or edit my user page. I can't find other ways to contact the person who blocked me as well. [[User:Красный Октябрь|Красный Октябрь]] ([[User talk:Красный Октябрь|talk]]) 16:30, 2 January 2025 (UTC) |
|||
:::::If there is no off-wiki means of contacting Russian Wikipedia administrators, I think you will need to contact a steward(https://meta.wikimedia.org/wiki/Stewards) who may at least be able to advise you better than we can. [[User:331dot|331dot]] ([[User talk:331dot|talk]]) 16:42, 2 January 2025 (UTC) |
|||
::::::Thank you! [[User:Красный Октябрь|Красный Октябрь]] ([[User talk:Красный Октябрь|talk]]) 17:12, 2 January 2025 (UTC) |
|||
:::::::I messaged one of the stewards [https://meta.m.wikimedia.org/wiki/User_talk:Mykola7#%D0%9C%D0%B5%D0%BD%D0%B5_%D0%B7%D0%B0%D0%B1%D0%BB%D0%BE%D0%BA%D1%83%D0%B2%D0%B0%D0%BB%D0%BE_%D0%B2_%D1%80%D1%83%D1%81%D1%8C%D0%BA%D0%BE%D1%97_%D0%92%D1%96%D0%BA%D1%96%D0%BF%D0%B5%D0%B4%D1%96%D1%97_%D0%B7%D0%B0_%D1%82%D0%B5,_%D1%87%D0%BE%D0%B3%D0%BE_%D1%8F_%D0%BD%D0%B5_%D1%80%D0%BE%D0%B1%D0%B8%D0%BB%D0%B0:_%D0%B1%D1%83%D0%B4%D1%8C_%D0%BB%D0%B0%D1%81%D0%BA%D0%B0_%D0%B4%D0%BE%D0%BF%D0%BE%D0%BC%D0%BE%D0%B3%D1%82%D0%B8 here]. Thank you for helping me find where to ask! [[User:Красный Октябрь|Красный Октябрь]] ([[User talk:Красный Октябрь|talk]]) 17:44, 2 January 2025 (UTC) |
|||
:Responding to the "why did this happen" aspect, the most likely explanation is that your IP at the time you edited your userpage is similar to or within a range used by someone evading blocks on ru.wiki around the same time. Either that, or a more random human error on the blocking admin's part. <sub>signed, </sub>[[User:Rosguill|'''''Rosguill''''']] <sup>[[User talk:Rosguill|''talk'']]</sup> 16:50, 2 January 2025 (UTC) |
|||
== Notice of geological updates == |
|||
== Using PGP signed messages from the person the article is about as a source == |
|||
Hi, in the past few months, I was working on pages regarding the periods, ages and other units of the geological timescale, and noticed a [https://stratigraphy.org/ICSchart/ChronostratChart2024-12.pdf recent update in the ICC Chronostratigraphic Chart (2024/12)], compared to the [https://stratigraphy.org/ICSchart/ChronostratChart2023-09.pdf previous version (2023/09)], that affects the time boundaries of a lot of ages. I didn't know which talk page or portal I could specifically notify of this change, so I came here to speak about it. |
|||
Hello, I am currently considering making changes to the [[Cicada 3301|Cicada 3301 wikipedia article]]. I was planning on using direct statements made by 3301 as a source, however I have run into an issue. Every step of the puzzle and other miscellaneous statements made by 3301 have been clearsigned using [[Pretty Good Privacy|PGP]] in some way or another. tl;dr, its a way to mathematically prove that the content can only be from 3301 themselves, and the signed message content is unmodified. Due to the complex math involved in generating the RSA keypairs and signatures, it is considered practically impossible to bruteforce/forge within our lifetime, and many lifetimes after that. Upon a signature being verified in a program like GPG, you also get all sorts of information about the message such as the author, date, and time the message was created. Because of this I believe their PGP signed messages would fall in the category of direct quotations, with 3301 themselves being the source. |
|||
Now, I don't want to linger on a change like this here, since that feels like advertising which I KNOW Wikipedia is not about (see |
|||
The issue I have ran into though, is that due to the formatting of PGP signed messages(example of one [https://pastebin.com/yEiTHhvF here]) it is often easiest to upload them to pastebin for people to download and verify themselves using GPG software. 3301 did this for a few of their signed messages. I have been informed by the wonderful folks in the help IRC channel that pastebin is still not considered a reliable source regardless of document contents and verifiability, due to lack of editorial oversight etc. I was wondering if there would be a way to still cite/precedent for citing a PGP signed message confirmed to be authored by the subject of the wikipedia article, without the issues associated with using pastebin. This is may be an uncommon issue without a clear answer, but if there's one thing I've learned in the past several years it's that 3301 is ''really'' good at creating unique problems for solvers. Thank you in advance for your time and consideration responding to this post. [[User:Ctvrty|Ctvrty]] ([[User talk:Ctvrty|talk]]) 18:43, 21 January 2024 (UTC) |
|||
[[WP:SOAPBOX]]). But I know a change like this is going to require a lot of edits to catch up with, and I for sure can't do it alone. So does anyone know a good project/portal page I can discuss this more on? — [[User:Alex26337|Alex26337]] ([[User talk:Alex26337|talk]]) 05:05, 2 January 2025 (UTC) |
|||
:Hi [[User:Alex26337|Alex26337]], thank you for your question. [[Wikipedia talk:WikiProject Geology]] would be the best venue to discuss this. I don't think this is soapbox like if you are advertising on-Wiki collaboration. The policy refers to more so advertising on Wiki things off-wiki. Discussions with other community members that share the same interest is what these Wikiprojects are for :D [[User:Justiyaya|'''<span style="color:#1d556d">Just</span>''']][[Special:Contributions/Justiyaya|'''<span style="color:#000000">i</span>''']][[User talk:Justiyaya#top|'''<span style="color:#6d351d">yaya</span>''']] 05:58, 2 January 2025 (UTC) |
|||
:Hi @[[User:Ctvrty|Ctvrty]]! Taking a step back, I would be wary of [[WP:Original research]] here. Wikipedia is [[WP:TERTIARY|a tertiary source]], so the idea is that we summarize information that secondary sources (like newspapers and books) have found notable to write about. Using info directly from Cicada 3301 (a primary source) would be more appropriate for a secondary source than for us. If no newspaper has found the info sufficiently important to write about, then I question whether it's [[WP:BALASP|important enough]] for us to cover in the article. |
|||
:There are [[WP:ABOUTSELF|some exceptions]] where using primary sources is OK. For instance, we like to have birth dates for all biographies, so if the only source for 3301's birthday is a statement they made themselves over PGP, we'd want to find a way to cite the PGP message. I'm afraid I'm not techy enough to be able to give you advice on that. But that's downstream of the original research issue, which should be worked through first. Hope that's helpful! Cheers, <span style="color:#AAA"><small>{{u|</small><span style="border-radius:9em;padding:0 5px;background:#088">[[User:Sdkb|<span style="color:#FFF">'''Sdkb'''</span>]]</span><small>}}</small></span> <sup>[[User talk:Sdkb|'''talk''']]</sup> 21:33, 21 January 2024 (UTC) |
|||
::Ah, I was afraid that would be an issue. Thank you for the thorough reply. |
|||
::Re:secondary sources, one of our community members suggested that recordings of talks at DEFCON conferences/presentation slides from the talks when a/v recording isn't available(the files are usually hosted by DEFCON or DEFCON Villages, so you know for sure that the slides are legit) that cover the history of 3301 could be an acceptable secondary source. There are currently 3 separate talks that were either approved by DEFCON as a main track talk, or by the Crypto & Privacy Village at DEFCON in previous years that cover the history of 3301 and cultural impact to varying degrees. DEFCON is a world-renowned hacking and cybersecurity conference hosted every year(except when its cancelled), with both the DEFCON main track and Crypto & Privacy Village talks being subject to review and revision before being accepted. Would these be acceptable as reliable, secondary sources? [[User:Ctvrty|Ctvrty]] ([[User talk:Ctvrty|talk]]) 22:49, 21 January 2024 (UTC) |
|||
:::I'd say conference proceedings <em>can</em> be acceptable sources, but it depends on who gave the talk, the extent to which the proceedings were published, and other factors. Editors more familiar with the subject area would be better positioned to answer than me. Feel free to try using them as sources and see how others watching the article react to it. If you're confident that the information belongs in the article, then including it with some source is always better than doing so with no source. Cheers, <span style="color:#AAA"><small>{{u|</small><span style="border-radius:9em;padding:0 5px;background:#088">[[User:Sdkb|<span style="color:#FFF">'''Sdkb'''</span>]]</span><small>}}</small></span> <sup>[[User talk:Sdkb|'''talk''']]</sup> 00:03, 22 January 2024 (UTC) |
|||
::::Awesome! I figured it might be a bit nuanced, but I'll still give it a try. Thank you so much for your help! [[User:Ctvrty|Ctvrty]] ([[User talk:Ctvrty|talk]]) 00:45, 22 January 2024 (UTC) |
|||
== Request to Update Image == |
|||
==Need help with my drafts== |
|||
Hi,I have made 3 pages that already exist in the Turkish Wiki so these are translations but they are quite different from the original ([[Draft:Erenköy, Kadıköy]], [[Draft:Göztepe Park]], [[Draft:Caddebostan, Kadıköy]]). I am slowly improving them and the best one out of the three is the Caddebostan article with over 20 sources. Is it possible for experienced editors to give me advice on these drafts and how can I improve them? It would be an honor for me to get an article published. Thank you. [[User:Youprayteas|Youprayteas]] ([[User talk:Youprayteas|talk]]) 19:22, 21 January 2024 (UTC)youprayteas |
|||
I noticed that the current image used at [[Dadvan Yousuf]] is from 2021, but there are more recent images from 2024 available on Wikimedia Commons. |
|||
:@[[User:Youprayteas|Youprayteas]], I went through the drafts and made various cleanups. I'd suggest looking in the [[Help:HISTINTRO|revision history]] to see what I've done. The biggest issue is establishing the notability of the neighborhoods ([[WP:POPULATED]]), which needs to be done through in-depth sourcing. The quality of sources is what matters, not the quantity. [[WP:NONENGLISH|It's okay]] if these sources are not in English, but we're looking for media coverage (or, even better, scholarly books). The other thing I'd pay attention to is [[WP:NOTTRAVELGUIDE]]. We don't want a [[Wikivoyage]] travel guide, but rather an encyclopedia article, and things like a destination list or overly detailed description of which bus lines run through a neighborhood are questionable. Hope that helps! Cheers, <span style="color:#AAA"><small>{{u|</small><span style="border-radius:9em;padding:0 5px;background:#088">[[User:Sdkb|<span style="color:#FFF">'''Sdkb'''</span>]]</span><small>}}</small></span> <sup>[[User talk:Sdkb|'''talk''']]</sup> 21:26, 21 January 2024 (UTC) |
|||
::I established the notability of Caddebostan and Erenköy through the sources. I added the bus lines because it was also on the Turkish version. [[User:Youprayteas|Youprayteas]] ([[User talk:Youprayteas|talk]]) 21:32, 21 January 2024 (UTC) |
|||
:::@[[User:Youprayteas|Youprayteas]], editorial standards differ between different language editions of Wikipedia. The fact that the neighborhoods are considered notable for Turkish Wikipedia doesn't necessarily mean they will be here. Likewise, the bus lines being appropriate to add there doesn't mean they necessarily will be here. I would look to [[Wikipedia:Featured_articles#Cities, towns and villages|high-quality examples]] of articles about places to see what we're aiming toward. Cheers, <span style="color:#AAA"><small>{{u|</small><span style="border-radius:9em;padding:0 5px;background:#088">[[User:Sdkb|<span style="color:#FFF">'''Sdkb'''</span>]]</span><small>}}</small></span> <sup>[[User talk:Sdkb|'''talk''']]</sup> 21:38, 21 January 2024 (UTC) |
|||
::::PS: shouldn’t all legally declared neighborhoods have an article, no matter how small? Less is better than none, I presume. [[User:Youprayteas|Youprayteas]] ([[User talk:Youprayteas|talk]]) 21:41, 21 January 2024 (UTC) |
|||
:::::Not necessarily. We are pretty strict about [[WP:Notability]] here compared to other language editions of Wikipedia. <span style="color:#AAA"><small>{{u|</small><span style="border-radius:9em;padding:0 5px;background:#088">[[User:Sdkb|<span style="color:#FFF">'''Sdkb'''</span>]]</span><small>}}</small></span> <sup>[[User talk:Sdkb|'''talk''']]</sup> 21:48, 21 January 2024 (UTC) |
|||
::I just read the changes and thank you for your help. I would like to ask you, do you think my Caddebostan article is ready for being an article? I have multiple sources declaring Caddebostan is a neighborhood. The other ones are work in progresses. [[User:Youprayteas|Youprayteas]] ([[User talk:Youprayteas|talk]]) 21:38, 21 January 2024 (UTC) |
|||
:::@[[User:Youprayteas|Youprayteas]], that's for the reviewer to say, not me. Looking at the 20 or so sources in the article, which [[WP:THREE|three]] do you consider to be the strongest? Are any of them from media outlets/publishers that have an article? <span style="color:#AAA"><small>{{u|</small><span style="border-radius:9em;padding:0 5px;background:#088">[[User:Sdkb|<span style="color:#FFF">'''Sdkb'''</span>]]</span><small>}}</small></span> <sup>[[User talk:Sdkb|'''talk''']]</sup> 21:49, 21 January 2024 (UTC) |
|||
::::The first source is used in literally every neighborhood article in Turkey so I would definetly include that. The second source is for the km2. The third source is for the population and the list of neighborhoods in Kadıköy. The 8th and 9th sources are for the history. The 12nd source is for the Barlar Street which isn't mentioned on the other sources. The 14th article is probably the most inclusive for destinations in Caddebostan. 19th source is needed for the mansion being sold. The 21st (last, for now) article is for the Göztepe Park. So in summary these 9 sources are enough. I wanted there to be as much references as possible because I really want this article to get published, since Caddebostan is a very notable neighborhood. [[User:Youprayteas|Youprayteas]] ([[User talk:Youprayteas|talk]]) 22:06, 21 January 2024 (UTC) |
|||
:::::[[User:Youprayteas|Youprayteas]], you say "I wanted there to be as much references as possible"; but {{U|Sdkb}} has already commented on this: "The quality of sources is what matters, not the quantity." Sdkb asked you "Looking at the 20 or so sources in the article, which [[WP:THREE|three]] do you consider to be the strongest? Are any of them from media outlets/publishers that have an article?" Please answer these questions. -- [[User:Hoary|Hoary]] ([[User talk:Hoary|talk]]) 23:53, 21 January 2024 (UTC) |
|||
::::::It is impossible to choose 3 sources. I need at least 9-10 sources or the information will be unsourced. [[User:Youprayteas|Youprayteas]] ([[User talk:Youprayteas|talk]]) 06:08, 22 January 2024 (UTC) |
|||
:::::::We're not asking you to remove the other sources from the article. But establishing [[WP:NOTABILITY|notability]] is separate from ensuring information in the article is sourced. We want sources that establish notability. <span style="color:#AAA"><small>{{u|</small><span style="border-radius:9em;padding:0 5px;background:#088">[[User:Sdkb|<span style="color:#FFF">'''Sdkb'''</span>]]</span><small>}}</small></span> <sup>[[User talk:Sdkb|'''talk''']]</sup> 06:17, 22 January 2024 (UTC) |
|||
::::::::My sources establish notability though. [[User:Youprayteas|Youprayteas]] ([[User talk:Youprayteas|talk]]) 06:40, 22 January 2024 (UTC) |
|||
::::::to reply to your second comment. I added five sources legally recognizing Caddebostan's neighborhood-ity. The first source is used in literally all subdivisions of Turkey and it is a database where you can search for legal divisions. I thought it as enough. but everyone kept saying we have no proof this is a real neighborhood even though it is and you can see from the references so I added 5 references just to prove that Caddebostan exists. I think it is unnecesarry too but eh. [[User:Youprayteas|Youprayteas]] ([[User talk:Youprayteas|talk]]) 06:13, 22 January 2024 (UTC) |
|||
:::::::No one is doubting that the neighborhood is real. But again, that's separate from establishing [[WP:NOTABILITY|notability]]. <span style="color:#AAA"><small>{{u|</small><span style="border-radius:9em;padding:0 5px;background:#088">[[User:Sdkb|<span style="color:#FFF">'''Sdkb'''</span>]]</span><small>}}</small></span> <sup>[[User talk:Sdkb|'''talk''']]</sup> 06:18, 22 January 2024 (UTC) |
|||
::::::::Caddebostan and Erenköy are notable enough for Wikipedia. They are redlinked in some places too, as I checked. I listed many attractions about the neighborhood and overall it is notable because there are plenty of sources about it too. [[User:Youprayteas|Youprayteas]] ([[User talk:Youprayteas|talk]]) 06:43, 22 January 2024 (UTC) |
|||
::::::::The Erenköy article is a work in progress though so don't review that one but I think [[Draft:Caddebostan, Kadıköy]] has importance regarding Wikipedia and is notable enough [[User:Youprayteas|Youprayteas]] ([[User talk:Youprayteas|talk]]) 06:45, 22 January 2024 (UTC) |
|||
::::::To further add to your second comment, the population of Caddebostan is over 21 thousand. There are many countries below this population, for example. I am not mentioning Erenköy because it is a work in progress, but it has over 30k which is incredibly a lot. [[User:Youprayteas|Youprayteas]] ([[User talk:Youprayteas|talk]]) 06:15, 22 January 2024 (UTC) |
|||
::::::@[[User:Youprayteas|Youprayteas]] ''Vis a vis'' notability of "legally declared neighborhoods", [[WP:Notability (geographic features)]] addresses this. (Well, I"m not even sure that we generally have "legally declared neighborhoods" in the U.S. Typically, we have plat books which may name subdivisions and the like, but I'm doubtful that meets this criterion. <!-- Template:Unsigned --><small class="autosigned">— Preceding [[Wikipedia:Signatures|unsigned]] comment added by [[User:Fabrickator|Fabrickator]] ([[User talk:Fabrickator#top|talk]] • [[Special:Contributions/Fabrickator|contribs]]) 19:12, 21 January 2024 (UTC)</small> |
|||
I kindly request an extended-confirmed user to review and consider updating the image to |
|||
== George Van Driem's Source for Maghrebi Mint Tea History == |
|||
"Dadvan Yousuf with Kurdistan Flag.jpg", Image is from recent - i dont recommend the images with the bitcoin flag. |
|||
Thanks.. [[User:Ayohama|Ayohama]] ([[User talk:Ayohama|talk]]) 07:44, 2 January 2025 (UTC) |
|||
Hi there! I'm here because my edit on Maghrebi mint tea got undone. The reason given by the user was that the source's author, George Van Driem, is a linguist, not a historian. However, considering George's specialization in historical linguistics and his book "The Tale of Tea: A Comprehensive History of Tea from Prehistoric Times to the Present Day" delving into tea's history from prehistoric times, should his source still be excluded from the history section? Thanks! [[User:MoroccanTeaEnjoyer|MoroccanTeaEnjoyer]] ([[User talk:MoroccanTeaEnjoyer|talk]]) 19:54, 21 January 2024 (UTC) |
|||
:{{u|MoroccanTeaEnjoyer}}, since the book has received positive reviews, such as [https://www.jstor.org/stable/26916454 this one in an academic journal], and I have found no negative reviews, I think the book should be considered a reliable source. Many academics branch out from their original field of study and do excellent work in related fields. The best place for a source analysis, though, is [[Wikipedia:Reliable sources/Noticeboard]]. [[User:Cullen328|Cullen328]] ([[User talk:Cullen328|talk]]) 21:13, 21 January 2024 (UTC) |
|||
::Thank you very much, Cullen. Your answer was helpful. [[User:MoroccanTeaEnjoyer|MoroccanTeaEnjoyer]] ([[User talk:MoroccanTeaEnjoyer|talk]]) 21:34, 21 January 2024 (UTC) |
|||
:Hi [[User:Ayohama|Ayohama]], welcome to the teahouse! Although the image [[:File:Dadvan Yousuf with Kurdistan Flag.jpg]] is more recent, I don't think it provides the same quality and clarity as the image currently in the article. I don't think this change should be made. [[User:Justiyaya|'''<span style="color:#1d556d">Just</span>''']][[Special:Contributions/Justiyaya|'''<span style="color:#000000">i</span>''']][[User talk:Justiyaya#top|'''<span style="color:#6d351d">yaya</span>''']] 07:53, 2 January 2025 (UTC) |
|||
== Adding "Democratic School" and editing "Democratic Education" accordingly == |
|||
::Hey, thanks for the solid feedback! fully understood [[User:Ayohama|Ayohama]] ([[User talk:Ayohama|talk]]) 07:54, 2 January 2025 (UTC) |
|||
:::{{ping|Ayohama}} I'd hold off in trying to use [[:File:Dadvan Yousuf with Kurdistan Flag.jpg]] or any of the other images of Yousuf uploaded to Commons by [[:c:User talk:Lonely34|Lonely34]] until the claims of "[[:c:COM:Own work|own work]]" can be verified by [[:c:COM:VRT]]. All the images that were uploaded by Lonely34 can be found being used online on sites like X, Facebook, Instagram, etc, from May 2024 onward prior to their being uploaded to Commons, some even with [[:watermarks]]. This doesn't necessarily mean they weren't taken by Lonely34, but the general licensing of those sites is too restrictive for Commons per [[:c:COM:L]] and in such cases [[c:COM:CONSENT|copyright holder consent]] needs to be verified. If the licensing of any or all of those images ends up being verified, you can add them to the body of the Wikipedia article about Yousuf if you want but probably should propose changing the infobox image on the article's talk page to see what others think. -- [[User:Marchjuly|Marchjuly]] ([[User talk:Marchjuly|talk]]) 08:30, 2 January 2025 (UTC) |
|||
::::{{u|Ayohama}}, even if the issues that {{u|Marchjuly}} describes can be ironed out, I would oppose the use of the image that you are proposing for the infobox. That is a mountaineering summit style photo, and this person is not a notable mountaineer even if he paid a lot of money to climb the standard tourist route on Mount Everest with the help of Sherpa guides. He is a cryptocurrency investor not a serious mountaineer and it is not appropriate to portray business people with golf clubs or tennis rackets in their hands. Or flags on mountain tops. This is not 1952 and rich people climbing Everest is not a notable accomplishment these days. [[User:Cullen328|Cullen328]] ([[User talk:Cullen328|talk]]) 08:56, 2 January 2025 (UTC) |
|||
:::::One of the references used to verify the claim about him climbing Everest says (in English translation) {{tpq|I had never climbed a mountain before, Everest was my first}}. This is not indicative of a serious mountaineer but rather a rich person willing and able to pay actual Sherpa mountaineers to help him get to the top for publicity purposes. [[User:Cullen328|Cullen328]] ([[User talk:Cullen328|talk]]) 09:07, 2 January 2025 (UTC) |
|||
::::::Although it doesnt seem to be that easy for me to climb everest as it seems for you. I think its still very dangerous and a notable achievement [[User:Ayohama|Ayohama]] ([[User talk:Ayohama|talk]]) 09:21, 2 January 2025 (UTC) |
|||
:::::Agree with you [[User:Ayohama|Ayohama]] ([[User talk:Ayohama|talk]]) 09:20, 2 January 2025 (UTC) |
|||
::::Thanks for the hint, i thought the Metadata is correct [[User:Ayohama|Ayohama]] ([[User talk:Ayohama|talk]]) 09:27, 2 January 2025 (UTC) |
|||
:::::{{ping|Ayohama}} The uploader or someone else has emailed Commons VRT regarding these files and is currently being processed by VRT. Assuming things check out, the [[:c:Template:Permission received]] added to each file's page will be changed to [[:c:Template:PermissionTicket]] by the VRT member reviewing the email. Once this happens, the files will be fine from a copyright standpoint; so, in principle, they're OK to be used. From an encyclopedic standpoint, though, things might still need to be sorted out on the article's talk page, particularly if others feel (like expressed above) that they shouldn't be used in the main infobox. One representative image in the body of the article should, however, be OK. -- [[User:Marchjuly|Marchjuly]] ([[User talk:Marchjuly|talk]]) 10:07, 2 January 2025 (UTC) |
|||
::::::Was thinking after our conversations, the infobox is not the right place there. MAYBE at Personal life, where context is about Everest is fine. Max [[User:Ayohama|Ayohama]] ([[User talk:Ayohama|talk]]) 10:10, 2 January 2025 (UTC) |
|||
:::::::According to [[List of Mount Everest summiters by frequency]], more than 6,000 people have summited Everest (as of 2022), so agree that if any placement at all, then at Personal life. [[User:David notMD|David notMD]] ([[User talk:David notMD|talk]]) 11:14, 2 January 2025 (UTC) |
|||
:::::::::And the record for a non-Sherpa (actually an Englishman) is 18 (exceeded by 9 individual Sherpas). {The poster formerly known as 87.81.230.195} [[Special:Contributions/94.6.84.253|94.6.84.253]] ([[User talk:94.6.84.253|talk]]) 05:40, 3 January 2025 (UTC) |
|||
::::::::[[User:Lonely34|Lonely34]] claims of "own work" are now verified by [[commons:Commons:Volunteer_Response_Team|c:COM:VRT]]. If you want, you can go ahead and place the [[:File:Dadvan Yousuf with Kurdistan Flag.jpg]] under "Personal Life". I guess the article itself is interesting to work on in the future- the tone is sometimes too promotional and the text layout weird- Reads like a chronology rather then a WP Article. [[User:Ayohama|Ayohama]] ([[User talk:Ayohama|talk]]) 13:39, 2 January 2025 (UTC) |
|||
== Help with improving references and notability for Eugene Vollmer draft == |
|||
Hi Everyone, |
|||
Hello! I submitted a draft for [[Draft:Eugene Vollmer|Eugene Vollmer]] but it was declined because the references didn't show significant coverage. Could anyone assist me in finding reliable sources or improving my draft to meet Wikipedia's notability guidelines? I appreciate any guidance! [[User:Sammy Tremlinn|Sammy Tremlinn]] ([[User talk:Sammy Tremlinn|talk]]) 08:45, 2 January 2025 (UTC) |
|||
I'd like to improve the article [[democratic education]] and have read in the Talk article discussion that the article should be split into Democratic School and Democratic Education, with a new definition for the latter. I wrote a draft for the new "Democratic School" article here in my Sandbox ( https://en.wikipedia.org/wiki/User:Altiflash/sandbox ) and also recommended a few changes for Democratic Education. However, I wouldn't want to delete the parts from "Democratic Education" that I adopted into the Democratic School article until the "Democratic School" article has been reviewed and approved. If I publish "Democratic School" now, though, there may be criticism that the topic is already covered by "Democratic Education". What do you recommend I do? |
|||
:{{u|Sammy Tremlinn}}, your draft fails to make the case that Vollmer is a truly notable athlete, although it seems that he was once highly ranked in an amateur track and field event in a country of less than a million residents. At first glance, that does not seem to be a plausible claim of notability to me. But I could be wrong. In the end, it is all about whether or not multiple indisputably reliable sources independent of Vollmer have devoted significant coverage to him. Your current sources are not independent of Vollmer and therefore do not meet that threshold. [[User:Cullen328|Cullen328]] ([[User talk:Cullen328|talk]]) 09:25, 2 January 2025 (UTC) |
|||
::Thank you for the feedback, Cullen328. I understand the importance of independent and significant coverage for establishing notability. I will work on finding additional sources that meet these criteria and provide substantial information about Eugene Vollmer's achievements. If you have any suggestions for where I might look or types of sources to prioritize, I’d greatly appreciate it! [[User:Sammy Tremlinn|Sammy Tremlinn]] ([[User talk:Sammy Tremlinn|talk]]) 10:01, 2 January 2025 (UTC) |
|||
:::[[List of newspapers in Fiji]] might be a starting point: many of those entities are likely to be [[Wikipedia:Reliable sources|Reliable sources]]. {The poster formerly known as 87.81.230.195} [[Special:Contributions/94.6.84.253|94.6.84.253]] ([[User talk:94.6.84.253|talk]]) 05:48, 3 January 2025 (UTC) |
|||
== New page == |
|||
<nowiki>https://en.wikipedia.org/wiki/Wikipedia:Teahouse/About</nowiki> [[User:Altiflash|Altiflash]] ([[User talk:Altiflash|talk]]) 20:34, 21 January 2024 (UTC) |
|||
:{{reply|Altiflash}} What I suggest you do is create the new article, using only material you pull out from the original article. Then, add any additional material. ~[[User:Anachronist|Anachronist]] <small>([[User talk:Anachronist|talk]])</small> 20:52, 21 January 2024 (UTC) |
|||
Hello, I was thinking of making a new page about the New Zealand retail company PB Technologies, but I think I'll be going in too far over my head if I start it myself, so I was wondering where I could propose the Idea so that I could join forces with a more experienced Wikipedian to make the page a reality. [[User:Sup3rG33k08|Sup3rG33k08]] ([[User talk:Sup3rG33k08|talk]]) 08:51, 2 January 2025 (UTC) |
|||
== photograph on the Wikipedia page for Kathy Ellis == |
|||
:Hello. You can go to [[WP:RA|Requested articles]], but it is backlogged to the point of uselessness, any request you make there will likely not be acted on for some time, if ever. You are wise to be cautious in attempting to create a new '''article''', but if you want to see one created, it likely won't happen unless you do it yourself. You can use the [[WP:WIZARD|Article Wizard]] to create and submit a draft for review by more experienced editors. You would first need to gather independent [[WP:RS|reliable sources]] to summarize in an article, and review the [[WP:ORG|definition of a notable business]] to see if this business meets it(most do not). |
|||
The photograph on the Wikipedia page for Kathy Ellis, swimmer, is incorrect, the photo is of Donna deVarona, not Kathy Ellis. [[Special:Contributions/2601:805:C100:ADA0:156A:FB3C:E694:E76B|2601:805:C100:ADA0:156A:FB3C:E694:E76B]] ([[User talk:2601:805:C100:ADA0:156A:FB3C:E694:E76B|talk]]) 22:06, 21 January 2024 (UTC) |
|||
:If you are associated with this business, that needs to be disclosed, see [[WP:COI]] and [[WP:PAID]]. [[User:331dot|331dot]] ([[User talk:331dot|talk]]) 10:05, 2 January 2025 (UTC) |
|||
::Articles about companies need to comply with [[WP:NCORP]] and call for attention to being neutral point of view, not in any way promotional. Teahouse Hosts are here to advise, not to be coauthors. [[User:David notMD|David notMD]] ([[User talk:David notMD|talk]]) 12:09, 2 January 2025 (UTC) |
|||
== Adding unnumbered tracks to an upcoming album == |
|||
:Hello, I looked at the [https://digital.library.ucla.edu/catalog/ark:/21198/zz0002sq35 original image], and it seems like it is Kathy Ellis. The image on Wikipedia is just a cropped version of the image. - <span style="font-family: Monospace;">Dents</span> ([[User talk:Dentsinhere43|talk2me 🖂]]) he/him btw!!! 23:19, 21 January 2024 (UTC) |
|||
::I think the poster is correct. The original says "Caption ...Donna deVarona, left, individual medley, and Cathy Ellis, freestyle, pose prettily." But the displayed photo is mirrored, for example seen by a mirrored "EXIT" in the right side. I guess the caption was made for the correct orientation. Compare also to other photos of [https://oldlifemagazine.com/october-09-1964-life-magazine.html Donna deVarona] and [https://ishof.org/happy-birthday-kathy-ellis/ Kathy Ellis]. [[User:PrimeHunter|PrimeHunter]] ([[User talk:PrimeHunter|talk]]) 00:48, 22 January 2024 (UTC) |
|||
:::Thanks for bringing this to our attention, and to {{U|PrimeHunter}} for the further research. I have removed the photo from the [[Kathy Ellis]] article, citing this conversation in the edit summary. Pinging {{U|Holly Cheng}}, who uploaded [[:File:Donna de Varona and Kathy Ellis.jpg]] and extracted [[:File:Kathy Ellis, 1964.jpg]]. [[User:GoingBatty|GoingBatty]] ([[User talk:GoingBatty|talk]]) 02:09, 22 January 2024 (UTC) |
|||
::::This happens occasionally with photos in the UCLA archive. I'll fix it. <span style="font-family:Verdana; ">—'''[[User:Holly Cheng|<span style="color:#33C;">holly</span>]]''' <small>{[[User talk:Holly Cheng|chat]]}</small></span> 02:26, 22 January 2024 (UTC) |
|||
:::::Hmm, the tool server seems to be down, so I can't redo the Kathy Ellis pic via CropTool, but I'll get it when it's available again. <span style="font-family:Verdana; ">—'''[[User:Holly Cheng|<span style="color:#33C;">holly</span>]]''' <small>{[[User talk:Holly Cheng|chat]]}</small></span> 17:20, 22 January 2024 (UTC) |
|||
:::Good detectiving! [[User:Gråbergs Gråa Sång|Gråbergs Gråa Sång]] ([[User talk:Gråbergs Gråa Sång|talk]]) 09:35, 22 January 2024 (UTC) |
|||
Hello, |
|||
== Standards for meeting validity bar on new pages == |
|||
This artist has been teasing some track titles (and snippets) from their upcoming album, [[So Close to What|''So Close To What'']]. Some titles are speculated, some were directly teased and are confirmed. How would I go about adding those tracks, assuming I don't know their numbers? The album has a Spotify countdown page, so the two pre-released singles have already been added in track list format. [[User:Lashyurn|Lashyurn]] ([[User talk:Lashyurn|talk]]) 10:39, 2 January 2025 (UTC) |
|||
Hi, I've created two pages that I though had enough third party references to meet the criteria for a page on wikipedia. https://en.wikipedia.org/wiki/Draft:DFJ_Growth and https://en.wikipedia.org/wiki/Draft:Randy_Glein. I'm not being paid, but doing to help DFJ (I have done some paid work for some of the companies in their portfolio) and to learn more about the Wikipedia ecosystem. How much more third party validity to these pages need or is there something else? they are both factually accurate, Randy Glein is a well known and important person in the venture community and DFJ has evolved into DFJ Growth and Threshhold Ventures (you can see this on DFJ.com). thank you! [[User:Scottfasser|Scottfasser]] ([[User talk:Scottfasser|talk]]) 00:30, 22 January 2024 (UTC) |
|||
:[[User:Lashyurn|Lashyurn]], I read that this album "scheduled to be released on February 21, 2025". I suggest that you simply wait until February 21, 2025. -- [[User:Hoary|Hoary]] ([[User talk:Hoary|talk]]) 12:24, 2 January 2025 (UTC) |
|||
::Oh, that's fine. I was just following what I saw was done with SZA's ''[[Lana (album)|LANA]]'' 10 days prior to its release. Thank you! :) [[User:Lashyurn|Lashyurn]] ([[User talk:Lashyurn|talk]]) 12:32, 2 January 2025 (UTC) |
|||
:::Hi [[User:Lashyurn|Lashyurn]], in that article, [[Special:Permalink/1263529594#Track listing|there were reliable sources]] that reported on the teasers for the singles. If that is the case for this album, I think a mention in the Promotion section could be nice. Things that are not being reported on by reliable, independent sources or are pure speculation are usually not included here, especially if they are self published or original research. Feel free to drop sources here for us to take a look :D [[User:Justiyaya|'''<span style="color:#1d556d">Just</span>''']][[Special:Contributions/Justiyaya|'''<span style="color:#000000">i</span>''']][[User talk:Justiyaya#top|'''<span style="color:#6d351d">yaya</span>''']] 12:45, 2 January 2025 (UTC) |
|||
::::These are my sources (and titles): |
|||
::::"Dear god" - Played at listening party and posted snippets on TikTok |
|||
::::"Revolving door" - Played at listening party and posted snippets on TikTok |
|||
::::"Moments" - Title posted on her management's Instagram as a collaboration with Spotify on December 13, 2024 |
|||
::::"Greenlight" - Played at listening party and title shown in album trailer |
|||
::::"Star child" - Title shown at the beginning of the "[[It's OK I'm OK|It's ok I'm ok]]" music video |
|||
::::Would these sources be considered reliable? I have links/pictures/videos to each of those [[User:Lashyurn|Lashyurn]] ([[User talk:Lashyurn|talk]]) 13:03, 2 January 2025 (UTC) |
|||
:::::Unfortunately none of those are secondary sources, @[[User:Lashyurn|Lashyurn]]. What we're looking for is mainstream music journalists who are reporting on the album. <span style="background-color: RoyalBlue; border-radius: 1em; padding: 3px 3px 3px 3px;">'''[[User:Qcne|<span style="color: GhostWhite">qcne</span>]]''' <small>[[User talk:Qcne|<span style="color: GhostWhite">(talk)</span>]]</small></span> 17:35, 2 January 2025 (UTC) |
|||
::::::Oh, I thought I could take them from the artist themselves. Thank you! I'll wait until release. [[User:Lashyurn|Lashyurn]] ([[User talk:Lashyurn|talk]]) 18:00, 2 January 2025 (UTC) |
|||
:::::::No, sources from the person that the article is about are considered [[WP:PRIMARY|primary sources]]. <span style="font-family:Arial;background-color:#fff;border:2px dashed#69c73e">[[User:Cowboygilbert|<span style="color:#3f6b39">'''Cowboygilbert'''</span>]] - [[User talk:Cowboygilbert|<span style="color:#d12667"> (talk) ♥</span>]]</span> 22:02, 2 January 2025 (UTC) |
|||
:I suggest to wait till release too [[User:Ayohama|Ayohama]] ([[User talk:Ayohama|talk]]) 13:33, 2 January 2025 (UTC) |
|||
== English Version of the German Article Judith Kakon == |
|||
:@[[User:Scottfasser|Scottfasser]] The issue is not factual accuracy (that's at [[WP:V]]), it's ''notability'' ([[WP:N]]). You need to show that these are notable people/things in the way that Wikipedia defines notability ("he is well known in the venture community" is a good indicator that someone ''might'' be notable, but that's all). You need to show that your topics have significant coverage in multiple independent, reliable sources. -- [[User:Asilvering|asilvering]] ([[User talk:Asilvering|talk]]) 00:45, 22 January 2024 (UTC) |
|||
Hello there, I just want to create the same article in english, I already created this article in german, all I want is to have a translated version. Could you please help me? My request was already denied twice. |
|||
== Hey Guys! == |
|||
This is the german article: Judith Kakon. [[User:Avaa Malula|Avaa Malula]] ([[User talk:Avaa Malula|talk]]) 14:25, 2 January 2025 (UTC) |
|||
:@[[User:Avaa Malula|Avaa Malula]] It appears as if German Wikipedia has the same concerns as English Wikipedia; that the article does not sufficiently prove its subject's [[WP:NOTABILITY|notability]]. Please read [[WP:YFA]] and [[WP:42]]. [[User:CommissarDoggo|<b style="font-family:Helvetica Neue;color:#fc1008">Commissar</b><b style="font-family:Helvetica Neue;color:#0363ff">Doggo</b>]]''[[User talk:CommissarDoggo|<sup style="font-family:Helvetica Neue;color:#0363ff">Talk?</sup>]]'' 14:30, 2 January 2025 (UTC) |
|||
Toothy Was Voiced By Dean MacDonald In Banjo Frenzy (Happy Tree Friends) [[User:Helpmechoosehappytf|Helpmechoosehappytf]] ([[User talk:Helpmechoosehappytf|talk]]) 01:34, 22 January 2024 (UTC) |
|||
: |
::[[Draft:Judith Kakon]] has no references. The content that is now under a description of her approach to art (my wording) comes across as either your or her description. All of this should be deleted unless it can be referenced to published descriptions of her and her work. [[User:David notMD|David notMD]] ([[User talk:David notMD|talk]]) 14:50, 2 January 2025 (UTC) |
||
:Hey @[[User:Avaa Malula|Avaa Malula]] |
|||
:It’s great that you’re interested in contributing to the English Wikipedia by translating an article. However, there are a few things to consider to ensure the article meets the standards required on the English Wikipedia: |
|||
:1. Improving the Original Article: The [[:de:Judith_Kakon|German article]] you created currently has notices indicating issues with missing citations and important information about the subject’s works. These issues suggest that the article may not yet meet Wikipedia's notability and verifiability standards. Before translating the article, it’s a good idea to improve it by: |
|||
:- Adding reliable and independent sources that demonstrate Judith Kakon's notability. |
|||
:- Including details about her important works or achievements. |
|||
:2. Meeting English Wikipedia Standards: The English Wikipedia has specific requirements for biographies. It’s important to: |
|||
:- Demonstrate notability according to the General Notability Guideline ([[Wikipedia:Notability|WP:GNG]]). |
|||
:- Provide inline citations from reliable sources to back up claims. |
|||
:- Write in a neutral tone without promotional language. |
|||
:3. Translation Guidelines: When translating articles, you must include a note on the article’s talk page to credit the original German version. See the [[Wikipedia:Translation|Wikipedia:Translation guidelines]] for details on how to do this properly. |
|||
:4. Steps to Resubmit: Once you’ve improved the content and addressed the above points: |
|||
:- Draft the translated article in your sandbox. |
|||
:- Add sources that meet English Wikipedia’s standards. |
|||
:- Submit it for review through the Articles for Creation (AfC) process. |
|||
:If you need help improving the article or drafting it in English, feel free to ask here. [[User:Ayohama|Ayohama]] ([[User talk:Ayohama|talk]]) 07:53, 3 January 2025 (UTC) |
|||
== Last aired date in the future == |
|||
For the curious: [[Happy Tree Friends]] mentions Toothy (a cartoon beaver) voiced by Warren Graff. The History section does mention ''Banjo Frenzy'' as in effect a pilot episode in which Toothy and two others are killed by being hit with a banjo. Characters are maimed/killed in every episode, but appear unharmed in subsequent episodes (to again be maimed/killed). Dean MacDonald is not named as a voice provider for any other character. [[User:David notMD|David notMD]] ([[User talk:David notMD|talk]]) 11:46, 22 January 2024 (UTC) |
|||
This article revison [[Special:Permalink/1266730608|SAS: Rogue Heroes revison of 02:04, 2 January 2025]] has a last aired date for series 2 which is in the future. What is probably meant is that the last episode is scheduled for broadcast on that date, but as it was worded it is factually incorrect. I've seen this on other such articles in the past. Has there been any discussions about wether this is acceptible or not? If so, can you direct me to any official decisions? [[User:Dubidubno|Dubidub]] ([[User talk:Dubidubno|talk]]) 14:49, 2 January 2025 (UTC) |
|||
== Edits not showing up == |
|||
:See [[Wikipedia:What Wikipedia is not|WP:FUTURE]] - Specifically, see the section NOTCRYSTALBALL, which states that Wikipedia is not a crystal ball. Articles should not speculate on future events unless they are verifiable and sourced. The correct phrasing should reflect that the episode is "scheduled to air" on that date to maintain accuracy and neutrality. [[User:Ayohama|Ayohama]] ([[User talk:Ayohama|talk]]) 07:45, 3 January 2025 (UTC) |
|||
Hi, I recently tried to make an edit on the page [[Timeline of abolition of slavery and serfdom]], giving more info on the Kingdom of Hungary. I made my edits, hit publish, got the little pop-up saying "Your edits have been published," scrolled down, and they weren't there? I then went back in to the edit page to see if something went wrong, and maybe re-do my edits, but my edits did show up on the editing page - they just wouldn't show up on the article page. Does anyone know what's going on? Thanks, [[User:Rj1255|Rj1255]] ([[User talk:Rj1255|talk]]) 01:42, 22 January 2024 (UTC) |
|||
== How do I clean up formatting of tables? == |
|||
:hi @[[User:Rj1255|Rj1255]] and welcome to the Teahouse! are you referring to the Kingdom of Hungary entry? it didn't display due to some errors with the table. in a table, <code>|-</code> denotes a split in the row of a table, and since your entry was after a <code>|-</code>, the code parser thought it was just a row break instead of a full entry which meant it did not display. happy editing! 💜 <span style="border-radius:4px;background:#edf"> [[User:Melecie|<span style="color:#471a7a">'''mel'''ecie</span>]] </span> [[User talk:Melecie|<span style="color:#471a7a">talk</span>]] - 01:59, 22 January 2024 (UTC) |
|||
::Oh, I see, thank you. Yes, that was the one. I am new to this and was trying to imitate the layout of other entries. It worked for a couple entries but I guess I made a mistake on that one. Thanks a lot! [[User:Rj1255|Rj1255]] ([[User talk:Rj1255|talk]]) 02:10, 22 January 2024 (UTC) |
|||
On the [[Columbia_University#Academics|Columbia University]] article, the ''Rankings'' section has quite poor formatting; the ranking tables are being displayed fully below the Library and Barnard images on the right side. How would I go about cleaning up table formatting in wikitext to remove the large blank gap? [[User:Doawk7|Doawk7]] ([[User talk:Doawk7|talk]]) 21:13, 2 January 2025 (UTC) |
|||
== Media bias == |
|||
:I solved it by removing those images:) [[User:DMacks|DMacks]] ([[User talk:DMacks|talk]]) 07:38, 3 January 2025 (UTC) |
|||
== How to ask ArbCom to lower protection status on Armenia-Azerbaijan == |
|||
Is there any place in Wikipedia where we can discuss media bias of popular newspapers and news channels, where many experienced editors can discuss and come to a conclusion? [[User:Nightingagleyt|Nightingagleyt]] ([[User talk:Nightingagleyt|talk]]) 04:13, 22 January 2024 (UTC) |
|||
I was going to submit an ArbCom request to lower protection of Armenia-Azerbaijan due to the conflict cooling and amends being made, however I changed my mind and decided to ask here first. How would I get the Arbitration Committee or whoever imposed the sanction on Ar-Az to hear on the matter? [[User:SimpleSubCubicGraph|SimpleSubCubicGraph]] ([[User talk:SimpleSubCubicGraph|talk]]) 00:21, 3 January 2025 (UTC) |
|||
:If I am interpreting your question correctly, [[Wikipedia:Reliable sources/Noticeboard]] and [[Wikipedia:Reliable sources/Perennial sources]] (for sources that are commonly used) are the closest pages you are going to get to Wikipedians assessing source bias and reliability. Hope that helps! [[user:HistoryTheorist|<span style="font-family:Courier;color:#2F7E98">❤History</span>]][[User talk:HistoryTheorist|<span style="font-family:Courier;color:lightpurple">Theorist❤</span>]] 04:31, 22 January 2024 (UTC) |
|||
*That's not an ArbCom matter; you can ask for that at [[Wikipedia:Requests for page protection]], [[User:SimpleSubCubicGraph|SimpleSubCubicGraph]]. Are you asking about [[Nagorno-Karabakh conflict]]? You can edit that--after you've made a few more article edits. [[User:Drmies|Drmies]] ([[User talk:Drmies|talk]]) 01:34, 3 January 2025 (UTC) |
|||
::Thanks for your reply. I know about reliable sources. My doubt is about bias among reliable sources. CNN, BBC are reliable sources. Example-In any Argentina political issue they are supporting one party while locals who read Argentina newspaper know that politicians from both parties are corrupt and violent. [[User:Nightingagleyt|Nightingagleyt]] ([[User talk:Nightingagleyt|talk]]) 05:23, 22 January 2024 (UTC) |
|||
**@[[User:Drmies|Drmies]] I mean the consensus to make everything Armenia and Azerbaijan, even if its a tragic plane crash extended confirmed protected. I feel like it should be knocked down due to calming geopolitical tensions in the Caucasus. [[User:SimpleSubCubicGraph|SimpleSubCubicGraph]] ([[User talk:SimpleSubCubicGraph|talk]]) 03:04, 3 January 2025 (UTC) |
|||
:::While the linked articles do discuss bias a bit, it doesn't go in to too much depth about bias. To my knowledge (I could be totally wrong), there is no dedicated noticeboard for bias only. [[user:HistoryTheorist|<span style="font-family:Courier;color:#2F7E98">❤History</span>]][[User talk:HistoryTheorist|<span style="font-family:Courier;color:lightpurple">Theorist❤</span>]] 05:27, 22 January 2024 (UTC) |
|||
***Also I have 251 edits, that is nowhere near 500. [[User:SimpleSubCubicGraph|SimpleSubCubicGraph]] ([[User talk:SimpleSubCubicGraph|talk]]) 03:04, 3 January 2025 (UTC) |
|||
****In that case, [[User:SimpleSubCubicGraph|SimpleSubCubicGraph]], you're talking about [[Wikipedia:Requests for arbitration/Armenia-Azerbaijan 2]] and you'd have to appeal that. I just looked over the page and I have no idea how to do that--and I used to be on ArbCom. Read over [[Wikipedia:Contentious topics]], and good luck! [[User:Drmies|Drmies]] ([[User talk:Drmies|talk]]) 17:20, 3 January 2025 (UTC) |
|||
== Referencing an article on Wikipedia == |
|||
== No Control (1927 film) == |
|||
I am reviewing an article and came across a source that cited another article from Wikipedia. This is the first time I have seen this. This isn’t acceptable is it? [[User:BigChrisKenney|BigChrisKenney]] ([[User talk:BigChrisKenney|talk]]) 00:35, 3 January 2025 (UTC) |
|||
:Hello @[[User:BigChrisKenney|BigChrisKenney]]. You are correct. Sources that lead to Wikipedia or any Wikimedia project are unreliable. See [[WP:CIRCULAR]]. [[User:Tarlby|<span style="color:cyan;font-family:Comic Sans MS;">''Tarl''</span><span style="color:orange;font-family:Comic Sans MS;">''by''</span>]] <sup>([[User talk:Tarlby|''t'']]) ([[Special:Contributions/Tarlby|''c'']])</sup> 01:33, 3 January 2025 (UTC) |
|||
::Thank you @[[User:Tarlby|Tarlby]] for the speedy reply and the direction to the exact policy! [[User:BigChrisKenney|BigChrisKenney]] ([[User talk:BigChrisKenney|talk]]) 01:43, 3 January 2025 (UTC) |
|||
:::You can jsut delete the ref to a Wikipedia article, but a better practice would be to look within that Wikipedia article to find a ref (refs) that support the content, and then copy those refs to the article you have concerns about. If you do this, in your Edit summary consider stating that you copied content from - naming the source article. Wikipedia does allow copying content and refs from Wikipedia articles with attribution. [[User:David notMD|David notMD]] ([[User talk:David notMD|talk]]) 14:17, 3 January 2025 (UTC) |
|||
== Writer or Screenwriter? == |
|||
:With the distinguish template, you don't need to add the wikilink brackets. The template does that itself, so you should remove the wikilink brackets you added. Hope that helps! [[user:HistoryTheorist|<span style="font-family:Courier;color:#2F7E98">❤History</span>]][[User talk:HistoryTheorist|<span style="font-family:Courier;color:lightpurple">Theorist❤</span>]] 05:28, 22 January 2024 (UTC) |
|||
::no that doesn't fix it. please look at this page and see what i mean. it breaks the annotated subsequent text if i remove the brackets as you advised. [[User:Iljhgtn|Iljhgtn]] ([[User talk:Iljhgtn|talk]]) 05:56, 22 January 2024 (UTC) |
|||
:::See: [[No Control (1927 film)]] [[User:Iljhgtn|Iljhgtn]] ([[User talk:Iljhgtn|talk]]) 05:57, 22 January 2024 (UTC) |
|||
::::@[[User:Iljhgtn|Iljhgtn]]: To resolve the issue, I replaced {{tl|Distinguish}} with {{tl|for}}. You could have also used {{tl|Distinguish}} with {{para|text}} to do something like this: |
|||
:::::<code><nowiki>{{Distinguish|text=[[No Control (2015 film)]], a film made in 2015 by Jessica Solce about gun policy in the United States}}</nowiki></code> |
|||
::::which would generate: |
|||
:::::{{Distinguish|text=[[No Control (2015 film)]], a film made in 2015 by Jessica Solce about gun policy in the United States}} |
|||
::::[[User:GoingBatty|GoingBatty]] ([[User talk:GoingBatty|talk]]) 06:04, 22 January 2024 (UTC) |
|||
:::::ok so pipe text was what was needed. thank you goingbatty [[User:Iljhgtn|Iljhgtn]] ([[User talk:Iljhgtn|talk]]) 06:05, 22 January 2024 (UTC) |
|||
::::::@[[User:Iljhgtn|Iljhgtn]]: I didn't know how to make {{tl|Distinguish}} work either until I read the documentation at [[Template:Distinguish]]. [[User:GoingBatty|GoingBatty]] ([[User talk:GoingBatty|talk]]) 06:07, 22 January 2024 (UTC) |
|||
Quick question: creating a stub-starter on Donald Ross: [https://en.wikipedia.org/wiki/User:Maineartists/sandbox#Personal_life] who was a television screenwriter, celebrity game show panelist (with his wife [[Patti Deutsch]]) and producer. Since there are other [[Donald Ross]] articles, would you accompanying his name with (writer) or (screenwriter) for the published article? He has no distinguishing middle name / initial. Deutsch's article refers to him as "comedy screenwriter and playwright", but Ross never wrote any plays. TIA. [[User:Maineartists|Maineartists]] ([[User talk:Maineartists|talk]]) 02:16, 3 January 2025 (UTC) |
|||
== user 98Tigerius imposes his will == |
|||
:[[User:Maineartists|Maineartists]], "Donald Ross (writer)" seems OK to me. -- [[User:Hoary|Hoary]] ([[User talk:Hoary|talk]]) 03:50, 3 January 2025 (UTC) |
|||
:(writer) seems OK to me too [[User:Ayohama|Ayohama]] ([[User talk:Ayohama|talk]]) 07:23, 3 January 2025 (UTC) |
|||
Great. Thanks all. [[User:Maineartists|Maineartists]] ([[User talk:Maineartists|talk]]) 13:47, 3 January 2025 (UTC) |
|||
== What is the opinion on wikipedia writing services? == |
|||
user 98Tigerius imposes his will.. user 98Tigerius imposes his will.. all the changes I made were based on credible sources AGB Nielsen website on [[Korean drama]] page but he changed my changes after that he made changes as he pleased [[User:Michaelelijahtanuwijaya|Michaelelijahtanuwijaya]] ([[User talk:Michaelelijahtanuwijaya|talk]]) 05:37, 22 January 2024 (UTC) |
|||
How would I detect if an article has been secretly written and paid for? There are services that you can reach just by searching on google and I would like to know if there are any ways to detect them or do something about them. [[User:SimpleSubCubicGraph|SimpleSubCubicGraph]] ([[User talk:SimpleSubCubicGraph|talk]]) 04:32, 3 January 2025 (UTC) |
|||
:Hello! If there is an editor who might be paid and performing edits, raise the issue at [[Wikipedia:Administrators' noticeboard/Incidents|WP:ANI]] or [[Wikipedia:Conflict of interest/Noticeboard|WP:COIN]] noticeboards. If there are any private details you have found about the editor or services, please send an email to paid-en-wp@wikipedia.org . '''[[User:TNM101|<span style="color:red;">TNM</span><span style="color:black;">101</span>]]''' ([[User talk:TNM101|<span style="color:blue;">chat</span>]]) 05:24, 3 January 2025 (UTC) |
|||
== Babel for WikiProject Userboxes? == |
|||
:Hi, SSCG, did you have any specific articles in mind? I would say that there are heuristics I know of, but there's nothing definitive overall. Heustics I've identified for undisclosed paid/COI include (''please'' do not use any of these to make definitive accusations; these observations are potential starting points, not endpoints): |
|||
:* The article has an image of an otherwise obscure person which is usually marked as "own work". This image is posed, not candid, indicating that either the subject themself uploaded it or they've given it to somebody else to use as effectively a PR photo on Wikipedia. Some subjects presumably will enthusiastically freely license their image even when they have no part in an article because they're just jazzed to have their own article, but what can be really suspect is when the uploader of the image is the same editor who created and/or has heavily edited the article itself. |
|||
:* The article was already very long, well-formatted, and essentially complete right out of the gate. For new editors, sometimes articles after going through the draft process can come out looking alright, but these edits skip the draft process altogether yet still seem like they've been written out ahead of time off-site. Whereas I take an incremental approach to my writing, experienced editors very often will pre-write everything out in their sandbox and then transfer their work into the article space once they're happy with it, but in my experience, this is highly uncommon for newer editors who organically create articles about subjects they're interested in. |
|||
:* The article was created or heavily modified by a [[WP:SPA]], which seemingly has not interacted elsewhere on Wikipedia. |
|||
:* The editor tries to wikilink this article absolutely everywhere they possibly can once it's created, presumably for a mix of SEO and for drawing as many eyes to that subject as possible on-wiki. |
|||
:* The article is mostly primary references, and a lot of it is [[WP:PUFFERY]]. For companies, a dead giveaway for something like this is when there's an entire section on some stupid PR BS the company did which is only cited to press releases from that company about what a great thing(TM) they did. Basically, consider [[WP:DUE]]. |
|||
:* The article goes into excruciating depth about a subject (sometimes this is just fans being fans, but for example, there was one some years ago about a Lithuanian(?) alt-rock band whose primary editor I thought to be just an overly enthusiastic fan but who I discovered was secretly the band's manager or something to that effect). |
|||
:* The 'External links' section contains an inordinate amount of links to social media (this one to me generally indicates that it's autobiographical, whereas I assume you specifically want contracted third parties). |
|||
:* The editor has a username-violating policy which indicates they're part of the subject organization. |
|||
:* The subject likely doesn't meet the [[WP:GNG]] but has been [[WP:REFBOMB]]ed to give the appearance that it does. |
|||
:* The subject is a very niche a) product/service/work, b) company/organization, or c) person. Depending on the nature of the organization (e.g. a club), this may not be "paid" as much as it is [[WP:COI]]. |
|||
:* The subject is not written about in [[WP:NPOV|a neutral way]]. If there is negative coverage of the subject in reliable sources, it is strangely absent. |
|||
:If you strongly suspect that an editor is engaging in undisclosed paid editing, the boards {{u|TNM101}} suggests are great resources. I would caution too however that if your suspicion isn't that strong, you can start by just asking them at their talk page. Sometimes they're earnestly trying to improve the encyclopedia about something they're very familiar with because they're a part of it, but they've failed out of ignorance of forgetfulness to disclose that on their user page. <b>[[User:TheTechnician27|<span style="color: #00a9ff"><i>TheTechnician27</i></span>]]</b> [[User talk:TheTechnician27|<span style="color: blue">(Talk page)</span>]] 06:03, 3 January 2025 (UTC) |
|||
::@[[User:TheTechnician27|TheTechnician27]]@[[User:TNM101|TNM101]] I wasn't talking about COIs or paid editing, I already know and have remembered both WP pages. I was just wondering if Wikipedia could take action against those services aka take them to court. [[User:SimpleSubCubicGraph|SimpleSubCubicGraph]] ([[User talk:SimpleSubCubicGraph|talk]]) 06:59, 3 January 2025 (UTC) |
|||
:::That doesn't seem to be within our scope, and I am pretty sure that the Wikimedia Foundation doesn't take legal action against anyone since it's a non-profit. '''[[User:TNM101|<span style="color:red;">TNM</span><span style="color:black;">101</span>]]''' ([[User talk:TNM101|<span style="color:blue;">chat</span>]]) 07:12, 3 January 2025 (UTC) |
|||
:::I can't think of a single viable avenue by which Wikipedia could take successful legal action against undisclosed paid editors. If you tried to bring a civil suit, you'd be trying to prove what's called a [[tort]]. The WMF would have to prove that 1) the editor had a legal duty to act in a certain way (they didn't; nothing legally says you have to identify an employer when you contribute to a website); 2) that the defendent breached this duty (i.e. if you could somehow prove 1 (not possible), then you would still need to prove in court that your suspicions are [[preponderance of evidence|more probable than not]]); and 3) that the WMF suffered injury or loss as a result of this undisclosed paid editing (short of some absolutely enormous conspiracy that blows up and damages the WMF's reputation, this is functionally impossible, because the reality is that any loss is negligible at best). <b>[[User:TheTechnician27|<span style="color: #00a9ff"><i>TheTechnician27</i></span>]]</b> [[User talk:TheTechnician27|<span style="color: blue">(Talk page)</span>]] 07:18, 3 January 2025 (UTC) |
|||
Deep thanks to [[User:TheTechnician27]] for putting all these undeclared paid flags in one place. If this is not already the subject of an essay, it should be. I do see that [[Wikipedia:Identifying PR]] is an existing essay flagging behavior that draws content from press releases. [[User:David notMD|David notMD]] ([[User talk:David notMD|talk]]) 11:45, 3 January 2025 (UTC) |
|||
: |
:Thank you very much for all the responses, is there anyway I can see of all the times when paid editors or undisclosed COIs were caught in the act? [[User:SimpleSubCubicGraph|SimpleSubCubicGraph]] ([[User talk:SimpleSubCubicGraph|talk]]) 04:33, 4 January 2025 (UTC) |
||
:@[[User:CanonNi|CanonNi]]: When you visit the user pages of other users and like their layout, you can click "Edit source" to see how they formatted their page. I've copied code from other users, pasted it on my user page, and then played around with it until it was something I liked. [[User:GoingBatty|GoingBatty]] ([[User talk:GoingBatty|talk]]) 15:53, 22 January 2024 (UTC) |
|||
== Improving a page through including my own research == |
|||
== User made information == |
|||
The 'icing the kicker' page can benefit from the following recent addition in regards to the effectiveness of this practice: |
|||
I have read through all the guidelines(I could find) of Wikipedia and as user made information becomes more prevalent I imagine there must be some way user made information(e.g Youtube) may be used if confirmed by numerous sources. I will accept a simple yes or no. [[User:Sputnik274|Sputnik274]] ([[User talk:Sputnik274|talk]]) 09:53, 22 January 2024 (UTC) |
|||
"Most recently and extensively, Professor Nadav Goldschmied and Tyler Ratkovich from the University of San Diego and Mike Raphaeli from Nanning Vocational and Technical University (China) found that in 25 NFL seasons (1999-2024) the success for "pressure kicks" (two minutes or less to the end of the game when -3, −2, −1, tied or in overtime) when iced was 74.1% as opposed to 79.8% otherwise which represented a significant decline in performance. |
|||
:{{ping|Sputnik274}} If confirmed by numerous reliable sources, just use those sources instead. User-generated sources are nearly always not suitable for use in any article. If all of the numerous sources you're referring to are also user-generated, the material should not be used. See [[WP:USERGENERATED]]. [[User:Tollens|Tollens]] ([[User talk:Tollens|talk]]) 09:57, 22 January 2024 (UTC) |
|||
::That is a solution, thanks. [[User:Sputnik274|Sputnik274]] ([[User talk:Sputnik274|talk]]) 09:59, 22 January 2024 (UTC) |
|||
:::{{u|Sputnik274}}, there is no simple yes or no answer. Sorry. The vast majority of YouTube videos are of no value as references because they do not comply with our policies in a variety of ways. On the other hand, the small percentage of YouTube videos on the official channels of established reliable sources are also reliable sources. It is your obligation to personally verify the reliability of any source you plan to use on Wikipedia, whether it is a video or anything else. [[User:Cullen328|Cullen328]] ([[User talk:Cullen328|talk]]) 10:28, 22 January 2024 (UTC) |
|||
'''Goldschmied, N.''', Ratkovich, T*., & Raphaeli, M. (2024) NFL field goal kicking under pressure: An expanded replication of icing the kicker strategy. ''Journal of Applied Sport Psychology.'' |
|||
== Way to know source is blacklisted beforehand == |
|||
Thanks, |
|||
It seems to me that there is no way to see a source is blacklisted, other than if it seems false. Which is not good for one's own time nor effort. It would help greatly to see a source is blacklisted immediately. I also find no guideline for this myself despite my searches through the pages. Thank you. [[User:Sputnik274|Sputnik274]] ([[User talk:Sputnik274|talk]]) 10:03, 22 January 2024 (UTC) |
|||
Nadav Goldschmied |
|||
:{{ping|Sputnik274}} You are likely looking for [[WP:RSP]]. [[User:Tollens|Tollens]] ([[User talk:Tollens|talk]]) 10:07, 22 January 2024 (UTC) |
|||
::Thanks! [[User:Sputnik274|Sputnik274]] ([[User talk:Sputnik274|talk]]) 10:08, 22 January 2024 (UTC) |
|||
::::{{u|Sputnik274}}, one of the most important skills of a Wikipedia editor is the ability to assess the reliability of a potential source on your own without asking. Who is the publisher and what is the publisher's reputation? Who are the members of their editorial team and what is their editorial policy? What is the reputation of the specific source, and have they won major journalistic awards or have they been consistently criticized for false reporting? These are only a handful of the questions that should run through your mind when evaluating the reliability of a source. You cannot ask about every single source. You must make judgments on your own most of the time. [[User:Cullen328|Cullen328]] ([[User talk:Cullen328|talk]]) 10:19, 22 January 2024 (UTC) |
|||
:::::That definetly does clears things up for me, thank you. [[User:Sputnik274|Sputnik274]] ([[User talk:Sputnik274|talk]]) 10:22, 22 January 2024 (UTC) |
|||
== How do I add an Old Boy to the list of Alumni at St Bees in Cumbria == |
|||
[[User:Ngoldschmied11|Ngoldschmied11]] ([[User talk:Ngoldschmied11|talk]]) 05:42, 3 January 2025 (UTC) |
|||
:The place to suggest this, [[User:Ngoldschmied11|Ngoldschmied11]], is [[Talk:Icing the kicker]]. (Better, I think: "A study found that in 25 NFL seasons..." Interested readers will identify the authors from the reference.) -- [[User:Hoary|Hoary]] ([[User talk:Hoary|talk]]) 05:48, 3 January 2025 (UTC) |
|||
:Please read [[WP:ALUMNI]]. People should only be added if there is a Wikipedia article about them. [[User:Shantavira|Shantavira]]|[[User talk:Shantavira|<sup>feed me</sup>]] 10:39, 22 January 2024 (UTC) |
|||
::Fine by me! so do I add the modification or is it done by another editor since it is my work? |
|||
::...just to add that it may be OK to add a name of an alumnus if there isn't an article about them, but only if one or more citations are added after their entry. This is both to prove that they did indeed attend that school or college ''and'' to demonstrate that they do meet our [[WP:NBIO|Notability Criteria]]. In that case, it is OK to add them as a [[WP:REDLINK|REDLINK]] in the expectation that there will be an article about them in due course. However, no suitable citations; no entry on the list! [[User:Nick Moyes|Nick Moyes]] ([[User talk:Nick Moyes|talk]]) 10:52, 22 January 2024 (UTC) |
|||
::Thanks, |
|||
:::{{u|Gnidwod}}, it is always optimal to mention the ''specific'' article that you are talking about, since we have 6,773,292 of them, and no human can possibly remember all of them. After searching around, I am guessing that you are talking about [[List of Old St. Beghians]]. Please let us know if it is another article. Briefly, if the Wikipedia biography of a notable person verifies that they attended [[St Bees School]], then add them to that list article. If the person is not the subject of a Wikipedia article, then either write an acceptable Wikipedia biography first, or add references verifying their notability, or don't add them to that list at all. After all, the school has been around for 440 years, and it would be inappropriate and of no value to try to list everyone who ever attended. We list only notable alumni, which in effect means that they are the subject of a Wikipedia biography in most cases. While you are at it, you could alphabetize that list. That would be useful. [[User:Cullen328|Cullen328]] ([[User talk:Cullen328|talk]]) 10:56, 22 January 2024 (UTC) |
|||
::Nadav [[User:Ngoldschmied11|Ngoldschmied11]] ([[User talk:Ngoldschmied11|talk]]) 06:15, 3 January 2025 (UTC) |
|||
::::Since the list has an existing order (note the parenthetical "by order of birth date" in the heading), rearranging it alphabetically could be a controversial move, requiring prior discussion on the talk page. [[User:Deor|Deor]] ([[User talk:Deor|talk]]) 14:48, 22 January 2024 (UTC) |
|||
:::@[[User:Ngoldschmied11|Ngoldschmied11]] pretty sure this falls under [[WP:No original research]] [[User:SimpleSubCubicGraph|SimpleSubCubicGraph]] ([[User talk:SimpleSubCubicGraph|talk]]) 07:00, 3 January 2025 (UTC) |
|||
::::{{u|SimpleSubCubicGraph}} Not quite. If the observations on kicking success/failure were unpublished, that would be original research. If published in a reliable source, such as a science journal article, OK to use. However, Wikipedia advises researchers who are also Wikipedia editors to not cite their own work. [[User:David notMD|David notMD]] ([[User talk:David notMD|talk]]) 12:05, 3 January 2025 (UTC) |
|||
:::Hello, @[[User:Ngoldschmied11|Ngoldschmied11]]: in case it's not clear from the other responses: you are discouraged from citing your own work, as that is regarded as a [[WP:conflict of interest|conflict of interest]], but you are welcome to suggest an edit which cites your own work, so that another editor decides what to do with your suggestion. If you use the [[WP:edit request wizard|edit request wizard]] that will guide you. Please make your suggestion as precise as possible (eg "Inset XXX at the end of the paragraph beginning YYY") [[User:ColinFine|ColinFine]] ([[User talk:ColinFine|talk]]) 13:10, 3 January 2025 (UTC) |
|||
== |
== Shortened forms list == |
||
Hello. Is there any place I could get the shortened forms of all Wikipedia guidelines and a brief definition (e.g. [[WP:GNG]], [[WP:NPV]])? I usually find it difficult to use the shortened forms when commenting and I just go for the full name. <span style="color:#9400D3;">[[User:Josedimaria|Jõsé]]<sup>[[User talk:Josedimaria|''hola'']]</sup></span> 07:13, 3 January 2025 (UTC) |
|||
:Hello @[[User:Josedimaria|Josedimaria]]. See [[WP:WP]]. [[User:Tarlby|<span style="color:cyan;font-family:Comic Sans MS;">''Tarl''</span><span style="color:orange;font-family:Comic Sans MS;">''by''</span>]] <sup>([[User talk:Tarlby|''t'']]) ([[Special:Contributions/Tarlby|''c'']])</sup> 07:20, 3 January 2025 (UTC) |
|||
:@[[User:TheProEditor11|TheProEditor11]] It depends on how many entries you want to cover. For example, [[J. K. Rowling]] has a large table, where other authors have a simple bulleted list (e.g. [[Coral Bell]]). There is no reason to include absolutely everything someone published: a "selected publications" list is fine. [[User:Michael D. Turnbull|Mike Turnbull]] ([[User talk:Michael D. Turnbull|talk]]) 13:31, 22 January 2024 (UTC) |
|||
:: |
:Hey, how about [[Wikipedia:Shortcut index]] or [[Wikipedia:List of policies and guidelines]] and might be an idea to work out something for [[Wikipedia:Glossary of shortcuts]] [[User:Ayohama|Ayohama]] ([[User talk:Ayohama|talk]]) 07:21, 3 January 2025 (UTC) |
||
:::@[[User:TheProEditor11|TheProEditor11]], there are citation templates you can use. I believe the relevant one here would be a template like {{t|Cite book}} which you can use to format these references. a general and in-depth guide to referencing can also be viewed at [[WP:Citing sources|Citing sources]]. happy editing! 💜 <span style="border-radius:4px;background:#edf"> [[User:Melecie|<span style="color:#471a7a">'''mel'''ecie</span>]] </span> [[User talk:Melecie|<span style="color:#471a7a">talk</span>]] - 13:57, 22 January 2024 (UTC) |
|||
::::Thanks for the help!!!!!! [[User:TheProEditor11|TheProEditor11]] ([[User talk:TheProEditor11|talk]]) 14:01, 22 January 2024 (UTC) |
|||
:::Note that in this case you would put the {{tl|cite book}} directly after a bullet (*), without any <code><nowiki><ref></nowiki></code> tages around it, so it will be part of the main text and not included in the footnotes/references. [[User:Michael D. Turnbull|Mike Turnbull]] ([[User talk:Michael D. Turnbull|talk]]) 14:01, 22 January 2024 (UTC) |
|||
::::I will make it sure! Thnx! [[User:TheProEditor11|TheProEditor11]] ([[User talk:TheProEditor11|talk]]) 14:01, 22 January 2024 (UTC) |
|||
== |
== Creating a new page == |
||
I want to make a new Wikipedia page but the name already exists so what should I do [[User:Chetan4u1|Chetan4u1]] ([[User talk:Chetan4u1|talk]]) 10:05, 3 January 2025 (UTC) |
|||
If I have a reference in the end of the sentence or phrase, should I put it after comma/dot or before? |
|||
:Can you tell us the name of the article so we can better understand the situation? [[User:Ivebeenhacked|Hacked]] ([[User talk:Ivebeenhacked|Talk]]|[[Special:Contributions/Ivebeenhacked|Contribs]]) 10:10, 3 January 2025 (UTC) |
|||
For example: ...was established on October 30, 1923[1], to the 32nd Government...<br> |
|||
:@[[User:Chetan4u1|Chetan4u1]] Welcome to the Teahouse. You don't need to worry about that at this stage. You can create a draft article by following the process at [[WP:AfC]]. If and when it is accepted into the encyclopedia (most draft articles are not), the reviewer will attend to the naming, which might require a [[WP:DISAMBIG|disambiguation page]], but if you haven't written a Wikipedia article before you'll have plenty of other hoops to jump through before this becomes an issue. [[User:Shantavira|Shantavira]]|[[User talk:Shantavira|<sup>feed me</sup>]] 10:15, 3 January 2025 (UTC) |
|||
or <br> |
|||
::@[[User:Chetan4u1|Chetan4u1]] Your attempt to create an article about Isabel Myers who is an actress has been deleted and a redirect created to [[Isabel Briggs Myers]], a different person. To start over, use [[WP:YFA]] to create a draft named Isabel Myers (actress). That said, common advice here is to gain experience improving existing articles before attempting to create an article. [[User:David notMD|David notMD]] ([[User talk:David notMD|talk]]) 12:18, 3 January 2025 (UTC) |
|||
...was established on October 30, 1923,[1] to the 32nd Government...<br> |
|||
Thanks. [[User: |
:::Thanks Brother but if I make a draft then how can I change the name of the page if it already exists. [[User:Chetan4u1|Chetan4u1]] ([[User talk:Chetan4u1|talk]]) 12:33, 3 January 2025 (UTC) |
||
::::@[[User:Chetan4u1|Chetan4u1]] If you're working on a draft on an individual who shares their name with someone who already has an existing article, you're supposed to differentiate/distinguish your draft from the existing article. You cannot 'delete' or 'change the name' of an article that has been reviewed and exists. In your case, name your draft '''Isabel Myers (actress)''', since the subject of the existing article, [[Isabel Briggs Myers]], is a writer. <span style="background-color: black; padding: 2px 3px 1px 3px;">[[User:Dissoxciate|<span style="color: greenyellow">'''Dissoxciate'''</span>]] [[User talk:Dissoxciate|<span style="color: turquoise">(talk)</span>]]</span> 13:19, 3 January 2025 (UTC) |
|||
:::::<small>@[[User:Dissoxciate|Dissoxciate]], I disagree with you. When creating a draft, you ''may'' disambiguate the title, but you don't need to, and you certainly aren't "supposed to". New editors probably don't know our disambiguation rules anyway, so I would not encourage them to try. |
|||
:::::For your second point, you can [[WP:move|move]] an existing article to a new title, and that sometimes needs to happen when a new article is created about something with the same name. But I agree that if Chean4u1 is asking how to change the name of an existing article in order to hijack it, they shouldn't. </small> [[User:ColinFine|ColinFine]] ([[User talk:ColinFine|talk]]) 13:58, 3 January 2025 (UTC) |
|||
::::Hello, @[[User:Chetan4u1|Chetan4u1]], and welcome to the Teahouse. |
|||
::::The answer to your specific question is that you change the name of an article or other page by [[WP:MOVE|moving]] it; but I suggest you forget about this for the moment because 1) as a new user you don't have access to that tool yet, and 2) at your present stage it is irrelevant to what you want to do. |
|||
::::{{User:ColinFine/PractiseFirst}} |
|||
::::When you are ready to create the draft, you can completely ignore the titles of any existing articles. You will be creating [[Draft:Isabel Myers]] (which doesn't exist at present, which is why it is in red). After you have developed an acceptable draft for an article (which is '''much''' harder than it looks to new editors, and will almost certainly give you frustration and disappointment if you don't take David's and my advice above), you will submit it for review, and if the reviewer accepts it, they will sort out ''all'' the issues about naming it. OK? [[User:ColinFine|ColinFine]] ([[User talk:ColinFine|talk]]) 13:20, 3 January 2025 (UTC) |
|||
:::::Thanks I will try my best. [[User:Chetan4u1|Chetan4u1]] ([[User talk:Chetan4u1|talk]]) 14:17, 3 January 2025 (UTC) |
|||
== adding a clarification in response to a user request == |
|||
I recommend doing it before a comma or period. [[User:Cometkeiko]] <!--Template:Undated--><small class="autosigned">— Preceding [[Wikipedia:Signatures|undated]] comment added 13:16, 22 January 2024 (UTC)</small> <!--Autosigned by SineBot--> |
|||
Hello, in doing what described in subject, should one delete the [[Graphite bomb|request]] for clarification, or leave it there? [[User:1808Eolo3|1808Eolo3]] ([[User talk:1808Eolo3|talk]]) 10:14, 3 January 2025 (UTC) |
|||
:@[[User:Aredoros87|Aredoros87]] [[WP:REF]] gives the full guidelines, from which you can see that references always come ''after'' punctuation. @[[User:Cometkeiko|Cometkeiko]] please don't comment here at the Teahouse if you don't know the correct guidance. [[User:Michael D. Turnbull|Mike Turnbull]] ([[User talk:Michael D. Turnbull|talk]]) 13:24, 22 January 2024 (UTC) |
|||
::I’ll see. [[User:Cometkeiko|Cometkeiko]] ([[User talk:Cometkeiko|talk]]) 13:30, 22 January 2024 (UTC) |
|||
:::Oh ok, got it. [[User:Cometkeiko|Cometkeiko]] ([[User talk:Cometkeiko|talk]]) 13:31, 22 January 2024 (UTC) |
|||
::::I remember there were articles who added references before commas and periods? [[User:Cometkeiko|Cometkeiko]] ([[User talk:Cometkeiko|talk]]) 15:35, 22 January 2024 (UTC) |
|||
:::::@[[User:Cometkeiko|Cometkeiko]]: You may see some articles where editors have incorrectly added references before commas and periods. Feel free to fix them when you see them. Happy editing! [[User:GoingBatty|GoingBatty]] ([[User talk:GoingBatty|talk]]) 15:43, 22 January 2024 (UTC) |
|||
::::::I've seen this specific typo/guideline get fixed by bots before. Is it possible to make a direct request for a bot to make this fix on a specific page? Do you know which bots can do it? [[User:Reconrabbit|<span style="color:#6BAD2D">Recon</span>]][[User talk:Reconrabbit|<span style="color:#2F3833">rabbit</span>]] 18:33, 22 January 2024 (UTC) |
|||
:@[[User:1808Eolo3|1808Eolo3]] Welcome to the Teahouse. If you mean resolving a {{how}} template (with adequate referencing) then you may remove the template. [[User:Shantavira|Shantavira]]|[[User talk:Shantavira|<sup>feed me</sup>]] 10:36, 3 January 2025 (UTC) |
|||
== Publishing Article - EBTRON == |
|||
== What is sandbox and how should I use it? == |
|||
Hello. I can not seem to figure out how to publish this article. I moved it out of my sandbox - by hitting "Move" (article). [[User:Lisahickey|Lisahickey]] ([[User talk:Lisahickey|talk]]) 13:39, 22 January 2024 (UTC) |
|||
How can I use sandbox [[User:Juamisi25|Juamisi25]] ([[User talk:Juamisi25|talk]]) 10:54, 3 January 2025 (UTC) |
|||
:hi @[[User:Lisahickey|Lisahickey]] and welcome to the Teahouse! are you referring to [[Lisahickey/sandbox]]? firstly, that article is currently named that instead of [[EBTRON]], which you can edit by moving the page again to a different title. however, instead of that I recommend you to go through the [[WP:Articles for creation|Articles for creation]] process (instead of publishing the page immediately) by moving the article back to User and following the steps in that page, since they could provide you with more advice to improve your article. editing! 💜 <span style="border-radius:4px;background:#edf"> [[User:Melecie|<span style="color:#471a7a">'''mel'''ecie</span>]] </span> [[User talk:Melecie|<span style="color:#471a7a">talk</span>]] - 13:54, 22 January 2024 (UTC) |
|||
::@[[User:Lisahickey|Lisahickey]] According to [[Special:Contributions/Lisahickey|your contribution history]] you have moved things around a bit an ended up with [[Lisahickey/sandbox]] in article space. As a new user and one that is a declared [[WP:PAID|paid editor]] (thanks for the declaration) you should not move drafts into the main encyclopaedia but should use the [[WP:AfC]] process: see that link for the details. A brief look tells me that there are all sorts of problems with your draft and it is likely to be speedily deleted if you don't follow our accepted process. [[User:Michael D. Turnbull|Mike Turnbull]] ([[User talk:Michael D. Turnbull|talk]]) 13:55, 22 January 2024 (UTC) |
|||
:::It was moved to [[EBTRON]] and I have tagged for speedy deletion, you need to go through the [[WP:AFC]] process. [[User:Theroadislong|Theroadislong]] ([[User talk:Theroadislong|talk]]) 16:54, 22 January 2024 (UTC) |
|||
:::Thank you. This is all a bit foreign to me. I had seen that other companies are listed in Wikipedia and wanted to get EBTRON listed. I did read the article links. I thought I removed the company promotion copy. Not sure what you mean by paid editor? [[User:Lisahickey|Lisahickey]] ([[User talk:Lisahickey|talk]]) 16:55, 22 January 2024 (UTC) |
|||
::::@[[User:Lisahickey|Lisahickey]] Sorry, I noticed that you had declared a [[WP:COI]] and I just assumed this was because you were employed by or an intern with EBTRON. The requirements to use the [[WP:AfC]] process is advised in any case. [[User:Michael D. Turnbull|Mike Turnbull]] ([[User talk:Michael D. Turnbull|talk]]) 18:42, 22 January 2024 (UTC) |
|||
:@[[User:Juamisi25|Juamisi25]] Welcome to the Teahouse! All should be explained at [[H:SAND|this help page]]: basically your personal sandbox is a place for you to experiment with editing without messing anything up in the main encyclopedia. [[User:Michael D. Turnbull|Mike Turnbull]] ([[User talk:Michael D. Turnbull|talk]]) 10:58, 3 January 2025 (UTC) |
|||
== Please help me rewrite. == |
|||
:Short and simple: [[Special:MyPage/sandbox| follow this link]] — you can do whatever with it as long as it's productive Wikipedia Stuff, use it to practice edits, put together rough drafts etcetera --[[User:Slowking Man|Slowking Man]] ([[User talk:Slowking Man|talk]]) 01:15, 4 January 2025 (UTC) |
|||
== Error with interlanguage link template == |
|||
Help me with this caution: https://en.wikipedia.org/wiki/Gnoppix [[User:Tumrabert|Tumrabert]] ([[User talk:Tumrabert|talk]]) 15:10, 22 January 2024 (UTC) |
|||
asdas |
|||
So while editing [[Guangdong Public Security Department]], I added an interlanguage link in the infobox, and for whatever reason in the infobox right after the interlanguage link are two brackets(]]). I entered the interlanguage link properly from both experience and directions, may I ask what I did wrong and how to solve this issue? [[User:Thehistorianisaac|Thehistorianisaac]] ([[User talk:Thehistorianisaac|talk]]) 11:18, 3 January 2025 (UTC) |
|||
:@[[User:Tumrabert|Tumrabert]]: Welcome to the Teahouse! I see the article has been moved to [[Draft:Gnoppix]]. As stated at the top of the draft, all the sources are primary sources, and Wikipedia articles are based on secondary sources. If you haven't done so already, I suggest reading [[Help:Your first article]] and [[Wikipedia:Notability (software)]]. Then, gather multiple sources that meet the four criteria listed at the top of your draft: "(1) reliable (2) secondary (3) independent of the subject (4) talk about the subject in some depth." (Click the links at the top of the draft for more information about each criteria.) Then rewrite your draft based on what the independent sources have written. Hope this helps, and happy editing! [[User:GoingBatty|GoingBatty]] ([[User talk:GoingBatty|talk]]) 15:41, 22 January 2024 (UTC) |
|||
:Hi {{u|Thehistorianisaac}}. I think this might have something to do with the syntax of [[:Template:Infobox law enforcement agency]]. If you look at the syntax for the {{para|Secondary governing body}} parameter, it appears things are set up to convert whatever is added for that parameter into a Wikilink. If you remove the {{tlx|ill}} template for that parameter, it will format it as a [[:WP:REDLINK]]. You could try asking about this on the template's talk page or at [[:WP:VPT]]; however, I don't think you can remove the brackets without removing them from the infobox template's syntax, and doing that would affect other articles where the template is being transcluded. You might be just have to [[:WP:HIDDEN|hide]] the parameter for the time being until a fix can be found or just go with the red link. -- [[User:Marchjuly|Marchjuly]] ([[User talk:Marchjuly|talk]]) 12:09, 3 January 2025 (UTC) |
|||
::Ok thank you [[User:Thehistorianisaac|Thehistorianisaac]] ([[User talk:Thehistorianisaac|talk]]) 12:14, 3 January 2025 (UTC) |
|||
:::{{ping|Thehistorianisaac}}. The documentation for the parameter at [[Template:Infobox law enforcement agency]] says "Do not link. The template will link." I looked at the implementation and it doesn't appear possible to add an interlanguage link there. If the link is important then you can add it to the opening paragraph instead. [[User:PrimeHunter|PrimeHunter]] ([[User talk:PrimeHunter|talk]]) 12:19, 3 January 2025 (UTC) |
|||
::::thanks [[User:Thehistorianisaac|Thehistorianisaac]] ([[User talk:Thehistorianisaac|talk]]) 12:20, 3 January 2025 (UTC) |
|||
:::::{{ping|Thehistorianisaac}} If you really want to include the interlanguage link, you might replace the linked "provincial government" in the second sentence of the article with the ILL you were using in the infobox. [[User:Deor|Deor]] ([[User talk:Deor|talk]]) 23:47, 3 January 2025 (UTC) |
|||
== Can someone help me with archiving bot? == |
|||
[[Draft:Gnoppix]] Speedy deleted (again!) and Tumrabert indefinitely blocked. [[User:Gnoppix]] also indefinitely blocked. [[User:David notMD|David notMD]] ([[User talk:David notMD|talk]]) 16:58, 22 January 2024 (UTC) |
|||
You can edit my talk page [[User:Cactusisme|'''<span style="color:#0D98BA;">Cactus</span><span style="color:#013220;">🌵</span>''']] <sup>[[User talk:Cactusisme|<i style="color:green">spiky</i>]]</sup> <sup>[[Special:Contributions/Cactusisme|<i style="color:green">ouch</i>]]</sup> 12:27, 3 January 2025 (UTC) |
|||
== Wikipedia's criteria for speedy deletion == |
|||
:@[[User:Cactusisme|Cactusisme]]: Done. I have set up auto-archiving on your talk page. Lowercase Sigmabot III will now archive threads that are 7 days old, keeping at least 5 threads and archiving at least 1. If you want to customise it further, you can check out [[Help:Archiving a talk page#Example with sequentially numbered archives]] or just ping me for assistance. – [[User:DreamRimmer|<span style="color:black">'''DreamRimmer'''</span>]] <small>([[User talk:DreamRimmer|'''talk''']])</small> 14:41, 3 January 2025 (UTC) |
|||
Via my article on this page. |
|||
::Thanks!!! [[User:Cactusisme|'''<span style="color:#0D98BA;">Cactus</span><span style="color:#013220;">🌵</span>''']] <sup>[[User talk:Cactusisme|<i style="color:green">spiky</i>]]</sup> <sup>[[Special:Contributions/Cactusisme|<i style="color:green">ouch</i>]]</sup> 01:21, 4 January 2025 (UTC) |
|||
== Images == |
|||
'''Link to article:[[Gnoppix|Gnoppix - Wikipedia]]''' |
|||
Is there a way one can add copyrighted images/images not in the public domain to articles, if necessary? Maybe something to do with fair use? Can someone walk me through the process, given there's one? Thanks! <span style="background-color: black; padding: 2px 3px 1px 3px;">[[User:Dissoxciate|<span style="color: greenyellow">'''Dissoxciate'''</span>]] [[User talk:Dissoxciate|<span style="color: turquoise">(talk)</span>]]</span> 12:34, 3 January 2025 (UTC) |
|||
:Hi @[[User:Dissoxciate|Dissoxciate]]: if you haven't yet read [[WP:Non-free content]], that seems a good place to start. -- [[User:DoubleGrazing|DoubleGrazing]] ([[User talk:DoubleGrazing|talk]]) 13:38, 3 January 2025 (UTC) |
|||
Please help me with any suggestions.(rewrite,appeal etc.) |
|||
:Hello! See the instructions at [[Special:Upload]] if you want to upload the file after reading [[WP:Non-free content]] as said by [[User:DoubleGrazing|DoubleGrazing]] '''[[User:TNM101|<span style="color:red;">TNM</span><span style="color:black;">101</span>]]''' ([[User talk:TNM101|<span style="color:blue;">chat</span>]]) 13:41, 3 January 2025 (UTC) |
|||
== Is there another method besides WP:RSP to check if a source is reliable? == |
|||
Wikipedia shown that |
|||
Are there other ways to check if a source is reliable besides using [[WP:RSP]]? For example, are there guidelines, tools, or methods that can help evaluate a source's trustworthiness when RSP doesn’t give a clear answer? [[User:Silkroadster|Silkroadster]] ([[User talk:Silkroadster|talk]]) 13:33, 3 January 2025 (UTC) |
|||
'''''This article may meet Wikipedia's [[Wikipedia:Criteria for speedy deletion|criteria for speedy deletion]]'' because in its current form it serves only to [[Wikipedia:Spam|promote]] or publicise an entity, person, product, or idea, and would require a fundamental rewrite in order to become encyclopedic.''' ''However, the mere fact that a company, organization, or product is a page's subject does not, on its own, qualify that page for deletion under this criterion. This criterion also does not apply where substantial encyclopedic content would remain after removing the promotional material as [[Wikipedia:DINC|deletion is not cleanup]]<nowiki>; in this case please remove the promotional material yourself, or add the {{</nowiki>[[Template:Advert|advert]]<nowiki>}} tag to alert others to do so</nowiki>''. '''See [[Wikipedia:Criteria for speedy deletion#G11|CSD G11]].''' |
|||
:@[[User:Silkroadster|Silkroadster]]: You could read [[WP:RS]] yourself to determine if a source meets its criteria, or ask on [[WP:RSN|RSN]]. [[User:JJPMaster|JJP]]<sub>[[User talk:JJPMaster|Mas]]<sub>[[Special:Contributions/JJPMaster|ter]]</sub></sub> ([[She (pronoun)|she]]/[[Singular they|they]]) 13:34, 3 January 2025 (UTC) |
|||
If this article does not meet the criteria for speedy deletion, or you intend to fix it, please remove this notice, but '''do not remove this notice from pages that you have created yourself'''. If you created this page and you disagree with the given reason for deletion, you can click the button below and leave a message explaining why you believe it should not be deleted. You can also [[Talk:Gnoppix|visit the talk page]] to check if you have received a response to your message. |
|||
:There is a userscript at [[User:Novem Linguae/Scripts/CiteHighlighter]] that takes ratings from various sources and highlights them in green, yellow or red depending on their reliability. Also, [[User:Novem Linguae/Scripts/CiteHighlighter#Original source lists|User:Novem Linguae/Scripts/CiteHighlighter#Original source lists]] is very helpful. – [[User:DreamRimmer|<span style="color:black">'''DreamRimmer'''</span>]] <small>([[User talk:DreamRimmer|'''talk''']])</small> 13:48, 3 January 2025 (UTC) |
|||
::@[[User:DreamRimmer|DreamRimmer]] Thank you. I realized that early on in my editing, but I appreciate it again as I used it as an argument in an AFD. [[User:Silkroadster|Silkroadster]] ([[User talk:Silkroadster|talk]]) 14:21, 3 January 2025 (UTC) |
|||
:::While looking at the [[Wikipedia:WikiProject Film/Indian cinema task force#Guidelines on sources]]. I found Hindu Businessline to be a reliable source. Does this reliability apply to news related to companies as well? [[User:Silkroadster|Silkroadster]] ([[User talk:Silkroadster|talk]]) 14:24, 3 January 2025 (UTC) |
|||
:@[[User:Silkroadster|Silkroadster]] Sometimes searching the [[WP:RSN]] archives is helpful. [[User:Gråbergs Gråa Sång|Gråbergs Gråa Sång]] ([[User talk:Gråbergs Gråa Sång|talk]]) 16:39, 3 January 2025 (UTC) |
|||
:Hi @[[User:Silkroadster|Silkroadster]], |
|||
:When [[Wikipedia:Reliable sources/Perennial sources|WP:RSP]] isn’t clear, assess the publisher’s reputation, the author’s expertise, editorial standards, and independence. Check for corroboration in other reliable sources and be cautious of bias or promotional tones. [[User:Ayohama|Ayohama]] ([[User talk:Ayohama|talk]]) 17:21, 3 January 2025 (UTC) |
|||
== Company Information == |
|||
Note that this article may be deleted at any time if it unquestionably meets the speedy deletion criteria, or if an explanation posted to the talk page is found to be insufficient. |
|||
I'm needing assistance to add my small business to the Wikipedia results. I started a page, but was presented with a message that stated my content as 'non-constructive'. What do I need to do or where can I find guidelines on how to add our information ?? [[User:Rdmind|Rdmind]] ([[User talk:Rdmind|talk]]) 15:35, 3 January 2025 (UTC) |
|||
: <small>'''''Nominator''': Please consider placing the [[Help:Template|template]]:''</small> |
|||
: <small><code>''<nowiki>{{subst:</nowiki>[[Template:Db-spam-notice|db-spam-notice]]<nowiki>|Gnoppix|header=1}} ~~~~</nowiki>''</code></small> |
|||
: ''<small>on the talk page of the author.</small>'' |
|||
:{{u|Rdmind}} Hello and welcome. You have no edits other than this one, so I can't look at your prior attempt. You seem to have a common misunderstanding about Wikipedia. It is [[WP:NOTDATABASE|not a database]] of things that exist, and is also not a place for companies to tell about themselves and what they do. Wikipedia articles about companies must summarize what independent [[WP:RS|reliable sources]] with significant coverage have chosen on their own to say about the company, showing how it meets our [[WP:ORG|special Wikipedia definition of a notable company]]. The vast majority of the millions of companies on this planet do not merit Wikipedia articles(even some that may have them, but we just haven't removed yet). Also note that a Wikipedia article [[WP:PROUD|is not necessarily desirable]]. There are good reasons to not want one. Our articles are typically written by independent editors wholly unconnected with the topic. I would suggest that you go on about the work of your company; if your company is truly notable, someone will eventually write about it. [[User:331dot|331dot]] ([[User talk:331dot|talk]]) 15:41, 3 January 2025 (UTC) |
|||
Note to page author: you have not edited the article talk page yet. If you wish to contest this speedy deletion, clicking the button above will allow you to [https://en.wikipedia.org/enwiki/w/index.php?title=Talk:Gnoppix&action=edit leave a talk page message] explaining why you think this article should not be deleted. |
|||
::{{ping|331dot}} see EF. [[User:DMacks|DMacks]] ([[User talk:DMacks|talk]]) 16:45, 3 January 2025 (UTC) |
|||
:::Ah, I see now- I didn't realize the filter blocked things as "non constructive". I now see that their username violates policy. [[User:331dot|331dot]] ([[User talk:331dot|talk]]) 16:48, 3 January 2025 (UTC) |
|||
:To add your business to Wikipedia, it must meet [[WP:Notability (organizations)]], with significant coverage in independent, reliable sources. Avoid promotional content and write neutrally. If you’re connected to the business, disclose your [[WP:COI|conflict of interest]] and suggest edits instead of directly creating or editing. Use the [[WP:Article wizard]] to draft and submit for review. But honestly speaking: take the advice from @[[User:331dot|331dot]]- go on about the work of your company [[User:Ayohama|Ayohama]] ([[User talk:Ayohama|talk]]) 17:17, 3 January 2025 (UTC) |
|||
== Weatherbox placement == |
|||
<small>If you have already posted to the talk page but this message is still showing up, try '''[https://en.wikipedia.org/enwiki/w/index.php?title=Gnoppix&action=purge purging the page cache]'''.</small> |
|||
Can't help but feel the weatherbox on the [[Cherry Hill, New Jersey]] article is in a space that leaves a lot of blank space behind. Can it be moved easily? (I should clarify I'm unsure how to do this) [[User:Finitestory|Finitestory]] ([[User talk:Finitestory|talk]]) 00:34, 4 January 2025 (UTC) |
|||
[[User:Tumrabert|Tumrabert]] ([[User talk:Tumrabert|talk]]) 15:13, 22 January 2024 (UTC) |
|||
:Yes, the weatherbox can be moved easily. Edit the article, find the weatherbox code, and relocate it to a section where it fits better, like "Climate" or below another relevant subsection, to reduce blank space. [[User:Ayohama|Ayohama]] ([[User talk:Ayohama|talk]]) 07:22, 4 January 2025 (UTC) |
|||
:Hello, Tumrabert, and welcome to the teahouse. |
|||
:As the notice says, you can context the speedy deletion by picking the button. But reading the article, the first paragraph is entirely promotional. |
|||
:{{HD/WINI}} |
|||
:An article should be based almost entirely on what those independent sources say, but not one of the sources you've cited is independent. Please read about [[WP:NSOFTWARE|notability]], and [[WP:your first article|your first article]]. |
|||
:A more general suggestion: {{HD/LTCF}} [[User:ColinFine|ColinFine]] ([[User talk:ColinFine|talk]]) 15:19, 22 January 2024 (UTC) |
|||
== New page reviewer questions == |
|||
::{{reply|Tumrabert}} I have moved your article to draft space ([[Draft:Gnoppix]]) to give you time to improve it before submitting it for review. In draft space, you don't need to worry about someone coming along and deleting it. ~[[User:Anachronist|Anachronist]] <small>([[User talk:Anachronist|talk]])</small> 15:32, 22 January 2024 (UTC) |
|||
:::@[[User:Anachronist|Anachronist]] I think the history of the numerous attempts to create this article has gone beyond a joke. Content was not only highly promotional, but also ''only'' sourced to Gnoppix-related sites. I have therefore [[WP:SALT]]ed the article, meaning it will require administrator approval should anyone wish to create it in the future. [[User:Nick Moyes|Nick Moyes]] ([[User talk:Nick Moyes|talk]]) 21:18, 22 January 2024 (UTC) |
|||
:Articles can be speedily deleted for a variety of different reasons, including the ones you mentioned. It is customary to place a notice on pages that have been nominated for speedily deletion while also notifying the creator of a nominated page. Speedy deletion is used to bypass the longer process of voting on whether or not to delete an article. [[User:Ishitomo|Ishitomo]] ([[User talk:Ishitomo|talk]]) 05:10, 23 January 2024 (UTC) |
|||
So I'm currently applying for NPR rights, and I have a couple questions about what I can do with NPR rights: <br> |
|||
[[Draft:Gnoppix]] Speedy deleted (again!) and Tumrabert indefinitely blocked. [[User:Gnoppix]] also indefinitely blocked. Both for advertising/promotional. Sockpuppeting possible, but not raised as a blocking cause. [[User:David notMD|David notMD]] ([[User talk:David notMD|talk]]) 16:58, 22 January 2024 (UTC) |
|||
{{bullet}} Am I allowed to review pages I created? <br> |
|||
{{bullet}} Do people with NPR rights only review articles or do they also review redirects? <br> |
|||
{{bullet}} Can reviews be undone (that is, can an article be unreviewed if need be)? [[User:RedactedHumanoid|RedactedHumanoid]] ([[User talk:RedactedHumanoid|talk]]) 02:11, 4 January 2025 (UTC) |
|||
:Hello, {{u|RedactedHumanoid}}! In order, no, both, and yes. A little more detail: |
|||
:*You can’t review pages you created, but creators who create a lot of unproblematic articles are often assigned the [[WP:AUTOPATROLLED|autopatrolled]] right so that reviewers don’t have to check extra articles unnecessarily. |
|||
:*NPPers can and do review both articles and redirects. In fact, there’s currently a [[WP:JAN25|backlog elimination drive]] aimed at reviewing both. |
|||
:*Articles can be unreviewed. |
|||
:Hope this helps! Happy editing, [[User:Perfect4th|Perfect4th]] ([[User talk:Perfect4th|talk]]) 03:02, 4 January 2025 (UTC) |
|||
::Alright, thank you. [[User:RedactedHumanoid|RedactedHumanoid]] ([[User talk:RedactedHumanoid|talk]]) 03:11, 4 January 2025 (UTC) |
|||
== Notability of Isa Isarb == |
|||
:{{reply|David notMD}} This seems wrong. User:Gnoppix is a very old account with no edits in 11 years (it created the mainspace article [[Gnoppix]] then) and there was no need to block it. The article was created in mainspace even before then, and prod-deleted and restored. For some reason Tumrabert, a fairly new account created last October, moved it all over the place between draft, main space, and User:Gnoppix for some reason. I attempted to clean up the mess, deleting the user page, moving the main space version back to draft. |
|||
:While it was in main space, it was tagged for G11 speedy deletion and administrator [[User:Sj]] declined it, so it was '''ineligible''' for G11 speedy deletion again, but [[User:Theroadislong]], who shouldn't have reversed Sj's administrative decision without prior discussion, deleted it. |
|||
:And Tumrabert was deleted as an advertising-only account by [[User:HJ Mitchell]] when it seems to me like overkill given the history here. |
|||
:What a mess. ~[[User:Anachronist|Anachronist]] <small>([[User talk:Anachronist|talk]])</small> 04:49, 23 January 2024 (UTC) |
|||
:: The poor original Gnoppix account... Minor Linux distros are often a mess. (Gnoppix these days is built on [[Kali Linux]], a significantly more notable distro, which article is nevertheless only slightly more contentful.) In general I'm for less drama and letting new accounts work on drafts, but given the history of title-dancing + past deletion I can't imagine that draft being accepted, on closer look there really are no modern sources to be found. Serious interest in the topic could start by adding a section to the Kali article. <span style="color:#666">– [[User:Sj|SJ]][[User Talk:Sj|<span style="color:#f90;"> +</span>]]</span> 05:56, 23 January 2024 (UTC) 06:11, 23 January 2024 (UTC) |
|||
Besides Nigel Ng, Isa Isarb is one of the most famous non-brand Youtube Channel with over 2,7 million subscribers. Cited by many and also mainly using Malay language [[User:Protocetus1|Protocetus1]] ([[User talk:Protocetus1|talk]]) 04:04, 4 January 2025 (UTC) |
|||
== Editing a co workers page == |
|||
:Teahouse is the wrong place to suggest an article should be created. There is a requests site but it is pretty much a "dead letter", meaning few if any article suggestions are acted upon. [[User:David notMD|David notMD]] ([[User talk:David notMD|talk]]) 05:41, 4 January 2025 (UTC) |
|||
::{{u|Protocetus1}}, being {{tpq|one of the most famous}} and having {{tpq|over 2,7 million subscribers}} is not in itself evidence of notability. What is required to establish notability are references to several published reliable sources that are entirely independent of Isarb and that devote significant coverage to Isarb. Nothing else matters. [[User:Cullen328|Cullen328]] ([[User talk:Cullen328|talk]]) 08:27, 4 January 2025 (UTC) |
|||
== Page text length and "stub" status == |
|||
Hi! I work at a music studio and a couple people that I work with want me to edit their Wikipedia's but when I tried in the past it either didn't save it or told me that if I tried doing it again it would delete their page. How can I edit it without any of those things happening? [[Special:Contributions/38.142.212.186|38.142.212.186]] ([[User talk:38.142.212.186|talk]]) 15:14, 22 January 2024 (UTC) |
|||
At present time, is a page developed to a length of, say, 400 words—''not'' counting many listed references—still considered a stub?[[User:Joel Russ|Joel Russ]] ([[User talk:Joel Russ|talk]]) 04:16, 4 January 2025 (UTC) |
|||
:Hello and welcome to the Teahouse. You should review the [[WP:COI|conflict of interest policy]] and [[WP:ER|how to make edit requests]] if you intend to contribute about co-workers. If you are doing so as part of your job duties, the Terms of Use require you to [[WP:PAID|declare as a paid editor]]. These things are easier to do with an account, but you must do them even if you choose to not create an account. [[User:331dot|331dot]] ([[User talk:331dot|talk]]) 15:22, 22 January 2024 (UTC) |
|||
:Hello, and welcome to the Teahouse. Thanks for asking. |
|||
:Please note that, as a colleague, you have a [[WP:conflict of interest|conflict of interest]], and as you are employed there, you almost certainly count as a [[WP:paid editor|paid editor]], even if you are not specifically employed to edit Wikipedia. |
|||
:You '''must''' make a formal declaration of your status, and then you may make [[WP:edit request|edit request]]s for changes to articles where you have a conflict of interest. |
|||
:Please remember that Wikipedia's articles about your colleagues do not belong to them, are not controlled by them, and will not necessarily say what they want them to say. If you provide [[WP:42|independent, reliably published sources|]] for any information you want to add, then it is likely that whichever volunteer editor deals with your edit request will carry it out. But if the material you want to add is at all promotional, or is not adequately sourced, then it will not be inserted into the article. [[User:ColinFine|ColinFine]] ([[User talk:ColinFine|talk]]) 15:23, 22 January 2024 (UTC) |
|||
:You cannot edit your coworkers page because you have a conflict of interest. Wikipedia takes combating promotional editing very seriously and if you persist, you will likely be blocked. [[User:Ishitomo|Ishitomo]] ([[User talk:Ishitomo|talk]]) 05:11, 23 January 2024 (UTC) |
|||
:Stub status depends on content depth and coverage, not just word count. A 400-word page could still be a stub if it lacks comprehensive information, see [[Wikipedia:Stub]] [[User:Ayohama|Ayohama]] ([[User talk:Ayohama|talk]]) 07:20, 4 January 2025 (UTC) |
|||
== Professor Mary Mellor == |
|||
== Saving a template for re-use in high volume == |
|||
Mary Mellor – UK sociologist who moved to ecofeminist ideas from an interest in cooperatives. Her books Breaking the Boundaries and Feminism and Ecology are grounded in a materialist analysis.[citation needed] |
|||
I use the "r from alternative language" template all the time. I would like to know if there is a way to save a template or have like a quick-use list of 3 to 5 templates that I use frequently when creating valid redirects from varied different languages etc.? |
|||
This is part of a post on econofeminism. I would like to create a page for Professor Mary Mellor, can you help? [[Special:Contributions/2A02:C7C:7A06:5400:D88A:556D:ACEB:979E|2A02:C7C:7A06:5400:D88A:556D:ACEB:979E]] ([[User talk:2A02:C7C:7A06:5400:D88A:556D:ACEB:979E|talk]]) 15:52, 22 January 2024 (UTC) |
|||
So for example. Lets say I see a town in Palestine or Israel that has an Arabic or Hebrew language word that could use a redirect that then requires the template "r from alternative language", I want to be able to just click that "r from alternative language" quickly without needing to type that full text in or even partial text in a search for the template again each and every time to use it. [[User:Iljhgtn|Iljhgtn]] ([[User talk:Iljhgtn|talk]]) 04:21, 4 January 2025 (UTC) |
|||
:Hi there, {{ping|2A02:C7C:7A06:5400:D88A:556D:ACEB:979E}} you should read [[Help:Your first article]]. It will tell you everything you need to know about creating an article. If you have a [[WP:COI]] with the subject, you should disclose it, and also make sure it follows our [[wp:n|notability guidelines]]. [[User:The Corvette ZR1|<b style="color:#ff6600;">'''''The 🏎 Corvette 🏍 ZR1'''''</b>]]<sup>[[User talk:The Corvette ZR1|<b style="color:#0a0a0a;">''(The Garage)''</b>]]</sup> 16:27, 22 January 2024 (UTC) |
|||
:Signing up for an account is recommended but not required. [[User:David notMD|David notMD]] ([[User talk:David notMD|talk]]) 17:02, 22 January 2024 (UTC) |
|||
:Welcome to the Teahouse! Creating a new Wikipedia article can be quite challenging, especially if you do not have a lot of experience editing existing Wikipedia articles. To learn how to edit, I suggest you start at [[Help:Introduction]], and then spend a significant amount of time editing existing articles to hone your skills. Once you're ready to create an article, you would gather multiple [[WP:IS|independent]] [[WP:RS|reliable sources]] that have provided [[WP:SIGCOV|significant coverage]] of the subject, and determine whether it meets Wikipedia's criteria for inclusion, called "[[WP:NBIO|notability]]". If so, you could follow the instructions at [[Help:Your first article]] and summarize what the sources have published, and be prepared for a process that may include waiting for review, declines, and rewrites before an article is accepted. Hope this helps, and happy editing! [[User:GoingBatty|GoingBatty]] ([[User talk:GoingBatty|talk]]) 17:18, 22 January 2024 (UTC) |
|||
:You should create an account, make ten edits, and wait four days. After that, you will be able to create articles without going through the review process. Note that your article will still be reviewed by patrollers gunning for adminship, hoping to boost their AfD and CSD stats. To give your article a better chance of surviving this process, make sure it has a lot of sources to make the subject appear notable. [[User:Ishitomo|Ishitomo]] ([[User talk:Ishitomo|talk]]) 05:14, 23 January 2024 (UTC) |
|||
:@[[User:Iljhgtn|Iljhgtn]]: One really good but rarely-mentioned tool that can help you with this is [[m:TemplateScript|TemplateScript]]. I'll be {{Working}} on an implementation of that tool specifically for this purpose. [[User:JJPMaster|JJP]]<sub>[[User talk:JJPMaster|Mas]]<sub>[[Special:Contributions/JJPMaster|ter]]</sub></sub> ([[She (pronoun)|she]]/[[Singular they|they]]) 05:24, 4 January 2025 (UTC) |
|||
== Template removal question == |
|||
::Thank you. That would be great! [[User:Iljhgtn|Iljhgtn]] ([[User talk:Iljhgtn|talk]]) 05:28, 4 January 2025 (UTC) |
|||
:::I figured it out; add the following to [[Special:MyPage/common.js]]: |
|||
Hello, Teahouse. Today my question is: If an expansion template on an article is there, but upon research there is no reliable sources for citation/ expansion on the topic, how should one proceed in regards to the template? Best regards, [[User:UnexpectedSmoreInquisition|UnexpectedSmoreInquisition]] aka USI ([[User talk:UnexpectedSmoreInquisition|talk]]) 17:15, 22 January 2024 (UTC) |
|||
:Hi there, {{reply|UnexpectedSmoreInquisition}} welcome to the teahouse! I would suggest leaving it there, because who knows, maybe a reliable source will pop up one day, or is already there. Also, can you show me where you saw this? [[User:The Corvette ZR1|<b style="color:#ff6600;">'''''The 🏎 Corvette 🏍 ZR1'''''</b>]]<sup>[[User talk:The Corvette ZR1|<b style="color:#0a0a0a;">''(The Garage)''</b>]]</sup> 17:23, 22 January 2024 (UTC) |
|||
::I didn't see it anywhere in particular, just in general. Thanks! [[User:UnexpectedSmoreInquisition|UnexpectedSmoreInquisition]] aka USI ([[User talk:UnexpectedSmoreInquisition|talk]]) 17:25, 22 January 2024 (UTC) |
|||
:{{ec}} @[[User:UnexpectedSmoreInquisition|UnexpectedSmoreInquisition]]: Welcome to the Teahouse! You have several options: |
|||
:* Do nothing, as someone else may find sources you cannot (such as offline sources) |
|||
:* Discuss the issue on the article talk page, explaining the details of where you researched |
|||
:* If there are not enough reliable sources to keep it as an article, consider the [[Wikipedia:Articles for deletion]] process (especially [[WP:BEFORE]]) |
|||
:* If there are enough reliable sources to keep it as an article, remove the template and the unsourced information (with descriptive edit summaries) |
|||
:Happy editing! [[User:GoingBatty|GoingBatty]] ([[User talk:GoingBatty|talk]]) 17:26, 22 January 2024 (UTC) |
|||
::That's another thing- I'm sure that there's no perfect solution to this problem, but I've found that talk page visibility has been low- I don't believe that I've gotten any responses from questions there yet. I know that this is what project pages are for, but I've only gotten a reply once there in the 5-7ish times I've tried that. Should I come here in that scenario? Anyways, thanks Batty. Best regards, [[User:UnexpectedSmoreInquisition|UnexpectedSmoreInquisition]] aka USI ([[User talk:UnexpectedSmoreInquisition|talk]]) 17:52, 22 January 2024 (UTC) |
|||
:::@[[User:UnexpectedSmoreInquisition|UnexpectedSmoreInquisition]]: If no one responds to your article talk page conversation, you can invite editors involved in a related WikiProject to the conversation. [[User:GoingBatty|GoingBatty]] ([[User talk:GoingBatty|talk]]) 19:13, 22 January 2024 (UTC) |
|||
:Templates can be added to articles to notify readers of any problems the article has or any maintenance it may need. There are many different kinds of templates, which have different appearances depending on how you are accessing Wikipedia. If you want to change the appearance of templates, try adjusting your settings and reconfiguring your browser firewall. [[User:Ishitomo|Ishitomo]] ([[User talk:Ishitomo|talk]]) 05:16, 23 January 2024 (UTC) |
|||
== Finding low-importance articles marked with advert template == |
|||
Hello! I saw a page recently that read like an article and had little encyclopedic value, in my opinion. I've marked that one for deletion, but I think there are likely a lot of articles like it. Mainly low-importance articles, or articles not part of a wikiproject. I figured out how to use deepcats to search for low importance articles with the advert template, but the search errored because there were too many categories. Is there a way to look through a subset of the results at a time to avoid that error? |
|||
My goal right now is to mark pages for deletion that have little to no encyclopedic value or relevance at present, not to revise larger, more relevant articles that happen to have some content that reads as an advert. From my experience using Wikipedia just as a reader, I've seen quite a few that don't seem to have much use for Wikipedia, and seem to exist only as an attempt to say "we have a Wikipedia article!". Written either by the subject of the article or by a fan or PR person. |
|||
So I want to look for articles that are ''more likely'' to be spam/adverts in their entirety so that I can mark them for deletion, without having to sort through as many articles that are more appropriate for a rewrite or revision. [[User:Dvallin|Dvallin]] ([[User talk:Dvallin|talk]]) 17:46, 22 January 2024 (UTC) |
|||
:I could be wrong, but unless you download the database yourself and devise your own search to combine [[:Category:Articles with a promotional tone]] with low-importance assessments, there isn't really a good way to search for what you want. |
|||
:One approach would simply to be go through the articles in that category, improving or tagging as you go. ~[[User:Anachronist|Anachronist]] <small>([[User talk:Anachronist|talk]])</small> 18:00, 22 January 2024 (UTC) |
|||
:The difficulty in finding those articles that exist solely as advertisements that haven't been deleted already is that they probably are out there with little to no maintenance tags on them. Anachronist has pointed out the most straightforward way of finding what you're looking for. It may do some good to limit your search to articles that are start-class, stub-class, or unassessed. [[User:Reconrabbit|<span style="color:#6BAD2D">Recon</span>]][[User talk:Reconrabbit|<span style="color:#2F3833">rabbit</span>]] 18:18, 22 January 2024 (UTC) |
|||
:@[[User:Dvallin|Dvallin]]: Welcome to the Teahouse! You could try reviewing those articles where you have an instance and/or knowledge. You could try adding another category to deepcats, such as [[:Category:Software]] and its subcategories. Another option is the [https://bambots.brucemyers.com/cwb/bycat/Software_Free_and_open-source_software_task_force.html Cleanup listing for WikiProject Software/Free and open-source software task force] report bundles articles by templates on the articles. Hope these help, and happy editing! [[User:GoingBatty|GoingBatty]] ([[User talk:GoingBatty|talk]]) 19:12, 22 January 2024 (UTC) |
|||
:There are many ways to do this on Wikipedia, mainly by searching through projects where these things are curated. Your best bet is to post your question at [[WP:ANI]] or another noticeboard such as [[WT:ACN]]. [[User:Ishitomo|Ishitomo]] ([[User talk:Ishitomo|talk]]) 05:19, 23 January 2024 (UTC) |
|||
== Adding contributions to top navigation bar? == |
|||
Currently the buttons at the top of the screen (when scrolled all the way up) are alerts, notifications, watchlist, then a dropdown with more options. I'd like to add contributions to the list so I don't have to navigate through the dropdown each time. I know it's something to do with css but I have no experience with it, and I'd like someone to provide me with the code to do so, if it is possible. [[User:Redoct87|Redoct87]] ([[User talk:Redoct87|talk]]) 19:40, 22 January 2024 (UTC) |
|||
:Hi [[User:Redoct87|Redoct87]], welcome to the Teahouse. Saving the below in [[Special:MyPage/vector-2022.js]] should add a new link on "C" when you have the skin Vector 2022 at [[Special:Preferences#mw-prefsection-rendering]]. [[User:PrimeHunter|PrimeHunter]] ([[User talk:PrimeHunter|talk]]) 20:30, 22 January 2024 (UTC) |
|||
::It works, thanks! [[User:Redoct87|Redoct87]] ([[User talk:Redoct87|talk]]) 22:13, 22 January 2024 (UTC) |
|||
:The top navigation bar can be customized to include a number of user specific interface options that account for run time variability in nested virtual algorithms. Simply reset your data input formatting to synchronize processing speed. [[User:Ishitomo|Ishitomo]] ([[User talk:Ishitomo|talk]]) 05:21, 23 January 2024 (UTC) |
|||
<syntaxhighlight lang="javascript"> |
<syntaxhighlight lang="javascript"> |
||
//<nowiki> |
|||
$( document ).ready( function() { |
|||
$.ajax('//tools-static.wmflabs.org/meta/scripts/pathoschild.templatescript.js', { dataType:'script', cache:true }).then(function() { |
|||
mw.util.addPortletLink( |
|||
pathoschild.TemplateScript.add([ |
|||
'p-vector-user-menu-overflow', |
|||
{ |
|||
mw.util.getUrl( 'Special:Contributions/Redoct87' ), |
|||
category: 'Redirect templates', |
|||
'C', |
|||
name: 'R from alternative language (Hebrew to English)', |
|||
null, |
|||
template: '{{R from alternative language|he|en}}', |
|||
'Contributions' |
|||
position: 'after', |
|||
); |
|||
editSummary: 'Adding redirect template' |
|||
}, |
|||
{ |
|||
category: 'Redirect templates', |
|||
name: 'R from alternative language (Arabic to English)', |
|||
template: '{{R from alternative language|ar|en}}', |
|||
position: 'after', |
|||
editSummary: 'Adding redirect template' |
|||
} |
|||
]); |
|||
}); |
}); |
||
//</nowiki> |
|||
</syntaxhighlight> |
</syntaxhighlight> |
||
Then, disable the "Syntax" button in the editing window (if it's enabled). Then, on the sidebar, you should see a new section called "Redirect templates" with the two templates I've seen you use in [[Special:NewPagesFeed]]. If you ever need to use more, you can add a similar [[JSON]] object to the existing ones, or check out [[WP:CAPRICORN|Capricorn]]. [[User:JJPMaster|JJP]]<sub>[[User talk:JJPMaster|Mas]]<sub>[[Special:Contributions/JJPMaster|ter]]</sub></sub> ([[She (pronoun)|she]]/[[Singular they|they]]) 06:13, 4 January 2025 (UTC) |
|||
== |
== Hang on... == |
||
Hi, could you please tell me which citation is not reliable in this article? |
|||
I cited everything. am not sure what am i missing here to make the article public. [[User:Nishikanthprabhu|Nishikanthprabhu]] ([[User talk:Nishikanthprabhu|talk]]) 19:57, 22 January 2024 (UTC) |
|||
*Courtesy link: [[Draft:Mynampally Rohit]]. [[User:331dot|331dot]] ([[User talk:331dot|talk]]) 20:21, 22 January 2024 (UTC) |
|||
*:Thank you for clarifying, i will remove the awards and put it under general information.' |
|||
*:The doctor is a known personality in my community. As the youngest member of legislative assembly i would like to contribute his article on Wikipedia. [[User:Nishikanthprabhu|Nishikanthprabhu]] ([[User talk:Nishikanthprabhu|talk]]) 20:57, 22 January 2024 (UTC) |
|||
:{{u|Nishikanthprabhu}} Hello and welcome to the Teahouse. I would ask you what your connection is with this doctor, as you took his picture and he posed for you. |
|||
:Any article about him must summarize what independent [[WP:RS|reliable sources]] with significant coverage have chosen on their own to say about him, showing how he meets the special Wikipedia definition of [[WP:BIO|a notable person]]. None of the sources you have do that. The award does not contribute to notability as the awards itself does not merit an article(like [[Academy Award]] or [[Nobel Peace Prize]]). [[User:331dot|331dot]] ([[User talk:331dot|talk]]) 20:24, 22 January 2024 (UTC) |
|||
::Thank you for clarifying, i will remove the awards and put it under general information.' |
|||
::The doctor is a known personality in my community. As the youngest member of legislative assembly i would like to contribute his article on Wikipedia. [[User:Nishikanthprabhu|Nishikanthprabhu]] ([[User talk:Nishikanthprabhu|talk]]) 21:42, 22 January 2024 (UTC) |
|||
:None of the citations you provided meet [[WP:RS]] criteria, indicating that the subject of your article isn’t notable. [[User:Ishitomo|Ishitomo]] ([[User talk:Ishitomo|talk]]) 05:22, 23 January 2024 (UTC) |
|||
== Help with notability guidelines on companies == |
|||
Hello all, I am working on creating a draft about a online casino, Gamdom. I notice that there are a lot of independant, popular sources in both the news and casino news space that report on it, though I just have a bad feeling it's not notable enough (although it is promoted quite a lot on user-created content websites such as YouTube, and several sponsorships are/were active with them). Could I get some advice on that all? <span style="text-shadow:7px 7px 8px blue; font-family:Papyrus color:#3399ff">[[User:OnlyNano|OnlyNano]]</span> 20:34, 22 January 2024 (UTC) |
|||
: {{re|OnlyNano}} Check out [[WP:NCORP]] for the guidance on this. [[User:RudolfRed|RudolfRed]] ([[User talk:RudolfRed|talk]]) 20:51, 22 January 2024 (UTC) |
|||
::Thanks! <span style="text-shadow:7px 7px 8px blue; font-family:Papyrus color:#3399ff">[[User:OnlyNano|OnlyNano]]</span> 21:30, 22 January 2024 (UTC) |
|||
:::Hello, {{u|OnlyNano}}. I would have major doubts about the reliability and independence of sources in the {{tpq|casino news space}}. As [[WP:NCORP]] says, {{tpq|Attention solely from local media (e.g., the weekly newspaper for a small town), or media of limited interest and circulation (e.g., a newsletter exclusively for people with a very unusual job), is not an indication of notability. At least one regional, statewide, provincial, national, or international source is necessary.}} [[User:Cullen328|Cullen328]] ([[User talk:Cullen328|talk]]) 21:52, 22 January 2024 (UTC) |
|||
::::Hey there, would you mind checking on the draft's sources and letting me know if I need additional ones? Yahoo Finance reported on their partnership with [[Usain Bolt]], which I would say is pretty big for information. The others are either primary sources (about their legalization, etc), and independent sources (such as HLTV, a CS:GO news source) reported on a tournament they sponsored. It's seeming to be a bit more notable as I go, though I'd love other editor's input on this. Thanks! ([[Draft:Gamdom|Draft]]) <span style="text-shadow:7px 7px 8px blue; font-family:Papyrus color:#3399ff">[[User:OnlyNano|OnlyNano]]</span> 22:31, 22 January 2024 (UTC) |
|||
:::::Hello, OnlyNano. Just answering one point: no, their partnership with Usain Bolt is not, of itself, of any great significance. If the article about the partnership talks in some depth about the company (and is not just regurgitating a press release) then it might be useful. [[User:ColinFine|ColinFine]] ([[User talk:ColinFine|talk]]) 22:39, 22 January 2024 (UTC) |
|||
::::::Yes, the article does talk about what Gamdom is. I have found some other interesting sources which describe it, and the founder is an editor of CoinTelegraph, which provides some background information on the company's foundings. I believe the sources should be good, but I'm going to find a bunch more to beef up the information and notability, as I do believe the company is quite notable. We'll see how it goes, though. <span style="text-shadow:7px 7px 8px blue; font-family:Papyrus color:#3399ff">[[User:OnlyNano|OnlyNano]]</span> 22:47, 22 January 2024 (UTC) |
|||
:::::::{{u|OnlyNano}}, Liquipedia is user generated content so not a reliable source. Yahoo Finance is a reprint of a press release so not an independent source. You need references to reliable, independent sources that devote significant coverage to the company. So far, you don't have any. [[User:Cullen328|Cullen328]] ([[User talk:Cullen328|talk]]) 22:52, 22 January 2024 (UTC) |
|||
::::::::Thanks, I'll keep digging! <span style="text-shadow:7px 7px 8px blue; font-family:Papyrus color:#3399ff">[[User:OnlyNano|OnlyNano]]</span> 23:17, 22 January 2024 (UTC) |
|||
:You are strongly encouraged to disclose any potential conflict of interest you may have with the company article you intend to edit. Failure to do so may result in a COI investigation. [[User:Ishitomo|Ishitomo]] ([[User talk:Ishitomo|talk]]) 05:23, 23 January 2024 (UTC) |
|||
== User boxes (the continuation) == |
|||
hi everyone! |
|||
I know this question has been asked before but I'll say it again how do you make a User box? [[User:Jude marrero|Jude Marrero \=D]] ([[User talk:Jude marrero|talk]]) 21:04, 22 January 2024 (UTC) |
|||
:[[Help:Userbox Maker]] [[User:Babysharkboss2|<span style="color: red; font-family: Comic Sans MS;">Babysharkboss2!!</span>]] <sup>([[User talk:Babysharkboss2|<span style="color:black">Big scary floating text!! (Talk Page btw)</span>]])</sup> 21:09, 22 January 2024 (UTC) |
|||
== Tone accents in Chinese pinyin: == |
|||
Tone accents in Chinese pinyin: The idea I propose is to add the tonal accents in the Latin alphabet (pinyin) used to transcribe Chinese. There are 4 tonal accents. Please consider making it a rule when using pinyin Chinese so as to convey that Chinese is a tonal language. [[Special:Contributions/2603:7000:4300:DBE:7C7D:9C01:EA30:A2C4|2603:7000:4300:DBE:7C7D:9C01:EA30:A2C4]] ([[User talk:2603:7000:4300:DBE:7C7D:9C01:EA30:A2C4|talk]]) 21:08, 22 January 2024 (UTC) |
|||
:Hello, and welcome to the Teahouse. The policy is stated at [[MOS:CHINA#Romanization]]: {{tq|English Wikipedia uses Hanyu Pinyin without tone marks as the default method of romanising Chinese characters}}, and at [[WP:NC-CHINA#Orthography]]: {{tq|The titles of Chinese entries should follow current academic conventions, which generally means Hanyu Pinyin without tone marks}}. |
|||
:These policies are established by consensus, and you are welcome to try and argue to change the consensus, but I don't think you'll have much success. The place to discuss this is the talk page of one of those, probably [[WT:Manual of Style/China- and Chinese-related articles]]. Please search this and its archives first, to see if there are any previous discussions on the question. [[User:ColinFine|ColinFine]] ([[User talk:ColinFine|talk]]) 21:29, 22 January 2024 (UTC) |
|||
== Making an article "Aziz Alili" in Bosnian == |
|||
Hello, I started making the article "Aziz Alili" in my Bosnian language, after seeing no such article in Bosnian or English. Only just now have I found that the article does indeed exist in the [[:sq:Aziz_Alili|sq.wiki domain]] (don't know the language). It is a fairly small article while I wanted to do a medium sized article about it. Now I'm wondering if I should translate that article or just make it anew in Bosnian like I have already started. Thanks a lot in advance. [[User:A flurry of stars|A flurry of stars]] ([[User talk:A flurry of stars|talk]]) 21:31, 22 January 2024 (UTC) |
|||
:Hello, {{u|A flurry of stars}}. Each language version of Wikipedia operates independently, so we cannot give you detailed advice about the [[Bosnian Wikipedia]]. You will have to ask there. Any articles created here must be in English. Good luck. [[User:Cullen328|Cullen328]] ([[User talk:Cullen328|talk]]) 21:46, 22 January 2024 (UTC) |
|||
:Welcome to the Teahouse, [[User:A flurry of stars|A flurry of stars]]. That is Albanian Wikipedia and the article is sourced to a single database entry which does not exist. It would be unwise simply to translate it into English. We have pretty tough criteria for notability (see [[WP:NBIO]]), and require good, reliable sources as the basis from which you should write any content (not vice versa). |
|||
:Because this is English Wikipedia, it is not the right place to start drafting any article in another language. You should do that on the relevant language Wikipedia. Again, base what you write on published sources, not a translation of (potentially) nonsense that you might read on any Wikipedia page. |
|||
:For English Wikipedia see [[WP:YFA|this page]] for guidance on creating an article from scratch (never an easy task, especially for a beginner). Regards, [[User:Nick Moyes|Nick Moyes]] ([[User talk:Nick Moyes|talk]]) 21:46, 22 January 2024 (UTC) |
|||
:Thanks to the both of you, I appreciate the help. Cheers [[User:A flurry of stars|A flurry of stars]] ([[User talk:A flurry of stars|talk]]) 22:56, 22 January 2024 (UTC) |
|||
== Question about COI == |
|||
Ruth Ashton Taylor was my grandmother-in-law. Am I allowed to edit her page? I did so before realizing I should ask first. Sorry. Thanks, Nirva20 [[User:Nirva20|Nirva20]] ([[User talk:Nirva20|talk]]) 22:10, 22 January 2024 (UTC) |
|||
::Others may disagree, but my interpretation of [[WP:CoI]] is that you probably do have a "conflict of interest" (given that she was a relative by marriage whom presumably you knew personally). |
|||
::That doesn't mean you ''must not'' edit the article, but you should declare this COI on your User page (which you haven't created yet, but that is trivially easy – just click on your red signature name and type something to start it). |
|||
::You ''also'' must ensure that ''anything'' you add to the article is [[Wikipedia:Citing sources|cited]] to ''published'' [[Wikipedia:Reliable sources|Reliable sources]] that are [[Wikipedia:Independent sources|independent]] of her or her friends/family/associates. Doubtless you know a great deal about her, but unless the information has been published, and you cite its published source, you must not add it. |
|||
::If there's something in the article you know to be false that is ''not'' cited, you may remove it: but if it's false and cited (maybe the published source made a mistake), you should discuss its removal on the article's Talk page, be prepared to have your removal reverted, and then discuss the matter with the reverter – this is called the [[Wikipedia:BOLD, revert, discuss cycle]] and is a normal part of Wikipedia editing. It may be that we end up presenting information from two sources that disagree ("A says this, but B says this.") allowing readers to make up their own minds. |
|||
::If you have an edit and citable source for something but are hesitant about editing the article with it, present it as a proposal on the Talk page and add an [[Wikipedia:Edit requests|Edit request template]] so another more experienced editor can evaluate it and make it if they think it's appropriate. |
|||
::Please click through all of the blue links I've included above and at least skim the material they lead to. Hope this helps, and happy editing! {The poster formerly known as 87.81.230.195} [[Special:Contributions/176.24.47.60|176.24.47.60]] ([[User talk:176.24.47.60|talk]]) 22:47, 22 January 2024 (UTC) |
|||
:{{Nirva20}} As an editor with a conflict of interest, your best approach is to propose any substantive changes on the article talk page. You can preface your proposal with the tag <nowiki>{{Edit COI}}</nowiki> to cause your request to be listed on a category page that is monitored by some editors. |
|||
:Generally, you can edit the article yourself to make minor corrections to spelling, grammar, names, dates, and numbers. You can add citations to reliable sources that are independent of the article topic. You can revert obvious vandalism. But anything more substantive, you should propose on the talk page. ~[[User:Anachronist|Anachronist]] <small>([[User talk:Anachronist|talk]])</small> 03:08, 23 January 2024 (UTC) |
|||
==Burns and Porter== |
|||
[[Burns and Porter]] ARTICLE was written by the subject, Sharon Burns [[User:3MRB1|3MRB1]] ([[User talk:3MRB1|talk]]) 23:15, 22 January 2024 (UTC) |
|||
:{{reply|3MRB1}} And your point is? I note that the article was created over a dozen years ago, back in 2011. Do you feel the topic isn't sufficiently notable to merit an article here? Do you see any evidence of non-neutral prose? ~[[User:Anachronist|Anachronist]] <small>([[User talk:Anachronist|talk]])</small> 03:04, 23 January 2024 (UTC) |
|||
== Review == |
|||
Can someone look up at my recent assessments made on the article talk pages. I would like to know if I am placing the class and importance parameters at the right place or not. And ensure that I am not missing anything to add. Thank you in advance. (Please ping when replying) [[User:456legend|<span style="background:#ADD8E6; color:black; padding:5px; box-shadow:darkgray 2px 2px 2px;">456legend</span>]][[User talk:456legend|<span style="background:#FF474C;color:black; padding:2px; box-shadow:darkgray 2px 2px 2px;">talk</span>]] 23:20, 22 January 2024 (UTC) |
|||
:@[[User:456legend|456legend]] I just spot-checked a few, looks fine. By the way, you can also use [[WP:RATER]] for this, if you would like an easier method. -- [[User:Asilvering|asilvering]] ([[User talk:Asilvering|talk]]) 23:24, 22 January 2024 (UTC) |
|||
::Hi @[[User:Asilvering|Asilvering]], Thank you for the insights. This looks good but I need some time to understand the coding I believe. I will better do it manually for now. Thank you [[User:456legend|<span style="background:#ADD8E6; color:black; padding:5px; box-shadow:darkgray 2px 2px 2px;">456legend</span>]][[User talk:456legend|<span style="background:#FF474C;color:black; padding:2px; box-shadow:darkgray 2px 2px 2px;">talk</span>]] 00:15, 23 January 2024 (UTC) |
|||
:::@[[User:456legend|456legend]] Of course, feel free to do it manually, but I do want to point out that there's no coding (not even wikicode) involved with rater. All you need to do is copy this exact text: |
|||
:::<nowiki>{{subst:lusc|User:Evad37/rater.js}}</nowiki> |
|||
:::onto [[Special:MyPage/common.js]]. Once you hit "publish changes", everything else is done for you. -- [[User:Asilvering|asilvering]] ([[User talk:Asilvering|talk]]) 00:50, 23 January 2024 (UTC) |
|||
::::@[[User:Asilvering|Asilvering]], Okayy. I should have cared to read the entire content properly. Anyways thank you. [[User:456legend|<span style="background:#ADD8E6; color:black; padding:5px; box-shadow:darkgray 2px 2px 2px;">456legend</span>]][[User talk:456legend|<span style="background:#FF474C;color:black; padding:2px; box-shadow:darkgray 2px 2px 2px;">talk</span>]] 01:14, 23 January 2024 (UTC) |
|||
== Proper way to cite online lecture slides == |
|||
Hey everybody, |
|||
I found an online PDF of some lecture slides presented at Johnson Spaceflight Center back in 2017, and I want to use it as a resource for the [[STS-41-B]] article. I'm not sure how I would cite it though, specifically with the CS1 Style Templates. Would it fall under {cite web} or is there a more specific template for lectures/presentations? Also, since this presentation was made and presented by and for NASA employees, would the images inside, which were presumably prepared by NASA employees, be in the public domain? [[User:SpacePod9|SpacePod9]] ([[User talk:SpacePod9|talk]]) 23:29, 22 January 2024 (UTC) |
|||
:Hello, {{u|SpacePod9}}. Perhaps [[Template:Cite conference]] might work for you. You are correct that text and images created by employees of the US federal government as part of their job duties are in the public domain. This does not apply to all state and local governments. [[User:Cullen328|Cullen328]] ([[User talk:Cullen328|talk]]) 00:08, 23 January 2024 (UTC) |
|||
:Hi @[[User:SpacePod9|SpacePod9]]. I tend to use "cite web", but see [[Template:Citation#Conference papers and public lectures]] for another way of doing it. As far as copyright, don't assume anything about who made the images. They could be using images from other sources. It would help if you provided a link to the source in question. [[User:StarryGrandma|StarryGrandma]] ([[User talk:StarryGrandma|talk]]) 00:21, 23 January 2024 (UTC) |
|||
:: Thanks y'all, this (https://ntrs.nasa.gov/enwiki/api/citations/20170002910/downloads/20170002910.pdf) is the link to the website/pdf/slides in question. <!-- Template:Unsigned --><small class="autosigned">— Preceding [[Wikipedia:Signatures|unsigned]] comment added by [[User:SpacePod9|SpacePod9]] ([[User talk:SpacePod9#top|talk]] • [[Special:Contributions/SpacePod9|contribs]]) 01:54, 23 January 2024 (UTC)</small> <!--Autosigned by SineBot--> |
|||
== Is an official transit map fair-use? == |
|||
Hi! I'm working on this article on the [[REM de l'Est]], a cancelled public transport project in Montreal, and I'm wondering if the official route map of the system could be considered fair use and usable in the article. Right now I've included a simple outline of the map that I made myself and it's obviously not great. I've noticed the [[New York City Subway]] page and other similar articles have maps but I'm a bit confused as to whether or not they're the official one or just really good recreations. Thanks for any help! [[User:WikiFouf|WikiFouf]] ([[User talk:WikiFouf|talk]]) 23:37, 22 January 2024 (UTC) |
|||
:Hello, {{u|WikiFouf}}. Wikipedia's policy on using copyrighted images is much more restrictive than [[fair use]]. Please read the requirements at [[WP:NFCI|Non-free images]]. In most cases, we do not use copyrighted maps, because it is a relatively easy matter to create a freely licensed version. [[User:Cullen328|Cullen328]] ([[User talk:Cullen328|talk]]) 00:14, 23 January 2024 (UTC) |
|||
::Understood, thanks:) [[User:WikiFouf|WikiFouf]] ([[User talk:WikiFouf|talk]]) 00:25, 23 January 2024 (UTC) |
|||
:::I want to add that copyright licenses can differ between transit agencies, and there is accurate transit mapping data out there that ''does'' fall under acceptable free use licenses. For example, in the United States, the Bureau of Transportation Statistics offers [https://geodata.bts.gov/datasets/21e74298378646789afb00db32b31a10_0/explore transit mapping data across the US] that is licensed under CC-BY-3.0, which is [[Wikipedia:File_copyright_tags/Free_licenses#Creative_Commons|an acceptable free license for Wikipedia]]. <span style="font-variant:small-caps">[[User:Orange Suede Sofa|<span style="color:DarkGreen;">Orange Suede Sofa</span>]]</span> ([[User talk:Orange Suede Sofa|talk]]) 01:16, 23 January 2024 (UTC) |
|||
::::Yes, my original answer was based on the assumption that the specific transit map in question is copyright protected. If it can be established through specific evidence that the map is in the public domain or is freely licensed, then what I said above about this specicic image does not apply. But it applies more broadly to images that are legitimately protected by copyright, which is a very low bar for anything published in recent decades. The bottom line is that we must assume that ''anything'' published in the last 95 years is copyright protected, unless we have ''solid'' evidence to the contrary. [[User:Cullen328|Cullen328]] ([[User talk:Cullen328|talk]]) 05:56, 23 January 2024 (UTC) |
|||
== a new article == |
|||
Hi, I'm working on an article and I don't seem to understand how that the reference isn't indepth as it was stated, I'd appreciate you help me figure it out [[User:Starheroine|Starheroine]] ([[User talk:Starheroine|talk]]) 00:10, 23 January 2024 (UTC) |
|||
:Hi @[[User:Starheroine|Starheroine]], you need sources that comply with [[WP:NCORP]]. That means, among other things, that they need to be from [[WP:RS]] and they cannot be interviews. -- [[User:Asilvering|asilvering]] ([[User talk:Asilvering|talk]]) 00:52, 23 January 2024 (UTC) |
|||
== Language issues with making a new article == |
|||
Hello! I am a very new editor, and I wanted to try my hand at making a new wiki page. Some context: a wikipedia page about my grandfather (a hebrew scholar) is written in Spanish, I cannot fluently write wikipedia-worthy articles in Spanish however. I want to write the conjoining page for my grandmother, also a hebrew professor and author, but I am unsure if I am allowed to write her page in english. Thanks! [[User:Dryforester|Dryforester]] ([[User talk:Dryforester|talk]]) 00:59, 23 January 2024 (UTC) |
|||
:hi @[[User:Dryforester|Dryforester]] and welcome to the Teahouse! an article doesn't need to be related to existing articles in English or non-English languages to be eligible for a page, however what it does require is that it falls under the [[WP:notability|notability]] criteria (with [[WP:reliable sources|reliable sources]] to back up these claims), whether or not it is related to existing articles. however do note that since you are their grandchild, you also need to take note of the [[WP:conflict of interest|conflict of interest]] policy here (as you are related to them) before writing the article. happy editing! 💜 <span style="border-radius:4px;background:#edf"> [[User:Melecie|<span style="color:#471a7a">'''mel'''ecie</span>]] </span> [[User talk:Melecie|<span style="color:#471a7a">talk</span>]] - 01:15, 23 January 2024 (UTC) |
|||
::awesome! I will keep the conflict of interest policy in mind as I write :) They are both notable hebrew scholars with multiple published works and awards, I would not have thought wikipedia worthy, but I was doing a bit of googling for fun and found that a complete stranger had already made a page for one of them. My grandmother was just as accoladed and published as her husband, so I do not think notability will be a problem if his page has been kept up! [[User:Dryforester|Dryforester]] ([[User talk:Dryforester|talk]]) 01:21, 23 January 2024 (UTC) |
|||
:::@[[User:Dryforester|Dryforester]] I didn't notice anybody providing a link to the COI page, so here it is: [[WP:COI]]. In very short summary, for existing articles, you should generally make a request on the article talk page for any edits to be made. For new articles, you should use the [[WP:AfC|articles for creation]] process. [[User:Fabrickator|Fabrickator]] ([[User talk:Fabrickator|talk]]) 02:05, 23 January 2024 (UTC) |
|||
::::Sheesh, I now see that there was already a link to the COI page, though there's certainly no harm in mentioning it twice. ;-) [[User:Fabrickator|Fabrickator]] ([[User talk:Fabrickator|talk]]) 02:10, 23 January 2024 (UTC) |
|||
== large non-free file Tag == |
|||
Hello, I recently uploaded a non-free promotional poster [[:File:EXchange (Transit Love) Promotional Poster.webp]] for article [[EXchange (TV series)]]. It got the tag '''large non-free''' file. Can anyone guide me to the correct help article about how to fix it? Thank you in advance! [[User:Shenaall|Shenaall]] ([[User talk:Shenaall|talk]]) 01:58, 23 January 2024 (UTC) |
|||
:{{reply|Shenaall}} you can fix it by uploading a smaller version, like 320 pixels wide. My understanding, however, is that a bot comes along and does this eventually for large non-free images. ~[[User:Anachronist|Anachronist]] <small>([[User talk:Anachronist|talk]])</small> 02:59, 23 January 2024 (UTC) |
|||
::{{u|Shenaall}}, it is best that you downsize the file yourelf. That will show that you understand the standards imposed on non-free images. If an image is freely licensed or in the public domain, we want the most accurate and highest resolution version available. On the other hand, if the image is non-free, we quite deliberately want a low resolution version, as part of our ongoing efforts to comply with copyright laws. [[User:Cullen328|Cullen328]] ([[User talk:Cullen328|talk]]) 05:39, 23 January 2024 (UTC) |
|||
== Unicode or… something? == |
|||
How come every time I expand an article, the view count suddenly spikes? [[User:TrademarkedTWOrantula|<span style="color: darkgreen"><span style="font-family: Georgia">TWOrantula</span></span>]]<sup><small>TM</small></sup> <small>([[User talk:TrademarkedTWOrantula|enter the web]])</small> 05:00, 4 January 2025 (UTC) |
|||
Hello! I was wondering if anyone knows what the Unicode-like system is in which %20 represents a space character. |
|||
:Hey @[[User:TrademarkedTWOrantula|TrademarkedTWOrantula]] Expanding an article can trigger increased views due to it appearing in more searches, being featured on watchlists, or drawing attention from other editors and readers curious about the changes I guess. [[User:Ayohama|Ayohama]] ([[User talk:Ayohama|talk]]) 07:18, 4 January 2025 (UTC) |
|||
Thank you! [[Special:Contributions/98.97.36.1|98.97.36.1]] ([[User talk:98.97.36.1|talk]]) 07:52, 23 January 2024 (UTC) |
|||
::{{ping|TrademarkedTWOrantula}} If "spikes" just means a significant percentage increase from a very low starting point then it could be your own views while editing the article. For example, you edited [[Pictoword]] three times yesterday and it went from around six daily views to 16 that day.[https://pageviews.wmcloud.org/?project=en.wikipedia.org&platform=all-access&agent=user&redirects=0&range=latest-30&pages=Pictoword] [[User:PrimeHunter|PrimeHunter]] ([[User talk:PrimeHunter|talk]]) 08:56, 4 January 2025 (UTC) |
Latest revision as of 08:56, 4 January 2025
Tenryuu, a Teahouse host
Your go-to place for friendly help with using and editing Wikipedia.
Note: Newer questions appear at the bottom of the Teahouse. Completed questions are archived within 2–3 days.
Technical question about the long hyphen
Hi!
I've been editing the timeline of Polermo where the long hyphen dominates, but I can't seem to generate one.Typing a regular hyphen, gives me just that - a regular hyphen, typing two hyphens gives me two hyphens (--) and trying to make one through the keboard shortcut which I found on internet forums (Alt+0151), just gives me one that's too long (—). So far I've been copying and pasting existing long hyphens which is kind of annoying, does anyone have any better solutions?
Thanks! Moonshane1933 (talk) 14:38, 22 December 2024 (UTC)
- Hello, @Moonshane1933. I think you're talking about an em-dash. See MOS:EMDASH ColinFine (talk) 14:52, 22 December 2024 (UTC)
- Yes! That's what I meant! Thank you! Moonshane1933 (talk) 15:15, 22 December 2024 (UTC)
- I don't think you could find a better character in "unicode table".
- This "article" is listing the most common characters.
- There are also the "Unicode block" entry on Wikipedia that can be maybe helpful. Anatole-berthe (talk) 14:54, 22 December 2024 (UTC)
- Excellent. Thank you too! Moonshane1933 (talk) 15:16, 22 December 2024 (UTC)
- I don't think ressources I shared with you will help you but I hope it will. Anatole-berthe (talk) 15:45, 22 December 2024 (UTC)
- Excellent. Thank you too! Moonshane1933 (talk) 15:16, 22 December 2024 (UTC)
- Ignoring the Minus sign, there are three 'horizontal line' characters most commonly used in text, the hyphen, the N-dash and the M-dash. There are various ways to insert the latter two; usually I do so with [alt]+0150 and [alt]+0151. Despite being a former professional book editor, I have not previously encountered a "long hyphen" (a term not found anywhere in Wikipedia). Note that the lengths of all these characters may look different in different typefaces: I suspect your "long hyphen" is an N-dash. [Apologies for semi-overlap with answers above.] {The poster formerly known as 897.81.230.195} 94.1.223.204 (talk) 17:00, 22 December 2024 (UTC)
- @Moonshane1933 If you use the source editor, which you can do even if you mainly edit with the visual editor, you'll find that the N-dash and M-dash appear at the foot of the editing window, where you can click on them to insert them into text. Other useful tags like <ref></ref> are also available with a single click. Mike Turnbull (talk) 14:11, 24 December 2024 (UTC)
- OOOOOOOHHHH... THANK YOU! That makes life easier! I hadn't even thought of looking at the source editor, because it always looks headache inducing to me. I'll give it a try. Thank you so much. Moonshane1933 (talk) 13:07, 25 December 2024 (UTC)
- Yes, well, the "long hyphen" is a term that I coined, simply because I lacked the knowledge of its correct name, So I would have been very surprised if it had appeared in Wikipedia. Anyway, thank you, oh mysterious IP poster, I hope our paths cross again! Moonshane1933 (talk) 13:03, 25 December 2024 (UTC)
- @Moonshane1933, some Christmas goodies for you:
- — Merriam-Webster Dictionary has a nice clear explanation about the both kinds of dashes and the hyphen, with good examples.
- — The way the two kindts of dashes is written is em-dash (for —) and en-dash ( for – ), even though we pronounce the terms "M dash" and "N dash."
- — Why these terns? Because the em-dash is exactly the width of capital M and the en-dash is exactly the width of capital N.
- — If you have a Macintosh, there's a real simple way to make the dashes: the em-dash by pressing Control Option Hyphen at the same time, and the en-dash by pressing Option Hyphen at the same time.
- —Did you notice how Nick Moyes creatively renamed Dasher, one of Santa Claus's eight reindeer, in his "Seasonal Greetings from all at the Teahouse" post to fellow editors below?
- —You may be pleased to know that I found an online reference to a "long hyphen." So, then, you weren't completely alone in doing that. But as 94.1.223.204 commented above, in professional editing we just don't use it. Like ColinFine, )I think anyone who did say "long hyphen" would probably be thinking of the em-dash; though I also think what 94.1.223.204 said above is also technically correct, that the term would have to refer to the en-dash (that's the next size up for a hyphen, after all). Augnablik (talk) 06:08, 27 December 2024 (UTC)
- @Augnablik, What a great reply! I thoroughly enjoyed every bit of it! And I learned a lot (not to detract from the other contributors, each of whom taught me something new - thanks, everybody) - a special thank you for the meanings of the em-dash and en-dash (I love that type of thing), and for drawing my attention to Nick Moyes' "Seasonal Greetings", and of course for finding me an ally in calling the en-dash a "Long hyphen" (don't worry, now that I know the correct terminology I will use it and hopefully amaze my friends...). Thank you again and Merry Christmas! Moonshane1933 (talk) 12:58, 29 December 2024 (UTC)
- @Augnablik Not to be a naysayer, but I think the bit about em dashes being named for being 'M' width is a false etymology. I too would have loved if it were true, but I think it's actually based on the em unit as described in Dash#Em dash. — Kilvin the Futz-y Enterovirus (talk) 10:23, 30 December 2024 (UTC)
- Well, @Kilvin the Futz-y Enterovirus, not to be a counter-naysayer (!) but there are many online sites with support for the width of the em-dash equal to M and of the en-dash equal to N. Here’s just one, offered by Grammarly. (Scroll down to What Is an En-Dash? and What Is an Em-Dash?) Augnablik (talk) 12:06, 30 December 2024 (UTC)
- Hm... not to be a counter- counter- naysayer I mean I don't really think 'many online sites' and Grammarly (unsourced, could just be mirroring untrustworthy sources) are WP:RS. But you're right; my searches show similar results and don't really yield great answers to this matter (ideally there'd be a page like "Many people say that an em dash is named for X, but actually it is Y, and this misconception came from Z" linking to many reliable sources). I guess my personal bias is towards the people going "false etymology!" that acknowledge both X and Y rather than one group who simply assert the "fun" explanation with seemingly no awareness of the other explanation. I mean, as linked previously, the em and en are typographical units whose pages describe their origins, even addressing Em (typography)#Obsolete alternative definition. — Kilvin the Futz-y Enterovirus (talk) 22:53, 30 December 2024 (UTC)
- Oooh, @Kilvin the Futz-y Enterovirus, you’re acknowledging “personal bias” towards “false etymology?” That requires a declaration of COI! 🙂 Augnablik (talk) 18:09, 31 December 2024 (UTC)
- Aha! I found something! In the Historical thesaurus on the OED site. And I quote:
- "em dash1836- A long dash —, originally and usually the width of one em (see em, n.)." Moonshane1933 (talk) 13:13, 1 January 2025 (UTC)
- Oooh, @Kilvin the Futz-y Enterovirus, you’re acknowledging “personal bias” towards “false etymology?” That requires a declaration of COI! 🙂 Augnablik (talk) 18:09, 31 December 2024 (UTC)
- Hm... not to be a counter- counter- naysayer I mean I don't really think 'many online sites' and Grammarly (unsourced, could just be mirroring untrustworthy sources) are WP:RS. But you're right; my searches show similar results and don't really yield great answers to this matter (ideally there'd be a page like "Many people say that an em dash is named for X, but actually it is Y, and this misconception came from Z" linking to many reliable sources). I guess my personal bias is towards the people going "false etymology!" that acknowledge both X and Y rather than one group who simply assert the "fun" explanation with seemingly no awareness of the other explanation. I mean, as linked previously, the em and en are typographical units whose pages describe their origins, even addressing Em (typography)#Obsolete alternative definition. — Kilvin the Futz-y Enterovirus (talk) 22:53, 30 December 2024 (UTC)
- Well, @Kilvin the Futz-y Enterovirus, not to be a counter-naysayer (!) but there are many online sites with support for the width of the em-dash equal to M and of the en-dash equal to N. Here’s just one, offered by Grammarly. (Scroll down to What Is an En-Dash? and What Is an Em-Dash?) Augnablik (talk) 12:06, 30 December 2024 (UTC)
- @Moonshane1933 If you use the source editor, which you can do even if you mainly edit with the visual editor, you'll find that the N-dash and M-dash appear at the foot of the editing window, where you can click on them to insert them into text. Other useful tags like <ref></ref> are also available with a single click. Mike Turnbull (talk) 14:11, 24 December 2024 (UTC)
Add a page?
Hello - How can somebody submit a page for a notable person? My husband has one of the country's worst wrongful convictions in the United States and I'd love to have somebody neutral put information up regarding his wrongful conviction case. We believe he will be exonerated someday. His name is Temujin Kensu and you can google search his name to learn more about this horrible case. Thank you! 65.111.210.82 (talk) 06:21, 28 December 2024 (UTC)
- Based on my Google search, I consider it almost certain that Temujin Kensu is notable and that Wikipedia ought to have an article about him. Cullen328 (talk) 07:39, 28 December 2024 (UTC)
- For anyone interested in starting a draft some of these Google hits could easily be used to pass WP:GNG. Mike Turnbull (talk) 12:21, 28 December 2024 (UTC)
- Realistically, you may not get a volunteer. Teahouse Hosts volunteer here to advise, not to be authors or co-authors. David notMD (talk) 20:18, 28 December 2024 (UTC)
- For anyone interested in starting a draft some of these Google hits could easily be used to pass WP:GNG. Mike Turnbull (talk) 12:21, 28 December 2024 (UTC)
- If your husband is notable enough, he will be talked about. SimpleSubCubicGraph (talk) 05:37, 1 January 2025 (UTC)
- IP editor: I have now started to draft an article at Draft:Temujin Kensu. In view of your conflict of interest it would be best if you did not edit it directly (although that's not forbidden while it remains a draft). I would welcome your suggestions for additions and corrections at Draft talk:Temujin Kensu, especially where you can provide additional published sources I can use. Mike Turnbull (talk) 12:47, 1 January 2025 (UTC)
Isotopes lists download
Is there a to download these lists ? ( For example : the list in "https://en.wikipedia.org/wiki/Isotopes_of_sodium" )
I have written a c# application that describes the relations between elements, isotopes, decays, fusions ... etc.(originating from the question "Where the carbon atoms in the cafeine in your coffee come from ?")
When you make normal modifications to these lists, it takes me about 2 weeks to refresh my database for over 3000 isotopes and 5000 decays coming from 118 pages (and subject to typing errors...)
I have tried to download one of these pages but I get one of these mumbo-jumbo network message ( about security and the correction looks like "set the web_client.Tchic_Tchac to Fling_Flang" ... and none of them works... )
Do you have a suggestion ?
Thank you very much Michel Béliveau (talk) 17:49, 28 December 2024 (UTC)
- Describing a message as a "mumbo-jumbo network message" is not very helpful in determining what your problem is. If you quote the error message exactly it might be more useful. In any case, I can successfully download articles using curl like this: CodeTalker (talk) 19:52, 28 December 2024 (UTC)
curl -k https://en.wikipedia.org/wiki/Isotopes_of_sodium
- Thank you for this fast (and good) answer.
- The mumbo-jumbo error message was : "The request was aborted: Could not create SSL/TLS secure channel." I was not using the good approach to download the content of the web page.
- >>> However the CURL function does what I need.
- Here is want to do In my application :
- For each Element ( 118 !!! ) get the "List of Isotopes" for this Element. Then for each Isotope : get its mass, half-life, decay mode(s) and decay product(s). This yields for over 3500 isotopes and over 4500 decays. Refreshing the data took quite a long time.
- Analyzing the results of the curl command is not so hard and will eliminate typing mistakes. Even if I need a few days to program the analysis, it will be faster than re-typing the data.
- I will take a look at Wikidata.
- Thanks again. Michel Béliveau (talk) 23:02, 28 December 2024 (UTC)
- Without really understanding what you are trying to do, I would suspect that Wikidata was a more useful resource than Wikipedia for your purpose, as it is a database which contains relations between its elements. ColinFine (talk) 21:34, 28 December 2024 (UTC)
- @Michel Béliveau Wouldn't it be easier to download from the original sources, for example NUBASE? Mike Turnbull (talk) 15:48, 29 December 2024 (UTC)
- Thank you Mr. Turnbull.
- You are correct. It would be easier to download from the original sources.
- I have found (and used) a NUBASE file (namely for nucleus values) . So far, I have found only 1 NUBASE file that I could use ( coming from "The Ame2020 atomic mass evaluation (I)" by W.J.Huang, M.Wang, F.G.Kondev, G.Audi and S.Naimi - Chinese Physics C45, 030002, March 2021) .
- The purpose of my request to Wikipedia is to avoid re-typing the values. The NUBASE file allowed this.
- Do you know other NUBASE files ? Or other sources ? (I also found some data in PeriodicTable.com)
- Thanks again for your interest Michel Béliveau (talk) 19:19, 29 December 2024 (UTC)
- @Michel Béliveau Wouldn't it be easier to download from the original sources, for example NUBASE? Mike Turnbull (talk) 15:48, 29 December 2024 (UTC)
To editor Michel Béliveau: Hello! This kind of question is something you might want to take to the Computing reference desk, where you can get people with computing expertise to assist. --Slowking Man (talk) 20:39, 31 December 2024 (UTC)
Locked out of account
I got locked out of my DooplissTTYD account because I forgot the complex password and didn’t have an email address linked to it. Is there any way that account can be renamed to something else and I change this one to DooplissTTYD? TTYDDoopliss (talk) 21:50, 28 December 2024 (UTC)
- You would have to request a renaming from the account you want renamed. It can't be requested by in essence a third party(as we have no way to know who is on the other end of the computer). The best you can do is post on your current and previous user pages that you lost access to your old account and have a new one. 331dot (talk) 00:06, 29 December 2024 (UTC)
- This is why the first thing you want to ensure you do right now is verify an e-mail address for your account, so you can restore it: follow this link if you haven't. Step 2: I highly recommend using a password manager to save passwords—as well as generate secure passwords. I suggest trying out Bitwarden.
- Now in this particular case, your prior account has only a few edits; this means if you wait a bit, say 6 months, you can request to "usurp" it which will accomplish what you want. (Also, a very good game.) --Slowking Man (talk) 18:06, 31 December 2024 (UTC)
- I mean that’s what I did. I generated a password using a feature on my phone, and it kept saying the one I had on my phone was incorrect.
- and yes I have an email linked to this account now. 6 months is a very long time though… TTYDDoopliss (talk) 19:13, 31 December 2024 (UTC)
donating - would like to donate
Hail... Would like to donate 50 quid to the cause but stop at the name and address part. Don't really see the need for full name and address. Just old and not particularly wise. Any suggestions? 81.96.25.61 (talk) 12:28, 29 December 2024 (UTC)
- Hi IP editor, the Wikimedia Foundation deals with all donation issues and questions - editors here at the Teahouse don't have any input. Please direct your query to the email address at the bottom of donate.wikimedia.org/wiki/Ways_to_Give qcne (talk) 12:49, 29 December 2024 (UTC)
- Just to elaborate slightly: all editors here give their time entirely voluntarily and gain absolutely no financial benefit from any contributions made to keep the broader Wikipedia projects going. So we have little knowledge of how the donation systems work - despite being grateful for everyone's contributions. The advice above is sound. Nick Moyes (talk) 18:21, 29 December 2024 (UTC)
- Also I'm not certain (not WMF personnel), but I am pretty confident the reason for requiring that info is fraud protection and banking and money laundering laws; it's not something the Foundation has any control over. WMF is a registered 501(c)(3) organization in the US, meaning it has to comply with a bunch of laws and regulations. (For furrin types, this is near-synonymous with "non-profit/charity" in the US, being simply the section of the Internal Revenue Code setting out the criteria said organizations have to meet to get tax exemption.) Note, a way that one can make semi-anonymous donations, is to pass them through what's called a "donor-advised fund" in the US, but this is a bit more involved and requires going through a bank or brokerage that will arrange your transfer. Alternatively, I suppose if one is accepting of the fact there are no guarantees in life one could always send cash to the WMF's front door with no return address and leave its final disposition up to them. --Slowking Man (talk) 18:38, 31 December 2024 (UTC)
I don't find the ressource to add books wrote by someone
Hello ! I'd like to add a book on the article about "David Murphy (CIA)".
This is a book wrote by him not mentionned in the article.
I don't find the ressource explaining how to add the bibliography of someone. Anatole-berthe (talk) 16:17, 29 December 2024 (UTC)
- @Anatole-berthe The standard template to use for book citations is {{cite book}}. It is usual, but not essential, to use its |URL= parameter to link to Google Books for the convenience of our readers. In this case that would be this link, from which you can also find the ISBN and full list of authors. Mike Turnbull (talk) 16:27, 29 December 2024 (UTC)
- Hello, @Anatole-berthe. Why do you want to add that book to that article? Has the book been discussed by independent sources? If not, why is it significant ednough to feature in a Wikipedia article?
- More to the point, that article is woefully short of sources, and does not establish that Murphy meets the criteria for notability. ColinFine (talk) 17:42, 29 December 2024 (UTC)
- The book was wrote by himself as I said.
- The ISBN-10 is "0300107803" and the ISBN-13 is "978-0300107807" for the first edition.
- The title is "What Stalin Knew: The Enigma of Barbarossa" , it was published by "Yale University Press" in 2005.
- The ISBN-10 is "030011981X" and the ISBN-13 is "978-0300119817" for the second edition.
- The second edition was published in 2006 by the same publisher.
- In this edition , it's mentionned the review of the first edition by "Niall Ferguson" for "New York Time Book Review".
- The first edition was also reviewed by "Andrew Nagorski" for "The Weekly Standard".
- There are also a review by "Henry Kissinger" but I consider it doesn't count to add this book on Wikipedia.
- This person have to be considered as a reviewer among others even if he's notable accorded to criterias.
- He didn't wrote the review for a magazine or a journal and therefore the fact "Kissinger" wrote a review should not be taken into account. This is what I think.
- The same for the review by "Donald Kagan" for "Yale University" for a particular reason. This person worked for "Yale" and the book was published by "Yale University Press". Therefore , I consider it's like a review by the publisher itself.
- Conclusion : I think this book met the criteria n°1 for "Wikipedia:Notability_(books)" to create an article on this book.
- I don't want to create an article on the book itself.
- The "Threshold standards" is met because "Library of Congress" catalogued this book.
- LCCN is "2004065916".
- If it is considered as notable to create an article , I consider it is notable enough to mention this book on the article "David Murphy (CIA)". Anatole-berthe (talk) 12:03, 1 January 2025 (UTC)
- The book was wrote by himself as I said.
can you give me a lnk of the Roblox page
i need Roblox link to sign in 24.192.134.19 (talk) 01:57, 30 December 2024 (UTC)
- Welcome to the Teahouse. Did you have a question about editing or using Wikipedia? —Tenryuu 🐲 ( 💬 • 📝 ) 02:07, 30 December 2024 (UTC)
- @Tenryuu I think hes trolling. SimpleSubCubicGraph (talk) 02:50, 30 December 2024 (UTC)
- That's my standard reply to anyone who doesn't ask questions about Wikipedia here. —Tenryuu 🐲 ( 💬 • 📝 ) 02:58, 30 December 2024 (UTC)
- @Tenryuu I think hes trolling. SimpleSubCubicGraph (talk) 02:50, 30 December 2024 (UTC)
- Does this link help at all? If you have general computing questions that aren't Wikipedia-specific, go to the Computing reference desk. --Slowking Man (talk) 18:46, 31 December 2024 (UTC)
Good Sources for Articles
G-Day
I wanted to ask if there is any reliable source for Articles. I cannot afford my personal favourite, The "Britannica" Encyclopedia, since they seem to be rare and expensive. Thank you. (I know this isn't about a specific Article but I need to know this) PizzaFrank (talk) 13:51, 30 December 2024 (UTC)
- @PizzaFrank there are all sorts of reliable sources, not just Encyclopædia Britannica, and most don't require a purchase of any kind. See Wikipedia:Reliable sources for a detailed overview of RSes here on Wikipedia. '''[[User:CanonNi]]''' (talk • contribs) 13:55, 30 December 2024 (UTC)
- Thank you, I will check that out. PizzaFrank (talk) 14:03, 30 December 2024 (UTC)
- PizzaFrank Note that you don't have to personally own the source- it just needs to be publicly accessible, like online, or in a library. 331dot (talk) 14:01, 30 December 2024 (UTC)
- Understood Thank you for the help. PizzaFrank (talk) 14:04, 30 December 2024 (UTC)
- PizzaFrank I have used https://www.britannica.com a few times, and there is no fee to use the online version of Encyclopedia Britannica. Karenthewriter (talk) 18:27, 30 December 2024 (UTC)
- Understood Thank you for the help. PizzaFrank (talk) 14:04, 30 December 2024 (UTC)
- And besides public libraries which are great, Wikipedia editors with some experience can access The Wikipedia Library which grants free access to tons of "paywalled" "digital content"! Pretty neat! Also see there: even if your account is not "seasoned" enough yet, you can request experienced editors assist you with researching sources through it. --Slowking Man (talk) 17:45, 1 January 2025 (UTC)
Photograph attribution
I want to use a portrait of my grandfather taken in 1915 by a company that no longer exists. I can show an attribution in the caption, but there is nobody I can seek permission from. How do I proceed, please? Gangnam Woodford (talk) 16:52, 30 December 2024 (UTC)
- Hello. It depends on the laws of your country, but a photo taken in 1915 is likely in the public domain. According to https://commons.wikimedia.org/wiki/Commons:Copyright_rules_by_territory/United_States in the US anything published before 1929 is public domain(other than sound recordings), so it would just depend on your own country. 331dot (talk) 16:56, 30 December 2024 (UTC)
- Yes BUT (big but!), "publication" in the context of US copyright law has a specifical technical, legal definition. It doesn't mean "creation" or even "giving a copy to a particular person". Hirtle chart. A photograph of a private individual is almost certainly an "unpublished work" under the US copyright meaning of that term, which matters significantly for the length of its copyright term especially if an "older" work. Copyright is a subject that gets quite technical, so if not familiar and experienced with copyright issues, it is best to hesitate to advise others before researching things in detail and double-checking, and if in doubt, direct others to forums like Commons where experts can be found. --Slowking Man (talk) 20:02, 31 December 2024 (UTC)
- @Gangnam Woodford Is this an American picture? If so, you can upload it as public domain, see [1]. Gråbergs Gråa Sång (talk) 16:57, 30 December 2024 (UTC)
- Sorry, should have said: UK. Gangnam Woodford (talk) 16:58, 30 December 2024 (UTC)
- @Gangnam Woodford Still good, see this example: File:Archibald Joyce.jpg. Gråbergs Gråa Sång (talk) 17:02, 30 December 2024 (UTC)
- Thank you! Gangnam Woodford (talk) 17:06, 30 December 2024 (UTC)
- Please also see MOS:CREDITS ".... do not credit the image author or copyright holder in the article. .... as long as the appropriate credit is on the image description page." - Arjayay (talk) 17:16, 30 December 2024 (UTC)
- Thank you! Gangnam Woodford (talk) 17:06, 30 December 2024 (UTC)
- @Gangnam Woodford Still good, see this example: File:Archibald Joyce.jpg. Gråbergs Gråa Sång (talk) 17:02, 30 December 2024 (UTC)
- Sorry, should have said: UK. Gangnam Woodford (talk) 16:58, 30 December 2024 (UTC)
- To editor Gangnam Woodford: "Congratulations", you've just tripped onto one of the fun little copyright law landmines that are out there (alternately: "job security for lawyers"). Everything on Wikimedia Commons must be public domain in both the US, and origin country. This means you have to care about US copyright law and all its nuances: see Hirtle chart. Unless the picture was like put on display in an art gallery or something, it is an "unpublished" work and therefore still under copyright in the US until 2035! (120 yrs from creation date) Really! Welcome to copyright law!
- Here are the practical implications for you in this case: Is this image going to be used in an article? If so, upload it here on enwp (not Commons) by following this link, and tag it {{PD-UK-unknown}}, as well as adding a fair use rationale since it's still under US copyright.
- Add it to the article(s), nothing more needed—some bot should automatically transport it over to Commons when it finally lapses into PD-US. However, if you just, say, want to use the image on your user page, I'm afraid you aren't going to be able to for 10 more years. So stick a pin in the calendar for that. --Slowking Man (talk) 19:54, 31 December 2024 (UTC)
An Article About CK
Extended content
|
---|
I was asked by christen kuikoua representation to write an article about him and it was declined saying I didn't correctly reference it. Please if anyone wishes to jump in and help me with Will appreciate it Draft:Christen Kuikoua Silvernet123 (talk) 18:27, 30 December 2024 (UTC)
|
- Note that the account Christenkofficial has (correctly or not, I'm just reporting) now been blocked as a sockpuppet of Silvernet123, per Wikipedia:Sockpuppet investigations/Silvernet123. {The poster formerly known as 87.81.230.195} 94.1.223.204 (talk) 03:35, 31 December 2024 (UTC)
How to add sources
I need a simplified tutorial in how to add the number reference and the cite the source. M. Chris Tucker (talk) 19:02, 30 December 2024 (UTC)
- Hello M. Chris Tucker (talk · contribs), might I suggest Referencing For Beginners. -- D'n'B-📞 -- 19:06, 30 December 2024 (UTC)
- Hi @M. Chris Tucker and welcome to Wikipedia! The simplest way to add references is, directly after the sentence or paragraph that your source supports, add <ref>, followed by the text of your reference, followed by </ref>. The software will sort out the numbering for you. Others here should be able to give you more detailed advice, and the guide that D'n'B linked above looks like a great place to learn more. Thanks for your additions to Edward Dickson (Canadian politician)! Best, Wham2001 (talk) 19:07, 30 December 2024 (UTC)
- 2607:FEA8:7D00:95A0:FDD3:6EB9:68D5:D6D7 (talk) 00:38, 31 December 2024 (UTC)
- Thanks very much! I will follow up on your suggestion and wish you a Happy New Year. M. Chris Tucker (talk) 00:42, 31 December 2024 (UTC)
- 2607:FEA8:7D00:95A0:FDD3:6EB9:68D5:D6D7 (talk) 00:38, 31 December 2024 (UTC)
How do I add a "Main Article" to Wikipedia
I updated our "Cornhusker Council" section under Boy Scouts of America, Nebraska Scouting. and would like to create a main article for our council. how do I do that?
Cornhusker324 (talk) 21:15, 30 December 2024 (UTC)
- Cornhusker324 Hello and welcome to the Teahouse. First, please see conflict of interest, as that needs to be formally disclosed. For a standalone article about your specific council, you would need to show with significant coverage in independent reliable sources that your council meets the special Wikipedia definition of a notable organization. That coverage can't just be the reporting of its routine activities, but in depth coverage as to what makes your council important/significant/influential. Be aware that writing a new article is the most difficult task to attempt on Wikipedia, and it's even harder with a conflict of interest. Also be aware that an article is not necessarily desirable. 331dot (talk) 21:23, 30 December 2024 (UTC)
- Hello @Cornhusker324. I have just reverted your edits in accordance with our conflict of interest policy. Please request specific edits on the article's respective talk page. Thanks. Tarlby (t) (c) 21:25, 30 December 2024 (UTC)
- And I'm not sure why the council itself would merit a standalone article from the article about scouting in Nebraska. 331dot (talk) 21:31, 30 December 2024 (UTC)
- Many other Councils already have a standalone article. I would think this would be supported for consistency sake. Cornhusker324 (talk) 21:35, 30 December 2024 (UTC)
- @Cornhusker324, please see WP:42. Some council have articles. Others don't, and maybe they shouldn't. It all depends on the reliable sources that we need to use so that an article can be made. An article on Cornhusker Council won't be made if there are no such sources. Tarlby (t) (c) 21:40, 30 December 2024 (UTC)
- Many other Councils already have a standalone article. I would think this would be supported for consistency sake. Cornhusker324 (talk) 21:35, 30 December 2024 (UTC)
- I have changed my user name in hopes of complying with your COI policy. The stand-alone article seems too difficult at this point. Perhaps in the future.
- As such, I would simply like to update our section, "Cornhusker Council," in "Scouting in Nebraska." and will attempt that in the 'talk page,' as you suggested. Cornhusker324 (talk) 21:42, 30 December 2024 (UTC)
- And I'm not sure why the council itself would merit a standalone article from the article about scouting in Nebraska. 331dot (talk) 21:31, 30 December 2024 (UTC)
- Hello @Cornhusker324. I have just reverted your edits in accordance with our conflict of interest policy. Please request specific edits on the article's respective talk page. Thanks. Tarlby (t) (c) 21:25, 30 December 2024 (UTC)
Your name change does not show up yet, and regardless of a name change, you still have a COI. I see that you are proposing changes on the Talk page of the article, but much of what you wrote is not referenced. See Help:Referencing for beginners to understand how to insert inline refs. David notMD (talk) 05:30, 31 December 2024 (UTC)
- I provided four references which is more than what is currently listed. Those currently used are not authoritative and factually incorrect. 209.92.187.50 (talk) 15:34, 31 December 2024 (UTC)
- How exactly did you change your username? It should be done via either Special:GlobalRenameRequest or WP:CHUS. You should continue to use your original username until it is changed, at which time you can begin using the new name. 331dot (talk) 15:48, 31 December 2024 (UTC)
- Never mind, I see it is changed now. 331dot (talk) 15:49, 31 December 2024 (UTC)
- That is not how references are created. Also, you now appear to be editing as an IP address, i.e., not logged into an account versus a name-change. David notMD (talk) 16:54, 31 December 2024 (UTC)
- I've used two different means to source the information I provided, both of which were recommended by this site and/or a moderator. CCJLJ (talk) 17:04, 31 December 2024 (UTC)
- Again, see Help:Referencing for beginners to understand how to insert inline refs. What you posted on the Talk page of the article (below) are not properly formatted references. David notMD (talk) 21:22, 31 December 2024 (UTC)
- I've used two different means to source the information I provided, both of which were recommended by this site and/or a moderator. CCJLJ (talk) 17:04, 31 December 2024 (UTC)
- That is not how references are created. Also, you now appear to be editing as an IP address, i.e., not logged into an account versus a name-change. David notMD (talk) 16:54, 31 December 2024 (UTC)
References supporting change:
- Salistean, John, "A History of the Cornhusker Council 1940-1975," Houchen Bindery LTD of Utica, Nebraska, 1st ed., 2011.
- Golden Sun Lodge Website, www.goldensunlodge.org
- Cornhusker Council Website, www.cornhuskercouncil.org
- Brown, Elinor L., "History of Lancaster County, Then and Now," ASIN B0006CJTC2, Jan 1971.
Notability and Independence.
My grandmother, Ethel Margaret Streit Harrison, was the first woman elected as Clerk of the Montana Supreme Court, one of the founders of the Montana Association of Female Executives and one of the original board members of the Holter Museum of Art. There are printed sources that talk about her achievements but according to wikipedia the only thing notable about her was that she was married to John C. Harrison (judge). I understand that, as a relative, there is a potential conflict here, but I think it is important her contributions are documented on something other than microfiche. Any thoughts? / Thank you :) Mehap dwhx (talk) 22:40, 30 December 2024 (UTC)
- Mehap dwhx, none of those three roles that you describe automatically confers Wikipedia-defined "notability" on a person. And the sum of the three doesn't either. But it's possible that she's "notable" all the same. Regardless of your conflict of interest, you're free to create Draft:Ethel Margaret Streit Harrison. If this demonstrates her notability and follows Wikipedia's other policies, it will be promoted to an article (possibly with a slightly different title), whereupon you shouldn't continue to edit it but would be free to make suggestions and requests on its talk page. -- Hoary (talk) 23:12, 30 December 2024 (UTC)
- To expand an aspect of Hoary's excellent advice above: in Wikipedia, 'Notability' boils down to "is there enough substantial material, published independently of the subject, in multiple Reliable sources, to form the basis of an article about the subject. It doesn't necessarily require that the subject is 'famous', or in one of many possible senses 'important'.
- Also, the fact that your grandmother's achievements are not yet in Wikipedia doesn't mean Wikipedia asserts they aren't, or she as a whole isn't, notable; merely that no volunteer editor has yet gathered the necessary sources and added the information (with citations), whether in her husband's article or in a draft for her own. I encourage you to try. Good luck!
- While we're here, I notice that John C. Harrison's article is on shaky ground, because it's entirely cited to a single (though reliable) source: we usually prefer a minimum of three, so if you could flesh out that article with further referenced material, it would be a good thing. {The poster formerly known as 87.81.230.195} 94.1.223.204 (talk) 03:56, 31 December 2024 (UTC)
- Add Ethel Margaret Streit Harrison to: John_C._Harrison_(judge)#Personal_life
- 69.181.17.113 (talk) 04:45, 31 December 2024 (UTC)
- can you find at least 4 WP:RS sources ? 69.181.17.113 (talk) 04:47, 31 December 2024 (UTC)
- She's already mentioned in that section as Ethel Harrison, cited to the article's (only) reference. Even to add her middle names would require a further citation to a published reliable source that mentions them (the existing one does not: I've downloaded and searched it). {The poster formerly known as 87.81.230.195} 94.1.223.204 (talk) 06:56, 31 December 2024 (UTC)
- Being married to JCH can be mentioned in her Personal life section, but does not contribute to establishing her notability. David notMD (talk) 05:34, 31 December 2024 (UTC)
Restoring previously unmerged articles
I would like to create a new article but it once existed back in 2009 and got merged without consensus. Is it okay if I restore the article but with more and better sources from my sandbox later on? Because one user tried to restore a merged article and it got immediately undone because a consensus existed to merge it. I'm not saying that I'll do it right now as it's still unfinished but I need to ask because if I can't then I'll just cancel. Underdwarf58 (talk) 00:29, 31 December 2024 (UTC)
- There's very little about the original subject 'Cyprus–Jordan relations' in Foreign relations of Cyprus that it was merged into, so I think that if you think you have enough Reliably sourced material (bearing in mind there's also been another 15 years of history) you should go ahead and create a new Draft on the subject, via the normal Wikipedia:Articles for creation process, which can be submitted for assessment. Good luck! {The poster formerly known as 87.81.230.195} 94.1.223.204 (talk) 04:11, 31 December 2024 (UTC)
template
Hello, I'm need of some assistance with creating a Wikipedia page about a music artist. Can anyone advise which template to use? RATHOMP (talk) 02:45, 31 December 2024 (UTC)
- look at other Wikipedia pages about music artists ... 69.181.17.113 (talk) 04:40, 31 December 2024 (UTC)
- Hello, IP user, and welcome to the Teahouse. From your question, I suspect that you are focussing on the layout of your proposed article (I'm guessing that that's what you mean by a "template" - we use the word a bit differently here). But while the layout of an article is important, it is MUCH less important than the quality of the sources used. Until you have found adequate sources to establish that the artist meets Wikipedia's criteria for [[WP:notability|}], it's pointless spending any time thinking about the content or the layout. To use a house-building analogy, you may have an idea for what you want your house to look like, and even a plan; but until you've surveyed the site to make sure it's fit to build on, and checked that your plans meet local building regulations, it would be a waste of effort to start building.
- More generally, My earnest advice to new editors is to not even think about trying to create an article until you have spent several weeks - at least - learning about how Wikipedia works by making improvements to existing articles. Once you have understood core policies such as verifiability, neutral point of view, reliable, independent sources, and notability, and experienced how we handle disagreements with other editors (the Bold, Revert, Discuss cycle), then you might be ready to read your first article carefully, and try creating a draft. ColinFine (talk) 10:36, 31 December 2024 (UTC)
- Is there a specific place you're currently working on the article at? I can't seem to find it on your userpage as a subpage. For advice, I'd recommend looking at other music artist articles (specifically various quality articles from The Beatles [FA] to Sepultura [C-class]) for general outlines on how to write it. For infoboxes, use {{infobox musical artist}} and fill out the template using it's template page at Template:Infobox musical artist. For general advice on writing, see Wikipedia's content policies and guidelines, I'd recommend WP:Nutshell as a starting point.
- Thanks, Sparkle and Fade talkedits 08:23, 31 December 2024 (UTC)
how can I rigth a article in wikipedia
how can I rigth Daniel Muanga (talk) 03:49, 31 December 2024 (UTC)
- Wikipedia:Article wizard
- 69.181.17.113 (talk) 04:39, 31 December 2024 (UTC)
- New editors are strongly advised to first gain skills by doing time improving existing articles. References required. David notMD (talk) 05:39, 31 December 2024 (UTC)
Find a Grave = WP:RS ?
Is Find a Grave = WP:RS ?
69.181.17.113 (talk) 04:37, 31 December 2024 (UTC)
- Welcome to the Teahouse. Please consult Wikipedia:FINDAGRAVE. —Tenryuu 🐲 ( 💬 • 📝 ) 05:03, 31 December 2024 (UTC)
- The answer is "no" because Find a Grave consists of user generated content, and is specifically mentioned in the WP:USERGENERATED section of the reliable sources guideline as a source that should not be used as a reference on Wikipedia. That does not mean that Find a Grave is of no value to Wikipedia editors. You may be able to find nuggets in those listings that will inform your searches about various people. Some but not all Find a Grave listings include references to reliable sources, and those sources may be useful as Wikipedia references. One thing that Find a Grave can teach editors is that many people down through the years share the same name, and we need to be very careful to avoid including biographical details about one Andrew Wilson in an article about another Andrew Wilson. That's just one of countless examples. I have been working on Andrew Stephen Wilson today, so that's why I chose that example.. Cullen328 (talk) 06:22, 31 December 2024 (UTC)
Hello -- I would like to start helping, but I'm having trouble finding what to do!
I see lots of support for ultra basics, but I know what the concept is, and how to edit, and how to make italics and hyperlinks, and that I should use a neutral voice, etc. I am trying to find some guidance on WHAT to contribute. I found the Typo Team (or at least, this typo team), but I haven't found guidance on interacting with it. (Do I delete entries if I resolve them? Yesterday I found many entries to check, but today none of the articles seem to HAVE the potential typo that was listed, or even a fixed version). I have found this Growth page, but can't get the features working. For example, it says to enable the Help panel in the Editing tab, but I don't see such a thing in the Editing tab. I also can't find "Display newcomer homepage" in my user preferences. Similar with Suggested Edits -- how can I "use Special:NewcomerTasksInfo
"? Etc., etc. I must be missing some key piece of advice -- where can I figure out how to get things rolling? SKM (talk) 05:08, 31 December 2024 (UTC)
- Hello @Skmccormick. As a volunteer project, you're free to do whatever you wanna do best. Want to copyedit a bunch of articles and bring our grammar up to shape? Join the Guild of Copy Editors and go wild. Wanna fight vandalism? Go patrol Special:RecentChanges and stop those dang vandals! Wanna go help out that typo team? Go right ahead. It's your choice. Tarlby (t) (c) 05:26, 31 December 2024 (UTC)
- @Skmccormick
- You should find display user homepage at the bottom of Special:Preferences, "User profile" tab.
- If I wanted to interact with the typo team, my first place to try would be Wikipedia talk:Typo Team.
- Have you found WP:TASKS and WP:REQUEST? Gråbergs Gråa Sång (talk) 07:28, 31 December 2024 (UTC)
- Thank you! These links are helpful. I found "display user homepage" under User Profile. So maybe a silly question, but: how do I find this user homepage? I don't really go to Wikipedia generically, I usually jump straight to an article. Tasks and Requests seem like what I'm looking for. SKM (talk) 01:47, 1 January 2025 (UTC)
- The link should be right at the top of every page when you're logged in, in the same place/menu as the link to your userpage. -- asilvering (talk) 02:05, 1 January 2025 (UTC)
- @Skmccormick
- @Skmccormick, do you have any particular topics that you'd like to edit about? One way to find a lot of articles that need help is to go through our various maintenance backlogs (Gråbergs Gråa Sång has already linked you to WP:TASKS). Some people are content to plug away at a particular backlog chronologically, but if you prefer to edit on things you're generally interested in, it's helpful to filter these by wikiproject. Alternatively, do you have any particular skills or outside knowledge that might be helpful here? There's always demand for multilingual editors, the copyright folks are always backlogged, etc. -- asilvering (talk) 10:16, 31 December 2024 (UTC)
- Thank you. I would prefer to get into the groove with smaller contributions before I start throwing any weight around. Sadly, I cannot offer multilingual help; I am American. I'm mostly having trouble navigating the various pages and internal tools like TASKS or RecentChanges (anything labeled "Special:" is still new to me). SKM (talk) 01:50, 1 January 2025 (UTC)
- I'm a fan of Category:Wikipedia introduction cleanup as a newbie task - not small, exactly, but hey, no time like the present to learn to WP:BEBOLD. You don't (or shouldn't) need to do any research to fix these articles - most of them are here because they are tagged with "lead too short". Find one of those, read the article, then rewrite the lead so it summarizes it accurately. Then remove the tag. All this requires is good English literacy, and since the lead is what most people read and what is used in the google knowledge box etc, it's a high-impact change that requires very little wiki-knowledge. -- asilvering (talk) 02:16, 1 January 2025 (UTC)
- Thank you. I would prefer to get into the groove with smaller contributions before I start throwing any weight around. Sadly, I cannot offer multilingual help; I am American. I'm mostly having trouble navigating the various pages and internal tools like TASKS or RecentChanges (anything labeled "Special:" is still new to me). SKM (talk) 01:50, 1 January 2025 (UTC)
Different images for Light/Dark mode
Is there any way to tell Wikipedia to display different images / media for Light vs Dark mode users? Due to transparency, some SVG and PNG images have bad contrast when viewed in Dark mode. CrushedAsian255 (talk) 06:04, 31 December 2024 (UTC)
- @CrushedAsian255 This question is likely to have a better audience at WP:VPT. That board has a more technically oriented team than here. 🇺🇦 FiddleTimtrent FaddleTalk to me 🇺🇦 09:26, 31 December 2024 (UTC)
Wider vs specific consensus
Can a case-specific consensus triumph a wider topic consensus already established? Not that consensus over wider topic is changed but maybe because that case is viewed from a different perspective. ExclusiveEditor 🔔 Ping Me! 07:33, 31 December 2024 (UTC)
- @ExclusiveEditor I doubt it. Were that to be the case we would not have consistency of operation. This question is likely to have a better audience at WP:VPP. That board has a more policy oriented team than here 🇺🇦 FiddleTimtrent FaddleTalk to me 🇺🇦 09:28, 31 December 2024 (UTC)
- Hello, @ExclusiveEditor, and welcome to the Teahouse. While I agree with FiddleFaddle's advice, I would also remark that general questions like yours are very frustrating for people who attempt to answer questions here. If you explain the specific issue you want guidance on, you are much more likely to get a useful answer (and also be less likely to be suspected of wikilawyering).
- I am aware of the possible irony in my answer, given your question. ColinFine (talk) 10:43, 31 December 2024 (UTC)
- Thanks for replying! Being a half-host on Teahouse myself I get you. I was inspired to ask this question by the discussion happening here. The latter part of the discussion specifically focuses on if 'Nobel prize' should be added or not in Jimmy Carter's death blurb on Main page's ITN section. It was initially proposed by the nominator and many supported it (albeit not mentioning specifically the Nobel prize) and it got posted without the mention of the prize. However later there was some more scrambling, this time with more regard to the Nobel prize and so it currently updated to include the prize in the blurb. The opposers are generally arguing that it is editorializing and other things. I may not be very good in summarizing discussions, so I left it in the question. --ExclusiveEditor 🔔 Ping Me! 10:58, 31 December 2024 (UTC)
How to upload a >100 MB file to Wikipedia?
I've uploaded plenty of files to Commons before, but I've only uploaded a few (non-free) files to WP. I would like to upload a short film that will become public domain in the US at the start of 2025 (won't be PD in its country of origin for a few years, so no uploading it to commons), but the file is over 100 MB, the maximum file size listed on the upload page. I don't want to compress it any more than it already is, so how to I get around this? I've seen several films large than 100 MB on WP already (1, 2), so it must be possible.
Any help with this is greatly appreciated. Thanks. — Toast for Teddy (talk) 07:48, 31 December 2024 (UTC)
- @Toast for Teddy This question is likely to have a better audience at WP:VPT. That board has a more technically oriented team than here 🇺🇦 FiddleTimtrent FaddleTalk to me 🇺🇦 09:26, 31 December 2024 (UTC)
- @Toast for Teddy That first file you linked was uploaded with the aid of a userscript discussed at Commons:User talk:Rillke/bigChunkedUpload.js. I don't pretend to understand the details, but you may ;-) Mike Turnbull (talk) 14:27, 31 December 2024 (UTC)
- Even if you're uploading "locally", Commons:Upload tools is likely helpful. All the projects use the same MediaWiki software, all that needs adjustment is the destination for the upload. --Slowking Man (talk) 20:54, 31 December 2024 (UTC)
Editing a sandbox, references number is doubling
Hi there! I was editing my sandbox and for some reasons all the different references I am adding are doubling the number of them and the previous ones are not disappeared. I was following your suggestions to add COI edit to my text, eliminating the internal sources and the bold words. Can you help me or it is just a matter of viewing and once I publish the sand box they will all disappear? Andrea Biographer (talk) 09:10, 31 December 2024 (UTC)
- Hi @Andrea Biographer: it seems that with this edit you've duplicated the contents by pasting an earlier edit into the page, thus embedding a copy of the entire page within the page (if that makes sense). You should undo your most recent edits up to and including that one. Or if you'd like me to do it, let me know. -- DoubleGrazing (talk) 09:16, 31 December 2024 (UTC)
- Hi @DoubleGrazing, thanks for your prompt answer! I think you made sense, could you check and fix this for me? I don't want to commit any further mistakes in the editing process or in the COI different templates, thanks in advance! Andrea Andrea Biographer (talk) 09:31, 31 December 2024 (UTC)
- User:Theroadislong has already sorted this out. Also, your draft is now located at Draft:Gridspertise. -- DoubleGrazing (talk) 10:30, 31 December 2024 (UTC)
- Hi @DoubleGrazing, thanks for your prompt answer! I think you made sense, could you check and fix this for me? I don't want to commit any further mistakes in the editing process or in the COI different templates, thanks in advance! Andrea Andrea Biographer (talk) 09:31, 31 December 2024 (UTC)
how to give feedback
i had a question on why an article was written and sent it to info@ wikipedia. they told me that I needed to engage in the "talk" feature and ask the editors, since wikipedia is only a platform. I did that and not only was the answer not given I was ridiculed, because I am not actually sure why. I am not interested in editing wikipedia, I am only interested in engaging with the editors, to understand inconsistencies. How exactly would I do that, if I should not be using the talk feature. thanks Mommer264 (talk) 12:34, 31 December 2024 (UTC)
- Hello. Discussion about an article usually takes place on its talk page. Discussion with an editor directly can occur on their user talk page.
- Note that your only other edit was about the Israeli-Arab/Palestinian conflict, which is a topic area with special rules that I will notify you of on your user talk page. One of those is that you must be an experienced user in order to make edits to any type of page about it. Your account must be 30 days old with 500 edits. 331dot (talk) 12:53, 31 December 2024 (UTC)
- Also answering your query: :You asked a question that is now at Talk:State of Palestine/Archive 21. Talk pages of articles are not for 'open' questions. Instead, the proper method is to propose a specific change of text, as in replace A with B, and include a reference to support your proposal. Given that State of Palestine is a very controversial article, editors who participate there - at both article and talk - can be short on tact. David notMD (talk) 12:54, 31 December 2024 (UTC)
I am not interested in editing wikipedia
If you're not interested in contributing to the project, then you probably ought to go elsewhere. The purpose of Wikipedia is to produce an encyclopedia; notably, talk pages are for constructively discussing the project and its content, not general Internet forums for discussion. There are many many discussion forums elsewhere on the Internet, and Wikipedia can even help direct you to some of them. If you do wish to contribute to the encyclopedia, take a look at WP:Welcome. Thank you and I hope you have a good day. --Slowking Man (talk) 21:03, 31 December 2024 (UTC)
VisualEditor
I've just changed my Preferences > Editing
Turned on "Enable the visual editor" (which, I think, it's on by default in Wikipedia in Italian language)
But i can not find out how to edit with VisualEditor.
I've looked at Help:VisualEditor and it says (Help:VisualEditor#Opening VisualEditor to click on the "Edit" tab (in the picture I can see a "drop down" menu to choose the editor).
But I've not this choice. My tab is named "Edit source". Centrodiurnomilano (talk) 14:47, 31 December 2024 (UTC)
- I've found out, reading Help:VisualEditor#First step: enabling VE (which seems a bit outdated, by the way).
- After I've changed my Preferences > Editing > "Enable the visual editor" = ON
- and the press "save" button at the bottom.
- Only then, a "drop down menu" show up in the same section: "Editing mode", which is setted by default "Remember the last editor". I've changed it to "Show both editor tabs".
- It's not easy to find out. Centrodiurnomilano (talk) 15:04, 31 December 2024 (UTC)
- By the way, I've looked at the settings on Wikipedia in Italian language and "Enable the visual editor" it's enough there (there is no "Editing mode" option at all) Centrodiurnomilano (talk) 15:08, 31 December 2024 (UTC)
- Italian Wikipedia and English Wikipedia run on the same system, MediaWiki, but are two different Wikipedias. So if this is a question about itwiki, then you should go to a help board on itwiki. If this a question about enwiki, you should be able to switch between the source editor and visual editor by pressing the pencil button next to the preview button in the source editor. Cowboygilbert - (talk) ♥ 16:07, 31 December 2024 (UTC)
- But, @Centrodiurnomilano, do note that you can only use the VisualEditor is some namespaces but not all. You can in mainspace, userspace, draftspace but not in wikispace which is the space with the Wikipedia: prefix. Cowboygilbert - (talk) ♥ 16:09, 31 December 2024 (UTC)
- Thank you, @Cowboygilbert . I've tried in main space and the in user space. Centrodiurnomilano (talk) 16:29, 31 December 2024 (UTC)
- Were you able to fix it using the instructions that I put,
you should be able to switch between the source editor and visual editor by pressing the pencil button next to the preview button in the source editor.
? Cowboygilbert - (talk) ♥ 16:30, 31 December 2024 (UTC)
- Were you able to fix it using the instructions that I put,
- Thank you, @Cowboygilbert . I've tried in main space and the in user space. Centrodiurnomilano (talk) 16:29, 31 December 2024 (UTC)
- But, @Centrodiurnomilano, do note that you can only use the VisualEditor is some namespaces but not all. You can in mainspace, userspace, draftspace but not in wikispace which is the space with the Wikipedia: prefix. Cowboygilbert - (talk) ♥ 16:09, 31 December 2024 (UTC)
"No Such Number tone" / "crybaby tone" (type of intercept error)
Hello everyone. This is one of my first times writing a brand-new article, and I'm not sure if this is a notable enough topic for its own page or a subsection in the intercept message.
This technically isn't a message, but before the intercept message was used, a different type of tone could also be returned instead of connecting to an operator. This is known under many different names, such as the "no such number tone" or even the "crybaby tone." This tone would be returned if a caller attempts to dial a number that can't possibly exist according to the numbering plan. The call would not be allowed to go through because some of these numbers could be reserved for private use. The tone itself is continuous, sweeping from 200-400Hz back to 200Hz again over the course of one second.
The issue is that it's almost exclusively used in North America, if not, completely exclusively. It was introduced by the Bell System in 1941, but I've also heard of it in use by the 3CX Asterisk system. However, my intent is to preserve information about this tone, as next to no information seems to exist about it. An excerpt from the Bell Labs Record describes it here. It also seems to be exclusive to crossbar systems.
[2] Bell Labs record
[3] Article about this record
[4] Sample of this sound
What do you think of this? ZetaformGames (talk) 19:01, 31 December 2024 (UTC)
- Hi @ZetaformGames, welcome to the teahouse and welcome to Wikipedia! I think that this could be a good idea as a subsection in the article for Intercept message or some other related article. I don't think that there is enough for a full article with the sources you've posted but could definitely be an interesting paragraph! Justiyaya 07:12, 1 January 2025 (UTC)
- Alright, thank you! And thanks for the welcome. I made this account a while ago, but haven't felt confident enough in my editing skills until now to contribute. ZetaformGames (talk) 17:46, 1 January 2025 (UTC)
Maryam Mirzakhani
Can someone fix the error in the Maryam Mirzakhani article?
Mirzakhani solved this counting problem by relating it to the problem of computing volumes in moduli space—a space whose points correspond to different complex structures on a surface genus Failed to parse (SVG (MathML can be enabled via browser plugin): Invalid response ("Math extension cannot connect to Restbase.") from server "http://localhost:6011/en.wikipedia.org/v1/":): {\displaystyle g} . In her thesis, Mirzakhani found a volume formula for the moduli space of bordered Riemann surfaces of genus g {\displaystyle g} with n {\displaystyle n} geodesic boundary components. From this formula followed the counting for simple closed geodesics mentioned above, as well as a number of other results. This led her to obtain a new proof for the formula discovered by Edward Witten and Maxim Kontsevich on the intersection numbers of tautological classes on moduli space.
Thanks. 76.14.122.5 (talk) 19:28, 31 December 2024 (UTC)
Hi IP 76.14.122.5. Generally, the best place to discuss something like this is on the talk page for the article in question, which in this case would be Talk:Maryam Mirzakhani; however, if you truly believe there's an error in the article, you can be WP:BOLD and fix it yourself if you think you can. Please understand though that "fix it" in this context means to correct the article in accordance with relevant Wikipedia policies and guidelines, and the policies/guidelines most likely applicable in this case are going to be Wikipedia:Verifiability and Wikipedia:No original research. Ideally, you're going to need to find WP:SECONDARY reliable sources (as defined by Wikipedia) to cite in support of the change you want to make; even if you know such changes to be true, you're still going to need to cite reliable sources in support to allow others to verify the changes. If you just make a change without providing any citations to a reliable source in support, there's a good chance the change will be undone by another user. Given that this seems to be related to mathematics, you might argue that "proving" the information to be incorrect based on Mirakhani's academic thesis is more than sufficient in and of itself, but Wikipedia generally requires something more and a thesis is going to be, for the most part, considered a WP:PRIMARY source and could have other issues as explained in WP:SCHOLARSHIP. You could also try asking about this at Wikipedia:WikiProject Mathematics since that's where you're going to have a better chance of finding someone sufficiently versed in mathematics who might be able to help sort this out. -- Marchjuly (talk) 19:52, 31 December 2024 (UTC);post edited. -- 22:29, 31 December 2024 (UTC)- Please disregard my reply IP 76.14.122.5. I misunderstood what you were asking about. ColinFine's suggestion below seems to be the best course of action here. -- Marchjuly (talk) 22:29, 31 December 2024 (UTC)
- Hello, IP user. What you are reporting is a problem in the software, the network, or the user interface, and not in the content of the article. Generally, technical problems of this sort are better handled at WP:VPT than here. However, I'm not seeing that problem, either on the browser on my laptop, or on the Android app. Is it repeatable, or might it have been a temporary glitch?
- Actually, now I look at it, the URL above appears to be a local proxy, so it may be that whoever manages your local network has not configured the proxy in a way that Wikipedia requires. Again, WP:VPT is a better place to ask about this.
- @Marchjuly. The problem that the IP is reporting is obviously a technical one, so your answer is entirely off the point. ColinFine (talk) 20:21, 31 December 2024 (UTC)
- Thanks for catching my mistake and pointing the OP in the right direction ColinFine. I've stricken my reply so as to not confuse the OP or anyone else. -- Marchjuly (talk) 22:29, 31 December 2024 (UTC)
Stuck on Puerto Rico outages notability.
Hi, new to Wikipedia here and probably gonna create a draft on the Puerto Rico power outage crisis but wanted to ask: I’m stuck in the notability of this topic, so, is the Puerto Rico power outage crisis notable enough for Wikipedia? By power outage crisis I mean the beginning of the Puerto Rican outages from Hurricane Maria to now since it has lasted multiple years with sustained media coverage when an outage does occur. Cheers! 66.50.50.222 (talk) 20:29, 31 December 2024 (UTC)
- Rather than a new article, you could expand on the outages already documented in Puerto Rico Electric Power Authority and in LUMA Energy. Schazjmd (talk) 20:44, 31 December 2024 (UTC)
- A search within Wikipedia on "Puerto Rico power outage" yields a list of several articles, including Puerto Rico Electric Power Authority and in LUMA Energy David notMD (talk) 21:15, 31 December 2024 (UTC)
- Noted. Will expand those articles then instead. Thanks! 66.50.50.222 (talk) 21:40, 31 December 2024 (UTC)
- My first impression would be that an article is likely warranted. PR is an island of ~3 million people, in the same neighborhood as Los Angeles. If Los Angeles were having sustained multi-year-long Issues with electrical service, there would be an article about it. Non-English language sources are perfectly acceptable for citing in articles, as long as a little care is observed.
- Note, to the anonymous editor: if you create an account you get your own shiny neato userspace to use mostly at your leisure, where for instance you can work on draft articles with no hurry. I have one underway in mine in fact. And thank you again for being interested in contributing to Wikipedia!
- (Regarding LA: some may have had come to mind the California energy crisis, but, neat fact, LA actually escaped impact from that because of having its own municipal utility with its own generation capacity!) --Slowking Man (talk) 01:38, 1 January 2025 (UTC)
- Note to User:Slowking Man: The userspace is the wrong place to draft an article. Either use your Sandbox or else follow instructions at WP:YFA to create a draft. David notMD (talk) 03:33, 1 January 2025 (UTC)
- Is "userspace" not the colloquial term for "your user page and any and all subpages of it"? The "official" user sandbox link is Special:MyPage/sandbox, which takes you to the /sandbox subpage of your user page. --Slowking Man (talk) 05:29, 1 January 2025 (UTC)
- Hello, the IP editor (66.xx) here, just created my account, and I mainly refrained from creating an article due to being worried about a potential conflict-of-interest (I live in Puerto Rico myself). I might work on a draft later today and collect sources (as @BusterD suggested), thank you all btw for helping me clear up this question I’ve had for some time now! Atheions (talk) 04:11, 1 January 2025 (UTC)
- Note to User:Slowking Man: The userspace is the wrong place to draft an article. Either use your Sandbox or else follow instructions at WP:YFA to create a draft. David notMD (talk) 03:33, 1 January 2025 (UTC)
- On the other hand, User:Slowking Man is correct that an article might be created on this newsworthy subject and that one shouldn't necessarily rely strictly on English-language sources. Looking at the existing material, it certainly seems a sequence of outages could be established from some of the reliable sources already applied to pagespace. I agree with User:David notMD that WP:YFA is a place to consider how to start a new page. I would start collecting sources, online and in print. BusterD (talk) 03:58, 1 January 2025 (UTC)
- Apologies if jumping the gun or creating a draft too early, but I have created one. Won’t be able to do much progress today but will def collect sources to use. Atheions (talk) 07:32, 1 January 2025 (UTC)
Am I allowed to post something about a game I am making on Wikipedia?
Am i allowed to post somthing about a game i am making? On Wikipedia Aaronfart14 (talk) 22:09, 31 December 2024 (UTC)
- Allowed? Yes. Good idea? No. Likely to remain on Wikipedia? No. Writing an article is difficult, particularly for new editors. And since this is about your game, you have a conflict of interest in writing about it. See H:YFA and WP:COI. Simply posting information about your game rather than writing an article would be promotion. See WP:PROMOTION Meters (talk) 22:20, 31 December 2024 (UTC)
- Aaronfart14, the relevant content guideline is WP:Wikipedia is not for things made up one day. Cullen328 (talk) 02:36, 1 January 2025 (UTC)
- If your game goes public, and if it becomes so popular that people are publishing about it, then there is a chance that someone other than you will create an article about it. David notMD (talk) 03:37, 1 January 2025 (UTC)
- Aaronfart14, the relevant content guideline is WP:Wikipedia is not for things made up one day. Cullen328 (talk) 02:36, 1 January 2025 (UTC)
Writing about Childhood and Early Life
I am writing an article about Andrea Sheridan Ordin, who is already included in two existing wikipedia articles entitled, "List of first women lawyers and judges in California" and "United States District Court for the Central District of California." I am interviewing her personally and have reputable sources about her career notability, but I'm not sure how to write her "early life" section, since there are not many sources describing her childhood aside from her firsthand account. How to I write about her early life without secondary sources? Aharten97 (talk) 22:17, 31 December 2024 (UTC)
- @Aharten97: It's not possible without reliable secondary published sources. For the policies, see WP:V. Personal notes from an interview are neither published nor reliable, and self-published statements are rarely reliable. If it's noteworthy it will have been published somewhere. If not then it isn't. -- zzuuzz (talk) 22:45, 31 December 2024 (UTC)
- https://ethics.lacity.gov/news/murray-and-ordin-re-elected-as-ethics-commission-leaders/ provides some info for an Early life and education section. David notMD (talk) 04:24, 1 January 2025 (UTC)
What to write on your talk page?
Above question HELSINKI!233 (talk) 22:32, 31 December 2024 (UTC)
- Hi and welcome, @HELSINKI!233. Your talk page is where other editors can leave messages for you or begin conversations with you. You can learn more at the guidelines for user pages. Schazjmd (talk) 22:37, 31 December 2024 (UTC)
- It is not for chat, or your thoughts/opinions about stuff. Also, with a few exceptions, you are allowed to delete content from your Talk page, although some people prefer to archive older content instead. David notMD (talk) 04:19, 1 January 2025 (UTC)
Citations about movies
I'm trying to improve The Lincoln Lawyer (film)
1) Is IMDB considered a good citation for the cast list, producer name, etc?
2) How do I add a citation to an existing infobox? (Visual or Source editor)
Many thanks
Ben (talk) 22:36, 31 December 2024 (UTC)
- Hi and welcome, @Littenberg. Imdb is not considered a reliable source because much of its information is user-generated. You can learn how to add citations at Easier Referencing for Beginners. Schazjmd (talk) 22:39, 31 December 2024 (UTC)
- If you click on Edit at the top menu it allows you to edit the entire article, including the Infobox. David notMD (talk) 04:15, 1 January 2025 (UTC)
How do I create a Wikipedia article
How do I create an article on Wikipedia? Красный Октябрь (talk) 23:12, 31 December 2024 (UTC)
- Здравствуйте, @Красный Октябрь, and welcome to the Teahouse. My earnest advice to new editors is to not even think about trying to create an article until you have spent several weeks - at least - learning about how Wikipedia works by making improvements to existing articles. Once you have understood core policies such as verifiability, neutral point of view, reliable, independent sources, and notability, and experienced how we handle disagreements with other editors (the Bold, Revert, Discuss cycle), then you might be ready to read your first article carefully, and try creating a draft. ColinFine (talk) 23:33, 31 December 2024 (UTC)
- Can you help me learn how to make one? I want to make an article on the Kazan bombing that happened about a 1.5 week ago. Красный Октябрь (talk) 23:50, 31 December 2024 (UTC)
- Красный Октябрь, please be aware that the Russo-Ukrainian War is a designated contentious topic. You cannot write new content about that war until your account is Extended confirmed, which means the account is over a month old (it is) and has made over 500 constractive edits (you have a long way to go). Cullen328 (talk) 02:56, 1 January 2025 (UTC)
- Then can someone else make an article about it? Красный Октябрь (talk) 04:08, 1 January 2025 (UTC)
- Teahouse Hosts are here to advise, but not to be authors or co-authors. David notMD (talk) 04:13, 1 January 2025 (UTC)
- Who do I ask about it? Красный Октябрь (talk) 08:59, 1 January 2025 (UTC)
- Красный Октябрь, if you persist in discussing a designated contentious topic such as the Russo-Ukrainian War before you are extended confirmed, you may be blocked. So, please edit other topic areas until then. Cullen328 (talk) 09:16, 1 January 2025 (UTC)
- You could ask on Talk:Russo-Ukrainian War. Normally such talk pages are protected, but that one doesn't seem to be at this time. What might happen is that someone adds a paragraph about the incident to the existing article rather than create a new article.
- I'll add that editors who aren't extended-confirmed generally aren't permitted to use the talk pages of such articles either, and I have always disagreed with this, mainly because constructive edit requests end up in WP:RFED, making extra work for administrators. ~Anachronist (talk) 16:34, 1 January 2025 (UTC)
- Who do I ask about it? Красный Октябрь (talk) 08:59, 1 January 2025 (UTC)
- Teahouse Hosts are here to advise, but not to be authors or co-authors. David notMD (talk) 04:13, 1 January 2025 (UTC)
- Then can someone else make an article about it? Красный Октябрь (talk) 04:08, 1 January 2025 (UTC)
- Красный Октябрь, please be aware that the Russo-Ukrainian War is a designated contentious topic. You cannot write new content about that war until your account is Extended confirmed, which means the account is over a month old (it is) and has made over 500 constractive edits (you have a long way to go). Cullen328 (talk) 02:56, 1 January 2025 (UTC)
- Can you help me learn how to make one? I want to make an article on the Kazan bombing that happened about a 1.5 week ago. Красный Октябрь (talk) 23:50, 31 December 2024 (UTC)
Why are padlocks not automatically added when an article is protected?
Sorry if this is the wrong place. Heyaaaaalol (talk) 02:38, 1 January 2025 (UTC)
- @Heyaaaaalol Because an administrator needs to add a protection notice (the padlocks) to a page in order to show the padlock on pages. Sometimes the padlocks do not get placed on a page, especially if it's a user page, unless if the protection notice is placed by an administrator. NicePrettyFlower (talk) 03:35, 1 January 2025 (UTC)
- Heyaaaaalol, I have done some page protection myself. Usually when I apply protection, I wish to see the "padlock" icon applied. Sometimes when I use the protection script, I forget to click the toggle which leaves the padlock icon. In that case, there's a bot which usually fixes that mistake automatically. If you see a protected page without an icon, you might tell someone. Do our answers help? BusterD (talk) 03:44, 1 January 2025 (UTC)
- Hello, @Heyaaaaalol. To actually answer your question: because nobody has implemented that function in the software. I don't know whether there is a technical reason for that, or whether it's just that nobody's got round to it. Questions about the software and user interface are better asked at the Village pump: either WP:VPT or WP:VPR ColinFine (talk) 13:34, 1 January 2025 (UTC)
- @Heyaaaaalol: Protecting a page is a log action in the MediaWiki software which powers Wikipedia and thousands of other wikis. It does not make an edit to the page. Logs are separate from edits. Displaying a padlock with a link is a Wikipedia practice. We do it by editing the page and adding special code which places a padlock in the corner instead of the normal text area. Other wikis may use no or other symbols for protected pages, place them in other places, and make no or other links on them. A MediaWiki feature to automatically display a symbol on protected pages was recently added at phab:T12347 but it's disabled by default. I haven't examined how flexible it is but I guess it would be non-trivial for us to convert to using it when we already have a well-functioning system. There is so far only a single Wikimedia wiki which has set
wgEnableProtectionIndicators
to true in https://noc.wikimedia.org/conf/highlight.php?file=InitialiseSettings.php. PrimeHunter (talk) 15:08, 1 January 2025 (UTC)
Multiple non-free album covers on one page
For the album Breakfast with Girls, there's an associated EP located at the section Brunch. The EP has an infobox but is currently without cover artwork, so I'm wondering if it's appropriate for the article to have another non-free image when one is already in use for the album's artwork. If not within free use, Brunch's artwork (seen here) is mostly text on a black background, so would I be allowed to crop out the non-text part and use that as a public domain text logo image in an infobox? Koopastar (talk) 04:50, 1 January 2025 (UTC)
- Hi Koopastar, I do think that another cover would be within fair use. There's also been instances of this in the past like In Rainbows#In Rainbows Disk 2. Happy editing! Justiyaya 06:59, 1 January 2025 (UTC)
afd
I really like a certain article, but it's afd, if it is deleted, is there a way to still view it 🐢 (talk) 05:24, 1 January 2025 (UTC)
- History checker SimpleSubCubicGraph (talk) 05:26, 1 January 2025 (UTC)
- If the article is still "public" currently, you can save a copy for yourself: Download as PDF (maybe see also Help:Export). Otherwise go to: CAT:RESTORE. --Slowking Man (talk) 06:14, 1 January 2025 (UTC)
- This appears to be about X11 color names, which has been in existance since 2003 and has been edited more than 700 times since then. While it is unlikely that it will be deleted, as noted, you can save a copy to your computer. David notMD (talk) 13:55, 1 January 2025 (UTC)
Hypothetically, could it be possible to write a netural autobiography?
Hypothetically, would it be possible for someone on Wikipedia to write a neutral autobiography of themselves? Showing the good and bad and making no favor to either side, providing proof and checking all of the boxes. How would that go and would it be accepted? What if say a Wikipedia administrator that is not really well known becomes for example the president of the US? Can they have oversight over their own article? Are they removed from their position? Can they no longer edit anything involving US politics due to their inherent bias? And finally has there been any real examples of this over the past 20+ years? SimpleSubCubicGraph (talk) 05:25, 1 January 2025 (UTC)
- Hello, SimpleSubCubicGraph. Hypothetically, what you describe is possible but the president of the US example is implausible because credible candidates for that office would already be the subject of a Wikipedia article. More plausible would be a longtime editor elected to a state or provincial legislature. I see no reason why that person could not submit an autobiography through Articles for Creation with full disclosure, have the article accepted and continue as an editor or even as an administrator. They should certainly recuse from the current legislative affairs of their state or province but otherwise I would not see a broader problem. As for whether anything like that has ever happened, I do not know. Maybe another editor does. Cullen328 (talk) 06:49, 1 January 2025 (UTC)
- @SimpleSubCubicGraph welcome to the Teahouse! Interesting questions. I would say that while it is hypothetically possible, guidelines still strongly discourage creations of autobiographies and I haven't seen an instance of it being successful outside of early Wikipedia. This would have to go through articles for creation and the editor should declare their conflict of interest. Subjects do not have oversight of their own article.
- I would argue that if someone is elected president of the US they would have a financial relationship with the federal government and should stay away from editing those topics. I think they would have potentially less of a conflict of interest with state governments and historical united states politics. I don't think one would be removed as an administrator because they are elected to a public office here. I don't know of any Wikipedian that has been elected. CongressEdits is probably the closest to an example of this. Justiyaya 06:46, 1 January 2025 (UTC)
- Succeeding at autobiography is more likely to occur in an area such as WP:NACADEMIC. A senior professor at a university would have as models articles about other professors at their university. David notMD (talk) 14:02, 1 January 2025 (UTC)
- The only time I recall a good autobiography being written was when the CEO of a small business wrote one, submitted it for review, and it was accepted after some minor revisions. So it is possible.
- By now I've gained enough experience on Wikipedia that I could probably write a neutral biography about myself, but because I am not notable, there's no point. ~Anachronist (talk) 16:26, 1 January 2025 (UTC)
- a) It would probably go over reasonably well, presuming they followed the Correct way to do that kind of thing: don't create your own bio article (meaning here the literal "creating a new page in Main namespace") and don't edit it directly but put up stuff on the Talk page for others to evaluate, revise, and put into the article if they decide.
- b) No, no one "owns" articles and gets special "powers" over them. Since all Wikipedia content is "free as in freedom" you and anyone else can copy it put it up elsewhere and do whatever with the copies, as long as you credit the original creators.
- c) Why would they get adminship removed if they haven't misused it?
- d) They probably ought to stay away from US politics content yes, being rather WP:INVOLVED. Also realistically the POTUS is not going to have ample free time to devote to Wikipedia contributing, or to be inclined to devote what little precious free time they get to, doing more work.
- e) Have a look at WP:List of Wikipedians with articles. --Slowking Man (talk) 17:26, 1 January 2025 (UTC)
Wiki page written by subject or friend of subject?
Hello, I saw a page on Wikipedia that looks like it was, based on the way it's presented, mostly written by the person who is the subject of the page, or a close friend. What is the standard flag or way to raise this on the page's talk page? Thank you for your help. FireBatV (talk) 08:05, 1 January 2025 (UTC)
- Hello @FireBatV! Welcome to the Teahouse. The standard way is to tag the article with the Template:COI on the main article page. Alternatively, you can use Twinkle to tag it as well. TNM101 (chat) 12:17, 1 January 2025 (UTC)
- Thank you for the help! I've added the
{{Template:COI}}
to the page in question (Rhett Ayers Butler) and as per the instructions on the template page I am have also added{{Connected contributor}}
to the talk page, so editors can take it from there. Thank you again for the help and the friendly welcome, my issue has been resolved. FireBatV (talk) 21:13, 1 January 2025 (UTC)- No problem! Feel free to reach out if you need any more help TNM101 (chat) 06:46, 2 January 2025 (UTC)
- Thank you for the help! I've added the
Editing the Wikipedia page of "The crown"
i want to edit the Wikipedia page of "the crown" on netflix why i wont add the seventh season of "the crown" if you cant tell the staff at netflix please add aditional information like the royal wedding of prince william and kate Middleton and the queen involvement in the 2012 summer olympics in london england alongside James bond!!! 89.128.137.159 (talk) 14:08, 1 January 2025 (UTC)
- Hello IP Editor: there was no 7th season of The Crown, so your additions were inappropriate and were removed. Please don't add fake information to Wikipedia. qcne (talk) 14:18, 1 January 2025 (UTC)
- False information is vandalism, which if continued can lead to your IP address being blocked and any account your subsequently register being blocked. David notMD (talk) 15:20, 1 January 2025 (UTC)
- Look for verified information about future seasons. This isn't forums and speculations. Cwater1 (talk) 20:34, 2 January 2025 (UTC)
The show button doesn't work on phone
Hi. Some articles have this dialouge box on top of them. But when we touch the show button on phone (Chrome for Android for me), nothing happens. I don't know about desktop. Aminabzz (talk) 15:38, 1 January 2025 (UTC)
- What is an example of such an article? What happens when you use the Wikipedia app? Does it work then? ~Anachronist (talk) 16:21, 1 January 2025 (UTC)
- Hello, @Aminabzz, and welcome to the Teahouse.
- I agree. If I look at Amathus, I get that box (from template {{expand language}}) at the top, but on a browser on my Android phone, when I pick "Show" it replaces the word "Show" with "Hide", but doesn't expand the box, so I can't see additional information.
- However, the box has a "Learn more" button, and if I pick that it shows me a little more, but not the full information that I see on my browser on a computer.
- This looks like a bug in the user interface: WP:VPT is a better place to ask/report such things than here, and I suggest you post there. Thank you for pointing it out. ColinFine (talk) 18:04, 1 January 2025 (UTC)
- I note that I cannot find any of these problems when I use the Wikipedia app. ~Anachronist (talk) 15:59, 2 January 2025 (UTC)
How to find the full form of abbreviations on phone?
Hi. On Wikipedia, some abbreviation words have dotline underlines. When we hover the mouse cursor on them on PC we can see the full form. For example, TBA reveals to be "to be announced".
But on mobile phones (Chrome for Android for me) there is no mouse so that we hover the cursor on them! So how can we find the full form of abbreviations on these dotline-underlined words in cellphones? Holding the word doesn't work.
Look at this for seeing the dotlines. Aminabzz (talk) 15:40, 1 January 2025 (UTC)
- That's one of the limitations of using a smartphone as a browser. There is no notion of "hover". I have seen hovering implemented in some Samsung phones, in which holding your finger near to the screen without touching it is sensed as a hover, but this worked only in certain apps and wasn't a universal experience across all apps on the phone. Unless someone has a better answer, I'd say that features reliant on hovering are generally not accessible on smartphones. ~Anachronist (talk) 16:16, 1 January 2025 (UTC)
- Samsung devices with S Pen support actually support "hover" by well, hovering the S Pen close to the display. The display has an "active digitizer" that can sense the Pen with NFC. Shows a little cursor and pops up stuff and you can hit the Pen's button to do things, very neat actually. ...Buuuut still doesn't work for the abbr stuff in browsers because it's handled differently in the system software (as it's not "really a mouse", it's handled by different code) and so doesn't "pass through" a mouse hover event down to the browser software. Shucks. --Slowking Man (talk) 17:07, 1 January 2025 (UTC)
- Oh yeah this is a known problem for years. Abbr and friends use the standard HTML tags for such things, and browsers don't gaf apparently about making them "work" when there's no pointing device (mouse), and the attitude appears to be ¯\_(ツ)_/¯. WP could implement some sort of JavaScript "workaround" that pops up a thing, but, that requires deploying something "globally" site-wide and that is a Big Deal so it needs Official approval: ask around at WP:VPT whether there's any effort in this direction. (FYI templates can't have JavaScript, it needs to be JS to "dynamically" add new page elements and display them)
- In the meantime the "workaround" is to hit edit and look at the wikitext. Or you could also view the page HTML source and use the browser's "find" to go to the abbreviation which will show its definition. Also also there might be some userscript someone has made to make it pop up, which you can "install" to use while logged-in. (You could also always plug in a mouse/trackpad/etc or connect a wireless one ) --Slowking Man (talk) 17:07, 1 January 2025 (UTC)
- @Aminabzz, Anachronist, and Slowking Man: I couldn't find any such script, so I made it. JJPMaster (she/they) 05:08, 2 January 2025 (UTC)
- Is this something that needs to be deployed in the Wikipedia software, or is it something that a user can add to vector.js? ~Anachronist (talk) 15:52, 2 January 2025 (UTC)
- @Anachronist: The latter. JJPMaster (she/they) 17:46, 2 January 2025 (UTC)
- @Aminabzz: Well, JJP has a solution for you, but it requires that you be logged into Wikipedia to take advantage of it. Me, I use the Wikipedia app for reading Wikipedia on my phone, and I never encountered a "hover" situation in that app. ~Anachronist (talk) 17:58, 2 January 2025 (UTC)
- @Anachronist: The latter. JJPMaster (she/they) 17:46, 2 January 2025 (UTC)
- Is this something that needs to be deployed in the Wikipedia software, or is it something that a user can add to vector.js? ~Anachronist (talk) 15:52, 2 January 2025 (UTC)
- @Aminabzz, Anachronist, and Slowking Man: I couldn't find any such script, so I made it. JJPMaster (she/they) 05:08, 2 January 2025 (UTC)
GA Spotcheck
Apparently I might've misunderstood what is needed for a source spotcheck in a good article review. Could someone please explain what is needed? History6042😊 (Contact me) 15:53, 1 January 2025 (UTC)
- Appears that at Wikipedia talk:Good article nominations, date 1/1/25, there has been a question raised about GA nomination reviews conducted by History60432. David notMD (talk) 17:07, 1 January 2025 (UTC)
- Yes, that is why I am asking. History6042😊 (Contact me) 17:09, 1 January 2025 (UTC)
- @History6042, note 2 on the Wikipedia:Good article criteria page explains source checking. If you're concerned that your interpretation isn't in line with the community's, you might find it helpful to discuss with other editors involved with the good article process at Wikipedia talk:Good article nominations. Schazjmd (talk) 17:33, 1 January 2025 (UTC)
- So it means Verifiable with no original research:
- it contains a list of all references (sources of information), presented in accordance with the layout style guideline;
- reliable sources are cited inline. All content that could reasonably be challenged, except for plot summaries and that which summarizes cited content elsewhere in the article, must be cited no later than the end of the paragraph (or line if the content is not in prose);
- it contains no original research; and
- it contains no copyright violations or plagiarism? History6042😊 (Contact me) 19:25, 1 January 2025 (UTC)
- The note says
"Ideally, a reviewer will have access to all of the source material, and sufficient expertise to verify that the article reflects the content of the sources; this ideal is not often attained. At a minimum, check that the sources used are reliable (for example, blogs are not usually reliable sources) and that those you can access support the content of the article (for example, inline citations lead to sources that agree with what the article says) and are not plagiarized (for example, close paraphrasing of source material should only be used where appropriate, with in-text attribution if necessary)."
Schazjmd (talk) 20:40, 1 January 2025 (UTC)- Okay I think i did that but do I just need to like write it down? History6042😊 (Contact me) 21:06, 1 January 2025 (UTC)
- Saying in your review what you checked is helpful. Perhaps read through this discussion on spot checks. You might also find it useful to read through some GA reviews by experienced reviewers, see what they're doing in their written reviews that you can learn from. Schazjmd (talk) 21:47, 1 January 2025 (UTC)
- Thank you. History6042😊 (Contact me) 00:39, 2 January 2025 (UTC)
- Saying in your review what you checked is helpful. Perhaps read through this discussion on spot checks. You might also find it useful to read through some GA reviews by experienced reviewers, see what they're doing in their written reviews that you can learn from. Schazjmd (talk) 21:47, 1 January 2025 (UTC)
- Okay I think i did that but do I just need to like write it down? History6042😊 (Contact me) 21:06, 1 January 2025 (UTC)
- The note says
Creating an article heavily lacking sources :/
I've been working on trying create an article (Draft:Millennium Force's effects) but I've run into some issues; including after initial submission. The only sources I can find are mainly from various YouTube videos (not tied to the subject) and very few separate links; so it's no wonder why it wasn't accepted.
So I know what I've written down is true, but I don't have the secondary sources to prove it and that makes my info Original Research. What can I do? I was told I could try another Wiki (ex: Amusement Park Wiki) but is there anything I can do to keep it on Wikipedia? Thanks! Therguy10 (talk) 16:49, 1 January 2025 (UTC)
- @Therguy10 Welcome to theTeahouse. I'm afraid not. Reliable sources are fundamental to Wikipedia articles. See WP:42. Shantavira|feed me 18:01, 1 January 2025 (UTC)
- Hello, @Therguy10. I agree with Shantavira. Please see also No amount of editing can overcome a lack of notability ColinFine (talk) 18:07, 1 January 2025 (UTC)
- @Shantavira @ColinFine Thank you both! I do believe that the article has potential in the future, but I also understand it isn't notable enough as of now. With the article being a history of a subject, there is older information that I fear may not be written about. My hope is that the new changes made this year were important enough to spark some kind of interest for a source; I'll have to wait and see. Therguy10 (talk) 18:13, 1 January 2025 (UTC)
- well i guess technically the changes were made last year...lol Therguy10 (talk) 18:14, 1 January 2025 (UTC)
- If you can find reliable sources, why not improve the Ride experience section of Millennium Force? Schazjmd (talk) 18:27, 1 January 2025 (UTC)
- That was a strong consideration of mine. I certainly won't be able to fit everything inside of the main article but it might not hurt to add a little more than what I already have there. Once again my biggest issue would be collecting sources; this seems more doable. Thanks! Therguy10 (talk) 18:34, 1 January 2025 (UTC)
- If you can find reliable sources, why not improve the Ride experience section of Millennium Force? Schazjmd (talk) 18:27, 1 January 2025 (UTC)
- well i guess technically the changes were made last year...lol Therguy10 (talk) 18:14, 1 January 2025 (UTC)
How do i delete a Wikepedia page?
a school has a wikepedia page, (carlisle public schools) i have moved the page to inside the town page (carlisle masssachusetts) so how do i delete the old school page? it is useless (i moveed the page because it contains about only 1 paragraph, so why not make it a section) so yeah, if someone knows how to delete a page, please delete Carlisle Public Schools
how to find page - go to the education part of carlisle, massachusetts, press carlisle public schools, and you will be there. Theawezomefriend12 (talk) 19:39, 1 January 2025 (UTC)
- Carlisle Public Schools is an article. If you believe it should be deleted, the process is to nominate it for deletion via the WP:AFD process. What you added to Carlisle, Massachusetts was rightfully deleted because you did not include any references. See Maynard, Massachusetts for an example of a referenced education section. I was going to point you to Acton or Sudbury, but those have unreferenced content. There is also Concord-Carlisle High School, but that has its own problems. David notMD (talk) 20:02, 1 January 2025 (UTC)
- Okay, Thanks! Theawezomefriend12 (talk) 20:30, 1 January 2025 (UTC)
- @Theawezomefriend12: I'm going to offer some slightly different advice. For a small article in this situation: First copy the relevant text to the target article (the article about the town). If there are no references then be sure to add one or some. See WP:CWW for the correct process, if you are moving any text. Once that has been successfully done, turn the former article (the school district) into a redirect to the town as you did previously. There is no need to actually delete anything: redirects are useful for people trying to find things, such as information about Carlisle Public Schools, and can be safely left behind. -- zzuuzz (talk) 21:12, 1 January 2025 (UTC)
- Thanks for the advice, i will remove the deletion (i dont know if thats possible i am new) and make the page redierect to Carlisle, Massachusetts. Theawezomefriend12 (talk) 21:30, 1 January 2025 (UTC)
- @Theawezomefriend12: I've helped a bit with the deletion part. The best way to learn is to just do things, so I'll leave you to continue the process (and will probably pop back in a bit). I'll just mention that it appears that the only reason your previous attempt was reverted was because you didn't include any references for the content you were adding/moving to the town article. Sufficient references probably exist in the district article so they can just be copied. If not, I'm sure they won't be difficult to find. -- zzuuzz (talk) 21:38, 1 January 2025 (UTC)
- oh ok, i am propably going to be moving the main description and the new gallery part i added, i probably will not be added refrences and external links. Theawezomefriend12 (talk) 21:45, 1 January 2025 (UTC)
- @Theawezomefriend12: I've helped a bit with the deletion part. The best way to learn is to just do things, so I'll leave you to continue the process (and will probably pop back in a bit). I'll just mention that it appears that the only reason your previous attempt was reverted was because you didn't include any references for the content you were adding/moving to the town article. Sufficient references probably exist in the district article so they can just be copied. If not, I'm sure they won't be difficult to find. -- zzuuzz (talk) 21:38, 1 January 2025 (UTC)
- Thanks for the advice, i will remove the deletion (i dont know if thats possible i am new) and make the page redierect to Carlisle, Massachusetts. Theawezomefriend12 (talk) 21:30, 1 January 2025 (UTC)
- @Theawezomefriend12: I'm going to offer some slightly different advice. For a small article in this situation: First copy the relevant text to the target article (the article about the town). If there are no references then be sure to add one or some. See WP:CWW for the correct process, if you are moving any text. Once that has been successfully done, turn the former article (the school district) into a redirect to the town as you did previously. There is no need to actually delete anything: redirects are useful for people trying to find things, such as information about Carlisle Public Schools, and can be safely left behind. -- zzuuzz (talk) 21:12, 1 January 2025 (UTC)
- Okay, Thanks! Theawezomefriend12 (talk) 20:30, 1 January 2025 (UTC)
Hi! PARIS
Good morning, good afternoon or good evening. Happy new year 2025. Please how to move to the main space the Draft:Hi! PARIS. This research center in AI is now one of the most important in France : https://www.lesechos.fr/tech-medias/intelligence-artificielle/une-dotation-de-70-millions-pour-la-formation-de-lelite-de-lia-en-france-2096150 and https://www.lemondeinformatique.fr/actualites/lire-hec-et-l-institut-polytechnique-remportent-l-appel-a-projets-cluster-ia-93818.html. Many thanks in advance for your help. Have a great day. 2A01:CB00:B48:9900:1C67:231F:4C2F:9933 (talk) 21:57, 1 January 2025 (UTC)
- On the AoPS Wiki go to special:move. I don't know bout wikipedia tho 73.31.42.97 (talk) 22:22, 1 January 2025 (UTC)
- Sorry its Special:MovePage 73.31.42.97 (talk) 22:23, 1 January 2025 (UTC)
- Bonsoir, IP user.
- 73. is right that the technique for moving pages is to use the Move function (though most people do it from their user-interface, rather than going to Special:MovePage.
- However, not logged in users do not have access to that function.
- I was going to advise you to submit it for review. However, I see that the draft was created in mainspace user BobVillars (since blocked for sockpuppetry) was then moved to Draft space by @Rosguill, was submitted for review by an ipV6 user in the same range as you, then moved to main space again by McSyl, also blocked as a sockpuppet of BobVillars, and moved back to draft space again by @Janhrach.
- Neither you nor anybody else has since made any substantial edits to the draft.
- I am finding it very difficult to assume good faith. I will ask you directly: are you BobVillars/McSyl? If so, you are evading your block, which is not permitted. ColinFine (talk) 22:59, 1 January 2025 (UTC)
- Hi Colin. Thanks for your answer. I don’t know what you are talking about. I am using 5G access so my Ip range is used by million of users. But if it is too complicated no worries, please cancel my request. This AI cluster is one of the most important in France and one of the biggest in Europe. So soon or later it will be back to mainspace. Have a great day. 2A01:CB06:B064:82B5:2C9D:5034:3AC5:1BD4 (talk) 06:21, 2 January 2025 (UTC)
AoPS page
Why isn't there a page for Art of Problem Solving? 73.31.42.97 (talk) 22:15, 1 January 2025 (UTC)
- Hi IP 73.31.42.97. There could be a couple reasons why: (1) nobody thought enough about the book/series to try and create one; or (2) somebody did try to create one, but the subject wasn't deemed to meet Wikipedia:Notability (books) and was either deleted altogether or added to Richard Rusczyk. Art of Problem Solving current WP:REDIRECTs to the "Rusczyk" article and its page history shows that there once was a stand-alone article about the book/series, but it was "redirected" per Wikipedia:Articles for deletion/Art of Problem Solving (2nd nomination). If you feel something has changed since then, you might want to first explain why to the administrator who closed the Articles for Deletion discussion. That administrator's name is Liz. Perhaps by asking at User talk:Liz, Liz can tell you what is needed for the article to be recreated or restored. -- Marchjuly (talk) 22:30, 1 January 2025 (UTC)
Where is Wikipedia year in review for iOS?
I am looking for Wikipedia year in review in iOS where is it? 172.59.25.192 (talk) 22:18, 1 January 2025 (UTC)
- Welcome to the Teahouse, IP user. This isn't something I'm familiar with, but a Google search yields this link which I hope gives you what you seek: https://www.mediawiki.org/wiki/New_Engagement_Experiments/PES_1.3.1:_Wikipedia_insights. Regards, Nick Moyes (talk) 01:56, 2 January 2025 (UTC)
How to find vandalism on Wikipedia?
The closest thing I have found to vandalism is on the page for the number 3, where someone changed references to “3” with “2.” I’m wondering how to find vandalism, so I can revert it. Most of my edits are fixing grammar. Thank you. Heyaaaaalol (talk) 00:18, 2 January 2025 (UTC)
- @Heyaaaaalol Welcome to the Teahouse. If you'd care to read WP:VANDALISM, and especially the section linked from this shortcut: (WP:SPOTVAN), you should have all the answers you need. Monitoring 'Recent Changes' is the best way to spot ongoing vandalism, and you can choose to select only certain types of edits which highlight the most likely problematic edits for you to assess and respond to, and ignore all the honest ones. Thank you for your interest in helping out in this field. Regards, Nick Moyes (talk) 01:50, 2 January 2025 (UTC)
- Go to the “recent changes” link and there are filter options you can select. What you’ll want to do is choose “likely bad faith”, “very likely bad faith”, and “latest revision”. Then save that filter as your default. Grimaceshakeohio (talk) 04:35, 3 January 2025 (UTC)
"Series of small edits"
I find it difficult to follow and review edits for valid constructive contributions when an editor makes a "series of small edits" (example: 1, 2, 3) where several of the edits could, IMHO, easily and conceivably be made in a single edit with a single edit summary encompassing all of what is edited. Is there policy or guidelines about this practice that could help me in a talk page discussion I am currently engaged in? Iljhgtn (talk) 00:23, 2 January 2025 (UTC)
- Well, there's WP:Cautious, which touches on not making too large edits. There is probably better material than this, but I can't find it.
- The point has been discussed on the Help/Teahouse desks before, and various editors have said that more smaller edits are better than one all-encompassing one, because if only one or two of several small edits are objected to, they can be reverted individually, but if they're part of a larger edit the whole thing has to be reverted and then it becomes difficult to sort out which details are accepted and which disputed.
- I myself, when reading an article for its content, sometimes make small edits as I go along on noticing typos, misspellings, incorrect grammar, etc. If I were to be deliberately setting out to copyedit the article (I used to be a professional editor) I would likely make them in larger batches, perhaps one per Section or Subsection (basically, as often as [edit] is present in the text), because that makes for a smaller and easier chunk (with all its confusing reference codings) to navigate in the edit box.
- In general, I don't think an editor making good-faith edits need think primarily of the convenience of some hypothetical would-be reverter, and I wouldn't criticise anyone for making successive small edits. Were the edits to smell of some deliberate obfustication as a cover for vandalism, or edit-count inflation, it would be another matter. {The poster formerly known as 87.81.230.195} 94.6.84.253 (talk) 02:11, 2 January 2025 (UTC)
- Small or a series isn't the issue really. It is that several of the edits could fall under a single edit summary, and therefore could be made as one. I am curious about this comment though, "
Were the edits to smell of some deliberate obfustication as a cover for vandalism, or edit-count inflation, it would be another matter
", because that is kind of what I think is the heart of the matter. How can you differentiate between the two if we are not allowed to scrutinize editing behavior which is a mix perhaps of both lots of small, with some more impactful edits laced in there? Iljhgtn (talk) 14:33, 2 January 2025 (UTC)- The key being the "lots of small" could just as easily be one edit under one edit summary, and then more larger or more substantive edits separately under another. If the "lots of small edits" are all broken into separate edits, for me at least, it makes it harder to check on someone's editing in the first place to determine IF there is in fact a case of real vandalism, deliberate edit-count inflation, or some other intended obfuscation. Iljhgtn (talk) 14:35, 2 January 2025 (UTC)
- You can differentiate because if it is a cover for vandalism, you can point out the vandalism. Edit-count inflation these days is mostly people trying to game their way to extended confirmed status - the edits you point out here are from an editor who already has that status. There's no policy against editing this way - this editor isn't doing anything wrong.
- If you want to examine a set of contiguous edits at once, you can get a multi-edit diff by using the radio buttons on the history page of the article in question. MrOllie (talk) 14:40, 2 January 2025 (UTC)
- Understood, and no accusation of vandalism has been levied, but if several edits could be made under a single edit summary, is there any policy that they should be? That is really the essence of my question. Iljhgtn (talk) 14:43, 2 January 2025 (UTC)
- There is not. Schazjmd (talk) 14:46, 2 January 2025 (UTC)
- Ok. Thank you for answering my question then. Iljhgtn (talk) 14:56, 2 January 2025 (UTC)
- There is not. Schazjmd (talk) 14:46, 2 January 2025 (UTC)
- Understood, and no accusation of vandalism has been levied, but if several edits could be made under a single edit summary, is there any policy that they should be? That is really the essence of my question. Iljhgtn (talk) 14:43, 2 January 2025 (UTC)
- The key being the "lots of small" could just as easily be one edit under one edit summary, and then more larger or more substantive edits separately under another. If the "lots of small edits" are all broken into separate edits, for me at least, it makes it harder to check on someone's editing in the first place to determine IF there is in fact a case of real vandalism, deliberate edit-count inflation, or some other intended obfuscation. Iljhgtn (talk) 14:35, 2 January 2025 (UTC)
- Small or a series isn't the issue really. It is that several of the edits could fall under a single edit summary, and therefore could be made as one. I am curious about this comment though, "
- How isn't that benign? Is there an issue with the individual edits? My editing tends to match my own writing (on non-Wikipedia, Word, etc.): dump a bunch of text, and make then increasingly granular refining edits. -- Very Polite Person (talk) 00:37, 3 January 2025 (UTC)
Political Party Relations
Just like how we have Category:Sign language family tree templates, should we have that for political parties kind of like below but for whole countries? These would be based on the infobox political party preceded by/succeeded by/merger of/merged into. If so would these be their own article or would they be in the many "List of political parties in ___"? I am curious and want to know if these are wanted/needed.
SPD | |||||||||||||||||||||||||||
Spartacus League | |||||||||||||||||||||||||||
KPD | |||||||||||||||||||||||||||
SED | |||||||||||||||||||||||||||
PDS | WASG | ||||||||||||||||||||||||||
Die Linke | |||||||||||||||||||||||||||
BSW | |||||||||||||||||||||||||||
History6042😊 (Contact me) 00:39, 2 January 2025 (UTC)
- Hi History6042, that's an interesting idea, you might want to pitch this at Wikipedia talk:WikiProject Politics. Justiyaya 03:23, 2 January 2025 (UTC)
- Okay, I'll do that, thanks. History6042😊 (Contact me) 03:27, 2 January 2025 (UTC)
Freedom of Panorama Inquiries
This is regarding the article Liliget Feast House. The site where the restaurant once stood was torn down 2008, and I wanted to inquire regarding the best way to navigate Fair Use and Freedom of Panorama rules on Wikipedia in order to get photographs of the site on the page. This is probably best asked on Commons, but I'm not familiar with where I might be able to ask on there either. Ornithoptera (talk) 02:19, 2 January 2025 (UTC)
- Hi Ornithoptera. I think you can take a picture of the where the restaurant once stood and upload it here, which should be fine under Freedom of panorama#Canada which also allows incidental inclusion of copyrighted, non-architectural work (also commons:COM:FOP Canada). You would still be able to release a photograph that you took of publicly viewable buildings under a free license.
- If you are asking about whether an inclusion of a non free file is allowed, I'm not sure but I think that it meets all the criteria seeing that the building was demolished and if a suitable free photo cannot be located. Justiyaya 05:48, 2 January 2025 (UTC)
- Hi Justiyaya, thank you so much for your time! Truthfully I'm uncertain about using a photograph of where the restaurant once stood because it would be irrelevant to the content of the article (or at least for the intended use within the infobox). I am more asking about if I am able to use a non free file, yes, is there a venue where I can have a conversation with individuals who are more familiar with the matter? I'm not entirely sure if there is a relevant noticeboard/talk page/etc where I can bring this matter up with, if you or one of the editors here know one do let me know! Ornithoptera (talk) 05:52, 2 January 2025 (UTC)
- @Ornithoptera I think it would be a good use of an non-free file. Wikipedia:Media copyright questions would be a relevant noticeboard for asking these types of questions. Commons won't be the place to go because they don't deal with non-free stuff. Hope this helps and happy editing! :D Justiyaya 06:06, 2 January 2025 (UTC)
- Wonderful! Thank you so much for the assistance Justiyaya, didn't know there was a place like that! I'll go and send them a message hopefully within the week, I hope you have a wonderful day! Ornithoptera (talk) 07:03, 2 January 2025 (UTC)
- @Ornithoptera I think it would be a good use of an non-free file. Wikipedia:Media copyright questions would be a relevant noticeboard for asking these types of questions. Commons won't be the place to go because they don't deal with non-free stuff. Hope this helps and happy editing! :D Justiyaya 06:06, 2 January 2025 (UTC)
- Hi Justiyaya, thank you so much for your time! Truthfully I'm uncertain about using a photograph of where the restaurant once stood because it would be irrelevant to the content of the article (or at least for the intended use within the infobox). I am more asking about if I am able to use a non free file, yes, is there a venue where I can have a conversation with individuals who are more familiar with the matter? I'm not entirely sure if there is a relevant noticeboard/talk page/etc where I can bring this matter up with, if you or one of the editors here know one do let me know! Ornithoptera (talk) 05:52, 2 January 2025 (UTC)
Why am I blocked from doing stuff on Russian Wikipedia?
According to my global account information I've been indefinitely blocked from doing anything other than viewing the Russian Wikipedia because I'm "using multiple accounts or having others promote me (so what I say is seen more correct)", but I'm not? I don't know anyone else with a Wikipedia account and as far as I know I only have one. It's just on the Russian Wikipedia, none of the others. I haven't really done anything on the Russian Wikipedia anyways. I've only edited my own user page. Why did this happen and how do I fix it?
More information: The exact stated reason was обход блокировки (bypass blocking) and it linked here. It also gave me a link to here. Красный Октябрь (talk) 04:25, 2 January 2025 (UTC)
- Hi @Красный Октябрь, welcome to the teahouse. English and Russian Wikipedia are moderated differently, so a block there should not affect editing here nor could the community here fix anything there. From what I've been able to see with google translate and all, it appears that you've been checkuser blocked by @Q-bit array without email or talk page access. The administrator appears to monitor cross wiki notifications so hopefully they would respond here. They also appear to monitor meta:User talk:Q-bit array. Usually appeals without talk page access go through WP:UTRS but I can't seem to find a similar system in ru-wiki. Justiyaya 05:34, 2 January 2025 (UTC)
- Красный Октябрь, the English Wikipedia and the Russian Wikipedia are entirely separate projects. Each has its own policies and guidelines, and its own separate teams of administrators. Here at the Teahouse, we can do our best to help you with problems you may have on the English Wikipedia. But we have no knowledge, power nor influence over the Russian Wikipedia, which is an autonomous project. Cullen328 (talk) 08:38, 2 January 2025 (UTC)
- Do you know where I can go on the Russian Wikipedia to get it fixed? Красный Октябрь (talk) 14:22, 2 January 2025 (UTC)
- Hello again, @Красный Октябрь I think the relevant section is ru:ВП:РАЗБЛОК. ColinFine (talk) 15:24, 2 January 2025 (UTC)
- I went through the whole section you linked and it gives me instructions to request an unblock but I can't because I can't access my talk page or edit my user page. I can't find other ways to contact the person who blocked me as well. Красный Октябрь (talk) 16:30, 2 January 2025 (UTC)
- If there is no off-wiki means of contacting Russian Wikipedia administrators, I think you will need to contact a steward(https://meta.wikimedia.org/wiki/Stewards) who may at least be able to advise you better than we can. 331dot (talk) 16:42, 2 January 2025 (UTC)
- Thank you! Красный Октябрь (talk) 17:12, 2 January 2025 (UTC)
- I messaged one of the stewards here. Thank you for helping me find where to ask! Красный Октябрь (talk) 17:44, 2 January 2025 (UTC)
- Thank you! Красный Октябрь (talk) 17:12, 2 January 2025 (UTC)
- If there is no off-wiki means of contacting Russian Wikipedia administrators, I think you will need to contact a steward(https://meta.wikimedia.org/wiki/Stewards) who may at least be able to advise you better than we can. 331dot (talk) 16:42, 2 January 2025 (UTC)
- I went through the whole section you linked and it gives me instructions to request an unblock but I can't because I can't access my talk page or edit my user page. I can't find other ways to contact the person who blocked me as well. Красный Октябрь (talk) 16:30, 2 January 2025 (UTC)
- Hello again, @Красный Октябрь I think the relevant section is ru:ВП:РАЗБЛОК. ColinFine (talk) 15:24, 2 January 2025 (UTC)
- Responding to the "why did this happen" aspect, the most likely explanation is that your IP at the time you edited your userpage is similar to or within a range used by someone evading blocks on ru.wiki around the same time. Either that, or a more random human error on the blocking admin's part. signed, Rosguill talk 16:50, 2 January 2025 (UTC)
Notice of geological updates
Hi, in the past few months, I was working on pages regarding the periods, ages and other units of the geological timescale, and noticed a recent update in the ICC Chronostratigraphic Chart (2024/12), compared to the previous version (2023/09), that affects the time boundaries of a lot of ages. I didn't know which talk page or portal I could specifically notify of this change, so I came here to speak about it.
Now, I don't want to linger on a change like this here, since that feels like advertising which I KNOW Wikipedia is not about (see WP:SOAPBOX). But I know a change like this is going to require a lot of edits to catch up with, and I for sure can't do it alone. So does anyone know a good project/portal page I can discuss this more on? — Alex26337 (talk) 05:05, 2 January 2025 (UTC)
- Hi Alex26337, thank you for your question. Wikipedia talk:WikiProject Geology would be the best venue to discuss this. I don't think this is soapbox like if you are advertising on-Wiki collaboration. The policy refers to more so advertising on Wiki things off-wiki. Discussions with other community members that share the same interest is what these Wikiprojects are for :D Justiyaya 05:58, 2 January 2025 (UTC)
Request to Update Image
I noticed that the current image used at Dadvan Yousuf is from 2021, but there are more recent images from 2024 available on Wikimedia Commons.
I kindly request an extended-confirmed user to review and consider updating the image to "Dadvan Yousuf with Kurdistan Flag.jpg", Image is from recent - i dont recommend the images with the bitcoin flag.
Thanks.. Ayohama (talk) 07:44, 2 January 2025 (UTC)
- Hi Ayohama, welcome to the teahouse! Although the image File:Dadvan Yousuf with Kurdistan Flag.jpg is more recent, I don't think it provides the same quality and clarity as the image currently in the article. I don't think this change should be made. Justiyaya 07:53, 2 January 2025 (UTC)
- Hey, thanks for the solid feedback! fully understood Ayohama (talk) 07:54, 2 January 2025 (UTC)
- @Ayohama: I'd hold off in trying to use File:Dadvan Yousuf with Kurdistan Flag.jpg or any of the other images of Yousuf uploaded to Commons by Lonely34 until the claims of "own work" can be verified by c:COM:VRT. All the images that were uploaded by Lonely34 can be found being used online on sites like X, Facebook, Instagram, etc, from May 2024 onward prior to their being uploaded to Commons, some even with watermarks. This doesn't necessarily mean they weren't taken by Lonely34, but the general licensing of those sites is too restrictive for Commons per c:COM:L and in such cases copyright holder consent needs to be verified. If the licensing of any or all of those images ends up being verified, you can add them to the body of the Wikipedia article about Yousuf if you want but probably should propose changing the infobox image on the article's talk page to see what others think. -- Marchjuly (talk) 08:30, 2 January 2025 (UTC)
- Ayohama, even if the issues that Marchjuly describes can be ironed out, I would oppose the use of the image that you are proposing for the infobox. That is a mountaineering summit style photo, and this person is not a notable mountaineer even if he paid a lot of money to climb the standard tourist route on Mount Everest with the help of Sherpa guides. He is a cryptocurrency investor not a serious mountaineer and it is not appropriate to portray business people with golf clubs or tennis rackets in their hands. Or flags on mountain tops. This is not 1952 and rich people climbing Everest is not a notable accomplishment these days. Cullen328 (talk) 08:56, 2 January 2025 (UTC)
- One of the references used to verify the claim about him climbing Everest says (in English translation)
I had never climbed a mountain before, Everest was my first
. This is not indicative of a serious mountaineer but rather a rich person willing and able to pay actual Sherpa mountaineers to help him get to the top for publicity purposes. Cullen328 (talk) 09:07, 2 January 2025 (UTC)- Although it doesnt seem to be that easy for me to climb everest as it seems for you. I think its still very dangerous and a notable achievement Ayohama (talk) 09:21, 2 January 2025 (UTC)
- Agree with you Ayohama (talk) 09:20, 2 January 2025 (UTC)
- One of the references used to verify the claim about him climbing Everest says (in English translation)
- Thanks for the hint, i thought the Metadata is correct Ayohama (talk) 09:27, 2 January 2025 (UTC)
- @Ayohama: The uploader or someone else has emailed Commons VRT regarding these files and is currently being processed by VRT. Assuming things check out, the c:Template:Permission received added to each file's page will be changed to c:Template:PermissionTicket by the VRT member reviewing the email. Once this happens, the files will be fine from a copyright standpoint; so, in principle, they're OK to be used. From an encyclopedic standpoint, though, things might still need to be sorted out on the article's talk page, particularly if others feel (like expressed above) that they shouldn't be used in the main infobox. One representative image in the body of the article should, however, be OK. -- Marchjuly (talk) 10:07, 2 January 2025 (UTC)
- Was thinking after our conversations, the infobox is not the right place there. MAYBE at Personal life, where context is about Everest is fine. Max Ayohama (talk) 10:10, 2 January 2025 (UTC)
- According to List of Mount Everest summiters by frequency, more than 6,000 people have summited Everest (as of 2022), so agree that if any placement at all, then at Personal life. David notMD (talk) 11:14, 2 January 2025 (UTC)
- And the record for a non-Sherpa (actually an Englishman) is 18 (exceeded by 9 individual Sherpas). {The poster formerly known as 87.81.230.195} 94.6.84.253 (talk) 05:40, 3 January 2025 (UTC)
- Lonely34 claims of "own work" are now verified by c:COM:VRT. If you want, you can go ahead and place the File:Dadvan Yousuf with Kurdistan Flag.jpg under "Personal Life". I guess the article itself is interesting to work on in the future- the tone is sometimes too promotional and the text layout weird- Reads like a chronology rather then a WP Article. Ayohama (talk) 13:39, 2 January 2025 (UTC)
- According to List of Mount Everest summiters by frequency, more than 6,000 people have summited Everest (as of 2022), so agree that if any placement at all, then at Personal life. David notMD (talk) 11:14, 2 January 2025 (UTC)
- Was thinking after our conversations, the infobox is not the right place there. MAYBE at Personal life, where context is about Everest is fine. Max Ayohama (talk) 10:10, 2 January 2025 (UTC)
- @Ayohama: The uploader or someone else has emailed Commons VRT regarding these files and is currently being processed by VRT. Assuming things check out, the c:Template:Permission received added to each file's page will be changed to c:Template:PermissionTicket by the VRT member reviewing the email. Once this happens, the files will be fine from a copyright standpoint; so, in principle, they're OK to be used. From an encyclopedic standpoint, though, things might still need to be sorted out on the article's talk page, particularly if others feel (like expressed above) that they shouldn't be used in the main infobox. One representative image in the body of the article should, however, be OK. -- Marchjuly (talk) 10:07, 2 January 2025 (UTC)
- Ayohama, even if the issues that Marchjuly describes can be ironed out, I would oppose the use of the image that you are proposing for the infobox. That is a mountaineering summit style photo, and this person is not a notable mountaineer even if he paid a lot of money to climb the standard tourist route on Mount Everest with the help of Sherpa guides. He is a cryptocurrency investor not a serious mountaineer and it is not appropriate to portray business people with golf clubs or tennis rackets in their hands. Or flags on mountain tops. This is not 1952 and rich people climbing Everest is not a notable accomplishment these days. Cullen328 (talk) 08:56, 2 January 2025 (UTC)
- @Ayohama: I'd hold off in trying to use File:Dadvan Yousuf with Kurdistan Flag.jpg or any of the other images of Yousuf uploaded to Commons by Lonely34 until the claims of "own work" can be verified by c:COM:VRT. All the images that were uploaded by Lonely34 can be found being used online on sites like X, Facebook, Instagram, etc, from May 2024 onward prior to their being uploaded to Commons, some even with watermarks. This doesn't necessarily mean they weren't taken by Lonely34, but the general licensing of those sites is too restrictive for Commons per c:COM:L and in such cases copyright holder consent needs to be verified. If the licensing of any or all of those images ends up being verified, you can add them to the body of the Wikipedia article about Yousuf if you want but probably should propose changing the infobox image on the article's talk page to see what others think. -- Marchjuly (talk) 08:30, 2 January 2025 (UTC)
- Hey, thanks for the solid feedback! fully understood Ayohama (talk) 07:54, 2 January 2025 (UTC)
Help with improving references and notability for Eugene Vollmer draft
Hello! I submitted a draft for Eugene Vollmer but it was declined because the references didn't show significant coverage. Could anyone assist me in finding reliable sources or improving my draft to meet Wikipedia's notability guidelines? I appreciate any guidance! Sammy Tremlinn (talk) 08:45, 2 January 2025 (UTC)
- Sammy Tremlinn, your draft fails to make the case that Vollmer is a truly notable athlete, although it seems that he was once highly ranked in an amateur track and field event in a country of less than a million residents. At first glance, that does not seem to be a plausible claim of notability to me. But I could be wrong. In the end, it is all about whether or not multiple indisputably reliable sources independent of Vollmer have devoted significant coverage to him. Your current sources are not independent of Vollmer and therefore do not meet that threshold. Cullen328 (talk) 09:25, 2 January 2025 (UTC)
- Thank you for the feedback, Cullen328. I understand the importance of independent and significant coverage for establishing notability. I will work on finding additional sources that meet these criteria and provide substantial information about Eugene Vollmer's achievements. If you have any suggestions for where I might look or types of sources to prioritize, I’d greatly appreciate it! Sammy Tremlinn (talk) 10:01, 2 January 2025 (UTC)
- List of newspapers in Fiji might be a starting point: many of those entities are likely to be Reliable sources. {The poster formerly known as 87.81.230.195} 94.6.84.253 (talk) 05:48, 3 January 2025 (UTC)
- Thank you for the feedback, Cullen328. I understand the importance of independent and significant coverage for establishing notability. I will work on finding additional sources that meet these criteria and provide substantial information about Eugene Vollmer's achievements. If you have any suggestions for where I might look or types of sources to prioritize, I’d greatly appreciate it! Sammy Tremlinn (talk) 10:01, 2 January 2025 (UTC)
New page
Hello, I was thinking of making a new page about the New Zealand retail company PB Technologies, but I think I'll be going in too far over my head if I start it myself, so I was wondering where I could propose the Idea so that I could join forces with a more experienced Wikipedian to make the page a reality. Sup3rG33k08 (talk) 08:51, 2 January 2025 (UTC)
- Hello. You can go to Requested articles, but it is backlogged to the point of uselessness, any request you make there will likely not be acted on for some time, if ever. You are wise to be cautious in attempting to create a new article, but if you want to see one created, it likely won't happen unless you do it yourself. You can use the Article Wizard to create and submit a draft for review by more experienced editors. You would first need to gather independent reliable sources to summarize in an article, and review the definition of a notable business to see if this business meets it(most do not).
- If you are associated with this business, that needs to be disclosed, see WP:COI and WP:PAID. 331dot (talk) 10:05, 2 January 2025 (UTC)
- Articles about companies need to comply with WP:NCORP and call for attention to being neutral point of view, not in any way promotional. Teahouse Hosts are here to advise, not to be coauthors. David notMD (talk) 12:09, 2 January 2025 (UTC)
Adding unnumbered tracks to an upcoming album
Hello,
This artist has been teasing some track titles (and snippets) from their upcoming album, So Close To What. Some titles are speculated, some were directly teased and are confirmed. How would I go about adding those tracks, assuming I don't know their numbers? The album has a Spotify countdown page, so the two pre-released singles have already been added in track list format. Lashyurn (talk) 10:39, 2 January 2025 (UTC)
- Lashyurn, I read that this album "scheduled to be released on February 21, 2025". I suggest that you simply wait until February 21, 2025. -- Hoary (talk) 12:24, 2 January 2025 (UTC)
- Oh, that's fine. I was just following what I saw was done with SZA's LANA 10 days prior to its release. Thank you! :) Lashyurn (talk) 12:32, 2 January 2025 (UTC)
- Hi Lashyurn, in that article, there were reliable sources that reported on the teasers for the singles. If that is the case for this album, I think a mention in the Promotion section could be nice. Things that are not being reported on by reliable, independent sources or are pure speculation are usually not included here, especially if they are self published or original research. Feel free to drop sources here for us to take a look :D Justiyaya 12:45, 2 January 2025 (UTC)
- These are my sources (and titles):
- "Dear god" - Played at listening party and posted snippets on TikTok
- "Revolving door" - Played at listening party and posted snippets on TikTok
- "Moments" - Title posted on her management's Instagram as a collaboration with Spotify on December 13, 2024
- "Greenlight" - Played at listening party and title shown in album trailer
- "Star child" - Title shown at the beginning of the "It's ok I'm ok" music video
- Would these sources be considered reliable? I have links/pictures/videos to each of those Lashyurn (talk) 13:03, 2 January 2025 (UTC)
- Unfortunately none of those are secondary sources, @Lashyurn. What we're looking for is mainstream music journalists who are reporting on the album. qcne (talk) 17:35, 2 January 2025 (UTC)
- Oh, I thought I could take them from the artist themselves. Thank you! I'll wait until release. Lashyurn (talk) 18:00, 2 January 2025 (UTC)
- No, sources from the person that the article is about are considered primary sources. Cowboygilbert - (talk) ♥ 22:02, 2 January 2025 (UTC)
- Oh, I thought I could take them from the artist themselves. Thank you! I'll wait until release. Lashyurn (talk) 18:00, 2 January 2025 (UTC)
- Unfortunately none of those are secondary sources, @Lashyurn. What we're looking for is mainstream music journalists who are reporting on the album. qcne (talk) 17:35, 2 January 2025 (UTC)
- Hi Lashyurn, in that article, there were reliable sources that reported on the teasers for the singles. If that is the case for this album, I think a mention in the Promotion section could be nice. Things that are not being reported on by reliable, independent sources or are pure speculation are usually not included here, especially if they are self published or original research. Feel free to drop sources here for us to take a look :D Justiyaya 12:45, 2 January 2025 (UTC)
- Oh, that's fine. I was just following what I saw was done with SZA's LANA 10 days prior to its release. Thank you! :) Lashyurn (talk) 12:32, 2 January 2025 (UTC)
- I suggest to wait till release too Ayohama (talk) 13:33, 2 January 2025 (UTC)
English Version of the German Article Judith Kakon
Hello there, I just want to create the same article in english, I already created this article in german, all I want is to have a translated version. Could you please help me? My request was already denied twice. This is the german article: Judith Kakon. Avaa Malula (talk) 14:25, 2 January 2025 (UTC)
- @Avaa Malula It appears as if German Wikipedia has the same concerns as English Wikipedia; that the article does not sufficiently prove its subject's notability. Please read WP:YFA and WP:42. CommissarDoggoTalk? 14:30, 2 January 2025 (UTC)
- Draft:Judith Kakon has no references. The content that is now under a description of her approach to art (my wording) comes across as either your or her description. All of this should be deleted unless it can be referenced to published descriptions of her and her work. David notMD (talk) 14:50, 2 January 2025 (UTC)
- Hey @Avaa Malula
- It’s great that you’re interested in contributing to the English Wikipedia by translating an article. However, there are a few things to consider to ensure the article meets the standards required on the English Wikipedia:
- 1. Improving the Original Article: The German article you created currently has notices indicating issues with missing citations and important information about the subject’s works. These issues suggest that the article may not yet meet Wikipedia's notability and verifiability standards. Before translating the article, it’s a good idea to improve it by:
- - Adding reliable and independent sources that demonstrate Judith Kakon's notability.
- - Including details about her important works or achievements.
- 2. Meeting English Wikipedia Standards: The English Wikipedia has specific requirements for biographies. It’s important to:
- - Demonstrate notability according to the General Notability Guideline (WP:GNG).
- - Provide inline citations from reliable sources to back up claims.
- - Write in a neutral tone without promotional language.
- 3. Translation Guidelines: When translating articles, you must include a note on the article’s talk page to credit the original German version. See the Wikipedia:Translation guidelines for details on how to do this properly.
- 4. Steps to Resubmit: Once you’ve improved the content and addressed the above points:
- - Draft the translated article in your sandbox.
- - Add sources that meet English Wikipedia’s standards.
- - Submit it for review through the Articles for Creation (AfC) process.
- If you need help improving the article or drafting it in English, feel free to ask here. Ayohama (talk) 07:53, 3 January 2025 (UTC)
Last aired date in the future
This article revison SAS: Rogue Heroes revison of 02:04, 2 January 2025 has a last aired date for series 2 which is in the future. What is probably meant is that the last episode is scheduled for broadcast on that date, but as it was worded it is factually incorrect. I've seen this on other such articles in the past. Has there been any discussions about wether this is acceptible or not? If so, can you direct me to any official decisions? Dubidub (talk) 14:49, 2 January 2025 (UTC)
- See WP:FUTURE - Specifically, see the section NOTCRYSTALBALL, which states that Wikipedia is not a crystal ball. Articles should not speculate on future events unless they are verifiable and sourced. The correct phrasing should reflect that the episode is "scheduled to air" on that date to maintain accuracy and neutrality. Ayohama (talk) 07:45, 3 January 2025 (UTC)
How do I clean up formatting of tables?
On the Columbia University article, the Rankings section has quite poor formatting; the ranking tables are being displayed fully below the Library and Barnard images on the right side. How would I go about cleaning up table formatting in wikitext to remove the large blank gap? Doawk7 (talk) 21:13, 2 January 2025 (UTC)
- I solved it by removing those images:) DMacks (talk) 07:38, 3 January 2025 (UTC)
How to ask ArbCom to lower protection status on Armenia-Azerbaijan
I was going to submit an ArbCom request to lower protection of Armenia-Azerbaijan due to the conflict cooling and amends being made, however I changed my mind and decided to ask here first. How would I get the Arbitration Committee or whoever imposed the sanction on Ar-Az to hear on the matter? SimpleSubCubicGraph (talk) 00:21, 3 January 2025 (UTC)
- That's not an ArbCom matter; you can ask for that at Wikipedia:Requests for page protection, SimpleSubCubicGraph. Are you asking about Nagorno-Karabakh conflict? You can edit that--after you've made a few more article edits. Drmies (talk) 01:34, 3 January 2025 (UTC)
- @Drmies I mean the consensus to make everything Armenia and Azerbaijan, even if its a tragic plane crash extended confirmed protected. I feel like it should be knocked down due to calming geopolitical tensions in the Caucasus. SimpleSubCubicGraph (talk) 03:04, 3 January 2025 (UTC)
- Also I have 251 edits, that is nowhere near 500. SimpleSubCubicGraph (talk) 03:04, 3 January 2025 (UTC)
- In that case, SimpleSubCubicGraph, you're talking about Wikipedia:Requests for arbitration/Armenia-Azerbaijan 2 and you'd have to appeal that. I just looked over the page and I have no idea how to do that--and I used to be on ArbCom. Read over Wikipedia:Contentious topics, and good luck! Drmies (talk) 17:20, 3 January 2025 (UTC)
- Also I have 251 edits, that is nowhere near 500. SimpleSubCubicGraph (talk) 03:04, 3 January 2025 (UTC)
- @Drmies I mean the consensus to make everything Armenia and Azerbaijan, even if its a tragic plane crash extended confirmed protected. I feel like it should be knocked down due to calming geopolitical tensions in the Caucasus. SimpleSubCubicGraph (talk) 03:04, 3 January 2025 (UTC)
Referencing an article on Wikipedia
I am reviewing an article and came across a source that cited another article from Wikipedia. This is the first time I have seen this. This isn’t acceptable is it? BigChrisKenney (talk) 00:35, 3 January 2025 (UTC)
- Hello @BigChrisKenney. You are correct. Sources that lead to Wikipedia or any Wikimedia project are unreliable. See WP:CIRCULAR. Tarlby (t) (c) 01:33, 3 January 2025 (UTC)
- Thank you @Tarlby for the speedy reply and the direction to the exact policy! BigChrisKenney (talk) 01:43, 3 January 2025 (UTC)
- You can jsut delete the ref to a Wikipedia article, but a better practice would be to look within that Wikipedia article to find a ref (refs) that support the content, and then copy those refs to the article you have concerns about. If you do this, in your Edit summary consider stating that you copied content from - naming the source article. Wikipedia does allow copying content and refs from Wikipedia articles with attribution. David notMD (talk) 14:17, 3 January 2025 (UTC)
- Thank you @Tarlby for the speedy reply and the direction to the exact policy! BigChrisKenney (talk) 01:43, 3 January 2025 (UTC)
Writer or Screenwriter?
Quick question: creating a stub-starter on Donald Ross: [5] who was a television screenwriter, celebrity game show panelist (with his wife Patti Deutsch) and producer. Since there are other Donald Ross articles, would you accompanying his name with (writer) or (screenwriter) for the published article? He has no distinguishing middle name / initial. Deutsch's article refers to him as "comedy screenwriter and playwright", but Ross never wrote any plays. TIA. Maineartists (talk) 02:16, 3 January 2025 (UTC)
- Maineartists, "Donald Ross (writer)" seems OK to me. -- Hoary (talk) 03:50, 3 January 2025 (UTC)
- (writer) seems OK to me too Ayohama (talk) 07:23, 3 January 2025 (UTC)
Great. Thanks all. Maineartists (talk) 13:47, 3 January 2025 (UTC)
What is the opinion on wikipedia writing services?
How would I detect if an article has been secretly written and paid for? There are services that you can reach just by searching on google and I would like to know if there are any ways to detect them or do something about them. SimpleSubCubicGraph (talk) 04:32, 3 January 2025 (UTC)
- Hello! If there is an editor who might be paid and performing edits, raise the issue at WP:ANI or WP:COIN noticeboards. If there are any private details you have found about the editor or services, please send an email to paid-en-wp@wikipedia.org . TNM101 (chat) 05:24, 3 January 2025 (UTC)
- Hi, SSCG, did you have any specific articles in mind? I would say that there are heuristics I know of, but there's nothing definitive overall. Heustics I've identified for undisclosed paid/COI include (please do not use any of these to make definitive accusations; these observations are potential starting points, not endpoints):
- The article has an image of an otherwise obscure person which is usually marked as "own work". This image is posed, not candid, indicating that either the subject themself uploaded it or they've given it to somebody else to use as effectively a PR photo on Wikipedia. Some subjects presumably will enthusiastically freely license their image even when they have no part in an article because they're just jazzed to have their own article, but what can be really suspect is when the uploader of the image is the same editor who created and/or has heavily edited the article itself.
- The article was already very long, well-formatted, and essentially complete right out of the gate. For new editors, sometimes articles after going through the draft process can come out looking alright, but these edits skip the draft process altogether yet still seem like they've been written out ahead of time off-site. Whereas I take an incremental approach to my writing, experienced editors very often will pre-write everything out in their sandbox and then transfer their work into the article space once they're happy with it, but in my experience, this is highly uncommon for newer editors who organically create articles about subjects they're interested in.
- The article was created or heavily modified by a WP:SPA, which seemingly has not interacted elsewhere on Wikipedia.
- The editor tries to wikilink this article absolutely everywhere they possibly can once it's created, presumably for a mix of SEO and for drawing as many eyes to that subject as possible on-wiki.
- The article is mostly primary references, and a lot of it is WP:PUFFERY. For companies, a dead giveaway for something like this is when there's an entire section on some stupid PR BS the company did which is only cited to press releases from that company about what a great thing(TM) they did. Basically, consider WP:DUE.
- The article goes into excruciating depth about a subject (sometimes this is just fans being fans, but for example, there was one some years ago about a Lithuanian(?) alt-rock band whose primary editor I thought to be just an overly enthusiastic fan but who I discovered was secretly the band's manager or something to that effect).
- The 'External links' section contains an inordinate amount of links to social media (this one to me generally indicates that it's autobiographical, whereas I assume you specifically want contracted third parties).
- The editor has a username-violating policy which indicates they're part of the subject organization.
- The subject likely doesn't meet the WP:GNG but has been WP:REFBOMBed to give the appearance that it does.
- The subject is a very niche a) product/service/work, b) company/organization, or c) person. Depending on the nature of the organization (e.g. a club), this may not be "paid" as much as it is WP:COI.
- The subject is not written about in a neutral way. If there is negative coverage of the subject in reliable sources, it is strangely absent.
- If you strongly suspect that an editor is engaging in undisclosed paid editing, the boards TNM101 suggests are great resources. I would caution too however that if your suspicion isn't that strong, you can start by just asking them at their talk page. Sometimes they're earnestly trying to improve the encyclopedia about something they're very familiar with because they're a part of it, but they've failed out of ignorance of forgetfulness to disclose that on their user page. TheTechnician27 (Talk page) 06:03, 3 January 2025 (UTC)
- @TheTechnician27@TNM101 I wasn't talking about COIs or paid editing, I already know and have remembered both WP pages. I was just wondering if Wikipedia could take action against those services aka take them to court. SimpleSubCubicGraph (talk) 06:59, 3 January 2025 (UTC)
- That doesn't seem to be within our scope, and I am pretty sure that the Wikimedia Foundation doesn't take legal action against anyone since it's a non-profit. TNM101 (chat) 07:12, 3 January 2025 (UTC)
- I can't think of a single viable avenue by which Wikipedia could take successful legal action against undisclosed paid editors. If you tried to bring a civil suit, you'd be trying to prove what's called a tort. The WMF would have to prove that 1) the editor had a legal duty to act in a certain way (they didn't; nothing legally says you have to identify an employer when you contribute to a website); 2) that the defendent breached this duty (i.e. if you could somehow prove 1 (not possible), then you would still need to prove in court that your suspicions are more probable than not); and 3) that the WMF suffered injury or loss as a result of this undisclosed paid editing (short of some absolutely enormous conspiracy that blows up and damages the WMF's reputation, this is functionally impossible, because the reality is that any loss is negligible at best). TheTechnician27 (Talk page) 07:18, 3 January 2025 (UTC)
- @TheTechnician27@TNM101 I wasn't talking about COIs or paid editing, I already know and have remembered both WP pages. I was just wondering if Wikipedia could take action against those services aka take them to court. SimpleSubCubicGraph (talk) 06:59, 3 January 2025 (UTC)
Deep thanks to User:TheTechnician27 for putting all these undeclared paid flags in one place. If this is not already the subject of an essay, it should be. I do see that Wikipedia:Identifying PR is an existing essay flagging behavior that draws content from press releases. David notMD (talk) 11:45, 3 January 2025 (UTC)
- Thank you very much for all the responses, is there anyway I can see of all the times when paid editors or undisclosed COIs were caught in the act? SimpleSubCubicGraph (talk) 04:33, 4 January 2025 (UTC)
Improving a page through including my own research
The 'icing the kicker' page can benefit from the following recent addition in regards to the effectiveness of this practice:
"Most recently and extensively, Professor Nadav Goldschmied and Tyler Ratkovich from the University of San Diego and Mike Raphaeli from Nanning Vocational and Technical University (China) found that in 25 NFL seasons (1999-2024) the success for "pressure kicks" (two minutes or less to the end of the game when -3, −2, −1, tied or in overtime) when iced was 74.1% as opposed to 79.8% otherwise which represented a significant decline in performance.
Goldschmied, N., Ratkovich, T*., & Raphaeli, M. (2024) NFL field goal kicking under pressure: An expanded replication of icing the kicker strategy. Journal of Applied Sport Psychology.
Thanks,
Nadav Goldschmied
Ngoldschmied11 (talk) 05:42, 3 January 2025 (UTC)
- The place to suggest this, Ngoldschmied11, is Talk:Icing the kicker. (Better, I think: "A study found that in 25 NFL seasons..." Interested readers will identify the authors from the reference.) -- Hoary (talk) 05:48, 3 January 2025 (UTC)
- Fine by me! so do I add the modification or is it done by another editor since it is my work?
- Thanks,
- Nadav Ngoldschmied11 (talk) 06:15, 3 January 2025 (UTC)
- @Ngoldschmied11 pretty sure this falls under WP:No original research SimpleSubCubicGraph (talk) 07:00, 3 January 2025 (UTC)
- SimpleSubCubicGraph Not quite. If the observations on kicking success/failure were unpublished, that would be original research. If published in a reliable source, such as a science journal article, OK to use. However, Wikipedia advises researchers who are also Wikipedia editors to not cite their own work. David notMD (talk) 12:05, 3 January 2025 (UTC)
- Hello, @Ngoldschmied11: in case it's not clear from the other responses: you are discouraged from citing your own work, as that is regarded as a conflict of interest, but you are welcome to suggest an edit which cites your own work, so that another editor decides what to do with your suggestion. If you use the edit request wizard that will guide you. Please make your suggestion as precise as possible (eg "Inset XXX at the end of the paragraph beginning YYY") ColinFine (talk) 13:10, 3 January 2025 (UTC)
- @Ngoldschmied11 pretty sure this falls under WP:No original research SimpleSubCubicGraph (talk) 07:00, 3 January 2025 (UTC)
Shortened forms list
Hello. Is there any place I could get the shortened forms of all Wikipedia guidelines and a brief definition (e.g. WP:GNG, WP:NPV)? I usually find it difficult to use the shortened forms when commenting and I just go for the full name. Jõséhola 07:13, 3 January 2025 (UTC)
- Hello @Josedimaria. See WP:WP. Tarlby (t) (c) 07:20, 3 January 2025 (UTC)
- Hey, how about Wikipedia:Shortcut index or Wikipedia:List of policies and guidelines and might be an idea to work out something for Wikipedia:Glossary of shortcuts Ayohama (talk) 07:21, 3 January 2025 (UTC)
Creating a new page
I want to make a new Wikipedia page but the name already exists so what should I do Chetan4u1 (talk) 10:05, 3 January 2025 (UTC)
- Can you tell us the name of the article so we can better understand the situation? Hacked (Talk|Contribs) 10:10, 3 January 2025 (UTC)
- @Chetan4u1 Welcome to the Teahouse. You don't need to worry about that at this stage. You can create a draft article by following the process at WP:AfC. If and when it is accepted into the encyclopedia (most draft articles are not), the reviewer will attend to the naming, which might require a disambiguation page, but if you haven't written a Wikipedia article before you'll have plenty of other hoops to jump through before this becomes an issue. Shantavira|feed me 10:15, 3 January 2025 (UTC)
- @Chetan4u1 Your attempt to create an article about Isabel Myers who is an actress has been deleted and a redirect created to Isabel Briggs Myers, a different person. To start over, use WP:YFA to create a draft named Isabel Myers (actress). That said, common advice here is to gain experience improving existing articles before attempting to create an article. David notMD (talk) 12:18, 3 January 2025 (UTC)
- Thanks Brother but if I make a draft then how can I change the name of the page if it already exists. Chetan4u1 (talk) 12:33, 3 January 2025 (UTC)
- @Chetan4u1 If you're working on a draft on an individual who shares their name with someone who already has an existing article, you're supposed to differentiate/distinguish your draft from the existing article. You cannot 'delete' or 'change the name' of an article that has been reviewed and exists. In your case, name your draft Isabel Myers (actress), since the subject of the existing article, Isabel Briggs Myers, is a writer. Dissoxciate (talk) 13:19, 3 January 2025 (UTC)
- @Dissoxciate, I disagree with you. When creating a draft, you may disambiguate the title, but you don't need to, and you certainly aren't "supposed to". New editors probably don't know our disambiguation rules anyway, so I would not encourage them to try.
- For your second point, you can move an existing article to a new title, and that sometimes needs to happen when a new article is created about something with the same name. But I agree that if Chean4u1 is asking how to change the name of an existing article in order to hijack it, they shouldn't. ColinFine (talk) 13:58, 3 January 2025 (UTC)
- Hello, @Chetan4u1, and welcome to the Teahouse.
- The answer to your specific question is that you change the name of an article or other page by moving it; but I suggest you forget about this for the moment because 1) as a new user you don't have access to that tool yet, and 2) at your present stage it is irrelevant to what you want to do.
- My earnest advice to new editors is to not even think about trying to create an article until you have spent several weeks - at least - learning about how Wikipedia works by making improvements to existing articles. Once you have understood core policies such as verifiability, neutral point of view, reliable, independent sources, and notability, and experienced how we handle disagreements with other editors (the Bold, Revert, Discuss cycle), then you might be ready to read your first article carefully, and try creating a draft.
- When you are ready to create the draft, you can completely ignore the titles of any existing articles. You will be creating Draft:Isabel Myers (which doesn't exist at present, which is why it is in red). After you have developed an acceptable draft for an article (which is much harder than it looks to new editors, and will almost certainly give you frustration and disappointment if you don't take David's and my advice above), you will submit it for review, and if the reviewer accepts it, they will sort out all the issues about naming it. OK? ColinFine (talk) 13:20, 3 January 2025 (UTC)
- Thanks I will try my best. Chetan4u1 (talk) 14:17, 3 January 2025 (UTC)
- @Chetan4u1 If you're working on a draft on an individual who shares their name with someone who already has an existing article, you're supposed to differentiate/distinguish your draft from the existing article. You cannot 'delete' or 'change the name' of an article that has been reviewed and exists. In your case, name your draft Isabel Myers (actress), since the subject of the existing article, Isabel Briggs Myers, is a writer. Dissoxciate (talk) 13:19, 3 January 2025 (UTC)
- Thanks Brother but if I make a draft then how can I change the name of the page if it already exists. Chetan4u1 (talk) 12:33, 3 January 2025 (UTC)
- @Chetan4u1 Your attempt to create an article about Isabel Myers who is an actress has been deleted and a redirect created to Isabel Briggs Myers, a different person. To start over, use WP:YFA to create a draft named Isabel Myers (actress). That said, common advice here is to gain experience improving existing articles before attempting to create an article. David notMD (talk) 12:18, 3 January 2025 (UTC)
adding a clarification in response to a user request
Hello, in doing what described in subject, should one delete the request for clarification, or leave it there? 1808Eolo3 (talk) 10:14, 3 January 2025 (UTC)
- @1808Eolo3 Welcome to the Teahouse. If you mean resolving a [how?] template (with adequate referencing) then you may remove the template. Shantavira|feed me 10:36, 3 January 2025 (UTC)
What is sandbox and how should I use it?
How can I use sandbox Juamisi25 (talk) 10:54, 3 January 2025 (UTC)
- @Juamisi25 Welcome to the Teahouse! All should be explained at this help page: basically your personal sandbox is a place for you to experiment with editing without messing anything up in the main encyclopedia. Mike Turnbull (talk) 10:58, 3 January 2025 (UTC)
- Short and simple: follow this link — you can do whatever with it as long as it's productive Wikipedia Stuff, use it to practice edits, put together rough drafts etcetera --Slowking Man (talk) 01:15, 4 January 2025 (UTC)
Error with interlanguage link template
So while editing Guangdong Public Security Department, I added an interlanguage link in the infobox, and for whatever reason in the infobox right after the interlanguage link are two brackets(]]). I entered the interlanguage link properly from both experience and directions, may I ask what I did wrong and how to solve this issue? Thehistorianisaac (talk) 11:18, 3 January 2025 (UTC)
- Hi Thehistorianisaac. I think this might have something to do with the syntax of Template:Infobox law enforcement agency. If you look at the syntax for the
|Secondary governing body=
parameter, it appears things are set up to convert whatever is added for that parameter into a Wikilink. If you remove the{{ill}}
template for that parameter, it will format it as a WP:REDLINK. You could try asking about this on the template's talk page or at WP:VPT; however, I don't think you can remove the brackets without removing them from the infobox template's syntax, and doing that would affect other articles where the template is being transcluded. You might be just have to hide the parameter for the time being until a fix can be found or just go with the red link. -- Marchjuly (talk) 12:09, 3 January 2025 (UTC)- Ok thank you Thehistorianisaac (talk) 12:14, 3 January 2025 (UTC)
- @Thehistorianisaac:. The documentation for the parameter at Template:Infobox law enforcement agency says "Do not link. The template will link." I looked at the implementation and it doesn't appear possible to add an interlanguage link there. If the link is important then you can add it to the opening paragraph instead. PrimeHunter (talk) 12:19, 3 January 2025 (UTC)
- thanks Thehistorianisaac (talk) 12:20, 3 January 2025 (UTC)
- @Thehistorianisaac: If you really want to include the interlanguage link, you might replace the linked "provincial government" in the second sentence of the article with the ILL you were using in the infobox. Deor (talk) 23:47, 3 January 2025 (UTC)
- thanks Thehistorianisaac (talk) 12:20, 3 January 2025 (UTC)
- @Thehistorianisaac:. The documentation for the parameter at Template:Infobox law enforcement agency says "Do not link. The template will link." I looked at the implementation and it doesn't appear possible to add an interlanguage link there. If the link is important then you can add it to the opening paragraph instead. PrimeHunter (talk) 12:19, 3 January 2025 (UTC)
- Ok thank you Thehistorianisaac (talk) 12:14, 3 January 2025 (UTC)
Can someone help me with archiving bot?
You can edit my talk page Cactus🌵 spiky ouch 12:27, 3 January 2025 (UTC)
- @Cactusisme: Done. I have set up auto-archiving on your talk page. Lowercase Sigmabot III will now archive threads that are 7 days old, keeping at least 5 threads and archiving at least 1. If you want to customise it further, you can check out Help:Archiving a talk page#Example with sequentially numbered archives or just ping me for assistance. – DreamRimmer (talk) 14:41, 3 January 2025 (UTC)
- Thanks!!! Cactus🌵 spiky ouch 01:21, 4 January 2025 (UTC)
Images
Is there a way one can add copyrighted images/images not in the public domain to articles, if necessary? Maybe something to do with fair use? Can someone walk me through the process, given there's one? Thanks! Dissoxciate (talk) 12:34, 3 January 2025 (UTC)
- Hi @Dissoxciate: if you haven't yet read WP:Non-free content, that seems a good place to start. -- DoubleGrazing (talk) 13:38, 3 January 2025 (UTC)
- Hello! See the instructions at Special:Upload if you want to upload the file after reading WP:Non-free content as said by DoubleGrazing TNM101 (chat) 13:41, 3 January 2025 (UTC)
Is there another method besides WP:RSP to check if a source is reliable?
Are there other ways to check if a source is reliable besides using WP:RSP? For example, are there guidelines, tools, or methods that can help evaluate a source's trustworthiness when RSP doesn’t give a clear answer? Silkroadster (talk) 13:33, 3 January 2025 (UTC)
- @Silkroadster: You could read WP:RS yourself to determine if a source meets its criteria, or ask on RSN. JJPMaster (she/they) 13:34, 3 January 2025 (UTC)
- There is a userscript at User:Novem Linguae/Scripts/CiteHighlighter that takes ratings from various sources and highlights them in green, yellow or red depending on their reliability. Also, User:Novem Linguae/Scripts/CiteHighlighter#Original source lists is very helpful. – DreamRimmer (talk) 13:48, 3 January 2025 (UTC)
- @DreamRimmer Thank you. I realized that early on in my editing, but I appreciate it again as I used it as an argument in an AFD. Silkroadster (talk) 14:21, 3 January 2025 (UTC)
- While looking at the Wikipedia:WikiProject Film/Indian cinema task force#Guidelines on sources. I found Hindu Businessline to be a reliable source. Does this reliability apply to news related to companies as well? Silkroadster (talk) 14:24, 3 January 2025 (UTC)
- @DreamRimmer Thank you. I realized that early on in my editing, but I appreciate it again as I used it as an argument in an AFD. Silkroadster (talk) 14:21, 3 January 2025 (UTC)
- @Silkroadster Sometimes searching the WP:RSN archives is helpful. Gråbergs Gråa Sång (talk) 16:39, 3 January 2025 (UTC)
- Hi @Silkroadster,
- When WP:RSP isn’t clear, assess the publisher’s reputation, the author’s expertise, editorial standards, and independence. Check for corroboration in other reliable sources and be cautious of bias or promotional tones. Ayohama (talk) 17:21, 3 January 2025 (UTC)
Company Information
I'm needing assistance to add my small business to the Wikipedia results. I started a page, but was presented with a message that stated my content as 'non-constructive'. What do I need to do or where can I find guidelines on how to add our information ?? Rdmind (talk) 15:35, 3 January 2025 (UTC)
- Rdmind Hello and welcome. You have no edits other than this one, so I can't look at your prior attempt. You seem to have a common misunderstanding about Wikipedia. It is not a database of things that exist, and is also not a place for companies to tell about themselves and what they do. Wikipedia articles about companies must summarize what independent reliable sources with significant coverage have chosen on their own to say about the company, showing how it meets our special Wikipedia definition of a notable company. The vast majority of the millions of companies on this planet do not merit Wikipedia articles(even some that may have them, but we just haven't removed yet). Also note that a Wikipedia article is not necessarily desirable. There are good reasons to not want one. Our articles are typically written by independent editors wholly unconnected with the topic. I would suggest that you go on about the work of your company; if your company is truly notable, someone will eventually write about it. 331dot (talk) 15:41, 3 January 2025 (UTC)
- @331dot: see EF. DMacks (talk) 16:45, 3 January 2025 (UTC)
- Ah, I see now- I didn't realize the filter blocked things as "non constructive". I now see that their username violates policy. 331dot (talk) 16:48, 3 January 2025 (UTC)
- @331dot: see EF. DMacks (talk) 16:45, 3 January 2025 (UTC)
- To add your business to Wikipedia, it must meet WP:Notability (organizations), with significant coverage in independent, reliable sources. Avoid promotional content and write neutrally. If you’re connected to the business, disclose your conflict of interest and suggest edits instead of directly creating or editing. Use the WP:Article wizard to draft and submit for review. But honestly speaking: take the advice from @331dot- go on about the work of your company Ayohama (talk) 17:17, 3 January 2025 (UTC)
Weatherbox placement
Can't help but feel the weatherbox on the Cherry Hill, New Jersey article is in a space that leaves a lot of blank space behind. Can it be moved easily? (I should clarify I'm unsure how to do this) Finitestory (talk) 00:34, 4 January 2025 (UTC)
- Yes, the weatherbox can be moved easily. Edit the article, find the weatherbox code, and relocate it to a section where it fits better, like "Climate" or below another relevant subsection, to reduce blank space. Ayohama (talk) 07:22, 4 January 2025 (UTC)
New page reviewer questions
So I'm currently applying for NPR rights, and I have a couple questions about what I can do with NPR rights:
• Am I allowed to review pages I created?
• Do people with NPR rights only review articles or do they also review redirects?
• Can reviews be undone (that is, can an article be unreviewed if need be)? RedactedHumanoid (talk) 02:11, 4 January 2025 (UTC)
- Hello, RedactedHumanoid! In order, no, both, and yes. A little more detail:
- You can’t review pages you created, but creators who create a lot of unproblematic articles are often assigned the autopatrolled right so that reviewers don’t have to check extra articles unnecessarily.
- NPPers can and do review both articles and redirects. In fact, there’s currently a backlog elimination drive aimed at reviewing both.
- Articles can be unreviewed.
- Hope this helps! Happy editing, Perfect4th (talk) 03:02, 4 January 2025 (UTC)
- Alright, thank you. RedactedHumanoid (talk) 03:11, 4 January 2025 (UTC)
Notability of Isa Isarb
Besides Nigel Ng, Isa Isarb is one of the most famous non-brand Youtube Channel with over 2,7 million subscribers. Cited by many and also mainly using Malay language Protocetus1 (talk) 04:04, 4 January 2025 (UTC)
- Teahouse is the wrong place to suggest an article should be created. There is a requests site but it is pretty much a "dead letter", meaning few if any article suggestions are acted upon. David notMD (talk) 05:41, 4 January 2025 (UTC)
- Protocetus1, being
one of the most famous
and havingover 2,7 million subscribers
is not in itself evidence of notability. What is required to establish notability are references to several published reliable sources that are entirely independent of Isarb and that devote significant coverage to Isarb. Nothing else matters. Cullen328 (talk) 08:27, 4 January 2025 (UTC)
- Protocetus1, being
Page text length and "stub" status
At present time, is a page developed to a length of, say, 400 words—not counting many listed references—still considered a stub?Joel Russ (talk) 04:16, 4 January 2025 (UTC)
- Stub status depends on content depth and coverage, not just word count. A 400-word page could still be a stub if it lacks comprehensive information, see Wikipedia:Stub Ayohama (talk) 07:20, 4 January 2025 (UTC)
Saving a template for re-use in high volume
I use the "r from alternative language" template all the time. I would like to know if there is a way to save a template or have like a quick-use list of 3 to 5 templates that I use frequently when creating valid redirects from varied different languages etc.?
So for example. Lets say I see a town in Palestine or Israel that has an Arabic or Hebrew language word that could use a redirect that then requires the template "r from alternative language", I want to be able to just click that "r from alternative language" quickly without needing to type that full text in or even partial text in a search for the template again each and every time to use it. Iljhgtn (talk) 04:21, 4 January 2025 (UTC)
- @Iljhgtn: One really good but rarely-mentioned tool that can help you with this is TemplateScript. I'll be Working on an implementation of that tool specifically for this purpose. JJPMaster (she/they) 05:24, 4 January 2025 (UTC)
- Thank you. That would be great! Iljhgtn (talk) 05:28, 4 January 2025 (UTC)
- I figured it out; add the following to Special:MyPage/common.js:
- Thank you. That would be great! Iljhgtn (talk) 05:28, 4 January 2025 (UTC)
//<nowiki>
$.ajax('//tools-static.wmflabs.org/meta/scripts/pathoschild.templatescript.js', { dataType:'script', cache:true }).then(function() {
pathoschild.TemplateScript.add([
{
category: 'Redirect templates',
name: 'R from alternative language (Hebrew to English)',
template: '{{R from alternative language|he|en}}',
position: 'after',
editSummary: 'Adding redirect template'
},
{
category: 'Redirect templates',
name: 'R from alternative language (Arabic to English)',
template: '{{R from alternative language|ar|en}}',
position: 'after',
editSummary: 'Adding redirect template'
}
]);
});
//</nowiki>
Then, disable the "Syntax" button in the editing window (if it's enabled). Then, on the sidebar, you should see a new section called "Redirect templates" with the two templates I've seen you use in Special:NewPagesFeed. If you ever need to use more, you can add a similar JSON object to the existing ones, or check out Capricorn. JJPMaster (she/they) 06:13, 4 January 2025 (UTC)
Hang on...
How come every time I expand an article, the view count suddenly spikes? TWOrantulaTM (enter the web) 05:00, 4 January 2025 (UTC)
- Hey @TrademarkedTWOrantula Expanding an article can trigger increased views due to it appearing in more searches, being featured on watchlists, or drawing attention from other editors and readers curious about the changes I guess. Ayohama (talk) 07:18, 4 January 2025 (UTC)
- @TrademarkedTWOrantula: If "spikes" just means a significant percentage increase from a very low starting point then it could be your own views while editing the article. For example, you edited Pictoword three times yesterday and it went from around six daily views to 16 that day.[6] PrimeHunter (talk) 08:56, 4 January 2025 (UTC)